metzler physik

216
Joachim Grehn – Joachim Krause Metzler Physik Lösungen Schroedel

Upload: felix-nindl

Post on 05-Dec-2014

8.952 views

Category:

Documents


696 download

TRANSCRIPT

Page 1: Metzler Physik

Joachim Grehn ± Joachim Krause

Metzler PhysikLösungen

Schroedel

Page 2: Metzler Physik

Metzler Physik3. Auflage

Lösungen

Herausgegeben von Joachim Grehn und Joachim Krause

Bearbeitet von

Dr. Joachim BolzJoachim GrehnJoachim KrauseDr. Herbert Kurt SchmidtDr. Heiner Schwarze

Unter Mitarbeitder Verlagsredaktion

ISBN 3-507-10702-3

, 1999 Schroedel Verlag Hannover

Alle Rechte vorbehalten. Dieses Werk sowie einzelne Teile desselben sind urheberrechtlichgeschützt. Jede Verwertung in anderen als den gesetzlich zugelassenen Fällen ist ohne vor-herige schriftliche Zustimmung des Verlages nicht zulässig.

Druck 5 4 3 2 1/Jahr 2003 2002 2001 2000 1999

Zeichnungen: Günter Schlierf

Satz: Satz-Offizin Hümmer GmbH, WaldbüttelbrunnDruck: Oeding Druck GmbH, Braunschweig

Page 3: Metzler Physik

Inhaltsverzeichnis

Vorwort 5

1 Mechanik1.1 Kinematik 71.2 Dynamik; Impuls und Kraft 291.3 Energie und Energieerhaltung 411.4 Die Rotation starrer Körper 49

2 Gravitation2.1 Das Gravitationsgesetz 532.2 Das Gravitationsfeld 572.3 Bewegungen im Gravitationsfeld 61

3 Mechanische Schwingungen und Wellen3.1 Schwingungen 683.2 Überlagerung von Schwingungen 723.3 Entstehung und Ausbreitung von Wellen 743.4 Wechselwirkung von Wellen 77

4 Wärmelehre4.1 Grundlagen 814.2 Die kinetische Gastheorie 834.3 Energieumwandlungen 844.4 Die Strahlungsgesetze 864.5 Die Entropie 87

5 Ladung und elektrisches Feld5.1 Elektrische Ladung und elektrischer Strom 885.2 Das elektrische Feld 905.3 Elektrische Stromkreise 975.4 Bewegung geladener Teilchen im elektrischen Feld 103

6 Bewegte Ladungsträger und magnetisches Feld6.1 Kräfte im magnetischen Feld 1076.2 Ströme als Ursache von Magnetfeldern 1116.3 Elektromagnetische Induktion 114

7 Elektromagnetische Schwingungen und Wellen7.1 Wechselstromtechnik 1197.2 Elektrische Schwingungen und elektromagnetische Wellen 1277.3 Licht als klassische Welle 1337.4 Das elektromagnetische Spektrum 145

8 Chaotische Vorgänge8.3 Wege ins Chaos ± Verhulst-Dynamik und Feigenbaum-Szenario 147

Inhaltsverzeichnis 3

Page 4: Metzler Physik

9 Relativitätstheorie9.1 Von der klassischen Physik zur Relativitätstheorie 1499.2 Die Kinematik der speziellen Relativitätstheorie 1519.3 Die Dynamik der speziellen Relativitätstheorie 162

10 Einführung in die Quantenphysik10.1 Grundlagen 16910.2 Verteilung der Photonen 17210.3 Ausbreitung von Elektronen 17310.4 Quantenphysik und klassische Physik 174

11 Atomphysik11.1 Energieaustausch mit Atomen 17511.2 Entwicklung der Atommodelle 17711.3 Das Atommodell der Quantenphysik 18011.4 Leistung der Atommodelle 183

12 Elektronik und Festkörperphysik12.1 Halbleiterschaltungen 18712.2 Das quantenhafte Modell des Festkörpers 18912.3 Analoge Signalverarbeitung 19112.4 Digitalelektronik 192

13 Kernphysik13.1 Natürliche Radioaktivität 19513.2 Aufbau und Systematik der Atomkerne 19813.3 Kernumwandlung und radioaktive Strahlung 20013.4 Energie der Atomkerne; Kernmodelle 20213.5 Künstliche Kernumwandlung und Kernspaltung 20513.6 Nutzung der Kernenergie 208

14 Teilchenphysik14.1 Standardmodell 209

15 Astrophysik15.1 Die Erforschung des Universums 21215.2 Die Sterne 213

Inhaltsverzeichnis4

Page 5: Metzler Physik

Liebe Kollegin, lieber Kollege,

in dem vorliegenden Buch finden Sie die Lösungen der 771 Aufgaben aus der 3. Auflage derMetzler Physik. Darüber hinaus sind einige weitere Zusatzaufgaben mit ihren Lösungen aufge-führt, die aus der vorhergehenden Auflage übernommen sind und die Sie nach Belieben in IhremUnterricht einsetzen können.

Neben dem Ergebnis sind im allgemeinen auch ± auûer bei einfacheren Aufgaben ± die Lösungs-wege skizziert. Diese sind soweit aufbereitet, dass Sie zum einen in der Unterrichtsvorbereitungden Schwierigkeitsgrad und die Besonderheiten der Aufgaben leicht überblicken und zum an-deren bei der Besprechung der Hausaufgaben Ansatz, Lösungsweg und Ergebnis schnell nach-lesen können.

Bei manchen Aufgaben sind verschiedene Lösungswege gangbar, von denen hier jeweils nureiner aufgeführt ist. Je nach dem Vorgehen wird man zu Werten kommen, die möglicherweise inder Genauigkeit von den vorgelegten Ergebnissen abweichen. Darauf wird besonders derjenigezu achten haben, der ± ohne selbst die Aufgaben durchzurechnen (dies Ihnen zu ersparen, ist jagerade die Absicht dieses Lösungsbuches) ± die Lösungen des Lehrerbegleitheftes zur Kontrollez. B. der Hausaufgaben heranzieht. Die Genauigkeit in den Stellen nach dem Komma hängt jadavon ab, mit welcher Stellenzahl Ausgangswerte und Konstanten in die Rechnung eingehenoder Zwischenergebnisse abgerundet und weiter verwendet werden.

Schwierige Aufgaben sind besonders gekennzeichnet. Sie mögen selbst entscheiden, ob und inwelcher Form Sie Hilfen zur Lösung einzelner Aufgaben geben wollen. In jedem Fall lohnt sichein Blick auf den Lösungsweg, bevor man die Aufgaben stellt, um vor unerwarteten Schwierig-keiten sicher zu sein, vor denen sonst Ihre Schülerinnen und Schüler z. B. in ihrer häuslichenArbeit stehen könnten.

Inzwischen stehen in allen Schulen auch im Physikunterricht Computer zur Verfügung. So kön-nen in heutiger Zeit auch Methoden zur Bearbeitung von Problemen eingesetzt werden, diefrüher aufgrund ihrer mathematischen Schwierigkeiten einer Behandlung nicht zugänglich wa-ren.

Das betrifft insbesondere das iterative Verfahren, bei dem es sich im Grundsatz um die Methodeder graphischen Integration handelt. Sie wird in der Metzler Physik mit dem einfachen Ganz-schritt- oder Halbschrittverfahren nach Runge-Kutta im Abschnitt ¹Numerische Methodeª aufSeite 24 des Lehrbuches eingeführt. Zahlreiche interessante Fragestellungen lassen sich mitdiesem leicht verständlichen Iterationsverfahren behandeln und für den Unterricht fruchtbarmachen. Beispiele für Aufgaben, die mit dieser Methode bearbeitet werden können, findet derLeser an vielen Stellen. Wir haben Wert darauf gelegt, dass die Anwendung dieses Verfahrens anallen Stellen des Buches nach einem einheitlichen Muster geschieht.

Die Anwendung dieser eleganten Methode mag an einigen Stellen auch ausgespart werden kön-nen, ohne dass der Erkenntnisgewinn damit grundsätzlich eingeschränkt wird. Für die Behand-lung anderer Gebiete, wie sie hier in der Metzler Physik erstmals vorgestellt werden, sind sie aberunabdingbar, will man sich auf den Unterrichtsgang des Buches einlassen. Das betrifft z. B. dasKapitel 10, Einführung in die Quantenphysik, speziell hier die Behandlung der Schrödinger-Gleichung, die uns aufgrund dieser Methode dem Unterricht auf der Oberstufe zugänglich zusein scheint.

Vorwort 5

Page 6: Metzler Physik

Die Lösung der Aufgaben dieses Kapitels setzen hier doch eine sorgfältige unterrichtliche Be-handlung der mathematischen Methode voraus, damit der mathematische Formalismus den phy-sikalischen Inhalt nicht überdeckt. Darauf sollte man in der Unterrichtsplanung achten. Je früherdas Instrument wie hier in der Aufgabensammlung vorgeschlagen, eingesetzt und geübt wird,umso einfacher werden es die Schülerinnen und Schüler auch bei der Behandlung komplizierterAufgaben handhaben können.

Wir haben für die Aufgabensammlung des Buches aber auch Problemlösungen vorgeschlagen,die doch schon einige Kenntnis in der Behandlung von Programmen voraussetzen. Solche Me-thodenanwendungen finden Sie in Kapitel 7.3 und 7.4, Optik, wo es um die Intensitätsberech-nung von Beugungsfiguren geht, und weiter im Kapitel 8, Chaos. Wir waren der Meinung, dassSchülerinnen und Schüler anhand dieser Beispiele, die physikalisch von einigem Interesse sind,erfahren sollten, wie auf einfachen physikalischen Überlegungen aufbauend, ein Programm alsWerkzeug zu weitreichenden Erkenntnissen führen kann. Sie werden diese Aufgaben sicherlichnicht ohne Vorbesprechung im Unterricht bearbeiten lassen können.

Der Hinweis sei hier angebracht, dass Iterationsmethoden und Programme nicht nur an den in denAufgaben vorgeschlagenen Stellen eingesetzt werden können. Sie werden sicherlich an vielenanderen Stellen von der Möglichkeit ihres Einsatzes Gebrauch machen.

Ein letztes Wort, obwohl es nicht im engeren Sinne die Aufgaben des Lehrbuches betrifft, derenLösungen hier in diesem Lösungsbuch zusammengestellt sind, gilt den Zusatzinformationen, die± mit gelbem Strich gekennzeichnet ± über das ganze Lehrbuch verstreut sind. Sie erfüllen ver-schiedene Zwecke. Einmal zeigen sie als interessante Zusatzinformation, wie die vielleicht ab-strakte Physik doch in Technik und Umwelt überall Anwendung findet. Sie stellen aber auch ±und deshalb sind sie hier im Lösungsbuch erwähnt ± ¹Aufgabenª eigener Art dar. Der Auftrag aneinzelne oder die ganze Lerngruppe kann dann z. B. lauten, die Zusatzinformationen durchzu-arbeiten und durch eigenen Recherchen anzureichern.

Die Autoren haben sich gröûte Mühe bei der Bearbeitung der Aufgaben und Lösungen gegeben.Sie hoffen, Ihnen mit dem vorliegenden Buch einen zuverlässigen Helfer zur Verfügung zustellen. Der Teufel kann aber auch hier im Detail stecken. Daher sind die Herausgeber jedemdankbar, der sie auf Fehler und Unrichtigkeiten hinweist.

Kiel und Neumünster, im Herbst 1999

Joachim Grehn und Joachim Krause

Vorwort6

Page 7: Metzler Physik

1.1.1 Beschreibung einer Bewegung

1 In einem Zug (auf gerader Strecke mit konstanterGeschwindigkeit) lässt man einen Ball fallen. Be-schreiben Sie seine Bewegung vom Zug und vomGleis als Bezugssystem aus.

Lösung:Vom Zug aus: senkrecht nach unten;vom Gleis aus: Parabelbahn, wie beim waagerech-

ten Wurf.

1.1.2 Die geradlinige Bewegung mit konstanter Geschwindigkeit

1 Ein Wagen durchfährt eine 1,6 km lange Streckein 24 s. Wie groû ist die Geschwindigkeit in m/s,km=h, m=min?

Lösung:vDs=Dt 1,6 km/24 s 66,7 m=s 240 km=h 4000 m/min.

2 Wie groû sind die Geschwindigkeiten in m=s undkm=h, mit der a) die Erde um die Sonne, b) derMond um die Erde läuft? (Astronomische Datenim Anhang).

Lösung:a) Als (wahre) Umlaufszeit der Erde um die Sonne

legt man das siderische Jahr zugrunde, weilnach dieser Zeit die mittlere Sonne wieder amselben Punkt (Stern) (sidus (lat.), Stern) derHimmelskugel steht.

v 2 pRT 2p 149,6 109 m

3,1557 107 s

29,78 103 m=s 1,072 ´ 105 km=h.

b) Entsprechend hat nach einem siderischen Mo-nat der Mond wieder die gleiche Stellung vorden Sternen.

v 2 pRT 2p 3,844 108 m

27,3217 24 60 60 s

1,023 103 m=s 3,683 ´ 103 km=h.

3 Zu einer geradlinigen Bewegung gehört das rechtsstehende Zeit-Weg-Diagramm.a) Berechnen Sie die Intervallgeschwindigkeiten.b) Zeichnen Sie das Zeit-Geschwindigkeit-Dia-

gramm.

in h

Lösung:a) v0ÿ2 2,5 km=h, v2ÿ3 5,0 km=h,

v3ÿ4;35 0, v4;35ÿ5 ÿ 7,14 km=h,v5ÿ7 ÿ5,0 km/h.Die Indizes geben die Anfangs- und Endzeitender Intervalle in Sekunden an.

b) υin km/h

t in h0

2

4

–2

–4

–6

2 4 6 8 10 12

4 a) Zeichnen Sie das Zeit-Weg-Diagramm der fol-genden linearen Bewegung (zwischen denPunkten verläuft die Bewegung gleichförmig):

P0 P1 P2 P3 P4 P5

t in s 0,0 1,5 4,5 6,0 9,0 10,5

s in m 4,0 5,0 6,0 6,0 3,0 0,0

b) Berechnen Sie die Wege und Zeiten zwischenje zwei aufeinander folgenden Punkten und diezugehörigen Geschwindigkeiten.

c) Stellen Sie die Zeit-Weg-Funktionen für diefünf Wegabschnitte auf. Beachten Sie die Vor-zeichen.

1.1 Kinematik 7

Seite

13

Seite

17

Page 8: Metzler Physik

Lösung:

a)s in m

0 1 t in s5 100

5

P0

1

23

4

51

PP

P

P

P

b) Weg Zeit Geschwindigkeit

s01 1,0 m t01 1,5 s v01 0,67 m=s

s12 1,0 m t12 3,0 s v12 0,33 m=s

s23 0 m t23 1,5 s v23 0

s34 ÿ3,0 m t34 3,0 s v34 ÿ1,0 m=s

s45 ÿ3,0 m t45 1,5 s v45 ÿ2,0 m=s

c) Von P0 nach P1:von P1 nach P2:von P2 nach P3:von P3 nach P4:von P3 nach P4:

s 0,67 m=s t 4,0 m,s 0,33 m=s t 4,5 m,s 6,0 m,s ÿ1,0 m=s t 12,0 m,s ÿ2,0 m=s t 21,0 m.

5 In A startet um 9.00 Uhr ein LKW und fährt mit derGeschwindigkeit v1 50 km=h zum 80 Kilometerentfernten B. 30 Minuten später startet ein zweiterLKW mit der Geschwindigkeit v2 78 km=h vonB aus nach A.a) Wann und wo treffen sich die Fahrzeuge?b) Zeichnen Sie das Zeit-Weg-Diagramm und lö-

sen Sie die Aufgabe auch grafisch.

Lösung:a) Lkw I ist nach einer halben Stunde s1

v1 t1 50 km=h ´ 0,5 h 25 km weit von Aentfernt, hat also bis B noch 55 km zurückzu-legen.Jetzt startet Lkw II. Von 9 Uhr 30 ab haben Iund II bis zum Treffpunkt zusammen 55 kmzurückzulegen, also v1 Dt v2 Dt 55 kmoder Dt 25,8 min.Sie treffen sich also 25,8 min nach 9 Uhr 30,also 9 Uhr 55,8 min an der Stelle s0 25 km v1Dt 46,5 km von A nach B.

b)s in km

09.00

0,59.30

1,010.00

1,510.30

t in h0

10

50

80

A

B

Lkw II Lkw I

s0

t0

C

*6 Ein Wagen fährt von A nach B (Entfernung150 km) mit der Durchschnittsgeschwindigkeitv1 120 km=h:a) Wann muss ein zweiter Wagen von C (C liegt

zwischen A und B, und zwar 76 km vor B) ausstarten, wenn er mit der Geschwindigkeitv2 80 km=h gleichzeitig mit dem ersten Wa-gen in B ankommen will?

b) Wie weit ist der erste Wagen beim Start deszweiten noch vom Ziel entfernt?

c) Zeichnen Sie das Zeit-Weg-Diagramm und lö-sen Sie die Aufgabe auch zeichnerisch.

Lösung:a) W1 trifft nach

te sAB=v1 150 km=(120 km=h) 1,25 h 1 h 15 min in B ein.

W2 benötigt von C nach B:Dt2 sCB=v2 76 km=(80 km=h) 0,95 h 57 min.

Also startet W2 um t02 1,25 hÿ 0,95 h 0,3 h 18 min später als W1.

b) Zu dieser Zeit ist W1 die Strecke s1 v1t02

120 km=h ´ 0,3 h 36 km gefahren. W1 istalso zu diesem Zeitpunkt Ds1 150 km=h ÿ36 km 114 km von B entfernt.

1 Mechanik8

Page 9: Metzler Physik

c)s in km

0 0,5 1,0 1,5 t in h0

50

100

150

A

B

s1

76CW1

2W

2W

E

t02 te

Zeichnerische Lösung: t02 18 min, s1 34 km,also 116 km vor B. Wichtig ist die geeignete Wahlder Achseneinteilung, z. B. auf der t-Achse 1 hé 6,0 cm, auf der s-Achse 100 km é 5,0 cm.

Zusatzaufgaben

7 An der Straûe von A-Stadt nach der 20 km entfern-ten C-Stadt liegt 4 km von A-Stadt entfernt B-Dorf.Ein Radfahrer (I) startet in B-Dorf und kommt nach1,5 Std. in C-Stadt an. Zur gleichen Zeit wie Rad-fahrer I in B-Dorf fährt ein anderer Radfahrer (II) inC-Stadt los und erreicht zur selben Ankunftszeitwie Radfahrer I A-Stadt.a) Zeichnen Sie das Zeit-Weg-Diagramm

(1 km é 0,5 cm; 1 h é 6 cm).b) Berechnen Sie die Geschwindigkeit beider Rad-

fahrer.c) Konstruieren Sie das Zeit-Geschwindigkeit-

Diagramm (1 km=h é 0,5 cm; 1 h é 6 cm).d) Wann begegnen sich die Radfahrer? Zeichneri-

sche und rechnerische Lösung.

Lösung:

a)

C-Stadt

A-Stadt

B-Dorf 4

0

20

11,2

s in km

0

III

0,5 0,67 1,0 1,5 t in h

b) I: vDs=Dt 16 km=1,5 h 10,67 km=h 2,96 m=s,II: vDs=Dt 20 km=1,5 h 13,33 km=h 3,70 m=s.

c)υ

in km/h

t in h0

5

10

0 0,5 1,0 1,5 2,0

III

d) Zeichnerische Lösung aus a):t0 0,67 h, s0 11,2 km.Rechnerische Lösung:Zeit-Weg-Funktion I:s 4 km (10,67 km=h) t,Zeit-Weg Funktion II:s 20 km (± 13,33 km=h) t.

Gleichsetzen liefertt0 2

3 h 0,67 h, s0 11 19 km 11,1 km.

1.1.3 Die geradlinige Bewegungmit konstanter Beschleunigung

1 Ein Bob hat vom Start an die gleich bleibende Be-schleunigung von 2 m=s2.a) Wie schnell fährt er 5 Sekunden nach dem

Start?b) Welchen Weg hat er bis dahin zurückgelegt?c) Wie groû ist bis zu dieser Zeit seine

Durchschnittsgeschwindigkeit?d) Wie weit ist er gefahren, wenn seine Geschwin-

digkeit auf 20 m=s angewachsen ist?

Lösung:Mit a 2 m=s2 ist nach s 1

2 a t2 und v a t

a) v 10 m=s, b) s 25 m,c) v 5 m=s, d) für t 10 s wird s 100 m.

2 Ein Auto wird aus dem Stand in 10,2 s auf eineGeschwindigkeit von 100 km/h konstant beschleu-nigt und dann nach einem Bremsweg von 96 mwieder zum Stehen gebracht.

Seite

20

1.1 Kinematik 9

Page 10: Metzler Physik

a) Wie groû sind die Beschleunigungen (m=s2,km=h2)?

b) Wie groû sind der Weg beim Anfahren und dieZeit beim Bremsen?

Lösung:a) Beschleunigungsvorgang:

a 2,72 m=s2 3,53 ´ 104 km=h2,Bremsvorgang:a 4,02 m=s2 5,21 ´ 104 km=h2.

b) Beschleunigungsweg:s 1

2 a t2 141,5 m,Bremszeit:t v=a 6,91 s.

3 Eine U-Bahn legt zwischen zwei Stationen einenWeg von 3 km zurück. Berechnen Sie aus der mitt-leren (betragsmäûig gleichen) Anfahrts- undBremsbeschleunigung a 0,6 m=s2 und derHöchstgeschwindigkeit vmax 90 km=h Anfahr-weg, Bremsweg, Wegstrecke der gleichförmigenBewegung und die einzelnen Fahrzeiten.

Lösung:Anfahrtsweg Bremsweg:s0 v2=2 a 520,8 m,

Anfahrtszeit Bremszeit:t0 v=a 41,7 s,

Weg mit konst. Geschwindigkeit:s1 s ÿ 2 s0 1 958,3 m,

Zeit dort:t1 s1=v1 79,4 s,

Gesamtfahrzeit:t 2 t0 t1 162,8 s.

4 Welche Beschleunigung ergibt sich aus der Regel:¹Den Bremsweg in m erhält man, indem man dieGeschwindigkeit in km=h durch 10 teilt und dasErgebnis quadriertª?

Lösung:Mit der Geschwindigkeit vkm in km=h, der Ge-schwindigkeit vm in m=s, der Beschleunigung am

in m=s2 und dem Weg sm in m lautet die Regel

(vkm=10)2 sm.

(Für vkm, sm werden nur die Zahlenwerte der Grö-ûen eingesetzt; es ergeben sich auch nur die Zah-lenwerte.)

Es gilt nämlich mit vkm 3,6 vm und v2 2 a sfür den Weg sm v2

m=2am

und damit die Regel

3,6 vm

10

2

sm v2

m

2am.

Daraus ergibt sich die Beschleunigungam 3,86 m=s2.Die Regel geht also von einer geringen, aber reali-stischen Beschleunigung aus.

5 Welche der beiden Geschwindigkeiten v1 und v2 zuden Zeiten t1 und t2 ist in den drei Zeit-Weg-Kurvender folgenden Abbildung jeweils gröûer als die an-dere? Begründen Sie.

Lösung:a) v1 v2 (Die Steigung, die die Geschwindigkeit

angibt, ist konstant.)b) v1 > v2 (Die Steigung nimmt ab.)c) v1 < v2 (Die Steigung nimmt zu: erst negativ,

dann positiv.)

6 Entscheiden und begründen Sie, um welche Be-schleunigungen es sich bei den Zeit-Weg-Kurvenin der folgenden Abbildung handelt.

Lösung:a) a 0, b) a > 0, c) a < 0.

*7 Zu einer bestimmten Zeit t1 3,0 s bewegt sichein Körper mit der Geschwindigkeit v1 2,6 m=s;zur Zeit t2 8,0 s ist seine Geschwindigkeitv2 ÿ 1,5 m=s. Wie groû ist seine mittlere Be-schleunigung?

1 Mechanik10

Page 11: Metzler Physik

Lösung:

aDvDtÿ1,5 m=sÿ 2,6 m=s

8,0 sÿ 3,0 sÿ4,1 m=s

5,0 s

ÿ0,82 m=s2.

8 Zu einer geradlinigen Bewegung gehört das Zeit-Geschwindigkeit-Diagramm der folgenden Abbil-dung.a) Berechnen Sie die Beschleunigungen in den

drei Intervallen und zeichnen Sie das zugehö-rige Zeit-Beschleunigung-Diagramm.

b) Wie lauten die Zeit-Geschwindigkeit-Funktio-nen in den drei Intervallen?

c) Wie groû sind die Teilwege und der Gesamt-weg?

Lösung:a) Beschleunigung in den Intervallen

a0ÿ2 1,0 m=s2, a2ÿ5 0,33 m=s2,a5ÿ6 ÿ4,0 m=s2. (Die Indizes geben die Zeitin Sekunden an.)

in m/s2a

–4,0

–2,0

0,331,02,0

2 4 6 8 10 t in s

b) Zeit-Geschwindigkeit-Funktionen:v (1,0 m=s2) t 1 m=s für 0 s t 2 s,v (0,333 m=s2) t2,333 m=s für 2 s t 5 s,v (ÿ 4,0 m=s2) t 24 m=s für 5 s t 6 s.

c) Die Teilwege errechnen sich zu:D s0ÿ2 1

2 (1,0 m=s2) ((2 s)2ÿ 02) (1 m=s) (2 s ÿ 0) 4 m,

D s2ÿ5 12 (0,333 m=s2) ((5 s)2ÿ (2 s)2)

(2,333 m=s) (5 s ÿ 2 s) 10,5 m,D s5ÿ6 1

2 (ÿ 4,0 m=s2) ((6 s)2ÿ (5 s)2) (24 m=s) (6 s ÿ 5 s) 2 m.

Der Gesamtweg beträgt 16,5 m.

9 Die folgende Tabelle zeigt die Wertetabelle einesFahrbahnversuchs.a) Zeichnen Sie das Zeit-Weg-Diagramm.b) Berechnen Sie die Intervallgeschwindigkeiten

und zeichnen Sie das zu a) gehörige Zeit-Ge-schwindigkeit-Diagramm. Tragen Sie dazu dieGeschwindigkeiten über der Mitte der Inter-vallzeiten ab.

c) Beschreiben und charakterisieren Sie die Be-wegung.

t in s 0,0 0,2 0,4 0,6 0,8 1,0

s in cm 3,2 4,5 8,4 14,8 23,8 35,4

t in s 1,2 1,4 1,6 1,8 2,0

s in cm 49,6 65,1 80,6 96,0 111,5

Lösung:

a) s in cm

0,2

10

50

100

3,2

1,0 1,8 t in s

b) Intervallgeschwindigkeiten:

t in s 0,0 0,2 0,4 0,6 0,8 1,0

v in cm=s 6,5 19,5 32,0 45,0 58,0

t in s 1,0 1,2 1,4 1,6 1,8 2,0

v in cm=s 70,5 77,5 77,5 77,0 77,5

in cm/s

0,2

10

50

100

1,0 t in s2,0

υ

c) Von t0 0,0 s bis t1 1,2 s eine gleichmäûigebeschleunigte Bewegung, danach eine gleich-förmige Bewegung.

1.1 Kinematik 11

Page 12: Metzler Physik

*10 Die gleichmäûig beschleunigte Bewegung einesFahrbahnwagens wird mit Messlatte und elektri-scher Stoppuhr registriert. (In der letzten Zeile derTabelle wird die Zeit Dt angegeben, in der einDs 1,48 cm breiter Bügel des Fahrbahnwagenseine Lichtschranke passiert.)a) Berechnen Sie (Mittelwertbildung) die Be-

schleunigung und geben Sie die Bewegungs-gleichungen an.

b) Berechnen Sie die mit der Lichtschranke ge-messenen (Intervall-)Geschwindigkeiten.

c) Bestimmen Sie aus den (Intervall-)Geschwin-digkeiten in b) ebenfalls durch Mittelwert-bildung die Beschleunigung.

d) Geben Sie für a) und c) die mittleren Fehler anund überprüfen Sie, ob die Beschleunigungeninnerhalb der Fehlergrenzen der anderen Mes-sung liegen (! S. 29).

e) Überprüfen Sie die Geschwindigkeiten in b)mit den aus dem Weg s und der in a) rechne-risch ermittelten Beschleunigung a.

f) Zeichnen Sie das Zeit-Weg- und nach denRechnungen wie in b) das Zeit-Geschwindig-keit-Diagramm.

s in cm 0 10 20 30 40 50

t in s 0 1,05 1,51 1,83 2,13 2,37

Dt in ms ± ± 54,9 ± 39,7 ±

s in cm 60 70 80 90 100

t in s 2,63 2,85 3,04 3,23 3,37

Dt in ms 32,6 ± 28,3 ± 25,4

Lösung:

s in cm 0 10 20 30 40 50

a) a1 2 st2

in cm=s2 18,1 17,5 17,9 17,6 17,8

b) vm D sD t

in cm=s 27,0 37,3

c) a2 vm

tin cm=s2 17,9 17,5

d)vm

2a1spin cm=s 26,5 37,5

s in cm 60 70 80 90 100

a) a1 2 st2

in cm=s2 17,3 17,2 17,3 17,3 17,6

b) vm D sD t

in cm=s 45,4 52,3 58,3

c) a2 vm

tin cm=s2 17,3 17,2 17,3

d)vm

2a1spin cm=s 45,9 53,0 59,3

a) Die mittlere Beschleunigung ista1 17,56 cm=s2 mit dem Fehler Da1 0,30 cm=s2.Die Bewegungsgleichungen lautens (8,78 cm=s2) t2, v (17,56 cm=s2) t,a1 17,56 cm=s2.

b) Siehe oben.c) Die mittlere Beschleunigung, die sich aus dem

Messungen mit der Lichtschranke ergibt, ista2 17,44 cm=s2 mit dem Fehler Da2 0,28 cm=s2.

d) Siehe a) und c). Da Da a1ÿ a2 0,12 cm=s2, liegt jede mittlere Beschleunigunga1 und a2 innerhalb der Fehlergrenzen der an-deren.

e) Siehe oben.f)

υmin cm/s

100 t in 10 s–2200 300

s in cm

100 t in 10 s–2200 300

100

50

10

50

10

*11 Vor einem Zug, der mit der Geschwindigkeitv1 120 km=h dahinfährt, taucht plötzlich ausdem Nebel in 1 km Entfernung ein Güterzugauf, der in derselben Richtung mit v2 40 km=hfährt. Der Schnellzug bremst (konstante Be-schleunigung) und hält nach 4 km.a) Wie lange dauert der Bremsvorgang des Zu-

ges?b) Wie lässt sich mit der Antwort zu a) entschei-

den, ob beide Züge zusammenstoûen?c) Wie berechnet man Zeit und Ort eines mögli-

chen Zusammenstoûes mithilfe der Bewe-gungsgleichungen?

Lösung:a) Aus v2

1 2 as1 und v1 a t1 erhält man mita ÿ 1,389 ´ 10ÿ1 m=s2 die Bremszeit t1 240 s.

1 Mechanik12

Page 13: Metzler Physik

b) In dieser Zeit durchfährt der Güterzug dieStrecke s2 v2 t1 2 667 m; es gibt einenZusammenstoû.

c) Die Zeit ts des Zusammenstoûes ergibt sichmit a < 0 aussSchnellzug v1 ts 1

2 a t2s

sGÈuterzug v2 ts s0 für ts 240 s

über die quadratische Gleichung

t2s 2

v1ÿ v2

atsÿ

2 s0

a 0 zu

ts1 54,2s (ts2 265,8 s).

Der Zusammenstoû ereignet sich (für denSchnellzug) ss1 1602 m nach Beginn desBremsvorganges.

*12 Der ICE 3 beschleunigt in 50 Sekunden auf eineGeschwindigkeit von 100 km=h, in 130 s auf200 km=h, in 6 min auf 300 km=h und in 11 minauf seine Höchstgeschwindigkeit von 330 km=h.a) Berechnen Sie die (mittleren) Beschleunigun-

gen.b) Stellen Sie den zeitlichen Verlauf der Ge-

schwindigkeit grafisch dar und berechnenSie in den Intervallen jeweils die mittlere Be-schleunigung.

c) AusderHöchstgeschwindigkeitvon330km=hbremst der ICE 3, wenn Generator-, Wirbel-strom- und Druckluftbremsen 100%ig wirk-sam sind, mit der mittleren Beschleunigungvon 1,24 m=s2. Wie lang ist der Bremsweg?Nach welcher Zeit kommt der Zug zum Ste-hen?

Lösung:a) Von 0 bis 50 s a v=t 0,556 m=s2,

von 0 bis 130 s a v=t 0,427 m=s2,von 0 bis 300 s a v=t 0,231 m=s2,von 0 bis 660 s a v=t 0,139 m=s2.

b) Von 0 bis 50 s a v=t 0,556 m=s2,von 50 bis 130 s aDv=Dt 0,347 m=s2,von 130 bis 300 s aDv=Dt 0,163 m=s2,von 300 bis 660 s aDv=Dt 0,023 m=s2.

in km/hυ

50 130 300 660 t in s

100

200

300330

c) DtDva 330 km=h

1,24 m=s2

330 m=s3,6 1,24 m=s2

73,9 s,

Ds 12 aD t2 1

2 1,24 m=s2 73,9 s2

3 385,95 3,4 km.

Zusatzaufgaben

13 Ein Auto steigert seine Geschwindigkeit gleich-mäûig von v1 120 km=h auf v2 150 km=h.Wie groû ist die Beschleunigung, wenn die Ge-schwindigkeitserhöhunga) in der Zeit von 10 Sekunden,b) entlang einer Strecke von 500 m erfolgt.Berechnen Sie jeweils zusätzlich den Weg bzw.die Zeit des Beschleunigungsvorganges.

Lösung:

a) Nach aDv=Dt ist a 0,833 m=s2.

b) Aus v2i 2 a si mit i 1, 2 gewinnt man

(v22ÿv2

1) 2 a (s2ÿs1) oder

a (v22ÿv2

1)=2 Ds:

Mit Ds 500 m ist a 0,625 m=s2.

14 Reicht die bekannte Regel für den Sicherheitsab-stand ¹Tachometerabstand (Abstand in m wie dieGeschwindigkeit in km=h) haltenª bei einer Ge-schwindigkeit von 100 km=h, wenn zwei Wagenmit der polizeilich verlangten Verzögerung von2,5 m=s2 bremsen können und der Fahrer deszweiten Wagens eine Reaktionszeit von t1 1,0 s bzw. t2 4,0 s besitzt?

Lösung:Das Zeit-Weg-Diagramm zeigt, dass unabhängigvon der für beide Wagen geltenden Bremsbe-schleunigung ein Zusammenstoû immer dann er-folgt, wenn der zweite Wagen innerhalb der Re-aktionszeit einen Weg durchfährt, der gröûer alsder Sicherheitsabstand ist.

1.1 Kinematik 13

Page 14: Metzler Physik

Nach der Regel ist der Sicherheitsabstand 100 m.In der Zeit t1 1,0 s durchfährt der zweite Wagenden Weg s1 27,8 m, bevor er bremst.In t1 4,0 s durchläuft der zweite Wagen denWeg s2 111,0 m (bei gleicher Bremsbeschleu-nigung).

15 Ein Auto (A) startet bei Grün vor einer Ampel underreicht nach 5 Sekunden bei konstanter Be-schleunigung die Geschwindigkeit vA 60 km=h,mit der es weiterfährt. Im Moment des Starts wirdes von einem anderen Auto (B) mit der (konstan-ten) Geschwindigkeit vB 40 km=h überholt.a) Wie lange dauert es, bis A so schnell fährt wie

B?b) Welchen Vorsprung besitzt zu dieser Zeit B

vor A?c) Welcher Wagen liegt am Ende des Beschleu-

nigungsvorganges von A vorne? Wie weit?d) In welcher Zeit und in welcher Entfernung

von der Ampel holt A das andere Auto ein?e) Wie sieht der Vorgang im t-v-Diagramm und

im t-s-Diagramm aus? (Zeichnung in den Ein-heiten m und s).

Lösung:a) Aus vA aA tA (tA 5 s) wird aA = 3,33 m=s2

und aus aA t0 vB ergibt sich t0 3,33 s.b) Für A gilt sA 1

2 aA t20 18,52 m und für

B sB vB t0 37,04 m. B hat vor A den Vor-sprung sBÿ sA 18,52 m.

c) Für A gilt mit tA 5 s: sA 12 aA t2

A 41,67 m, für B gilt sB vB tA 55,56 m.Nach dem Beschleunigungsvorgang liegt Bnoch mit sBÿ sA 13,89 m vor A.

d) Beim Zusammentreffen gilt sA sB oder12 aA t2

A vA (tsÿ tA) = vB ts und daraus tS 7,50 s. Zu diesem Zeitpunkt sind beide Wagenum die Strecke sA sB 83,33 m von derAmpel entfernt.

e) t-v-Diagramm

in km/sυ

t in s

100

0

50

A

B

3,33 5,0t0 A

10t

t-s-Diagramm

t in s

100

0

50

A

B

3,33 5,0t sA

1t

s in m

100

83,33

7,5

0t

Zur Zeit t0 erreicht A dieselbe Geschwindigkeit wieB, zur Zeit tA hat A seinem Beschleunigungsvor-gang beendet, zur Zeit ts überholt A den Wagen vonB.

16 Ein Autofahrer, der sich auf einer Ausfallstraûe(zugelassene Höchstgeschwindigkeit 60 km=h)mit v1 100 km=h bewegt, entdeckt plötzlich150 m vor sich eine parkenden Streifenwagender Polizei.a) Mit welcher Beschleunigung müsste der Fah-

rer (Reaktionszeit t 0,5 s) bremsen, wenn erden Streifenwagen mit 60 km=h passierenmöchte?

b) Leider hatte der Fahrer einen ersten Polizei-posten übersehen, der 250 m vor dem zweitenstand. Welche Durchschnittsgeschwindigkeitmisst die Polizei, wenn das Auto am zweitenPosten mit der vorgeschriebenen Höchstge-schwindigkeit vorbeifährt?

c) Wie groû hätte die Bremsbeschleunigung(statt der in a) sein müssen, wenn die Polizeizwischen beiden Posten eine (Durchschnitts-)Geschwindigkeit von 70 km=h hätte messensollen?

Lösung:a) Zwischen der Bremsstrecke Ds

150 m ÿ v1 tr 136,11 m (tr Reaktionszeit),der Beschleunigung a (a < 0), der Anfangsge-schwindigkeit v1 sowie der Endgeschwindig-keit ve 60 km=h gilt die Beziehungv2

e v21 2 a Ds und daraus a ÿ1,81 m=s2.

1 Mechanik14

Page 15: Metzler Physik

b) Für das Bremsen benötigt der Fahrer nach derBeziehung v1 a Dt ve die Zeit Dt 6,12s.Damit erreicht er auf der 250 m langen Streckedie Durchschnittsgeschwindigkeitvm 250 m=(t1 trDt) 24,46 m=s 88,06 km=h,

wobei t1 3,60 s die Zeit zum Durchfahren derersten 100 m mit der Geschwindigkeit v1 ist.

c) Die Bremsstrecke Ds (siehe a)) muss derartmit einer Bremsbeschleunigung a (a < 0) inder Zeit D tM durchfahren werden, so dass imDurchschnitt auf der 250 m langen Strecke diegeforderte Durchschnittsgeschwindigkeit vonvm 70 km=h erreicht wird.

Mit vm 250 m=(t1 trDtM) wird D tM 8,76 s.In dieser Zeit geht die Geschwindigkeit vonv1 100 km=h auf die (noch unbekannte)Geschwindigkeit ve zurück. In derselben Zeitkönnte die Bremsstrecke auch mit der mitt-leren Geschwindigkeit aus v1 und ve durch-fahren werden: (v1 ve)=2 Ds=Dt, woraussich ve 3,31 m=s 11,91 km=h und mitv1 a DtM ve die Bremsbeschleunigungaÿ 2,79 m=s2 berechnen.

1.1.4 Bewegungsgesetze und mathematische Methoden

1 Ein Körper bewegt sich längs der s-Achse nach derZeit-Weg-Funktions 1=2 m=s3 t32 m=s2 t2 3 m:Bestimmen Sie Geschwindigkeit und Beschleuni-gung und berechnen Sie ihre Werte für a) t 2,0 s,b) t 3,0 s.

Lösung:Zeit-Weg-Funktion und ihre Ableitungens (t) (0,5 m=s3) t3 (2 m=s2) t2 3 m,v (t) (1,5 m=s3) t2 (4 m=s2) t,a (t) (3 m=s3) t (4 m=s2).

a) t 2,0 s: s 15 m; v 14 m=s; a10m=s2,b) t 3,0 s: s 34,5 m; v 25,5 m=s; a 13 m=s2.

2 Zu gleicher Zeit starten bei t0 0 zwei Körper nachden Zeit-Weg-Gesetzen s1 1

20 m=s2 t2 unds2 1

2 m=s t:a) Berechnen Sie Geschwindigkeit und Beschleu-

nigung und zeichnen Sie die Diagramme biste 12 s:

b) Ermitteln Sie analytisch und zeichnerisch:Wann und wo überholt der erste Körper denzweiten? Wo haben beide dieselbe Geschwin-digkeit?

Lösung:a) _s1 v1 (0,1 m=s2) t, s1 a1 0,1 m=s2

_s2 v2 0,5 m=s, s2 a2 0.(Anmerkung: In der Aufgabenstellung muss dieGleichung für s2 folgendermaûen lauten:s2 1

2 m=s ´ t.)

Zeit-Weg-Diagramm

10

s in m

ts

1,00

0 1 5 t in s

5,0

1

2

Zeit-Geschwindigkeit-Diagramm

10

in m/s

ta

2,00

0 1 5 t in s

5,0

υ

2

1

Zeit-Beschleunigung-Diagramm

10

in m/s0,1

00 1 5 t in s2

1a 2

b) s1 (0,05 m=s2) t2s s2 (0,5 m=s) ts ergibt

rechnerisch ts 10 s, und ausv1 (0,1 m=s2) ta v2 0,5 m=s wird ta 5 s.

Seite

22

1.1 Kinematik 15

Page 16: Metzler Physik

*3 Ein Körper bewegt sich mit konstanter Beschleuni-gung a0 ÿ3,0 m=s2. Zu Beginn hat er die Aus-gangslage s0 24 m und die Anfangsgeschwindig-keit v0 6 m=s.a) Stellen Sie die drei Bewegungsgleichungen

auf.b) Wo und zu welcher Zeit kehrt der Körper seine

Bewegungsrichtung um?c) Wann erreicht er wieder die Ausgangslage?d) Zeichnen Sie die drei Diagramme der t-s-, der

t-v- und der t-a-Funktion.

Lösung:a) s ÿ (1,5 m=s2) t2 (6 m=s) t 24 m,

vÿ (3,0 m=s2) t 6 m=s,aÿ (3,0 m=s2).

b) Der Körper kehrt um, wenn v 0 ist:tu 2,0 s, und daraus su 30 m.

c) Der Körper erreicht seine Ausgangslage, fallss0 24 m ist: ta1 0, ta2 4,0 s.

d) t-s-Diagramm

t in s0 1 2 3 424

25

26

27

28

29

30

31s in m

t-v-Diagramm

in m/sυ

t in s0

1 2 3 4

2

4

6

–2

–4

–6

t-a-Diagramm

in m/s

t in s0

1 2 3 4

a2

–1

*4 Die Beschleunigung eines Körpers auf der s-Achsenimmt ständig proportional mit der Zeit nach derGleichung a 4,0 m=s2ÿ2,0 m=s3 t vom An-fangswert a0 4,0 m=s2 ab. Zur Zeit t0 0 besitztder Körper im Nullpunkt der s-Achse die Ge-schwindigkeit v0 ÿ 5=3 m=s:a) Wie lauten die Bewegungsgleichungen?b) Wann und an welcher Stelle ändert der Körper

seine Bewegungsrichtung?c) Wann und mit welcher Geschwindigkeit pas-

siert der Körper den Nullpunkt der s-Achse?d) Zeichnen Sie die zugehörigen Diagramme. Wie

kann man an ihnen die Antwort zu b) erkennen?

Lösung:a) Nach Aufgabenstellung ist

a (4,0 m=s2) ÿ (2,0 m=s3) t.

Integration liefertv (4,0 m=s2) t ÿ (1,0 m=s3) t2 ÿ ( 5

3 m=s)

mit der Integrationskonstanten v0 ÿ ( 53 m=s).

Nochmalige Integration ergibt sich mit s0 0:

s (2,0 m=s2) t2 ÿ ( 13 m=s3) t3 ÿ ( 5

3 m=s) t.

b) Für v 0 erhält man über

t1;2 (2,0 s) (4,0 s2)ÿ ( 5

3 s2)q

t1 0,476 s, s1 ÿ 0,376 m;t2 3,528 s, s2 4,376 m.

c) Für s 0 ergeben sich die Lösungen t1 0,t2 1,0 s und t3 5,0 s mit v1 ÿ 5

3 m=s,v2 4

3 m=s und v3 ÿ6 23 m=s.

d) t-s-Diagramm

s in m

t in s1 2 3 4 501

23

45

–1

Seite

23

1 Mechanik16

Page 17: Metzler Physik

t-v-Diagramm

in m/s

t in s1 2 3 4 501

2

–1–2–3–4–5–6–7

υ

t-a-Diagramm

t in s1 2 3 4 502

–2

4in m/s

a2

–4

5 Berechnen Sie iterativ die Zeit-Weg-Tabelleder gleichmäûig beschleunigten Bewegung fürs0 0,2 m, v0 1,5 m=s und a0 ÿ0,4 m=s2 mitDt 1 s von t0 2 s bis t9 10 s und zeichnen Sieden Graphen.

Lösung:Iterationsvorschrift: v (tDt) v (t) aDt,

s (tDt) s (t) v (t)Dt.

Startwerte: a0 ÿ0,4 m=s2; v0 1,5 m=s;s0 0,2 m;

Dt 1 s mit t0 2 s und t9 10 s.

1. Interationsschritt

v (3 s) 1,5 m=s ÿ 0,4 m=s2 ´ 1 s 1,1 m=s

s (3s) 0,2 m 1,5 m=s2 ´ 1 s 1,7 m

t in s 2 3 4 5 6

v in m=s 1,5 1,1 0,7 0,3 ÿ 0,1

s in m 0,2 1,7 2,8 3,5 3,8

t in s 7 8 9 10

v in m=s ÿ 0,5 ÿ 0,9 ÿ 1,3 ÿ 1,7

s in m 3,7 3,2 2,3 1,0

1 2

s in m

t in s4 6 80

4

3

2

1

0

6 Berechnen Sie iterativ eine gleichmäûig beschleunigte Bewegung für s0 0,5 m, v0 0,25 m=s unda0 ÿ 0,1 m=s2 von t0 0 bis te 10 s. Wählen Sie a) Dt 1 s, b) Dt 0,1 s und c) Dt 0,001 s und ver-gleichen Sie.

Lösung:Iterationsvorschrift: v (tDt) v (t) aDt,

s (tDt) s (t) v (t)Dt.Startwerte: s0 0,5 m; v0 0,25 m=s2;

a0 ÿ 0,1 m=s2; t0 0, te 10 sa) Dt 1 s; b) Dt 0,1 s, c) Dt 0,01 s.

1. Interationsschritt

v (1 s) 0,25 m=s (ÿ 0,1 m=s2) ´ 1 s 0,15 m=s,

s (1s) 0,5 m 0,25 m=s2 ´ 1 s 0,75 m=s.

t in s 0 1 2 3 4 5 6 7 8 9 10

v in m=s 0,25 0,15 0,05 ÿ 0,05 ÿ 0,15 ÿ 0,25 ÿ 0,35 ÿ 0,45 ÿ 0,55 ÿ 0,65 ÿ 0,75

s in m 0,50 0,75 0,90 0,95 0,90 0,75 0,50 0,15 ÿ 0,30 ÿ 0,85 ÿ 1,50

Seite

24

1.1 Kinematik 17

Page 18: Metzler Physik

b) Dt 0,1 sTabelle mit den Werten bis 1 s

t in s 0 0,1 0,2 0,3 0,4 0,5 0,6 0,7 0,8 0,9 0,1

v in m=s 0,25 0,24 0,23 0,22 0,20 0,19 0,19 0,18 0,17 0,16 0,15

s in m 0,50 0,525 0,549 0,572 0,594 0,615 0,635 0,654 0,672 0,689 0,705

c) Dt 0,01 sTabelle mit den Werten bis 0,1 s

t in s 0 0,01 0,02 0,03 0,04 0,05 0,06 0,07 0,08 0,09 0,1

v in m=s 0,25 0,249 0,248 0,247 0,246 0,245 0,244 0,243 0,242 0,241 0,240

s in m 0,50 0,5025 0,50499 0,50747 0,50994 0,5124 0,51485 0,51729 0,51972 0,52214 0,52455

Verhalten im Straûenverkehr

1 Berechnen Sie den Sicherheitsabstand fürv 30 km=h, 50 km/h (Ortsverkehr) und fürv50km=h,80km=h,100km=h,140km=h(auûer-orts).

Lösung:

¹Ein-Sekunden-Abstandª

¹Zwei-Sekunden-Abstandª

v in km=h 30 50 50 80 100 140

s in m 8,3 13,9 27,8 44,4 55,6 77,8

2 Zeichnen Sie das s-v-Diagramm für den Sicher-heitsabstand s (siehe oben) von v 0 bis 50 km=hinnerhalb und bis 140 km=h auûerhalb geschlosse-ner Ortschaften.

Lösung:

in km/hυ

10 20

50

100

140

0s in m

30

80

40 60 80

in O

rtsch

afte

n

außerhalb

0

3 Berechnen Sie Reaktions-, Brems- und Anhalte-weg für einen Motorroller und einen PKW auftrockner Straûe, die bei 30 km=h bzw. 50 km=hzu bremsen beginnen.

Lösung:Reaktionsweg (in 1 s) s v Dt Dt 1 sBremsweg s 1

2 a v2

PKW Motorroller

beideRäder

Vorderrad Hinterrad

v im km=h 30 50 30 50 30 50 30 50

Reaktions-weg in m

8,3 13,9 8,3 13,9 8,3 13,9 8,3 13,9

Bremswegin m

4,3 12,1 4,3 12,1 6,7 18,5 12,4 34,4

Anhalte-weg in m

12,6 26,0 12,6 26,0 15,0 32,4 20,7 48,3

4 Stellen Sie tabellarisch die Anhaltewege zusam-mena) für die angegebenen Bremsmöglichkeiten desMotorrollers bei 30 km=h, 50 km=h, 70 km=h,90 km/h;b) für die obigen Beschleunigungen des PKWs bei30 km=h, 50 km=h, 80 km=h, 100 km=h, 140 km=h.

Lösung:Anhalteweg s v Dt v2

2 a, Angaben in m

a) v in km=h beide Räder Vorderrad Hinterrad

30 12,7 15,0 20,7

50 26,0 32,4 48,3

70 43,1 55,8 87,0

90 64,1 85,1 136,6

b) v in km=h trocken nass Schnee/Eis Glatteis

30 12,7 17,0 25,7 43,1

50 26,0 38,0 62,1 110,3

80 53,1 84,0 145,7 269,1

100 76,0 124,2 220,7 413,6

140 133,4 227,9 417,0 795,1

Seite

25

1 Mechanik18

Page 19: Metzler Physik

*5 Welche Beschleunigung ergibt sich aus der Regelfür den Anhalteweg bei a) 50 km=h, b) 100 km=h?

Lösung:Der Anhalteweg (in m) berechnet sich nach derRegel sa 3 (v=10) (v=10)2 (v in km=h):

Nach der Formel ist der Anhaltewegsa v Dt v2=(2a) mit Dt 1 s.

Daraus ergibt sich die Beschleunigung a zu

a v2

2 (saÿ v Dt).

a) v 50 km=h b) v 100 km=h

Anhalteweg in m 40 130Beschleunigungnach Formelin m=s2

3,69 3,77

6 Ein Motorroller überholt auf einer Landstraûe mit50 km=h einen Trecker, der mit 25 km=h dahinfährt.a) Berechnen Sie Überholzeit und Überholweg,

wenn der Motorroller beim Aus- und Einsche-ren den Sicherheitsabstand einhält.

b) Zeichnen Sie das t-s- und das t-v-Diagramm.

Lösung:a) Der Trecker hat bei Beginn des Überholvor-

gangs den Vorsprung ¹Sicherheitsabstandª,der sich berechnet zu s Èu vt 2 s 27,8 m.Am Ende des Überholungsvorgangs hat der Mo-torroller denselben Vorsprung s Èu.Die Überholzeit te ist die Zeit, in der der Motor-roller beim Start beider Fahrzeuge an derselbenStelle den Vorsprung 2 sü herausgeholt habenwürde: 2 s Èu vM teÿ vr te, also Überholzeitte 8 s und Überholweg se 111 m.(Länge von Trecker und Motorroller bleiben inder Aufgabe unberücksichtigt.)

b) s

te t tte

sü T

M = 50 km/h

υ

υ

= 25 km/h

υ

*7 Im Moment, als ein PKW einen anderen überholt,erkennen beide Fahrer ein Hindernis vor sich undbremsen. Der langsamere Wagen kommt kurz vordem Hindernis zum Stehen. Berechnen Sie die Ge-schwindigkeit, mit der der schnellere am Hindernis

vorbeifährt für die Geschwindigkeit v1 50 km=hdes langsameren und v2 60 km=h bzw. 70 km=hdes schnelleren bei gleicher Bremsbeschleunigunga 6 m/s2.

Lösung:Der Anhalteweg für den langsameren Wagen istse v2

0=(2a), also se 16,1 m.Längs der Strecke se vermindert sich die Geschwin-digkeit der schnelleren Wagen von v01 auf ve1

(bzw. von v02 auf ve2) nach v201ÿ v2

e1 2 ase.Da 2ase v2

0 ist, ergibt sichv2

01ÿ v2e1 v2

0, also ve1 v2

01ÿ v20

p.

Bei v01 70 km=h ist ve1 49 km=h,bei v02 60 km=h ist ve2 33 km=h,

*8 Ein PKW lA 5,0 m fährt einige Zeit hinter ei-nem LKW lB 18 m mit der GeschwindigkeitvA vB 90 km=h her und überholt dann denLKW mit der als konstant angesetztenBeschleuni-gung aA 1,86 m=s2. Der PKW schert mit dem ge-forderten Sicherheitsabstand ein bzw. aus.a) Berechnen Sie Überholzeit, Überholstrecke so-

wie die Geschwindigkeit des PKW am Endedes Überholens.

b) Zeichnen Sie das t-s-, t-v- und t-a-Diagrammdes Überholvorgangs im Bezugssystem Straûeund LKW.

c) Formulieren Sie die Zusammenhänge beimÜberholen im Bezugssystem Straûe und LKW.

Lösung:a) Der PKW mit der Geschwindigkeit vA

vB 90 km=h steigert sie von Beginn des Be-schleunigungsvorganges ab mit der Beschleuni-gung a 1,86 m=s2 und legt während der ge-samten Überholzeit te damit den Wegse vB te 1

2 a t2e zurück.

Bis zum Ende des Beschleunigungsvorgangesmuss der PKW dabei den Sicherheitsabstands Èu vor dem LKW und diesen noch einmal nachdem Vorbeifahren einholen, wobei der Sicher-heitsabstand die Strecke ist, die der LKW inzwei Sekunden zurücklegt: s Èu vB 2s; weitermuss er die Länge des LKWs und seine eigeneLänge abfahren und schlieûlich noch dieStrecke durchfahren, die der LKW währendder Überholzeit te mit seiner GeschwindigkeitvB weitergekommen ist: sB vB te, alsovB te 1

2 a t2e 2 s Èu lB lA vB te. Daraus be-

rechnen sich die Überholzeit te 11,5 s, derÜberholweg se 410,5 m und die Geschwindig-keit des PKW beim Ende des Überholvorgangesve 46,4 m=s oder ve 167,0 km=h.

1.1 Kinematik 19

Page 20: Metzler Physik

b) Bezugssystem Straûe

Pkwυ

aPkw

te

s

lA

Bl

sB

Ü

t

s

Lkwυ

te t

υ

t

aLkw

a

Bezugssystem LKW

aPkw

te

lA

Bl

t

s

Lkwυ′

te t

υ

t

aLkw

asÜ

= at

c) Für die Verhältnisse im Bezugssystem Straûesiehe a).Im Bezugssystem LKW berücksichtigt man nurdie Relativgeschwindigkeit vR a t des PKW

gegenüber dem LKW, mit der der PKW relativzum LKW nur den zweifachen Sicherheitsab-stand s Èu sowie die Länge vom LKW und vomPKW lA lB zurücklegen muss.

1.1.5 Der freie Fall

1 EinKörper fällt auseinerHöhevon130mfreiherab.a) Welche Strecke hat er nach 2 bzw. 4 Sekunden

durchfallen, welche Geschwindigkeit besitzt erdann?

b) Nach welcher Zeit und mit welcher Geschwin-digkeit trifft er auf dem Boden auf (ohne Luft-widerstand)?

Lösung:a) Der durchfallene Weg ist nach s 1

2 g t2 fürt1 2 s s1 19,62 m, also h1 110,38 mund fürt2 4 s s2 78,48 m, also h2 51,52 m;nach v g t ist für die Geschwindigkeitv1 19,63 m=s und v2 39,42 m=s.

b) Aus s 130 m wird te 5,15 s undve 50,50 m=s.

2 Von der Spitze eines Turms lässt man einen Steinfallen. Nach 4 Sekunden sieht man ihn auf den Bo-den aufschlagen. (Vom Luftwiderstand werde ab-gesehen.)a) Wie hoch ist der Turm?b) Mit welcher Geschwindigkeit trifft der Stein

auf dem Erdboden auf?c) Nach welcher Zeit hat der Stein die Hälfte sei-

nes Fallweges zurückgelegt?d) Welche Zeit braucht der Stein zum Durchfallen

der letzten 20 m?e) Nach welcher Zeit (seit dem Loslassen) hört

man den Stein aufschlagen? (Schallgeschwin-digkeit 320 m/s)

Seite

28

1 Mechanik20

Page 21: Metzler Physik

Lösung:a) h 78,48 m, b) ve 39,24 m=sc) t 2,83s, d) Dt 0,547 s, e) t 4,245 s.

3 Welche der unten dargestellten Kurven stellt dasZeit-Geschwindigkeit-Diagramm eines Steinesdar, der zur Zeit t 0 senkrecht in die Höhe gewor-fen wird und zur Zeit t te wieder den Boden er-reicht? Beschreiben Sie die Bewegungen in denbeiden anderen Diagrammen.

Lösung:Diagramm a) stellt die Fallbewegung des Steinesdar.Diagramm b): Die Bewegung beginnt mit kon-stanter Beschleunigung, erreicht bei t 0,5 te dieHöchstgeschwindigkeit und bremst nun mit glei-cher, aber negativer Beschleunigung.Diagramm c): Aus der Höchstgeschwindigkeitbremst hier ein Körper mit negativer konstanter Be-schleunigung, bis er zum Stehen kommt, um dannsogleich mit gleich groûer, aber positiver Be-schleunigung wieder anzufahren.

*4 Schreiben Sie ein Programm zur iterativen Berech-nung (Dt 0,01 s) für den Weg (Fallweg 20 m)und die Geschwindigkeit der Fallbewegung einesFuûballs in Luft cw 0,45, r 11 cm, m0,4 kg und stellen Sie beide grafisch dar. Verglei-chen Sie mit dem freien Fall.

Lösung:Iterationsvorschrift: v (tDt) v (t) a (t)Dt,

s (tDt) s (t) v (t)Dt,a (t)ÿg (1ÿ k v (t)2).

Für k gilt:k 1

2 cw rA=G 1

2 0,45 1,29 kg/m3 ´ 0,038 m2=

(3,924 m kg=s2) 2,81 ´ 10ÿ3 s2=m2.

Startwerte: a (0)ÿ 9,81 m=s2; v (0) 0;s (0) 20,0 m; Dt 0,01 s.

1. Iterationsschritt:v (0,01 s) 0 (ÿ 9,81 m=s2) ´ 0,01 s

ÿ 0,0981 m=s,s (0,01 s) 20,0 m 0 ´ 0,01 s 20,0 m,a (0,01 s) ÿ 9,81 m=s2 ´ [1ÿ k (ÿ 0,0981 m=s)2]

ÿ 9,8097 m=s2.

Endwerte der Iteration:

t in s a(t) in m=s2 v (t) in m=s s(t) in m

2,21 ÿ 3,2295 ÿ 15,4277 0,04663

2,22 ÿ 3,2019 ÿ 15,460 ÿ 0,1076

Beim freien Fall ergibt sich ausse 0 20,0 m ÿ 1

2 g t2e und ve ÿg te.

2,019 ÿ 9,81 ÿ 19,806 0,00

t-v-Diagramm

υ 0,5 1,0 1,50

–5

–10

–15

2,0l( ) t

t-s-Diagramm

s

0,5 1,0 1,50

5

10

15

20

0 t2,0

υ( )

5 Zur Bestimmung der Erdbeschleunigung lässt manmehrmals einen Stein von einem s 19,85 m ho-hen Turm herabfallen und bestimmt mit verschie-denen Stoppuhren jeweils die Fallzeit. BestimmenSie g aus dem Mittel von neun Zeitmessungen:ti 2,00 s; 2,04 s; 1,99 s; 2,02 s; 2,06 s; 1,96 s;1,97 s; 2,01 s; 2,07 s.

Lösung:Mit dem Mittel der Fallzeiten t 2,013 s mit demFehler Dt 0,038 s wird g 9,797 m=s2.

1.1 Kinematik 21

Page 22: Metzler Physik

*6 Messung der Reaktionszeit: Jemand drückt ein ver-tikal gehaltenes Lineal (Skala von unten nachoben) mit der einen Hand an eine Wand und ver-deckt sie mit der anderen durch ein Stück Papier.Die Testperson hält einen Finger wenige Millime-ter über der Nullmarke. Wenn die erste Person nunplötzlich das Lineal fallen lässt, muss die zweitezudrücken. Aus der (Fall-)Strecke zwischen Null-

marke und der Stelle, an der die Testperson dasLineal festhält, berechnet man die Reaktionszeit.Stellen Sie eine Tabelle von 0 cm bis 20 cm auf undüberprüfen Sie Ihre Reaktionszeit und die andererPersonen.

Lösung:Ð

1.1.6 Der Wurf; Überlagerung von Bewegungen

1 a) Berechnen Sie Wurfweiten und Fallwege derwaagerechten Würfe für v0 5 m=s bzw.10 m=s nach t 0,1 s; 0,5 s; 1,0 s; 1,5 s; 2,0 s.Zeichnen Sie die Bahnkurven.

b) Berechnen Sie dazu Wurfweiten und Wurfzei-ten, wenn die geworfenen Körper 5 m; 10 mgefallen sind.

Lösung:a) Wurfweite sx v t Fallweg sy 1

2 g t2

t in s 0,1 0,5 1,0 1,5 2,0

v0 5 ms sx

sy

0,50,05

2,51,23

5,04,91

7,511,04

10,019,62

v0 10 ms sx

sy

1,00,05

5,01,23

10,04,91

15,011,04

20,019,62

sy in m

5 10 15 20

10

20

+

+ +

++

+ +

sx

in m

υ = 5ms υ = 10

ms

b) Für die Fallzeit tw 2hg

rund die Wurfweite

sxw v0

2hg

rergibt sich:

v05 m 10 m

tw sxw tw sxw

5 ms 1,01 s 5,05 m 1,43 s 7,14 m

10 ms 1,01 s 10,10 m 1,43 s 14,3 m

*2 Ein Stein wird mit der Geschwindigkeitv0 20 m=s horizontal von der Höhe h aus abge-worfen. Er erreicht in der Horizontalen eine Wurf-weite von xE 40 m.

a) Wie groû sind die Abwurfhöhe und die Flug-zeit?

b) Mit welcher Geschwindigkeit und unter wel-chem Winkel zur Horizontalen trifft der Steinauf den Boden?

c) Wie lauten die Lösungen zu a) und b) allge-mein?

Lösung:

a) Aus x v0 t und yÿ 12 g t2 ergibt sich

yÿ g2v2

0

x 2, und daraus mit xE 40 m

yE ÿ519,62 m ÿ h und tE 2 hg

s 2,0 s.

b) Den Winkel zur Horizontalen und die Ge-schwindigkeit erhält man aus den Geschwin-digkeitskomponenten am Auftreffpunkt

vxE v0, vyE ÿg t mit tan aÿg tE

v0

und

vE v2

x v2y

q

v2

0 g2 t2E

p

zuaÿ 44,5 und vE 28,0 m=s.

c) Siehe Lösung zu b).

3 Ein Körper wird mit der Geschwindigkeitv0 18 m=s senkrecht nach oben geworfen.(Vom Luftwiderstand sehe man ab.)a) Stellen Sie die speziellen Bewegungsgleichun-

gen auf.b) Berechnen Sie die Wurfhöhe, Wurfzeit und die

Zeit bis zum Erreichen des höchsten Punktesder Bahn.

Lösung:a) s (18 m=s) tÿ 1

2 (9,81m=s2) t2;

v (18 m=s) ÿ (9,81m=s2) t:

Seite

33

1 Mechanik22

Page 23: Metzler Physik

b) sh v2

0

2g 16,51 m,

Steigzeit th v0

g 1,83 s,

Wurfzeit te 2 th 3,67 s.

4 Ein Geschoss verlässt ein Gewehr mit der Ge-schwindigkeit v0 780 m=s. (Vom Luftwider-stand soll abgesehen werden.)Welche Höhe und Weite erreicht das Geschoss,wenn es unter den Winkeln 90, 60, 45, 30 zurWaagerechten abgeschossen wird?

Lösung:

a 90 60 45 80

h in m 31 009 23 257 15 504 7 752

w in m 0 53 709 62 018 5 3709

*5 Ein Wasserstrahl, der unter einem Winkel von 40zur Horizontalen die Düse eines Gartenschlauchsverlässt, erreicht das in 30 m Entfernung stehendeBuschwerk in gleicher Höhe wie die Düse.a) Mit welcher Geschwindigkeit verlässt der

Strahl die Düse? (Vom Luftwiderstand abse-hen!)

b) Wie groû ist die Gipfelhöhe des Wasserstrahls?c) Welche Zeit benötigt ein einzelner Wassertrop-

fen vom Verlassen der Düse bis zum Auftref-fen?

Lösung:a) v0 17,3 m,b) h ys 6,29 m,c) te 2,27 s.

*6 Ein Körper wird von einer Klippe der Höhe h mitder Geschwindigkeit v0 unter dem Winkel a (vonder Horizontalen nach unten gemessen) abgeschos-sen.Stellen Sie die Gleichungen auf, die die Bewegungin horizontaler und vertikaler Richtung beschrei-ben, und ermitteln Sie die Gleichung der Wurf-parabel.

Lösung:Ersetzt man in den Bewegungsgleichungen adurch ÿasx v0 cos a t und sy ÿv0 sin a tÿ 1

2 g t2,

löst dann sx nach t auf und setzt t in sy ein, so erhältman als Gleichung der Wurfparabel

sy ÿg

2v20 cos2 a

s2xÿ tan a sx:

Nach Umformen

sy ÿg

2 v20 cos2 a

sx

v20 sin2a

2 g

2

v20 sin2 a

2g

liest man für den Scheitelpunkt (im 2. Quadraten)ab:

S

ÿ v2

0 sin2 a2g

v2

0 sin2 a2g

:

*7 Ein Motorrad fährt mit der Geschwindigkeit v0 aufeine unter dem Winkel a gegenüber der Horizon-talen ansteigenden Rampe an einen Graben mit derBreite b heran und landet auf der gegenüberliegen-den Seite des Grabens auf einem Plateau, das umdie Höhe h höher gelegen ist als die höchste Stelleder Absprungrampe.a) Bestimmen Sie bei gegebener Endgeschwin-

digkeit v0 50 km=h auf der Rampe, bei gege-benem a 30 und b 5,0 m die obere Grenzefür die Höhe h, bei der das Motorrad den Gra-ben noch überspringen kann.

b) Wie groû muss die Geschwindigkeit v0

mindestens sein, wenn die Höhe h 1,0 m beimWinkel a 20 und der Breite b 5,0 m er-reicht werden soll?Vernachlässigen Sie die Ausmaûe des Motor-rads und lösen Sie die Aufgabe allgemein undmit Zahlenwerten.

Lösung:a) Aus der Gleichung der Wurfparabel

sy sx tan aÿ 12

gv2

0 cos2 as2

x

erhält man die maximale Höhe sy h im Ab-stand sx b zu hmax 2,04 m.

b) Die Gleichung der Wurfparabel löst man nachv0 auf und erhält mit sx b und sy h über

v0 b

cosa

g

2 (b tanaÿ h)

r

die Mindestgeschwindigkeit v0 13,01 m=soder v0 46,85 km=h.

Zusatzaufgaben

8 Ein Stein fällt aus der Höhe h 40 m senkrecht zurErde. Gleichzeitig wird von unten ein zweiter Steinmit der Geschwindigkeit v0 20 m=s senkrechthoch geworfen.a) Nach welcher Zeit und in welcher Höhe fliegen

die beiden Steine aneinander vorbei?

1.1 Kinematik 23

Page 24: Metzler Physik

b) In welchem zeitlichen Abstand treffen die bei-den Steine auf dem Boden auf?

c) Welche Anfangsgeschwindigkeit müsste derzweite Stein haben, wenn beide zu gleicher Zeitauf dem Boden auftreffen sollen?

Lösung:a) Für den ersten Stein gilt s1 hÿ 1

2 g t2, für denzweiten s2 v0 tÿ 1

2 g t2: Sie fliegen aneinandervorbei für s1 s2, ausgerechnet t0 2,0 s in derHöhe s0 20,38 m.

b) Der erste Stein trifft zur Zeit te1 2 hg

s

2,86 s, der zweite zur Zeit te2 2v0

g 4,08 s

auf den Boden auf. Die Differenz ist alsoDt 1,22 s.

c) Dazu muss te1 te2 sein, also

2hg

s 2v0

goder

v0 gh2

r, also v0 14,01 m=s.

*9 Ein Sportler stöût die Kugel aus der Höhe h 1,8 munter dem Winkel a 30 zur Horizontalen underreicht die Weite von 19,3 m.

Mit welcher Anfangsgeschwindigkeit v0 hat er dieKugel gestoûen?

Lösung:Aus den beiden Bewegungsgleichungen

sx v0 cosa t und sy v0 sina tÿ 12 g t2

gewinnt man über die erste Gleichung mitsx xe 19,3 m und der Zeit te des gesamtenVorgangs die Geschwindigkeit

v0 xe

te cosa.

Setzt man dies in die zweite Gleichung mitsy ye ÿhÿ 1,8 m ein, so erhält man

te 2xe tana h

g

s 1,624 s

und über den obigen Ausdruck für v0 schlieûlichv0 13,72 m=s.

1.1.7 Die gleichförmige Kreisbewegung

1 Berechnen Sie die Winkelgeschwindigkeit des Se-kunden-, Minuten- und Stundenzeigers einer Uhr.

Lösung:Sekundenzeiger w 6,283 sÿ1;Minutenzeiger w 1,047 10ÿ1 sÿ1;Stundenzeiger w 1,745 10ÿ3 sÿ1.

2 Wie groû sind die Radialbeschleunigungen abso-lut und im Vergleich zur Erdbeschleunigungg 9,81 m=s2a) einerWäschetrommel (d32cm,3000U=min),b) einer Astronautentestmaschine (Abstand Dreh-

achse±Kabine 6,5 m, 20 U=min),c) auf der Erde am ¾quator bzw. auf 45 Breite

infolge der Drehung der Erde um ihre Achse,d) desMondes infolgeseinesUmlaufsumdieErde,e) der Erde infolge ihrer Bewegung um die Sonne?

Lösung:a) Wäscheschleuder

aR 1,579 104 m=s2 1,610 103 g,b) Astronautentestmaschine

aR 28,5 m=s2 2,91 g,

c) Erde am ¾quatoraR 3,369 10ÿ2 m=s2 3,434 10ÿ3 g,auf 45 BreiteaR 2,382 10ÿ2 m=s2 2,428 10ÿ3 g,

d) Mond (siderischer Monat)aR 2,724 10ÿ3 m=s2 2,777 10ÿ4 g,

e) Erde um Sonne (siderisches Jahr)aR 5,930 10ÿ3 m=s2 6,045 10ÿ4 g.

3 Berechnen Sie die Winkelgeschwindigkeit, dieBahngeschwindigkeit und die Radialbeschleuni-gung eines Punktes auf dem Radkranz d 875 mm eines ICE 3, der mit 330 km=h dahinfährt.

Lösung:Bahngeschwindigkeit des Radkranzesv 330 km=h als Ansatzv 330 km=h 91,67 m=s,w v=r 209,53 sÿ1,aR v2=r 19,208 103 m=s2 1,958 103 g.

*4 a) Wie ändert sich im weiteren Verlauf die Bahn-kurve in Abb. 35.2 bei der angegebenen Lagevon a?

Seite

35

1 Mechanik24

Page 25: Metzler Physik

b) Welche Bewegung liegt vor, wenn der Be-schleunigungsvektor von konstantem Betragständig a) parallel oder b) senkrecht zum Ge-schwindigkeitsvektor zeigt?

Lösung:Die Aufgabe bezieht sich auf Abb. 35.2.a) Die Komponente von a tangential an den Kreis

erhöht die Bahngeschwindigkeit; die Kompo-nente von a in Richtung auf den Mittelpunktverringert ± bei Vergröûerung dieser Kompo-nente ± oder vergröûert ± bei Verkleinerung ±den Bahnradius.

b) a) Lineare Bewegung mit konstanter Be-schleunigung,

b) Kreisbewegung mit konstantem Radius.

*5 Entwickeln Sie ein Computerprogramm zur itera-tiven Berechnung der Kreisbewegung: Radiusr 1 m, Bahngeschwindigkeit v 2p m=s, Start-punkt x0 1 m jy0 0 und Dt 0,001 s. (Hin-weis: Bei konstanter Beschleunigung steht der Be-schleunigungsvektor stets senkrecht zum Ge-schwindigkeitsvektor.)

Lösung:Iterationsvorschrift s. Abb. 24.1 mit

axt 4p2xt

x2t y2tp , ay ÿ

4p2ytx2t y2t

pStartwertex01;y00;vx0 0;vy0 2p;ax0ÿ4p2;ay00.

Zusatzaufgaben

6 Die Radien der Auûenrille bzw. Innenrille einerLangspielplatte mit 33 U=min betragen r1 16,2cm bzw. r2 6,8 cm.a) Wie groû sind die Winkel- und Bahngeschwin-

digkeiten eines Punktes der Auûen- bzw. derInnenrille?

b) Welche Radialbeschleunigungen ergeben sich?

Lösung:a) w1 w2 3;456 sÿ1, v1 5,598 10ÿ1 m=s,

v2 2,350 10ÿ1 m=s.b) a1 1,935 m=s2, a2 8,121 10ÿ1 m=s2.

7 Ein Körper durchläuft mit konstanter Winkelge-schwindigkeit einen Kreis. Drücken Sie die Um-laufszeit T, die Frequenz f und die Anzahl der Um-läufe n in einer Minute durch die Winkelgeschwin-digkeit aus.

Lösung:Umlaufzeit T 2 p=w,Frequenz f 1=T w=2p,Umlaufzahl pro Minute n (w=2p) 60 s=min.

1.1.8 Vektorielle Darstellung von Bewegungen

1 Ein Schiff steuert auf See mit 10 Knoten den KursN 100 O (1 Knoten 1 Seemeile=Stunde; 1 See-meile 1,852 km). Die See weist eine Strömungvon 3 Knoten in Richtung N 20 W auf.a) Zeichnen Sie den Weg des Schiffes in einer

Stunde über dem Grund (der See) und bestim-men Sie aus der Zeichnung Kurs und Ge-schwindigkeit (in Knoten) über dem Grund(1 Knoten é 1 cm).

b) Lösen Sie die Fragen zu a) rechnerisch und ge-ben Sie die Geschwindigkeit auch in km=h an.

Lösung:

a)

α

β

υ

w

s

100°

N 20° W

υ

N

γ

υg

N 100° O

O

W

S

N

Man liest ab vg 8,9 Knoten, g 17,2, alsoKurs über dem Grund a N 82,8 O.

b) ~vw~vs ~vg.c) Aus der Abb. ergibt sich für den der Seite vg

gegenüberliegenden Winkel b 60. Nachdem Cosinus-Satz erhält man ausv2

g v2w v2

s ÿ 2 vw vs cos b oder

Seite

37

1.1 Kinematik 25

Page 26: Metzler Physik

vg 8,89 Knoten 16,5 km=h.Den Winkel zwischen den Seiten vg und vw er-hält man nach dem Sinussatz über

sin g vs

vgsin b

zu g 17,00 und daraus a 100ÿ 17,0083,00.

*2 Ein Flugzeug startet von einem Flugplatz aus inwestlicher Richtung mit 300 km=h Fluggeschwin-digkeit und steigt dabei 500 m pro Minute. Nach20 Minuten Flugzeit dreht es nach Norden ab undsteigt weiter zu denselben Bedingungen.a) Welche Höhe hat das Flugzeug 30 Minuten

nach dem Start erreicht?b) In welcher direkten Entfernung und in welcher

Richtung vom Startpunkt aus gesehen (Höhen-winkel zur Horizontalen, Winkel zur Nordrich-tung) befindet sich das Flugzeug zu dieser Zeit?

c) Berechnen Sie die Orts- und Geschwindigkeits-vektoren vom Startpunkt bis zum Ort nach20 min und nach 30 min mit ihren drei Kompo-nenten.

Lösung:a) h 15,0 km.b) In westlicher Richtung über dem Boden sw

99,50 km, dann in nördlicher sN 49,75 km;damit über dem Boden vom Ausgangspunktentfernt s 111,24 km. Daraus der Kurswinkelüber dem Boden N 63,44 W. Entfernung Flug-zeug±Startpunkt e 112,25 km.

c) Nach 20 min hat das Flugzeug die Höhe (z-Komponente) h20 10 000 m erreicht, nachweiteren 30 min die Höhe h30 15 000 m.Die Ortsvektoren zu diesen Punkten sind (Ostené Richtung der positiven x-Achse, Norden éRichtung der positiven y-Achse)

~r20 ÿ99,50 km

010,00 km

!und ~r30

ÿ99,50 km49,75 km15,00 km

!,

die Geschwindigkeitsvektoren

~v20 297,31 km=h

040,09 km=h

!und~v30

0297,31 km=h

40,00 km=h

!.

*3 Die Spitze des Vektors~r beschreibt als Funktionder Zeit t einen Kreis mit dem Radius r um denNullpunkt eines x-y-Koordinatensystems (Abbil-dung unten)

~r r cos w tr sin w t

r cos w t ~ex r sin w t ~ey.

Der Vektor~r rotiert bei der gleichförmigen Kreis-bewegung mit der Winkelgeschwindigkeit w umden Nullpunkt des Koordinatensystems;~ex und~ey

sind so genannte Einheitsvektoren (Vektoren derLänge 1).a) Fertigen Sie eine Zeichnung an und tragen Sie

für die Zeiten t 0, 112 T , 1

4 T , 12 T den Vektor~r

ein.b) Einen Vektor leitet man nach der Zeit ab, indem

man die Komponenten nach der Zeit ableitet.Bilden Sie den Geschwindigkeitsvektor ~v _~rund den Beschleunigungsvektor~a _~v und zei-gen Sie, dass~aÿw2~r ist.

c) Berechnen Sie für t 12 T die Vektoren~v und~a

und ihre Komponenten und tragen Sie die Er-gebnisse in die Zeichnung zu a) ein T 3p s,r 5 cm).

x

y

P

ω tey

xe

ay

xa

aω t

ω t

υ

υ

x

υy

x = r cos

y = r sin

ω t

ω t

r

Lösung:a) Vektoren für t 1

12 T eingezeichnet

υ

r

y

x

T

0

r

r

r

14

T12

T112a t =

b) Man erhält durch Differentiation

~v _~r ÿw r sin w tw r cos w t

und

1 Mechanik26

Page 27: Metzler Physik

~a ~r ÿw2r cos w tÿw2r sin w t

ÿw2 r cos w t

r sinw t

,

also~aÿw2~r:c) Für t 1

2 T und T 3p s wird w 23 sÿ1 und da-

mit w t p und mit r 5,0 cm ergibt sich also

~v ÿ 103 cm sinp

103 cm cosp

0

3,33 cm

,

~a ÿ 209 cm cos p209 cm sinp

2,22 cm

0

.

Zusatzaufgaben

4 Ein Flugzeug fliegt mit der Geschwindigkeitvf 250 km=h in Richtung N 30 O. Es weht einWind aus Südwest mit einer Geschwindigkeit vonvw 60 km=h. Wie groû ist die Geschwindigkeitdes Flugzeuges, und welchen Kurs steuert es, je-weils orientiert an der Erdoberfläche? Die Aufgabeist zeichnerisch (vektoriell) und rechnerisch zulösen.

Lösung:Rechnerisch ergibt sich die Geschwindigkeit v0

über der Erde ausv2

0 v2f v2

wÿ 2 vf vw cos 165 zuv0 308 km=h und der Kurs über der Erde aussin a=vw sin 165=v0 und darausa 2,89 zu N 32,89 O.

*5 Ein Körper wird unter dem Winkel a zur Horizon-talen mit der Anfangsgeschwindigkeit v0 gewor-fen. Luftwiderstand usw. vernachlässige man. Be-stimmen Sie als Funktion der Zeit in Vektorforma) den Beschleunigungsvektor,b) daraus den Geschwindigkeitsvektor undc) daraus den Wegvektor.d) Berechnen Sie als Funktion der Zeit den Betrag

der Geschwindigkeit und den Tangens desWinkels, den die Bahnrichtung jeweils mitder Horizontalen bildet.

Lösung:

a) ~a 0ÿg

;

b) ~v 0ÿg t

v0 cos a

v0 sin aÿ g t

c) ~s v0 cos a tv0 sin a tÿ 1

2 g t2

d) j~vj v2

x v2y

q

v2

0ÿ 2 g v0 sin a t g2 t2p

;

tan a vy

vx v0 sin aÿ g t

v0 cos a

tan aÿ gv0 cos a

t:

*6 Der Ortsvektor einer Bewegung sei durch die zeit-abhängige Vektorform

~r (2 m=s) t(8m=s) tÿ (2 m=s2) t2

gegeben.a) Ermitteln Sie die Gleichung der Bahnkurve im

x-y-Koordinatensystem, und zeichnen sie (1 mé 1 cm).

b) Bestimmen Sie durch Ableiten den Geschwin-digkeitsvektor und den Beschleunigungsvek-tor.

c) Berechnen Sie den Orts-, den Geschwindig-keits- und den Beschleunigungsvektor für t 0,1 s, 2 s, . . . 5 s, und tragen Sie die Vektoren indie Zeichnung der Bahnkurve ein (1 m=s é1 cm; 1 m=s2 é 1 cm).

d) Beschreiben Sie die Bewegung, und verglei-chen Sie sie mit den Ihnen bekannten Bewe-gungsformen.

Lösung:

a) Aus~r liest man x (2 m=s) t und y (8 m=s) tÿ(2 m=s2) t2 ab. Durch Elimination von t gewinntman

y 4 xÿ 12

1m

x2 oder

yÿ12

1mxÿ 4 m2 8 m:

Das ist die Gleichung einer Parabel mit demScheitelpunkt S (4 m=8 m).

b) Differenzieren ergibt

~v _~r 2 m=s8 m=sÿ4 m=s2 t

und

~a ~r 0ÿ4 m=s2

:

c) Für t0 0, t1 1 s, t2 2 s, t3 3 s, t4 4 s,t5 5 s ergibt sich

~v0 2 m=s8m=s

,~v1 2m=s

4m=s

,~v2 2 m=s

0

;

~v3 2 m=sÿ4 m=s

,~v4 2 m=s

ÿ8 m=s

,

~v5 2 m=sÿ12 m=s

,

~a0 ~a1 ~a2 ~a3 ~a4 ~a5 0ÿ4 m=s2

:

1.1 Kinematik 27

Page 28: Metzler Physik

yin m

xin m

υ

1

2

3

4

0

P

a

υ

a υa

υ

a4

1

2

3

0

4

P

P

PP 3

0

5P

1 4 5 10

1

5

81 2

d) In x-Richtung ist die Geschwindigkeit kon-stant, in y-Richtung ergibt sich eine konstantenegative Beschleunigung.

1 Mechanik28

Page 29: Metzler Physik

1.2.1 Trägheitsprinzip

1 Beschreiben Sie das Verhalten eines Mitfahrersin einem Auto bei verschiedenen Verkehrssituatio-nen und begründen Sie es mit dem Trägheitsprin-zip.

Lösung:Ð

2 Geben Sie drei Beispiele aus anderen Bereichen alsoben erwähnt für die Gültigkeit des Trägheitsprin-zips an (mit Begründung).

Lösung:Ð

3 Legt man eine Postkarte zusammen mit einemGeldstück auf ein Glas, so fällt das Geldstück insGlas, wenn man die Karte ruckartig wegzieht. Füh-ren Sie den Versuch aus. Begründen Sie.

Lösung:Ð

4 Man legt ein dünnes Brett auf eine Tischkante undbreitet über dem auf dem Tisch liegenden Teil eineZeitung aus. Drückt man das über die Tischkanteherausragende Brett leicht herunter, so hebt mandie Zeitung. Schlägt man dagegen schnell und kräf-tig auf das Brett, so bleibt die Zeitung liegen unddas Brett zerbricht. Führen Sie den Versuch ausund begründen Sie.

Lösung:Ð

*5 In einem mit der konstanten Geschwindigkeit vZ

fahrenden Zug wird ein Ball mit der Geschwindig-keit v0

a) senkrecht hoch geworfen,b) unter dem Winkel a nach hinten geworfen.c) Wie lauten die Bewegungsgleichungen im

Bezugssystem eines Beobachters, der den Zugan sich vorbeifahren sieht, für a) und für b),wenn in diesem Fall der Beobachter den Ballsenkrecht nach oben geworfen sieht?

Lösung:c) Bewegung des Zuges in x-Richtung; die

y-Achse zeigt nach oben.a) Bewegungsgleichungen für den Ball, dervom fahrenden Zug senkrecht nach oben ge-worfen wirdsx vZ t; vx vt; ax 0sy v0 tÿ 1

2 g t2; vy v0ÿ g t; ay ÿgb) Bewegungsgleichungen für den Fall, daûder Beobachter von Bahndamm den Ball senk-recht nach oben geworfen sieht.Bedingung vx ÿv0 cosaÿvZ.sx 0; vx 0; ax 0sy v0 sin a tÿ 1

2 g t2; vy v0 sin aÿ g t;ay ÿg.

1.2.2 Masse und Impuls

1 Zwei Gleiter A mA 0,4 kg und B bewegen sichnach dem Abbrennen der Schnur (Versuch 1) mitden Geschwindigkeiten vA ÿ3,2 m=s undvB 4,8 m=s in entgegengesetzter Richtung von-einander fort.a) Berechnen Sie die Masse von B.b) Berechnen Sie die Impulse und zeichnen Sie

beide als Vektoren in eine Skizze des Versuchsein.

Lösung:a) vA=vB ÿmB=mA; mB 0,267 kg.

b) pA ÿ 1,28 m kg=spB 1,28 m kg=s.

1.2.3 Impuls und Impulserhaltung

1 Ein stehender Güterwagen m1 20 t wird durcheinen anderen Güterwagen m2 30 t mit einerGeschwindigkeit von v2 5 km=h gerammt. Wel-che Geschwindigkeit ergibt sich, wenn die Wagen

nach dem Zusammenstoû miteinander zusammen-gekoppelt sind?

Lösung:Aus m2 v2 (m1m2)v0 folgt v0 3 km=h.

Seite

39

Seite

41

Seite

45

1.2 Dynamik; Impuls und Kraft 29

Page 30: Metzler Physik

2 Ein Körper A mit der Masse mA 4 kg stöût un-elastisch und in einem zweiten Fall elastisch mitder Geschwindigkeit vA 2 m=s auf einen KörperB mit mB 6 kg, der sich in gleicher Richtung mitvB 1,5 m=s bewegt.a) Berechnen Sie die Geschwindigkeiten und Im-

pulse für beide Fälle vor und nach dem Stoû.b) Was ergibt sich, wenn sich die Körper mit den

obigen Geschwindigkeitsbeträgen aufeinanderzu bewegen?

Lösung:a) Beim unelastischen Stoû:

v0 pA pB

mAmB 1,7

ms

;

p0 (mAmB) vM17 m kgs ;

beim elastischen Stoû:v0A 1,4 m

s ; p0A 5,6 m kgs ,

v0B 1,9 ms ; p0B 11,4 m kg

s ;

b) mit entgegengesetzten Geschwindigkeiten,d. h. vB ÿ1,5 m

s ;

beim unelastischen Stoû:v0 ÿ 0,1 m

s ; p0 ÿ1 m kgs ;

beim elastischen Stoû:v0A 2,2 m

s ; p0A 8,8 m kgs ,

v0B 2,9 ms ; p0B 17,4 m kg

s

3 Von einem Rollwagen m1 20 kg, der sich miteiner Geschwindigkeit v1 2,0 m=s bewegt,springt ein Junge m 60 kg in Fahrtrichtungab. Beim Auftreffen auf den Bodena) bewegt der Junge sich mit der gleichen Ge-

schwindigkeit, die der Wagen hat, bevor derJunge abspringt;

b) ist der Junge gegenüber dem Boden in Ruhe;c) bewegt sich der Junge mit der doppelten An-

fangsgeschwindigkeit des Wagens.Wie ändert sich jeweils die Geschwindigkeit desWagens, nachdem der Junge abgesprungen ist?

Lösung:Allgemein gilt (m1m2) v1 m1 v01m2 v02.a) Aus v02 v1 folgt auch v01 v1,b) Aus v02 0 folgt v01 8,0 m=s,c) Aus v02 2 v1 folgt v01 4,0 m=s.

4 Zwei Wagen m1 200 g, m2 500 g) fahren mitv1 32 cm=s und v2 ÿ21 cm=s aufeinander zu.Bestimmen Sie die Geschwindigkeit nach einemelastischen und nach einem unelastischen Stoû.

Lösung:Die Wagen laufen aufeinander zu, weil v1 > 0 undv2 < 0.

a) v01 ÿ 43,7 cm=s, v02 0,29 cm=s.b) v0 ÿ 5,86 cm=s.

5 Ein Wagen m1 4 kg trifft mit einer Geschwin-digkeit v1 1,2 m=s auf einen zweitenm2 5 kg, der sich in gleicher Richtung mitv2 0,6 m=s bewegt.Wie groû sind ihre Geschwindigkeiten nach demzentralen elastischen Stoû?

Lösung:v01 0,533 m=s; v02 1,133 m=s

*6 Eine Explosion zersprengt einen Stein in drei Teile.Zwei Stücke (m1 1,0 kg, m2 2,0 kg) fliegenrechtwinklig zueinander mit v1 12 m=s bzw.mit v2 8,0 m=s fort. Das dritte Stück fliegt mitv3 40 m=s weg.Ermitteln Sie aus einem Diagramm Richtung undMasse des dritten Stücks und berechnen Sie seineMasse.

Lösung:

a) Aus der Zeichnung erhält man mit tan am1 v1

m2 v2

die Richtung von v2 gegenüber ÿv3 zua 36,87 und die von v2 entsprechend zub 90 a 126,87.Nach dem Impulssatz bewegen sich alle dreiStücke in einer Ebene.

b) Aus dem Impuls m3 v3 (m1 v1)2 (m2 v2)2

p

folgt m3 0,5 kg.

*7 Ein Satellit bewegt sich horizontal mit einer Ge-schwindigkeit v1 8 km=s relativ zur Erde. Er solleine Ladung in horizontaler Richtung rückwärtsausstoûen, sodass diese senkrecht auf die Erde fällt.Satellit und Ladung wiegen 450 kg, die Ladungallein beträgt 50 kg.Mit welcher Geschwindigkeit relativ zur Erdemuss die Ladung ausgestoûen werden? WelcheGeschwindigkeit relativ zur Erde besitzt danachder Satellit?

Lösung:Die Frage geht nach der Geschwindigkeit der La-dung relativ zum Satelliten beim Ausstoûen.Mit (450 kg) v1 50 kg v01 400 kg v02 folgt nachv01 0, v02 9,0 km=s. Der Satellit bewegt sichmit v02 gegenüber der Erde weiter. Im System desSatelliten, das sich mit der Geschwindigkeit vonSatellit und Ladung vor dem Anstoû bewegt, gilt0 u01 50 kg u02 400 kg und damit u02 ÿ8 u02und mit u02v02ÿv1 1,0 km=s schlieûlichu01 ÿ 8,0 km=h.

1 Mechanik30

Page 31: Metzler Physik

Die Ladung gewegt sich zum Satelliten mitu01 ÿ 8,0 km=h (wie man auch ohne Rechnungsofort einsieht).

*8 Führen Sie die Rechnungen zur Bestimmung derEndgeschwindigkeit beim zentralen elastischenStoû (S. 44) im Einzelnen durch.

Lösung:Ð

*9 Vater m1 70 kg und Tochter m2 30 kg ste-hen zusammen auf dem Eis und stoûen sich gegen-seitig ab. Wie weit sind beide nach 5 s voneinanderentfernt, wenn der Vater sich mit v1 0,3 m=swegbewegt?

Lösung:Aus m1 v1 m2 v2 folgt v2 0,7 m=s und auss v t schlieûlich s1 1,5 m und s2 3,5 m.

*10 Ein Gleiter der Masse m1 300 g trifft mit derGeschwindigkeit v1 64,3 cm=s elastisch auf ei-nen zweiten mit der Masse m2 220 g, der sich inRuhe befindet.Berechnen Sie im Schwerpunktsystem beide Ge-schwindigkeiten und Impulse nach dem Stoû.

Lösung:Die Geschwindigkeit des Schwerpunktsystemsist gleich der Geschwindigkeit nach einem ange-nommenen unelastischen Stoû zu v0 31,7 cm=s.Im Schwerpunktsystem ergibt sich

u1 v1ÿ v0 27,2 cm=s undu2 v2ÿ v0 ÿ 37,1 cm=s.

Nach den Formeln für den elastischen Stoû, be-zogen auf das Schwerpunktsystem, erhält man

u01 ÿ 27,2 cm=s ÿu1 undu02 37,1 cm=s ÿu2

sowie q01 ÿ 8160 cm g=sq02 8162 cm g=s 8160 cm g=s.

Vergleich mit q1 8160 cm g=s undq2 ÿ 8162 cm g=sÿ 8160 cm g=s:

Im Schwerpunktsystem tauschen beide Impulsesich gegenseitig aus.

*11 Zwei Gleiter mit m1 300 g und m2 180 g sto-ûen mit den Geschwindigkeiten v1 35,0 cm=sund v2 ÿ39,5 cm=s elastisch aufeinander.a) Bestimmen Sie Geschwindigkeiten und Im-

pulse im Schwerpunktsystem vor und nachdem Stoû.

b) Berechnen Sie damit im System der Fahrbahndie Geschwindigkeiten und Impulse nach demStoû.

Lösung:Die Geschwindigkeit des Schwerpunktesystemsist v0 7,06 cm=s.

Damit wirdu1 v1ÿ v0 27,94 cm=s;u2 v2ÿ v0 ÿ 46,56 cm=s.

Im Schwerpunktsystem ergibt sich (Aufgabe 10oder nachrechnen mit Formeln für die Geschwin-digkeiten nach dem elastischen Stoû)

u01 ÿu1 27,94 cm=s; u02 ÿu2 46,56 cm=s.

Daraus berechnen sich die Geschwindigkeitenim System der Fahrbahn nach u0i v0iÿ v0

oder v0i u0i v0 (i = 1,2) zu v01 ÿ 20,88 cm=s;v02 53,62 cm=s.

Diese Geschwindigkeiten erhält man auch durchunmittelbare Anwendung der Formeln für denelastischen Stoû auf die ursprünglich gegebenenGeschwindigkeiten v1 und v2.

*12 Beschreiben Sie, wieso beim Schieûen ein Rück-stoû entsteht. Schätzen Sie ab, wie groû er ist.

1

2

= = 0υ 2υ

1υ′ << υ′1υ′ 2υ′

1 2p′ = m1 1υ′ p′1

= p′

m1 m1

<< m2 m2

2υ′2 =p′ m2

Lösung:Ð

Zusatzaufgaben

13 Zwei Gleiter einer Luftkissenfahrbahn (A mit derMasse mA 120 g und B mit der unbekanntenMasse mB) bewegen sich mit den Geschwindig-keiten vA 0,12 m=s und vB 0,15 m=s aufein-ander zu. ± Welche Masse besitzt jeweils B, wennsich beide Gleiter nach dem unelastischen Stoûa) in der Richtung wie A vor dem Stoû mit

v0 0,08 m=s,b) in der Richtung wie B vor dem Stoû mit

v0 0,06 m=s weiterbewegen?

1.2 Dynamik; Impuls und Kraft 31

Page 32: Metzler Physik

c) Nunmehr trifft der Gleiter A unter denselbenBedingungen wie vorher wieder im unelasti-schen Stoû auf den ruhenden Gleiter B mit derMasse aus a) bzw. mit der Masse aus b).

Lösung:Es ist nach mA vAmB vB (mAmB)v0

mB mAvAÿ v0

v0 ÿ vB.

a) mB 21 g, b) mB 240 g,

c) Für v0B 0 ist v0 vAmA

mAmB, also im Fall

a) v0 0,102 m=s, b) v0 0,04 m=s.

14 Ein Wagen A der Masse m1 trifft in einem erstenVersuch in zentralem Stoû auf ein ruhenden Wa-

gen B der Masse m2 mit der Geschwindigkeitv1 50 cm=s. Nach dem Zusammenstoû bewegtsich A in umgekehrter Richtung mit der Ge-schwindigkeit v01 10 cm=s, während sich Bmit v02 30 cm=s bewegt. In einem zweiten Ver-such wird A mit einem Gewicht der Masse 1 kgbeladen und stöût wiederum auf B mit der Ge-schwindigkeit von v1 50 cm=s. Nach dem Stoûbleibt A in Ruhe, während B mit v02 50 cm=sweiterfährt. ± Man berechne die Masse beiderWagen.

Lösung:Aus m1 v1 ÿm1 v01m2 v02 und

(m1m0) v1 m2 v1

folgt mit m0 1 kg, m1 1,0 kg, m2 2,0 kg.

1.2.4 Erstes und zweites Newton'sches Axion; Grundgleichung der Mechanik

1 Ein PKW (m 900 kg) erfährt eine Beschleuni-gung a 4,5 m=s2. Welche Kraft muss dabei vonden Rädern auf den Wagen übertragen werden?

Lösung:F m a 900 kg ´ 4,5 m=s2 4050 N.

2 Ein Junge bringt einen Ball der Masse m 0,5 kgin der Zeit t 0,2 s auf die Geschwindigkeitv 8 m=s. Welche (durchschnittliche) Kraft übter auf den Ball aus?

Lösung:

F m vt 0,5 kg 8 m=s

0,2 s 20 N.

3 Ein Zug der Gesamtmasse m 600 t erreicht beimAnfahren von der Haltestelle aus auf der Streckevon 2,45 km die Fahrgeschwindigkeit 120 km/h.Wie groû ist die als konstant angenommene Kraft,mit der die Lokomotive den Zug zieht?

Lösung:

F m v2

2 s

600 103 kg 120

3,6

2m2=s2

2 2,45 103 m

1,361 105 N mit a v2=(2 s) in F m a.

4 Ein PKW mit der Masse m 600 kg wird auf einerStrecke von 50 m durch die konstante KraftF 900 N abgebremst. Wie groû war die Anfangs-geschwindigkeit?

Lösung:Aus F m a und v

2 a sp

wirdv 1,225 101 m=s 44,1 km=h.

5 Ein Körper der Masse m 2 kg wird geradlinignach der Zeit-Geschwindigkeit-Kurve der folgen-den Grafik bewegt. Berechnen Sie daraus fürdie einzelnen Intervalle der Bewegung die wir-kende Kraft und zeichnen Sie das Zeit-Kraft-Diagramm.

υ in ms

t in s0 1 2 3 4 50

1

2

3

Lösung:Von 0 s bis 1 s : F 0,von 1 s bis 3 s : F 1,0 N,von 3 s bis 4 s : F 2,0 N,von 4 s bis 5 s : F 0,von 5 s bis 6 s : F ÿ 6,0 N.

Seite

47

1 Mechanik32

Page 33: Metzler Physik

F in N

1 2 3 4

5 6

t in s

12

–1

–6

*6 Ein PKW (m 1000 kg) fährt bergan auf einerStraûe mit dem Steigungswinkel a 20. WelcheKraft erzeugt der Motor, wenn das Auto berganfährta) mit konstanter Geschwindigkeit;b) mit einer (konstanten) Beschleunigung von

0,2 m=s2?c) Mit welcher Kraft drückt das Auto in beiden

Fällen auf die Straûe?d) Wie lautet die Antwort, wenn das Auto unter

den Bedingungen a) und b) bergab fährt?

Lösung:a) Die Antriebskraft F ist gleich der Hangabtriebs-

kraft F G sin a 3,355 103 N.b) Die Antriebskraft wie in a) wird vermehrt um

die beschleunigende Kraft zuF m (g sin a a) 3,555 103 N.

c) Die Kraft, die auf die Straûe wirkt (Normal-kraft FN), ist FN G cos a 9,218 103 N.

d) Bei konstanter Geschwindigkeit müsste dasAuto mit der Kraft FB 3,355 103 N bremsen.Soll das Auto mit der Beschleunigung 0,2 m=s2

bergab fahren, muss es eine Bremskraft entwi-ckeln, die die Hangabtriebskraft teilweise auf-hebt, m amg sin aÿFB und darausFB 3,155 103 N.

*7 Ein PKW (m 720 kg) wird durch eine (konstante)Bremskraft F 4,37 kN auf einem Weg s 68 mauf die Hälfte seiner Geschwindigkeit abgebremst.a) Aus welcher Geschwindigkeit wurde er abge-

bremst?b) Wie lange dauert der Bremsvorgang?

Lösung:a) Aus a F=m 6,07 m=s2 errechnet man mit

v20ÿ ( 1

2 v0)2 2 a D s die Anfangsgeschwin-digkeit zu v0 33,2 m=s 119,5 km=h.

b) Dv=Dt 12 v0=Dt a ergibt die Zeit des

Bremsvorganges Dt 1=2 v0=a 2,74 s.

*8 Ein PKW (m 900 kg) soll auf einer Strecke vonl 150 m von der Geschwindigkeit v1 10 m=sauf die Geschwindigkeit v2 40 m=s beschleunigtwerden.a) In welcher Zeit geht der Beschleunigungsvor-

gang vor sich?b) Wie groû ist die Beschleunigung?c) Welche (konstante) Kraft ist erforderlich?

Lösung:a) Aus Dv=Dt a mit Dv v2ÿ v1 und

v22ÿ v2

1 2 a D s gewinnt man durch Gleichset-zen von aD t 2D s=(v1 v2) 6,0 s.

b) Die Beschleunigung a errechnet man direkt ausder zweiten Gleichung in (a) oder mit dem Er-gebnis aus (a) zu aDv=Dt oder zu a 5,0m=s2.

c) Die beschleunigende Kraft ist F m a 5,40 103 N.

*9 Über eine feste Rolle wird eine Schnur gehängt, andie an den beiden Enden zwei Körper mit den Mas-sen m1 und m2 (m1 <m2) gehängt werden. Was ge-schieht? Beschreiben und analysieren Sie den Be-wegungsvorgang.

Lösung:Auf die beiden Körper wirken einzeln die Ge-wichtskräfte

G1 m1 g und G2 m2 g:

Beide zusammen mit der Masse mm1m2 be-wegen sich unter dem Einfluû der Differenz derGewichtskräfte F m aG1ÿG2:

Daraus erhält man die Beschleunigung

am1ÿm2

m1m2g:

Das ist das Prinzip der Atwoodschen Fallmaschine(Georg Atwood, 1745±1807), mit der man bei ge-eigneter Wahl von m1 und m2 die Beschleunigung abequem messen und damit die Fallbeschleunigungg bestimmen kann.

1.2 Dynamik; Impuls und Kraft 33

Page 34: Metzler Physik

1.2.5 Dynamische und statische Kraftmessung

1 Eine Schraubenfeder wird mit Wägestücken belas-tet. Die Stellung x des unteren Federendes wird ge-messen.Bestimmen Sie aus den folgenden Messwerten dieFederkonstante.

Belastung Fin N

0,00 0,50 1,00 1,50 2,00 2,50

Stellung xin cm

49,6 51,1 52,6 54,2 55,7 57,3

Lösung:Die Auswertung in einer Tabelle ergibt:

F in N 0,00 0,50 1,00 1,50 2,00 2,50

x in cm 0,0 1,5 3,0 4,6 6,1 7,7

D in N=cm ± 0,333 0,333 0,326 0,328 0,325

Gemittelt wird D 0,329 N=cm 3,29 ´ 101 N=m.

2 Ein Körper wird auf einer schiefen Ebene (Stei-gungswinkel a) durch eine Kraft F im Gleichge-wicht gehalten. Wie muss F gewählt werden, wennsiea) parallel zur schiefen Ebene,b) horizontal,c) unter dem Winkel b zur Horizontalen wirkt?

Lösung:

α

β

α

α

–F

1

2

3

F

G

F

F

1

a) F1 G sin a,b) F2 G sin a,

c) F3 F2

cos (aÿb) G

sin acos (aÿb)

.

3 Überprüfen Sie die Gültigkeit des Hooke'schenGesetzes an einer Gummischnur. Was zeigt sich?

Lösung:Die Auslenkungen einer Gummischnur bei Bela-stung entsprechen nicht dem Hookeschen Gesetz.

*4 Zwei Schraubenfedern mit den FederkonstantenD1 und D2 werden nebeneinander bzw. hinterei-

nander zusammengefügt. Wie lauten die Gesetzefür die beiden Kombinationen der Schraubenfe-dern jeweils? Vergleichen Sie.

Lösung:Die Schrauben hängen nebeneinander.Beide werden um die gleiche Strecke x1 ausgelenktund erzeugen die Kräfte F1 D1 x1 undF2 D2 x1, die in ihrer Stimme der GewichtskraftG das Gleichgewicht halten:GD x1 F1F2 D1 x1D2 x1.

Daraus gewinnt man DD1D2:

Die Schrauben hängen aneinander.Auf beide Schraubenfedern wirkt die gleiche KraftF1 F2 G oderGD1 x1, GD2 x2.

Die Kombination wird um die Strecke x1 x2 aus-gelenkt:GD xD (x1 x2).

Löst man nach x, x1 und x2 auf, so erhält man ausx x1 x2

GD G

D1 G

D2oder

1D 1

D1 1

D2.

*5 Zur Messung des Volumens von Körpern unregel-mäûiger Form benutzt man das Archimedes'schePrinzip. Man hängt den Körper frei an eine Schrau-benfeder, Dehnung s1, und taucht ihn dann in eineFlüssigkeit bekannter Dichte rFl, z. B. Wasser,Verlängerung an der Schraubenfeder nunmehr s2.a) Zeigen Sie, dass damit für die Dichte des Kör-

pers gilt:r rFl s1=s1ÿ s2.

b) Ein Versuch ergibt s1 20,4 cm unds2 12,9 cm (rWasser 1,00 103 kg=m3). Be-stimmen Sie die Dichte des Körpers.

Lösung:a) Hängt man den Körper, Volumen V, Dichte r,

an die Schraubenfeder, giltG g r V D s1.

In der Flüssigkeit wirkt der AuftriebA g rFl V der Gewichtskraft entgegen:GÿA g (rÿrFl) V D s2.

Löst man beide Gleichungen nach D auf undsetzt die Terme gleich, so fällt g V heraus,und man erhält nach Auflösen nach rr rFl s1=(s1ÿ s2):

b) Man erhält r 2,72 10 kg=m3. Der Körperkönnte aus Aluminium sein.

Seite

49

1 Mechanik34

Page 35: Metzler Physik

1.2.7 Reibungskräfte

1 Legt man einen Besenstiel waagerecht auf die bei-den Zeigefinger bei vorgestreckten Armen und be-wegt die Hände nun langsam aufeinander zu, bleibtder Besenstiel stets im Gleichgewicht. ErklärenSie.

Lösung:Der Versuch demonstriert den Unterschied zwi-schen Haft- und Gleitreibung. Wo das kürzereStück des Besenstiels über die Hand hinausragt,ist die Auflagekraft auf die Hand und damit dieReibungskraft geringer als auf der anderen Seite.Der Stiel beginnt hier über die Hand zu gleiten,bis die Gleitreibung auf dieser Seite gröûer ist alsdie Haftreibung auf der anderen. Dann beginnt derStiel dort zu gleiten, bis sich die Verhältnisse zwi-schen gleitender und haftender Reibung wiederumkehren.Dieses Spiel setzt sich fort, bis sich die Händeschlieûlich genau unter dem Schwerpunkt des Be-senstiels befinden.

2 Ein Schlepper zieht vier hintereinander gekoppelteLastkähne (gleiche Masse, gleicher Widerstandbei der Fahrt durch das Wasser) mit der KraftF 2000 N.a) Wie groû ist jeweils die Kraft, die das Seil zwi-schen dem ersten und zweiten, dem zweiten unddritten und dem dritten und vierten Kahn belastet?b) Macht es einen Unterschied, ob der Schlepper denZug beschleunigt oder ihn mit konstanter Geschwin-digkeit zieht?

Lösung:a) Ist der Widerstand eines einzelnen Lastkahns

bei der Fahrt durch das Wasser Fw, so ziehtder Schlepper mit der Kraft F 4 Fw alleKähne. Die Zugkraft des Seils zwischen erstemund zweitem Kahn ist 3 Fw 1500 N, zwischenzweitem und drittem 2 Fw 1000 N und zwi-schen drittem und viertem Fw 500 N.

b) Bei Beschleunigung muss entsprechend derMasse der Kähne (und ihrer Beschleunigung)die Zugkraft gröûer sein; sie verteilt sich aberauf die Seile in gleicher Weise wie oben.

*3 Auf einer schiefen Ebene aus Holz (Neigungswin-kel a 30) ruht ein Holzklotz mit der Gewichts-kraft G 2,0 N, an dem zusätzlich parallel zurschiefen Ebene nach oben eine Kraft F angreift.

a) Wie groû muss F mindestens und darf F höch-stens sein, damit der Klotz auf der Unterlagehaften bleibt?

b) Was ergibt sich bei a 20?

Lösung:Der Holzklotz gleitet von selbst die schiefe Ebenehinab, wenn die Hangabtriebskraft FA G sin agröûer als die Haftreibungskraft FH fH G cos aist.In diesem Fall muss die äuûere, nach oben parallelzur schiefen Ebene wirkende Kraft mindestens dieGröûeFmin FAÿFH G (sin aÿ fH cos a)besitzen. Sie darf aber auch nicht gröûer werden alsdie Summe aus der Hangabtriebskraft FA und der(dann schräg nach unten wirkenden) Haftreibungs-kraft FH:Fmax FAFH G (sin a fH cos a):a) a 30: Fmin 0,134 N, Fmax 1,866 N.b) a 20: FAÿFH ÿ 0,256 N < 0.Der Körper gleitet von selbst die Ebene nicht hinab,hier wäre Fmin 0: Fmax 1,624 N.

Zu Aufgabe 4, 5 und 6 siehe den Exkurs Seite 53*4 Für ein Auto (m 1000 kg) mit Hinterradantrieb

betrage die Normalkraft auf die Hinterachse dieHälfte der Gesamtnormalkraft (fH 0,7). Wie groûist die Beschleunigung beim Anfahren auf ebenerStraûe und beim Anfahren auf einer Straûe mit20% Steigung?

Lösung:Es ist FA [ FH und FH fH FN1 fH

12 G:

Die Beschleunigung ist a [ FH=m oder mitFH fH

12 mg folgt a [ fH

12 g 3,43 m=s.

Bei einer Steigerung von 20% (arc tan 0,211,31)beträgt die Normalkraft FN1 1

2 G cos a und dieHangabtriebskraft FH G sin a, sodass fürdenBe-schleunigungsvorgang die Kraft F fH FN1 ÿFH

zur Verfügung steht.Daher ist a [ fH

12 g cos aÿ g sin a 1,44 m=s2.

*5 Welche Antriebskraft ist erforderlich, um ein Auto(m 1300 kg) auf horizontaler Straûe unter Be-rücksichtigung seines Luftwiderstandes (A1,9 m2, cW 0,4) und seiner Rollreibung(fR 0,03) auf der Geschwindigkeit v 50; 100;150; 200 km/h zu halten?

Seite

53

1.2 Dynamik; Impuls und Kraft 35

Page 36: Metzler Physik

Lösung:Die Antriebskraft FA muss der Bedingung genü-gen:FA FRFL fR G 1

2 cW A r v2.Die Kraft FA muû bei der Geschwindigkeit v min-destens betragen:

v in km=h 50 100 150 200

FA in N 477 762 1236 1899

*6 Ein Auto (m 1000 kg, A 2,0 m2, cW 0,4,fR 0,03) mit Allradantrieb fährt auf einer Straûe(fH 0,6) mit dem Steigungswinkel a 10 berg-auf. Berechnen Sie einschlieûlich Luftwiderstandund Rollreibung, welche Beschleunigung aus derGeschwindigkeit v 0; 50; 100; 150; 200 km/hnoch möglich wäre.(Allradantrieb heiût: Das gesamte Gewicht trägtzur Normalkraft bei.)

Lösung:Zur Beschleunigung steht die KraftF FAÿFRÿFL oderF (fHÿ fR) G cos aÿ 1

2 cW A r v2

zur Verfügung. Die mögliche Beschleunigung istalso a (fHÿ fR) g cos aÿ 1

2 cW A r v2. Dann istbei der Geschwindigkeit v die Beschleunigung amöglich.

v in km=h 0 50 100 150 200

Fa in m=s2 5,507 5,407 5,108 4,609 3,911

Zusatzaufgaben

7 Ein Klotz (m 0,85 kg) wird auf einer horizontalenTischfläche mit einer konstanten Kraft F 1,9 Ngezogen und legt dabei aus der Ruhe in t 6,2 sden Weg s 4,5 m zurück.a) Welche Beschleunigung erfährt der Körper?b) In welchem Verhältnis steht die Kraft F zur

Masse m?c) Die Antwort b) stimmt nicht mit der zu a) über-

ein. ± Was kann man daraus über die Bewegungschlieûen? Die Antwort ist möglichst zahlen-mäûig zu ergänzen.

d) Wie groû ist die Gleitreibungszahl fG?

Lösung:Die Kraft wirkt parallel zur Tischfläche.a) Die Beschleunigung der (gleichmäûig be-

schleunigten) Bewegung ist

a1 2 st2 0,234 m=s2.

b) Die Kraft F würde nach dem Grundgesetz derMechanik die Beschleunigung

a2 Fm 2,235 m=s2 bewirken.

c) Der antreibenden Kraft F muss eine Reibungs-kraft R entgegenwirken:

R FÿF1 Fÿma1 1,701 N.

Die Gleittreibungszahl fG FR=G 0,4 ent-spricht etwa der Gleitreibung eines Holzklotzesauf Holz (siehe Tab. 52.3).

*8 Für den Luftwiderstand eines Fallschirmes ist Aseine auf die Horizontale projizierte Fläche. Fürdie hier betrachtete Fallschirmsorte sei cW 2,0.a) Mit welcher (konstanten) Geschwindigkeit fällt

ein Springer vom 100 kg Masse, Radius desSchirms 1,8 m?

b) Aus welcher Höhe würde ein Körper im freienFall dieselbe Geschwindigkeit erreichen? Ver-gleichen Sie: Für einen geübten Springer ist einSprung aus einer Höhe bis zu 4 m noch unge-fährlich.

c) Wie groû ist der Fallschirm zu wählen, wennmit ihm ein Auto (m 1000 kg) abgesetzt wer-den soll? Dabei gehe man davon aus, dass dasAuto bei freiem fall aus einer Höhe von zu 3 mfast unbeschädigt bleibt.

Lösung:a) Es muss gelten G FL, also v2

2 m g=(cW A r) oder v 8,63 m=s. Vergleichmit freiem Fall aus 4 m Höhe: v0

2 g hp

8,86 m=s.b) Für 3 m Höhe ist v0

2 g hp 7,67 m=s.

Mit v [ v0 ist A 2 m gcW v2 r

oder A mcW h r

(mit v2 2 g h) und A 128,98 m2 oder r 6,41 m.

1 Mechanik36

Page 37: Metzler Physik

1.2.8 Kräfte bei der Kreisbewegung

1 Im einfachen Atommodell des Wasserstoffatomsumkreist das Elektron (mE 9,1 10ÿ31 kg) dasProton auf einer Kreisbahn mit dem Radiusr 0,5 10ÿ10 m mit einer Geschwindigkeit v2,2 106 m=s. Wie groû müsste danach die Kraftzwischen Elektron und Proton sein?

Lösung:Nach F m v2=r ist die Kraft F 8,81 ´ 10ÿ8 N.

2 Ein Körper (m 0,1 kg) wird an einer Schnur(l 0,5 m) auf einem Kreis herumgeschleudert,dessen Ebene senkrecht zur Erdoberfläche steht.a) Wie groû müssen mindestens die Winkelge-schwindigkeit und die Drehzahl pro Minute sein,damit der Körper im oberen Punkt seiner Bahnnicht herunterfällt?b) Welche Reiûfestigkeit (in N) muss die Schnurhaben?

Lösung:a) Für die Zentrifugalkraft gilt j FZ j > jG j oder

w2 r > g, d. h. w >g=r

p 4,43 sÿ1 oder

n 60 f > 42,3 1=minÿ1.b) Für die Zerreiûfestigkeit gilt FR > FgFZ

2 m g 1,96 N.

3 Ein Körper bewegt sich mit konstanter Geschwin-digkeit v auf einer Kreisbahn mit dem Radius r.a) Wie ändert sich die Radialbeschleunigung,wenn sich die Geschwindigkeit bzw. der Radiusverdoppelt?b) Warum kann sich kein Körper exakt recht-winkelig um eine Ecke bewegen?

Lösung:a) Bei Verdoppelung der Bahngeschwindigkeit

vervierfacht sich die Radialbeschleunigung,bei Verdoppelung des Radius halbiert sie sich.

b) Wegen r! 0 würde gelten a v2=r!1.

4 Ein Körper (m 0,4 kg) wird an einer 0,8 m lan-gen Schnur 80-mal in der Minute auf einem Kreis,der in einer waagerechten Ebene liegt, herumge-schleudert.a) Welche Zugkraft muss die Schnur aushalten?b) Bei welcher Umdrehungszahl reiût die Schnur,

wenn ihre Zugfestigkeit mit 500 N angegebenist?

Lösung:a) Mit T 60 s = 80 ist die Zugkraft ist Z

m (2 p=T)2 r 2,25 ´ 101 N.b) Mit n 60s=T folgt aus FZ 500 N und FZ <

m (2p n=60 s)2r für die Umdrehungszahl proMinute n < 377.

5 Welche Zentripetalkraft wird aufgrund der tägli-chen Drehung der Erde um ihre Achse auf einenmit ihr fest verbundenen Körper (m 70 kg) aufder geografischen Breite ò 0; 30; 60; 90 aus-geübt?

Lösung:Der Abstand von der Erdachse ist r R cos a,die Zentrifugalkraft also FZ m w2 R cos a mitw 2 p=T .

Man erhält für die Zentrifugalkräfte

ò 0ò 30ò 60ò 90

FZ 2,359 NFZ 2,043 NFZ 1,179 NFZ 0.

6 Ein Wagen (m 1200 kg) fährt mit einer Ge-schwindigkeit v 120 km=h durch eine Kurvemit einem Krümmungsradius von r 2500 m.Wie groû ist die Zentripetalkraft, die ihn auf derBahn hält? Wie groû ist das Verhältnis Zentripetal-kraft zu Gewichtskraft des Wagens? Was bedeutetdas für die Haftreibungskraft?

Lösung:a) FZ m v2=r 533,2 N.b) FG m g 11 772 N; FR 4,5=100 FG.

7 Ein Schnellzug durchfährt mit der Geschwindig-keit v 120 km=h eine Kurve vom Radiusr 2500 m. Wie groû muss man die Überhöhungü (in mm) der äuûeren Schiene (Spurweitew 1435 mm) wählen, damit beide Schienengleich belastet werden?

Lösung:Es gilt tan a FR

Gund sin a Èu

w, also

ü w sin (arc tan (FR=G))

w sin (arc tan(v2=(r g))) 64,9 mm.

*8 Eine Straûenkurve mit dem Radius 300 m sei nichtüberhöht, sodass ein Auto (m 900 kg) in derKurve allein durch die Haftreibungskraft zwischenReifen und Straûe gehalten wird. (Das reale Kur-venverhalten eines Autos kann auf diese Weiseallerdings nicht erfasst werden, da die Verhältnissekomplizierter als hier darstellbar sind.) Mit wel-cher Höchstgeschwindigkeit kann ein Auto dieKurve auf a) trockener (fH 0,8), b) nasser

Seite

55

1.2 Dynamik; Impuls und Kraft 37

Page 38: Metzler Physik

(fH 0,5) und c) vereister (fH 0,1) Straûe durch-fahren?

Lösung:Die Zentripetalkraft soll durch die Haftreibungs-kraft aufgebracht werden:

m v2=r [ fH m g oder v [fH rgp

.

a) Auf trockener Straûe v [48,5 m=s175 km=h,b) auf nasser Straûe v [38,4 m=s138 km=h,c) auf vereister Straûe v [17,2m=s61,8km=h.

Nach dieser Rechnung hat die Masse keinen Ein-fluû auf die Kurvengeschwindigkeit, solange sichdie Haftreibungszahl nicht ändert.Die Mechanik der Kurvenfahrt ist zu kompliziert,als dass man sie hier darstellen könnte.Die Rechnung liefert daher auch kein realistischesBild der wirklichen Verhältnisse.

*9 Ein Pkw (m 1300 kg) fährt mit konstanter Ge-schwindigkeit v 40 km=h über eine gewölbteBrücke. Der Radius des Brückenbogens beträgtR 50 m. Mit welcher Normalkraft belastet derPkw die Brückenmitte? Bei welcher Geschwindig-keit würde der Wagen abheben?

Lösung:Auf der Brückenmitte wirkt bei normaler Ge-schwindigkeit ein Teil der Gewichtskraft als Zen-tripetalkraft, so dass der Wagen die Brücke mit derKraft

F GÿFR m (gÿ v2=r) 9,54 ´ 103 N

statt mit der vollen Gewichtskraft vonG 1,2753 ´ 104 N belastet.Hebt der Wagen ab, wäre die Gewichtskraft gleichder Zentripetalkraft:

FR G oder v2 r g oder v 79,9 km=h.

*10 Stöût man ein Fadenpendel (Masse m, Pendel-länge l) so an, dass sich das Pendel mit der Win-

kelgeschwindigkeit w auf einem Kreis bewegt,wobei Pendelfaden und Drehachse den Winkela bilden, so gehört zu jeder Winkelgeschwindig-keit w ein fester Winkel a (Abbildung unten).a) Berechnen Sie die Zentripetalkraft F und zei-

gen Sie, dass cos a g=w2 l gilt.b) Berechnen Sie Winkelgeschwindigkeit,

Bahngeschwindigkeit und die Zugkraft aufden Pendelfaden (l 0,6 m), wenn das Kreis-pendel (m 2 kg) einen Winkel a 15 mitder Drehachse bildet.

α

υ α

ω

ra

RF

GF

R

Lösung:a) Zerlegung von Gm g in eine Komponente

längs des Pedalfadens und eine für die Zentri-petalkraft ergibt mit r l sin a undFZ G tan a aus m w2 l sin am g tan a

schlieûlich cos a gw2 l

.

b) Aus w2 gl cos a

folgt w 4,11 sÿ1 und

vw r l sin a 6,39 ´ 10ÿ1 m=s sowie

Z Gcos a

2,03 ´ 10ÿ1 N.

1.2.9 Trägheitskräfte im beschleunigten Bezugssystem: Galilei-Transformation undInertialsystem

1 Ein Fahrzeug durchfährt eine Kurve mit der kon-stanten Geschwindigkeit v 90 km=h. Ein Kraft-messer, an dem eine Kugel (m 500 g) hängt,zeigt während der Kurvenfahrt die KraftF 6,0 N an.Beschreiben Sie den Vorgang von beiden Bezugs-systemen aus und berechnen Sie den Kurvenradius.

Lösung:Im bewegten Bezugssystem registriert der Be-obachter, daû die Kugel mit der GewichtskraftG 4,905 N durch eine waagerecht wirkende KraftFZ

F2ÿG2p

3,456 N aus der Vertikalen her-ausgezogen wird. Er deutet FZ als Zentrifugalkraft.Im ruhenden Bezugssystem sieht der Beobachter,

Seite

58

1 Mechanik38

Page 39: Metzler Physik

dass sich die Kugel auf einem Kreis bewegt. Durchdie Auslenkung entsteht eine rücktreibende KraftFR

F2ÿG2p

3,456 N, die die Kugel als Zent-ripetalkraft auf der Kreisbahn hält.

Den Bahnradius erhält man zu rmv2

FR 90,43 m.

*2 Konstruieren Sie die Bahn einer Kugel, die aufeiner mit der Winkelgeschwindigkeit w 1

12 sÿ1

rotierenden Scheibe vom Mittelpunkt gegenüberdem Laborsystem mit der Geschwindigkeitv 6 cm=s abgeschossen wird, im Laborsystemund im System der drehenden Scheibe.

Lösung:Im Laborsystem bewegt sich die Kugel in ersterNäherung geradlinig mit der Geschwindigkeitv 6 cm=s vom Mittelpunkt weg.Im bewegten System konstruiert man eine Spiral-bewegung. (Für die Zeichnung wähle man einengeeigneten Maûstab, z. B. 6 m=s é 1 cm.)

*3 Ein Fahrstuhl bewegt sich mit konstanter Be-schleunigung a< g nach unten. Im Fahrstuhl hängtan einem Kraftmesser eine Kugel.Beschreiben und skizzieren Sie mit Angabe derKräfte,a) was ein Beobachter A im System 1, in dem derFahrstuhl beschleunigt wird, und b) ein Beobach-ter B im System 2, das mit dem Fahrstuhl verbun-den ist, beobachten und feststellen.

Lösung:a) Im System 1 folgert man, daû der Kraftmesser

die Kraft F m (gÿ a) deshalb anzeigt, weilder Fahrstuhl mit konstanter Beschleunigunga in der Richtung bewegt wird, in der auch dieSchwerkraft wirkt.

b) Im System 2 schlieût man auf eine Trägheits-kraft FT m a, nach oben wirkend, die derGewichtskraft Gm g entgegenwirkt, so daûdie Kugel den Kraftmesser mit der KraftF m (gÿ a) dehnt.

*4 In einem Fahrstuhl steht ein Mann auf einer Perso-nen-(Feder-)waage. Sie zeigt beim Anfahren desFahrstuhls F 950 N, beim Halten F 880 N an.a) In welcher Richtung fährt der Fahrstuhl an?b) Berechnen Sie die Beschleunigung.c) Was ergibt sich, wenn der Fahrstuhl mit einer

betragsmäûig gleichen Beschleunigung anhält?d) Zeichnen Sie die Verhältnisse beim Anfahren

und Anhalten.

Lösung:a) Der Fahrstuhl wird nach oben beschleunigt,

denn im System des Fahrstuhls wirkt aufden Mann zusätzlich zur GewichtskraftGm g die Trägheitskraft FT m a. DerKörper drückt daher mit der GesamtkraftF1 m (g a) 950 N auf die Waage.

b) Aus der Gewichtskraft Gm g, der Kraft G880 N beim Halten, berechnet sich die Massem 89,7 kg und daraus die Beschleunigungbeim Anfahren zu a (990 N ÿ 880 N)=m 0,78 m=s2.

c) Die Beschleunigung beim Anhalten wirkt inRichtung der Schwerkraft, also zeigt die Waagedie Kraft F2 Gÿ (F1ÿG) 770 N an.

d) ±

*5 Ein Körper wird mit der Geschwindigkeit v0 unterdem Winkel a zur Horizontalen abgeworfen. Vonwelchem Bezugssystem aus erscheint die Bewe-gunga) als freier Fall;b) als waagerechter Wurf;c) als senkrechter Wurf?

Lösung:In a) dürfte sich der Körper im neuen System inwaagerechter Richtung (orientiert am Laborsys-tem) nicht bewegen: Das neue Bezugssystemmüûte daher in dieser Richtung die Geschwindig-keit v0 cos a gegenüber dem Laborsystem besit-zen. In vertikaler Richtung dürfte man nur einereine Fallbewegung registrieren: In vertikalerRichtung müsste daher das neue System gegenüberdem Laborsystem mit der Geschwindigkeitv0 sin a bewegt werden.¾hnlich überlegt man für b): Das neue Systemmüsste sich gegenüber dem Laborsystem nachoben mit v0 sin a bewegen. Man registriert imneuen Bezugssystem die Bewegung eines waage-rechten Wurfes mit der horizontalen Anfangsge-schwindigkeit v0 cos a.Umgekehrt bewegt sich in c) das neue Bezugs-system gegenüber dem alten in waagerechter Rich-tung mit der Geschwindigkeit v0 cos a.Allgemein setzt man für die Galilei-Transforma-tion an, wenn x (t), y (t) die Gleichungen der Bewe-gungen im Laborsystem, x0 (t), y0 (t) die im neuenBezugssystem und x0 (t), y0 (t) diejenigen für denUrsprung des bewegten Systems sind:

x (t) x0 (t) x0 (t), y (t) y0 (t) y0 (t).

1.2 Dynamik; Impuls und Kraft 39

Page 40: Metzler Physik

Setzt man für x (t), y (t) die Gleichung des schrägenWurfs (! 1.1.6) und für x0 (t), y0 (t) die der ge-wünschten Bewegung in neuen Bezugssystemein, so ergeben sich für x0 (t), y0 (t) die gesuchtenBewegungsgleichungen für den Ursprung desneuen Systems gegenüber dem Laborsystem.In a) wäre x0 (t) 0,y0 (t)ÿ 1

2 g t2 zu setzen, esergibt sich x0 (t) v0 cos a t, y0 (t) v0 sin a t.In b) setzt man wieder y0 (t)ÿ 1

2 g t2 und weiterx0 (t) v00 (t) mit z. B. v00 v0 cos a und erhältx0 (t) 0 sowie y0 (t) v0 sin a t: Der Körperscheint (mit dem genannten v00 als Beispiel) imneuen Bezugssystem waagerecht mit der Ge-schwindigkeit v0 cos a abgeworfen worden zu sein.In c) erhält man mit dem Ansatz x0 (t) 0 undy0 (t) v00 tÿ 1

2 g t2 mit z. B. v00 v0 sin a die Lö-sung x0 (t) v0 cos a t und y0 (t) 0: Im neuenKoordinatensystem scheint der Körper (mit den ge-nannten v00 als Beispiel) nach oben mit v0 sin a ab-geworfen zu sein.

Zusatzaufgabe

*6 Auf einem Tanker, der 15 Knoten (s. Aufgabe 1,Seite 37) Fahrt macht, startet vom Heck aus einMoped mit konstanter Beschleunigung von0,8 m=s2. Wie lauten die Bewegungsgleichungenbeim Beschleunigungsvorgang, bezogen auf einmit dem Tanker bzw. dem Meeresgrund verbunde-nes System?

Lösung:Gegenüber dem Tanker gilt

s (0,4 m=s2) t2, v (0,8 m=s2) t, a 0,8m=s2,

gegenüber dem Meeresgrund mit vg 15 Knoten 7,72 m=s

s (0,4 m=s2) t2 (7,72 m=s) t,v (0,8 m=s2) t (7,72 m=s),a 0,8 m=s2.

1 Mechanik40

Page 41: Metzler Physik

1.3.1 Mechanische Energie

1 Ein Körper der Masse m werde auf lotrechtemWege von der Höhe h1 auf die Höhe h2 gebracht.Dabei gelte h1 < h2 (Fall I) und h1 > h2 (Fall II).Fertigen Sie für beide Fälle eine Zeichnung an undberechnen Sie die erforderliche mechanische Ener-gie.

Lösung:Fall I: Em g h2ÿ h1> 0;Fall II: Em g h2ÿ h1< 0:Zeichnung: ±

2 Ein Körper der Masse m wird um die Strecke s eineschiefe Ebene hinaufgezogen bzw. hinabgelassen.(Die schiefe Ebene bilde mit der Waagerechten denWinkel a.) Fertigen Sie eine Zeichnung an und be-rechnen Sie für beide Fälle die mechanische Ener-gie.

Lösung:Beim Hinaufziehen: F wirkt in Richtung von s

E F s G s sin a > 0.Beim Herablassen: F wirkt entgegengesetzt von s

E ÿF s ÿG s sin a < 0.

α αG

F

s

F = G sinα

*3 Berechnen Sie die mechanische Energie für Auf-gabe 1 und 2 mithilfe des skalaren Produktes, in-dem Sie für jeden einzelnen Fall eine Zeichnungmit den entsprechenden Vektoren anfertigen.

Lösung:Zu Aufgabe 1:Fall I: ~F ÿ~G zeigt wie~s nach oben.Fall II: Hier zeigt~s nach unten.Fall I: h2 > h1:

E~F~s jFj jsjcosû~F,~sm gh2ÿ h1cos 0 m g h2ÿ h1 > 0,

Fall II: h2 < h1:

E~F ~s jFj jsjcosû~F,~sm gh1ÿ h2cos 180

ÿm g h1ÿ h2 < 0.

Zu Aufgabe 2:Beim Hinaufziehen:Ansatz wie Fall I oben.E~F ~s mit cos û~F,~s cos (90 ÿa sin amit a Steigungswinkel wie in Abb. Lösung Auf-gabe 2.Damit wirdE jFj jsj sin a m gs sin a G s sin a > 0.Beim Herablassen:Ansatz wie Fall II obenE~F ~s mit cos û~F,~s cos 90a ÿ sin a.AlsoEjFj jsjÿsin a) =ÿm g s sin aÿG s sin a< 0.

*4 Berechnen Sie nach Abb. 61.1 die Teilenergienund bilden Sie angenähert die Gesamtenergie(1 cm é 1 N bzw. 1 m).

F

4

1

cos

α1

32

α

ααF2 F3

F4

∆s2

∆s3

∆s4

∆s1

F1 α1

cosF2 α2cosF3 α3

cosF4 α4

Zur Berechnung der Austausch-energie aus Teilsummen

Lösung:Summiert werden die Produkte aus den Strecken,die die Projektionen der Kräfte auf die Wegrichtun-gen darstellen, und aus denen, die die Wege bilden.E 1,0 N 2,0 m 1,2 N 1,8 m 0,8 N 1,7 m 0,6 N 1,3 m

2,0 J 2,16 J 1,36 J 0,78 J 6,3 J.

*5 Ein quaderförmiges Wasserbecken der Grundflä-che A und der Höhe h werde durch einen am Be-ckenboden befindlichen Einlauf mit Wasser ge-füllt.a) Erläutern Sie, dass die mechanische Energie,

um vom Einlauf auf dem Beckenboden ausdas Becken mit Wasser zu füllen, E12 rAh2 g 1

2 rV gh 12 mw gh ist, wobei V

das Volumen des Beckens und mW die Gesamt-menge des Wassers ist (Zeichnung).

b) Berechnen Sie die mechanische Energiemithilfe des Wegintegrals der Kraft, indemSie davon ausgehen, dass, um eine Wasser-schicht der Dicke Dx auf die Höhe x zu heben,die Teilenergie DE rgAxDx erforderlich ist(Zeichnung).

Seite

61

1.3 Energie und Energieerhaltung 41

Page 42: Metzler Physik

Lösung:a) Die Energie berechnet sich aus dem Weg, um

den sich der Schwerpunkt des Wassers im ge-füllten Becken gegenüber dem Boden erhebt,und der Gewichtskraft des gesamten Wassers:

Gw mw g r V g r A h gundE mw g 1

2 h 12 r g A h2.

b) Das Wegintegral liefert

E R

D ERh0

r g A x dx 12 r g A h2.

Das Energiediagramm kann man wie folgt in-terpretieren: Um das Becken von der Höhe xauf die Höhe x Dx aufzufüllen, muss eineWasserschicht der Dicke Dx vom GewichtDm g r g ADx um die Höhe x gehoben wer-

den. Dabei wird die Energie DE r g A x Dxbenötigt.

Das Energieintegral summiert dann sämtliche Teil-arbeiten.

Zusatzaufgabe

6 Berechnen Sie die Energie die eine konstante KraftF 12 N entlang einer horizontalen Strecke s 7 m, wenn Kraft und Weg einen Winkel von a) 0,b) 60, c) 90, d) 145, e) 180 einschlieûen.

Lösung:Aus E F s cos a folgta) für 0 E 84,0 J, d) für 145 Eÿ68,8 J,b) für 60 E 42,0 J, e) für 180 Eÿ84,0 J.c) für 90 E 0,

1.3.2 Existenzformen der Energie

1 Ein Auto (m 950 kg) wird in 4 s vonv1 50 km=h auf v2 90 km=h beschleunigt.a) Wie groû ist die dazu erforderliche Energie?b) Welche Geschwindigkeit hätte der Wagen mit

der gleichen Energie erreicht, wenn er aus demStand beschleunigt worden wäre?

Lösung:a) Es ist Ekin 1

2 m v22ÿ 1

2 m v21 2,052 105 J

unabhängig von der Zeit, in der die Beschleuni-gung erfolgt.

b) Aus Ekin 12 m v2

e 2,052 105 J folgt ve 2,079 101 m=s 74,8 km=h.

2 Ein Körper (m 25 kg) wird eine 5,0 m langeschiefe Ebene (Steigungswinkel a 30) hinauf-gezogen.a) Wie groû ist die aufzubringende Energie?b) Wie groû müsste eine horizontal wirkende Zug-

kraft sein? Von der Reibung sehe man ab.

Lösung:a) Die Energie ist Epot G l sin a 613,1 J.b) Mit FH G sin a und FZ FH=cos a ist FZ

G tan a 141,6 N.

3 Welche Energie ist erforderlich, eine Schraubenfe-der um 2 cm zu dehnen? Für die Feder gelte, dassein Körper der Masse 4 kg, an die Feder gehängt,diese um 1,5 cm verlängert.

Lösung:Mit D F s 2,616 103 N=m ist Es 1

2 Ds2 5,232 ´ 10ÿ1 J.

4 Eine Schraubenfeder wird durch eine KraftF 0,6 N um s 3,5 cm gedehnt. Wie groû istdie Spannenergie, um die Feder um weitere7,0 cm zu dehnen?

Lösung:Es ist D F=s 1,71 ´ 101 Nm undEs 1

2 D s22ÿ s2

1 8,4 ´ 10ÿ2 J.

5 Eine Schraubenfeder erhält schrittweise durchWägestücke der Masse m die Dehnung s.

m in kg 50 100 150 200 250 300

s in cm 1,6 3,15 4,8 6,3 7,85 9,35

a) Ermitteln Sie die Federkonstante D und berech-nen Sie die Spannenergie, die erforderlich ist,um die Feder auf die Endlänge zu dehnen.

b) Berechnen Sie zu jeder Teilstrecke Ds die Teil-energie DE, die beim Auflegen eines neuenWägestückes aufgebracht wird, und verglei-chen Sie die Summe der Teilenergien mit demErgebnis aus a).

Lösung:

a) m in g

s in cm

D in

N=m

50

1,6

30,7

100

3,15

31,1

150

4,8

30,7

200

6,3

31,1

250

7,85

31,2

300

9,35

31,5

b) Ds in

cm

DE in

10ÿ2 J

1,6

0,78

1,55

1,52

1,65

2,43

1,50

2,94

1,55

3,80

1,50

4,56

Seite

64

1 Mechanik42

Page 43: Metzler Physik

a) DÅ 31,1 N=m; Es 12 DÅ s2

e 0,1359 J.b) Die Teilenergien berechnen sich aus

DEs m g Ds, ihre Summe Es SDE0,1603 J ist um 18% gröûer als der Wert ausa), weil die Teilenergien jeweils mit der gröûtenKraft des Intervalls berechnet sind.

*6 Ein Körper der Masse 40 kg werde auf horizontalerFläche (Gleitreibungszahl fG 0,25;!1.2.7) übereine Strecke von 5 m mit konstanter Geschwindig-keit durch eine Kraft bewegt, die mit der Horizon-talen einen Winkel von 30 einschlieût.Wie groû ist die Kraft und welche Energie wirddurch sie übertragen?

Lösung:Die Gleitreibungskraft FG fG m g wird durch dieKraft F FG=cos a fG m g=cos a 113 N auf-gewogen. Unter ihrem Einfluû wird die EnergieE FG s 490,5 N aufgebracht.

*7 Ein Güterzug (m 500 t) erhöht auf einer Streckevon 1,8 km Länge und 8% Steigung die Geschwin-digkeit von 30 km/h auf 55 km/h (Rollreibungszahl0,005). Berechnen Sie die Reibungs-, die Hub-, dieBewegungsenergie sowie die gesamte übertrageneEnergie.

Lösung:Die aufgebrachte Reibungsenergie beträgtER FR s fR FN s fR G cos a s, alsoER 4,401 ´ 107 J.Die aufgebrachte potentielle Energie beträgtEpot G h G s sin a 7,035 ´ 108 J.Die aufgebrachte kinetische Energie istEkin 1

2 m(v22ÿ v2

1) 4,099 ´ 107 JDamit wird die aufgebrachte GesamtenergieEges EREpotEkin 7,885 ´ 108 J.Die Steigung ist, wie aus der Mathematik bekannt,definiert als Tangens des Steigungswinkels:m tan a. Hier ist also mit m 8%a arc tan 0,08 = 4,57.

8 Eine Diesellokomotive zieht mit der Kraft 60 000 Neinen Güterzug auf ebener Strecke mit konstanterGeschwindigkeit 50 km/h. Welche Energie bringtdie Maschine auf 1 km Länge auf und welche Lei-stung entwickelt sie (ohne Berücksichtigung derReibung usw.)?

Lösung:E F s 6,000 ´ 107 J, P F v 8,333 ´ 105 J.

9 Ein Mann (m 80 kg) geht mit einer Geschwindig-keit v 6 km=h auf einer Straûe mit der Steigunga 10 bergan. Wie groû ist seine Leistung?

Lösung:Die äuûere Kraft ist G m g, die mit der Wegrich-tung auf der schiefen Ebene den Winkel 90ÿaeinschlieût. Damit ist P G v0 cos (90 ÿ a) G v0 sin a 227,1 W.

10 Ein Bootsmotor besitzt die Leistung 3000 W. Ertreibt das Boot mit einer (konstanten) Geschwin-digkeit von v 9 km=h an. Wie groû ist die Kraft(Wasserwiderstandskraft), die der Bewegung ent-gegenwirkt?

Lösung:Aus P F v wird F 1200 N.

11 Ein Auto (m 1,2 t) wird vom Stand aus in 12 s auf100 km/h beschleunigt. Berechnen Sie die (kon-stante) beschleunigende Kraft, die Leistung desMotors und den Weg, auf dem die Endgeschwin-digkeit erreicht wird sowie die kinetische Energiedes Autos nach 12 s.

Lösung:Mit a 2,31 m=s2 aus v a t ist Fm a 2772 N,Ekin 1

2 m v2 4,63 106 J und se 12 a t2 166 m.

Die Leistung des Motors ist P F v; sie steigt bisauf P 7,70 ´ 104 W an.

12 Ein Lastwagen (m 3,5 t) fährt 4,8 km auf einerStraûe mit 8% Steigung mit 40 km/h bergauf.a) Berechnen Sie die erforderliche Energie.b) Wie groû ist die Leistung des Motors, wenn

man davon ausgeht, dass 1=3 für die potentielleEnergie und 2=3 (nicht 1=3) für Reibungsener-gie (Rollreibung, Luftwiderstand) zu veran-schlagen sind?

Lösung:a) Aus der Steigung von 8% erhält man über tan a 0,08 den Steigungswinkel a 4,57. Damitwird E G sin a s 1,314 ´ 107 J.

b) Der Wagen fährt 432 s, so dass die für die po-tentielle Energie aufzubringende Leistung Ph3,042 ´ 104 W 30,42 kW ist. Die gesamteLeistung beträgt dann P 3 Ph 9,127 ´ 104

91,27 kW.

13 Ein Körper der Masse m wird an eine Feder (Feder-konstante D) gehängt (Auslenkung s 0) und lang-sam bis zur Gleichgewichtsstellung (s se) herab-gelassen. Auf dem Wege nach unten wirken auf denKörper die Gewichtskraft G und die Federkraft FF.a) Fertigen Sie eine Zeichnung an.

1.3 Energie und Energieerhaltung 43

Page 44: Metzler Physik

b) Berechnen Sie die auf den Körper wirkendeKraft in Abhängigkeit von der nach unten posi-tiv gezählten Auslenkung s.

c) Berechnen Sie die Spannenergie als Weginteg-ral und zeigen Sie mithilfe der Gleichgewichts-bedingung für s se, dass sich E 1

2 Ds2e er-

gibt.

Lösung:a) Während die Feder durch das angehängte Ge-

wicht (langsam) ausgezogen wird, wirkt nachunten die stets gleiche Gewichtskraft G, nachoben die proportional zur Auslenkung wach-sende Federkraft, bis im Gleichgewichtszustandbeide (betrags-)gleich sind. Die auf den Körperwirkende (äuûere) Kraft, die die Feder auszieht,muss also stets nach unten gerichtet sein:~F ~GÿD~s:

b) Die aufzubringende Energie berechnet sich zu

ERse

0

~F d~sRse

0~GÿD~s d~s

Rse

0G dsÿ

Rse

0Ds dsGseÿ 1

2 Ds2e :

c) Mit ~GD~s im Gleichgewichtszustand istschlieûlichEDs2

e ÿ 12 Ds2

e 12 D s2

e .

Zusatzaufgaben

14 Der Boden eines quaderförmigen Pumpspeicher-sees (l 60 m, b 35 m, Wassertiefe t 6,4 m)liegt 72 m über dem Kraftwerk. ± Berechnen Siedie im Wasser enthaltene potentielle Energie.

Lösung:1. Lösungsweg:Die potentielle Energie einer Wasserschicht derDicke Dh in der Höhe h über dem Kraftwert istEpot l b g r h Dh, aus der sich durch Integrationvon h1 72,0 m bis h2 78,4 m die potentielleEnergie des gesamten Wassers im Becken ergibt

mit r 1 103 kgm3

fÈur Wasser

:

EpotRh2

h1

dEpot l b g r 12 h2

2ÿ h21 9,915 109 J.

2. Lösungsweg:Mit denselben Bezeichnungen h1 und h2 wie obenliegt der Schwerpunkt des Quaders, den das Wasserim Becken bildet,

h h1 h2

2Èuber dem Kraftwerk:

Sein Gewicht ist G l b h2ÿ h1 r g und damitgilt für seine potentielle Energie

Epot l b (h2ÿ h1) r gh1 h2

2 l b g r 1

2 (h22ÿ h2

1):

15 Eine Kraft F 30 N beschleunigt einen Körper(m 2,0 kg) aus der Ruhe auf einer Strecke von3,0 m auf ebener reibungsloser Unterlage. Dannändert die Kraft ihren Betrag auf F 15 N undwirkt weitere 2,0 m.a) Welche kinetische Energie besitzt der Körper

am Ende des Vorganges?b) Wie groû ist dann seine Geschwindigkeit?

Lösung:a) Die kinetische Energie ist gleich der längs des

Weges aufzubringenden mechanischen Ener-gie, also Ekin F1 s1 F2 s2 120 J.

b) Aus Ekin gewinnt man ve 10,95 m=s.

*16 Eine Feder (D 200 N=m) wird um s 0,10 mzusammengedrückt. Ein Körper (m 0,5 kg)wird vor ihr Ende gesetzt, und dann wird dieFeder losgelassen.a) Mit welchem Impuls verlässt der Körper die

Feder?b) Wie groû ist seine kinetische Energie?

Lösung:a) Mit 1

2 D s2 12 m v2

e 2,0 m=s und pm ve

1,0 kg m=s.b) Weiter ist Ekin 1

2 m v2e 1,0 J.

1.3.3 Energieerhaltungssatz

1 Ein Schlitten der Masse 60 kg startet aus der Ruhevon einem Hügel aus 20 m Höhe und erreicht denFuû des Hügels mit einer Geschwindigkeit von16 m/s. Welchen Betrag an Energie hat er durchReibung usw. verloren?

Lösung:Durch Reibung geht verloren DE EpotÿEkin

m g hÿ 12 m v2

0 4,092 103 J.

2 Ein Auto (m 800 kg) prallt mit der Geschwindig-keit v 60 km=h gegen eine Mauer.a) Wie groû ist seine kinetische Energie?

Seite

67

1 Mechanik44

Page 45: Metzler Physik

b) Aus welcher Höhe müsste das Auto frei fallen,um beim Auftreffen auf den Boden die gleichekinetische Energie zu besitzen?

Lösung:a) Es ist Ekin 1,11 105 J.b) Das Auto müsste aus der Höhe h 1

2 v2=g14,16 m frei fallen.

3 Ein Stein (m 0,2 kg) wird an einer 0,5 m langenSchnur mit 2 Umdrehungen pro Sekunde auf einerwaagerechten Kreisbahn herumgeschleudert.a) Welche kinetische Energie besitzt er?b) Welche Zentripetalkraft wirkt auf ihn?c) Wie groû ist die von der Zentripetalkraft bei

einer Umdrehung übertragene Energie?

Lösung:a) Ekin 1

2 m2 p r=T2 3,95 J:b) FR 15,8 N.c) E 0, da stets D~s senkrecht zu ~FR.

4 Ein Stein (m 100 g) wird von einem hohen Turmfallen gelassen. Er erreicht nach einem Fallwegvon 100 m die Geschwindigkeit v 20 m=s. Wel-che Energie ist an die Luft übertragen worden?

Lösung:Nach h 100 m Fallweg müsste der Stein die Ge-schwindigkeit ve

2 g hp 44,3 m=s statt v

20 m=s erreicht haben.DE 1

2 mv2e ÿ v2 78,1 J.

5 Eine Pendelkugel (d 2,4 cm, m 300 g) wird aneinem Band (l 1,20 m) auf die Höhe h 20 cmgehoben. Im untersten Punkt braucht sie zumDurchlaufen der Lichtschranke (Ds d) die ZeitDt 0,012 s. Berechnen Sie die kinetische Energieim untersten Punkt der Bahn auf zwei Arten.

Lösung:Die Angaben ermöglichen die Beantwortung aufzwei Wegen:Es ist Ekin 1

2 m Ds=Dt 6,0 10ÿ1 J,und nach dem EnergiesatzEkin Epot m g h 5,89 10ÿ1 J,also Ekin gemessen an Epot mit ÿ1,9% Fehler.

6 Rechnen Sie die Ergebnisse des senkrechten undschiefen Wurfs mithilfe des Energieerhaltungssat-zes nach.

Lösung:Nach dem Energiesatz lässt sich in beiden Fällendie Steighöhe berechnenSenkrechter Wurf: Ekin 1

2 m v20 Epot m g h er-

hält man die Wurfhöhe

h 12 v2

0=g.Schräger Wurf: Aus dem Ansatz12 m v0 sin a2 m g h erhält man die Steighöheh 1

2 v20 sin2a=g.

7 Ein Wagen (m 0,32 kg) rollt eine schiefe Ebene(Steigungswinkel a 30) herab.a) Welche Geschwindigkeit v1 und v2 hat er nach

Durchlaufen der Strecken s1 15 cm bzw.s2 65 cm erreicht?

b) Wie groû ist der Zuwachs an kinetischer Ener-gie von s1 nach s2?

Lösung:Die beschleunigende Kraft ist F G sin a und dieBeschleunigung ist a g sin a.a) Mit v2 2 a s ergibt sich nach 15 cm

v1 1;21 m=s und nach 65 cm v2 2,53 m=s.b) Der Zuwachs an kinetischer Energie ist

DE12 12 m v2

2ÿ v21m a s2ÿ s1 0,790 J.

8 Ein Fahrbahnwagen (m 0,5 kg) wird längs einerhorizontalen Strecke s 1,00 m durch Wäge-stücke (mw), die auf den Auflageteller (mT 5 g)gelegt werden, beschleunigt. Am Ende der Streckemisst ein Kurzzeitmesser die Zeit Dt, während derDs 1,45 cm breite Bügel des Wagens die Licht-schranke durchläuft:

mw in g 20 30 40 50

Dt in ms 15,12 12,54 11,38 10,09

Berechnen Sie für jeden Versucha) die Lageenergie von Wägestück und Auflage-

teller;b) die kinetische Energie am Ende der Strecke.c) Vergleichen Sie die Ergebnisse.

Lösung:a) Die potentielle Energie ist Epotmwmt g s.b) Die kinetische Energie ist

Ekin 12 mmwmtDs=Dt2.

c) Siehe letzte Zeile der folgenden Tabelle:

mw in g 20 30 40 50

Dt in 10ÿ3 s 15,12 12,54 11,38 10,09

EPot in 10ÿ1 J 2,45 3,43 4,41 5,40

Ekin in 10ÿ1 J 2,41 3,58 4,42 5,73

EpotÿEkin

Epotin % ± 1,6 ± 4,2 ± 0,2 ± 6,2

*9 Ein Körper der Masse 0,20 kg wird mit der An-fangsgeschwindigkeit von 50 m/s senkrecht nachoben katapultiert. Berechnen Sie die kinetische,die potentielle und die gesamte Energie

1.3 Energie und Energieerhaltung 45

Page 46: Metzler Physik

a) zu Beginn der Bewegung,b) nach 3 Sekunden,c) in 100 m Höhe,d) in einer Höhe, in der die kinetische Energie auf

80% ihres Ausgangswertes abgenommen hat.

Lösung:a) Für t0 0 ist Epot m g h 0,

Ekin 12 m v2 250 J.

b) Nach der Zeit t1 3 s ist mit v v0ÿ g t1 20,57 m=s und h1 v0 t1ÿ 1

2 g t21 105,9 m

dann Epot 207,8 J und Ekin 42,31 J.c) In h2 100 m Höhe ist mit v2

0ÿ v22 2 g h2

oder v2v2

0ÿ 2 g h2

p23,19 m=s

ÿ23,19 m=s Epot 196,2 J; Ekin 53,8 J.d) Für 80% der kinetischen Anfangsenergie

Ekin 200 J ist mit

v32 Ekin=m

p 44,72 m=s ÿ44,72 m=s

und h3 12 v2

0ÿv23=g 25,49 m

dann Epot 50,01 J, Ekin 200 J.Die Werte in Klammern in c) und d) gelten für denzweiten Teil des Bewegungsvorganges, wenn derKörper wieder fällt.

*10 Lösen Sie die Aufgabe 9 für den Fall, dass derAbschuss des Körpers unter einem Winkel von80 zur Horizontalen erfolgt.

Lösung:a) Wie in Aufgabe 9b) Nach der Zeit t1 3 s ist mit

v21 v0 cos a2 v0 sinaÿ g t12

v121,63 m=s ÿ21,63 m=s) undh1 v0 sina t1ÿ 1

2 g t21 oder h1 103,6 m und

damit Epot 203,3 J, Ekin 46,79 J.c) Für h2 100 m Höhe gilt mit

h2 v0 sin a t2ÿ 12 g t2

2 oder

t2 v0 sin a=gv0 sin a2ÿ 2 g h2

q=g

oder t2 2,827 s 7,212 s und

v2 v0 cos a2v0 sin aÿ g t22

q

oder v2 23,19 m=s ÿ23,19 m=s dannEpot 192,2 J, Ekin 53,78 J.

d) Bei 80% der anfänglichen kinetischen Ener-gie ist mitv3 44,72 m=s ÿ44,72m=s und ausv2

3 v0 cos a2ÿv0 sin aÿ g t32t3 v0 sin a=g

v2

3ÿ v20 cos2 a

p=g oder

t3 0,548 s 9,053 s dann Epot 50,05 Jneben Ekin 200 J.

Die Werte in Klammern gelten für den Bewe-gungsteil des Fallens.

*11 Eine lotrecht stehende Schraubenfeder wirddurch eine darauf gelegte Kugel (m 50 g) umDs 2 mm zusammengedrückt. Wie hoch (vomoberen Rand der entspannten Feder aus gemes-sen) fliegt die Kugel, wenn die Feder um weitere15 cm zusammengedrückt und dann plötzlich ent-spannt wird?

Lösung:Mit DG=Ds 2,45 102 N=m und Es 1

2 Ds2

Ekin 12 m v2

0 2,83 J sowie h 12 v2

0=g folgth 1

2 sDs2=Ds 5,78 m oder, gemessen ander Ausgangshöhe, heff hÿ 15,2 cm 5,63 m.

*12 Berechnen Sie mithilfe des Energiesatzes: Vonwelcher Höhe h (über dem tiefsten Punkt derSchleifenbahn) muss eine Kugel der Masse m ent-lang einer Schiene auf einer schiefen Ebene her-ablaufen, um danach eine kreisförmige Schlei-fenbahn (d 2 r 0,2 m) zu durchlaufen, ohneim oberen Punkt der Schleifenbahn herabzufal-len? (Von der Rotationsenergie sehe man ab.)

Lösung:Auf den obersten Punkt P1 der Bahn muss dieZentripedalkraft mindestens die Gewichtskraftder Kugel kompensieren. Daraus gewinnt mandie kinetische Energie in diesem Punkt der Bahn,die zusammen mit der potentiellen Energie andieser Stelle gleich der potentiellen Energie desStartpunktes P0 mit der Höhe h über dem tiefstenPunkt der Schleifenbahn sein muss.P1: Aus m v2=r mg wird Ekin1

2 mv2 12 m g r

und EkinEpot 12 m g rm g 2 r 5

2 m g r.P0: Hier ist Epot m g h.Vergleich beider Energien ergibt h 5

2 r.

*13 Ein Pendel der Länge l 95 cm wird um 15 cmangehoben und dann losgelassen. Im tiefstenPunkt der Bahn wird der Pendelkörper(m 150 g) durch eine Rasierklinge vom Fadengetrennt und fällt auf den 1,6 m tiefer gelegenenBoden.a) Wo trifft der Körper auf?b) Welche Geschwindigkeit (insgesamt und ho-

rizontal) besitzt er beim Auftreffen auf demBoden?

Lösung:a) Nach dem Energiesatz hat der Pendelkörper

im tiefsten Punkt seiner Bahn die Geschwin-digkeit v0

2 g Dhp 1,72 m=s.

Er fällt dann mit sy 12 g t2

e nach te 0,571 s,in dem er in x-Richtung der Streckesx v0 te 0,98 m zurücklegt.

1 Mechanik46

Page 47: Metzler Physik

b) Nach dem Energiesatz gilt für die Gesamtge-schwindigkeit v2 2 gDh sy oder

vv2

0g te2q

; also v 5,86 m=s.In waagerechter Richtung bleibt die Ge-schwindigkeit v v0 1,72 m=s.

14 Hakt man eine Kugel an eine entspannte Schrau-benfeder, die an einem Stativ hängt, und lässt dieKugel los, so zieht sie die Feder um das Doppelteder Strecke nach unten, die sich ergibt, wenn mandie Kugel langsam mit der Hand nach unten führt,bis sie ihre Gleichgewichtslage, an der Feder hän-gend, erreicht hat. Begründen Sie Ihre Lösung miteiner Energiebetrachtung.

Lösung:Hängt man die Kugel an die entspannte Feder undlässt sie dann los, so fällt sie um die Strecke h, hatalso die potentielle Energie Epot m g h inSpannenergie Es 1

2 Dh2 der Feder umgesetzt.In der Gleichgewichtslage ± die Feder ist um dieStrecke s gestreckt ± ist die Federkraft gleich derGewichtskraft: m gD s.Aus dem Vergleich folgt h 2 s: Im ersten Fallwird die Feder doppelt soweit gedehnt (Ver-such!).

1.3.4 Stoûvorgänge und Erhaltungssätze

1 Ein Wagen (Masse m1 4 kg) prallt mit einer Ge-schwindigkeit v1 1,2 m=s auf einen zweiten(m2 5 kg), der sich in gleicher Richtung mit derGeschwindigkeit v2 0,6 m=s bewegt.a) Wie ändern sich die kinetischen Energien beim

zentralen elastischen Stoû?b) Wie lauten die Lösungen, wenn die Wagen auf-

einander zulaufen?

Lösung:a) Mit v01 0,533 m=s, v02 1,133 m=s

wird Ekin1 2,88 J, Ekin2 0,9 J.E0kin1 0,596 J, E0kin2 3,211 J,DE1 ÿ2,311 J, DE2 2,311 J.

b) Es gilt v1 1,2 m=s und v2 ÿ0,6 m=s.v01 ÿ0,8 m=s, v02 1,0 m=s.Ekin1 2,88 J, Ekin2 0,9 J.E0kin1 1,28 J, E0kin2 2,5 J,DE1 ÿ1,6 J, DE2 1;6 J.

*2 Eine Stahlkugel der Masse m stöût zentral mit derGeschwindigkeit v gegen mehrere gleiche, die hin-tereinander liegen und sich berühren.Weshalb fliegt nur eine Kugel fort?

Lösung:Aus dem Ansatz, daû beim Auftreffen einer Kugelmit dem Impuls pm v diese Kugel nach demStoû in Ruhe bleibt v01 0 und dabei n Kugelnmit der Geschwindigkeit v02 fortfliegen, folgt nachden Gesetzen des elastischen Stoûes für v010m v n mÿv, also n 1; das bedeutet:nur eine Kugel rollt fort, und für v02v0 m2 v=2 m v,sie rollt mit der Geschwindigkeit v der ersten Kugelfort.

Physikalisch kann man sich den Versuch wie folgterklären: Jede einzelne Kugel gibt beim Auftreffenauf die Kugelreihe einzeln durch elastische Verfor-mung ihre Energie und ihren Impuls an die nächsteweiter, bis jeweils die letzte mit der gleichen Ge-schwindigkeit wie die angekommene sich weiter-bewegt.

*3 Ein leerer Güterwagen A mit der MassemA 2,5 104 kg rollt auf einer horizontalenStrecke mit vA 2,0 m=s gegen einen stehendenWagen B mit der Masse mB 5,0 104 kg. BeideWagen sind sogleich gekoppelt.Welches ist die gesamte Energie vor und nach demStoû? Die Reibung bleibe unberücksichtigt.

Lösung:v0B 0,67 m=s.EA 5 ´ 104 J, E0 1,70 ´ 104 J.Es liegt ein unelastischer Stoû vor.

*4 Eine Kugel mit der Masse m1 3 kg stöût mit derGeschwindigkeit v1 6 m=s gegen eine zweite mitder Masse m2 2 kg, die ihr mit der Geschwindig-keit v2 ÿ8 m=s genau entgegenkommt. Nachdem Stoû hat die erste die Geschwindigkeitv01 ÿ2,4 m=s, die zweite die Geschwindigkeitv02 4,6 m=s, beide in umgekehrter Richtung alsursprünglich.a) Berechnen Sie Impuls- und Energiesumme vor

und nach dem Stoû.b) Was für ein Stoû liegt vor? Geben Sie den mög-

licherweise vorhandenen Unterschied betrags-mäûig an.

Seite

68

1.3 Energie und Energieerhaltung 47

Page 48: Metzler Physik

Lösung:a) Der Impulssatz gilt: p p0 2,0 kg m=s.

Der Energiesatz gilt nicht:E 118 J, E0 29,8 J, also DE 88,2 J.

b) Bei vollkommen elastischem Stoû würde geltenv01 ÿ5,2 m=s, v02 8,8 m=s,bei einem rein unelastischen Stoûv0 0,4 m=s.Nach den Daten der Aufgabe besitzt dann dererste Körper eine um DE0 56,3 J kleinereEnergie.

Beim unelastischen Stoûe verlören die Körperdie Energie DE 117,6 J.Die Versuchsdaten beschreiben einen teilweiseelastischen Stoû, und damit den Normalfall wiees z. B. auch beim Zusammenstoû von Autosder Fall ist. Nur ein Teil der Energie geht inkinetische Energie nach dem Stoû über.

1 Mechanik48

Page 49: Metzler Physik

1.4.1 Die gleichmäûig beschleunigte Drehbewegung; Drehmoment

(Alle Aufgaben beziehen sich auf den Exkurs Seite 71)

*1 Berechnen Sie für ein Auto mita) Vorderradantrieb,b) Allradantrieb die maximale Beschleunigung

(ohne Rollreibung und Luftwiderstand).

Lösung:a) Für A als Drehpunkt lautet die Gleichgewichts-

bedingung

G l1ÿF2 lÿFH h 0.

Mit fH multipliziert, wobei fH F2 FH ist, erhältman

fH l1 Gÿ l FHÿ fH h FH 0

oder umgeformt

FH l1 fH Gl fH h

oder mit FH m a und Gm g die maximaleBeschleunigung

a l1 fH gl fH h

:

b) Beim Vierradantrieb wird das Auto durch diebeiden Haftreibungskräfte der Vorder- undHinterräder angetrieben:

FH fH F1 fH F2 fH G.

Damit wird die maximale Beschleunigung

a fH g.

Die Beschleunigung beim Hinterrad- bzw. Vor-derradantrieb wird damit:

aH fH gl2

lÿ fH hbzw. aV fH g

l1

l fH h.

Für l1 0,5 (l fH h) und l2 0,5 (lÿ fH h)(nachrechnen!) wäre die Beschleunigung fürbeide Fälle gleich.Im Fall b) kommt man ohne den Ansatz überdie Gleichgewichtsbedingung aus, sie führte imübrigen auch nicht weiter. Das Ziel ist ja auch inerster Linie, einen Ausdruck für FH zu gewin-nen, aus dem sich die Beschleunigung a berech-nen läût.

*2 Ein Pkw (m 800 kg) mit dem Radabstand 2,40 mhat seinen Schwerpunkt 0,55 m über dem Bodenund 1,00 m vor der Hinterachse. Die Haftreibungs-

zahl sei 0,6. Berechnen Sie die mögliche Beschleu-nigung beia) Hinterradantrieb, b) Allradantrieb.

Lösung:Nach Aufgabe 1 ist mit l 2,40 m, l1 1,00 m, alsol2 1,40 m, und weiter h 0,55 m sowie fH 0,6für dena) Hinterradantrieb amax 3,98 m=s2,b) Allradantrieb amax 4,89 m=s2.Die Masse m geht in die Beschleunigung nicht ein.

*3 Stellen Sie die Gleichgewichtsbedingung für einAuto auf und wählen Sie dabei als Drehpunkt denSchwerpunkt des Autos, und zwar für eine Fahrtsowohl auf ebener Straûe als auch bergan (Stei-gungswinkel a).

Lösung:Wählt man den Schwerpunkt S als Drehpunkt, sogilt l1 F1ÿ l2 F2ÿ h FH 0.Mit F2 FGÿF, und l1 lÿ l2 wird darausl F1ÿ l2 FGÿ h FH 0.Das aber ist die oben abgeleitete Gleichgewichts-bedingung.

*4 Zeigen Sie: Die maximale Steigung für ein Automit Allradbetrieb berechnet sich zutana fHÿ fR. (VomLuftwiderstandsehemanab.)Zum Vergleich: Für fH 0,8 und fR 0,015 wirdbei Allradantrieb tan a 0,78, d.h. 78% bzw.38 Steigung, bei Hinterradantrieb tan a 0,47,d. h. 47% bzw. 25 Steigung und bei Vorderradan-trieb tana 0,33, d. h. 33% bzw. 18 Steigung.(Anmerkung: Die Angaben für Hinter- undVorderradbetrieb können hier nicht begründet wer-den,dadieBerechnungauchbeieinfachstenAnnah-men zu kompliziert ist.)

Lösung:Die maximal wirksame Antriebskraft kann höchs-tens so groû sein wie die Haftreibungskraft FH.Bei Allradbetrieb ist die Haftreibungskraft insge-samt gleich der Summe der Haftreibungskräfte aufVorder- und Hinterrad FHv und FHh

FH FHvFHh fH FN fH G cos a.

Bei konstanter Geschwindigkeit, wenn man beiBerganfahrt die Luftwiderstandskraft vernachläs-sigt, muss die Antriebskraft FA die Rollreibungs-kraft FR und die Hangabtriebskraft (Steigungswi-derstandskraft) FS aufwiegen:

Seite

71

1.4 Die Rotation starrer Körper 49

Page 50: Metzler Physik

FA FH FSFR

mit FS G sin a undFR FRvFRh fR FN fR G cos a:Damit wirdFH FSFR

oderfH G cos aG sin a fR G cos a oderfHÿ fR tan a.

Der maximale Steigerungswinkel ist durch die Dif-ferenz von Haftreibungszahl und Rollreibungszahlgegeben.

F = G sin

F = G cos

GN

S

α

α

α

α

Zusatzaufgaben

5 Ein Radfahrer übt auf die Pedale (r 18 cm) eineKraft F 480 N aus. Berechnen Sie das Drehmo-ment für den Fall, dass der eingeschlossene Winkel0, 45, 90 und 180 beträgt.

Lösung:M r F sin a 84,4 Nm ´ sin a;M1 0, M2 61,1 Nm, M3 86,4 Nm, M4 0.

6 Was kann ein Radfahrer tun, um das Drehmomentauf die Pedale zu vergröûern?

Lösung:Man kann so treten, dass die Kraft senkrecht zurPedale wirkt. Man kann aufstehen, um das volleGewicht wirken zu lassen. Man kann den Körpernach unten ziehen, um eine zusätzliche Kraft zuerhalten.

*7 (Die Aufgabe setzt die Kenntnis des Exkurses Seite71 voraus.)a) Berechnen Sie das Drehmoment des Motors ei-

nes Autos mit der Leistung 90 kW bei 5100Umdrehungen pro Minute.

b) Berechnen Sie die Antriebskraft, die auf dieHinterräder (R 0,31 m) wirkt, wenn dieGangübersetzung i 1 (im 4. Gang) ist undder Wirkungsgrad des Getriebes usw. mith 0,8 angesetzt wird.

c) Wie groû ist die Haftreibungskraft an den Hin-terrädern und das Beschleunigungsvermögen,wenn der Radstand l 2,6 m, der Abstand Hin-terachse ± Schwerpunkt l1 1,1 m, die Höhedes Schwerpunktes h 0,58 m und die Massedes besetzten Wagens m 1500 kg betragen(Haftreibungszahl fH 0,7)?

Lösung:a) Nach PM w mit w 2 p=T und

T 60 s= 5100 ist M P=w 168,5 Nm.b) Antriebskraft FA, Wirkungsgrad des Getriebes

h:

FA h iMR 434,8 N.

c) Nach Aufgabe 1 Seite 71 ist mit fH 0,7,l 2,6 m, l1 1,1 m, also l2 1,5 m, undh 0,58 m bei Hinterradantrieb die maximaleBeschleunigung amax 4,69 m=s2.Danach beträgt die mögliche HaftreibungskraftFH 7035 N. Die Antriebskraft des Motorsdient voll der Beschleunigung.

1.4.2 Trägheitsmoment und Rotationsenergie

1 Eine Eiskunstläuferin dreht eine Pirouette. Ist ihrTrägheitsmoment gröûer oder kleiner als 20 kg m2?

Lösung:Eine Abschätzung ergibt, daû das Trägheitsmo-ment erheblich kleiner als 20 kg m2 ist: Befändesich die Gesamtmasse (maximal 80 kg) im maxi-malen Abstand von der Drehachse (höchstens30 cm), so ergäbe sich J m r2 7,2 kg m2.

*2 An den vier Ecken eines Rechtecks mit den Seiten-längen von 30 cm und 40 cm befinden sich Kugelnmit der Masse von je 200 g. Berechnen Sie das Ge-samtträgheitsmoment um eine Achse durch denMittelpunkt des Rechtecks senkrecht zur Recht-eckebene. (Der Durchmesser einer Kugel kann imVergleich zu den Seitenlängen des Rechtecks alsklein angesehen werden.)

Seite

73

1 Mechanik50

Page 51: Metzler Physik

Lösung:r 1

2

0,32 0,42p

m 0,5 m, m 0,8 kg;J m r2 0,2 kg=m2.

*3 Eine Vollkugel (r 4,4 cm, m 0,62 kg) rollt mitder Geschwindigkeit v 36 cm=s auf dem Tisch.Berechnen Sie ihre Translations- und ihre Rota-tionsenergie.

Lösung:Ekin 1

2 mv2 4 10ÿ2 J; JKu 25 mr2,w v=r.

Erot 12 J w2 1

5 m v2 1,6 ´ 10ÿ2 J;Verhältnis Ekin : Erot 5 : 2.

*4 Auf welche Höhe hat man die Kugel in einerSchleifenbahn (!Aufgabe 12 in 1.3.3) zu bringen,wenn man die Rotationsenergie berücksichtigt?

Lösung:Die potentielle Energie des Startpunktes muss diekinetische und die Rotationsenergie im höchstenPunkt der Schleifenbahn liefern:

m g h 12 m v2 1

5 m v2m g 2 r.

Dabei ist (Aufgabe 12, Seite 67)m v2=r ] m g, also m g h ] 7

10 r m g 2 r m g unddaraus h ] 2,7 r.

*5 Von einer schiefen Ebene (Länge l 1,2 m, Nei-gungswinkel 30) rollen eine Kugel, ein Hohlzylin-der und ein Vollzylinder von der gleichen Masse(m 300 g, r 2,1 cm) herab (von Reibungskräf-ten sehe man ab).a) Mit welcher Bahn- und welcher Winkelge-

schwindigkeit und nach welcher Zeit treffendie Körper am Ende der schiefen Ebene ein?

b) Wie groû ist dort ihre kinetische Energie derTranslations- und der Rotationsbewegung?

c) Welche Bahn- und welche Winkelbeschleuni-gung erfahren sie?

Lösung:

Kugel Hohl-zylinder

Voll-zylinder

a) Epot :v in m=sw in 102 sÿ1

te 2 sv

in s

0,7 m v2

2,901,38

0,828

m v2

2,431,16

0,988

0,75 m v2

2,801,33

0,857

b) Ekin in JErot in J

1,2620,505

0,8860,886

1,1760,588

c) a in m=s2

a in 102 s±2

3,501,66

2,461,17

3,271,55

Zu a): Aus s 12 a t2 und v a t ergibt sich

t 2 s=v.

Zusatzaufgaben

*6 An einem Reifenapparat (Reifen m 0,823 kg,r 0,31 m, Trommeldurchmesser d 2,0 cm)werden auf das Gewichtstellerchen (m0 0,54 g)nacheinander eine bzw. zwei Scheiben von 10 ggelegt (Ausgleichsmasse für die ReibungmR 0,65 g).Man misst jeweils für 2 Umdrehungen die Zeit tund danach die Geschwindigkeit mit einem Papier-streifen der Breite Ds 2,0 cm, der die Licht-schranke für die Zeit Dt unterbricht, nachdem derReifen unbeschleunigt weiterläuft.Eine Scheibe: t 45,1 s, Dt 1,17 ´ 10ÿ1 sZwei Scheiben: t 31,8 s, Dt 8,19 ´ 10ÿ2 s.a) Bestimmen Sie jeweils die Winkelbeschleuni-

gung a und die (End-)Winkelgeschwindigkeitw, und überprüfen Sie den Zusammenhang.

b) Berechnen sie jeweils das Drehmoment M undmit a) das Trägheitsmoment J.

Lösung:a) Die Winkelbeschleunigung a 2 ò=t2 und die

Winkelgeschwindigkeit am Ende des Be-schleunigungsvorganges w a t kann manvergleichen mit der mittels des Papierstreifens,Breite Ds, Zeit der Unterbrechung Dt, gemes-senen Winkelgeschwindigkeit w v=r (Ds=Dt)=r.

mA in g t in s ò a in10ÿ2sÿ2

w in10ÿ1sÿ1

1020

45,131,8

4 p

4 p

1,242,49

5,597,92

mA in g Dt in10ÿ2s

v in10ÿ1m=s

w in10ÿ1sÿ1

1020

11,78,19

1,712,44

5,517,87

Beide Messungen kommen zu übereinstimmendenErgebnissen mit 1,4% bzw. 0,6% Fehler.

b) mA in g mA in g FA in 10ÿ2 N

1020

9,919,9

9,719,5

mA in g M in 10ÿ2 Nm J in 10ÿ2 Nms2

1020

0,971,95

7,827,83

1.4 Die Rotation starrer Körper 51

Page 52: Metzler Physik

Dabei berechnet sichmA mScheibenmTellerchenÿmReibung.J erhält man mit den Ergebnissen aus a) für a mitJ M=a.

*7 Berechnen Sie das Trägheitsmoment des Reifens inAufgabe 6 und überprüfen Sie den Zusammenhangzwischen M, J und a.

Lösung:Das Trägheitsmoment lässt sich aus m 0,823 kgund r 0,31 m über J m r2 berechnen zuJ 7,91 N m=s2 mit einem am Ergebnis aus Auf-gabe 6 b) gemessenen Fehler von 1,1%.

*8 Berechnen Sie mit einem Energieansatz für das Ge-rät in ! Abb. 72.2 (m 1,15 kg, r 31 cm) dieWinkelgeschwindigkeit, die ein Gewicht (F 0,3 N) mit einem Kraftarm von r0 1,0 cm beimHerabsinken um h 34 cm bewirkt.

Lösung:

Epot Erot; F h 12 J w2; J m r2;

w2 F h=(m r2)

p 1,36 sÿ1.

1.4.3 Der Drehimpuls und seine Erhaltung

1 Ein Vollzylinder (m 350 g, r 2,7 cm) rollt miteiner konstanten Geschwindigkeit v 0,9 m=s aufder Tischfläche. Berechnen Sie seinen Drehimpuls.

Lösung:L J w, J 1

2 m r2, w v=r;L 1

2 m r v 4,25 ´ 10ÿ3 kg m2=s.

2 Wie groû ist der Drehimpuls eines Körpers derMasse 1 kg, der sich am Erdäquator auf Meeres-höhe befindet?

Lösung:L J w, J m r2,w 2 p=a 2 p=(24 60 60 s) 7,27 ´ 10ÿ5 sÿ1;L m r2 w 2,95 ´ 109 kg m2=s.

3 Vergleichen Sie den Drehimpuls der Erde aufgrundihrer täglichen Umdrehung mit dem Drehimpulsdes Mondes aufgrund seines Umlaufs um die Erde.Die Erde betrachte man dabei als homogene Kugel.

Lösung:L J w;JE 2

5 mE r2E,

wE 2 p=d 2p (24 3600) sÿ1 7,27 ´ 10ÿ5 sÿ1;JM mM r2, wM 2p=(27,3 d) 2,66 ´ 10ÿ6 sÿ1;LE 7,05 ´ 1033 kg m2=s,LM 2,88 ´ 1034 kg m2=s,LM=LE 4.

Seite

74

1 Mechanik52

Page 53: Metzler Physik

2.1.1 Das Sonnensystem

1 Berechnen Sie aus den Angaben für die groûenPlaneten (Anhang) die Konstanten nach dem3. Kepler'schen Gesetz und bestimmen Sie ihrenMittelwert.

Lösung:

T2=r3 in 10ÿ34 a2=m3 T2=r3 in 10ÿ19 s2=m3

MerkurVenusErdeMarsJupiterSaturnUranusNeptunPluto

2,973,053,022,983,012,982,982,992,98

2,963,043,012,972,992,962,962,982,97

Im Mittel: T2=r3 2,996 ´ 10ÿ34 a2=m3 oderT2=r3 2,983 ´ 10ÿ19 s2=m3

mit den Streuungen s 2,45 ´ 10ÿ36 a2=m3 oders 2,49 ´ 10ÿ21 s2=m3.

*2 Setzt man die mittlere Entfernung der Erde von derSonne gleich 1, so folgen die relativen Entfernun-

gen der groûen Planeten, Neptun ausgenommen,dem Titius-Bode'schen Gesetz: dn 0,4 0,3 2n

mit n ÿ1, 0, 1, 2, 4, 5, 6, 7. Berechnen und ver-gleichen Sie mit den Werten der Tabelle imAnhang.

Lösung:Vergleich der Planetenentfernungen nach Titius-Bode und nach Tabelle S. 576.

nnach

Titius-Bode

relativ absout in 109 m

nachTitius-Bode

nachTabelle

nachTitius-Bode

nachTabelle

MerkurVenusErdeMars

JupiterSaturnUranusNeptunPluto

ÿ1012

(3)456

7

0,40,71,01,6

5,210,019,6

38,8

0,3870,7231,0001,524

5,2059,576

19,281

39,880

59,8104,7149,6239,4

77814962932

5809

57,9108,2149,6227,9

77914322884

5966

2.1.2 Die Erforschung von Gestalt und Gröûe der Erde und der Planetenbewegung

1 Bestimmen Sie den Erdumfang nach Eratosthe-nes.

Lösung:Das Verhältnis des gemessenen Mittelpunktwin-kels von a 7,2 zum Umfangswinkel von 360ist gleich dem Verhältnis der Strecke Syene±Alexandria b 5000 ´ 148,5 m zum Erdumfangu 2 p R.Daraus ergibt sich für den Erdumfang ub ´ 360=7,20 37 125 km und für den Radiusder Erde R 5909 km, gegenüber den geltendenWerten ein Fehler von 7,3%.

2 Aristarch beobachtete bei einer Mondfinsternis,bei der der Mond durch die Mitte des Erdschattensging, dass die Zeit zwischen dem Eintritt desMondrandes in den Erdschatten und dem Beginnder totalen Mondfinsternis gleich der Dauer der to-talen Verfinsterung war.a) Erläutern Sie an der nachfolgenden Skizze die

Angaben Aristarchs und bestimmen Sie denDurchmesser von Mond und Sonne in Erdbahn-durchmessern.

b) Die scheinbaren Durchmesser von Mond undSonne nahm Aristarch als gleich groû zu0,5 an. Berechnen Sie die EntfernungenErde±Mond und Erde±Sonne in Erdbahndurch-messern und geben Sie die Distanzen mithilfedes Wertes für den Erdradius nach Eratosthe-nes in Kilometern an.

x e

d2d d 9d

Mond Erde Sonne

= 19e

E

MSeM

S

MM

=

Lösung:a) Aus 2 dM=x ds=(20 eM x) und ds 19 dM

gewinnt man x 4017 eM, und damit folgt aus ei-

nem weiteren Verhältnis der Figur 2 dM=xdE=(eM x) einerseits dM 20

57 dE 0,35 dE

und weiter aus einem dritten Verhältnis, näm-lich dS=(20 eMx) dE=(eM x), andererseitsdS 380

57 dE 6,67 dE.

2.1 Das Gravitationsgesetz 53

Seite

80

Seite

83

Page 54: Metzler Physik

b) Mit dE 11 800 km nach Eratosthenes erhältman nach a) dS 380

57 dE 78 800 km unddM 20

57 dE 4140 km. Für den Sehwinkelvon 0,5 oder 8,73 ´ 10ÿ3 im Bogenmaû gilt:Der Sehwinkel, unter dem ein Gestirn er-scheint, ist gleich dem Verhältnis von Durch-messer des Gestirns zur Entfernung zum Ge-stirn. Daraus ergibt sich für die Entfernungzur Sonne eS 78 700 km : 8,73 ´ 10ÿ3 9 026 000 km, und für die Entfernung zumMond eM 4140 km : 8,73 ´ 10ÿ3 47 400 km.

*3 a) Zeigen sie, dass die (mittlere) EntfernungErde±Sonne, die so genannte astronomischeEinheit (AE) einer Horizontalparallaxe derSonne von 8,7964 Bogensekunden entspricht(Abb. 82.1).

b) Die Parallaxe (! Abb. 81.3) des erdnächstenSterns Proxima Centauri beträgt 0,762 Bogen-sekunden. Welcher Entfernung in Metern bzw.Lichtjahren entspricht das? Bis zu welchen Ent-fernungen kann man den Weltraum mit derkleinstmöglichen (messbaren) Sonnenparal-laxe von 0,016 Bogensekunden ausmessen?

Lösung:a) Nach Abb. 82.1 ist p R=e 6,371 ´ 106 m :

1,496 ´ 1011 m 8,78400.b) Nach Abb. 81.3 ist p 1 AE=e oder

e 1 AE=p, wobei e die gesuchte Entfernungzu a-Centauri und p die zugehörige Sonnen-parallaxe im Bogenmaû ist mit 0,75400 é3,655 ´ 10ÿ6.Damit ist die Entfernung zu a-Centauri e 1,496 ´ 1011 km : 3,655 ´ 10ÿ6 oder e 4,326 Lj.Entsprechend ergibt sich als Entfernung, bis zuder man die Sonnenparallaxe noch ausmessenkann, e 2,04 ´ 102 Lj.

*4 a) Bestimmen Sie aus dem scheinbarenSonnendurchmesser die relative EntfernungErde±Sonne vom sonnennächsten (rmin, Peri-hel) zum sonnenfernsten (rmax, Aphel) Punktder Erdbahn (peri, apo, griech. bei, weg).

1. Januar 16,30 1. Juli 15,80

1. Februar 16,30 1. August 15,80

1. März 16,20 1. September 15,90

1. April 16,00 1. Oktober 16,00

1. Mai 15,90 1. November 16,10

1. Juni 15,80 1. Dezember 16,20

b) Die (numerische) Exzentrizität e der Ellipsen-bahn der Erde um die Sonne errechnet man ause rmaxÿ rmin=rmax rmin.Vergleichen Sie den Wert mit dem genauenWert e 0,01674. (Zur Bezeichnung von e! Abb. 97.2)

Lösung:a) Sei R der Sonnenradius, rA die Entfernung der

Sonne im erdfernsten Punkt (Aphel) und pA derscheinbare Sonnendurchmesser im Aphel undentsprechend rP sowie pP die entsprechendenGröûen im erdnächsten Punkt der Sonne (Peri-hel), dann giltrA 2 R=pA und rP 2 R=pP unddaraus die relative Entfernung der Sonne imPerihel zu der im AphelrP=rA pA=pP 15,80=16,30 0,969.

b) Mit rA rmax und rP rmin errechnet man

e (rmaxÿrmin)=(rmax rmin)(rAÿrP)=(rA rP)

1ÿ rP

ra1 rP

rA

0,01558, also (eÿ e0)=e0 100%

6,9% Fehler.

Zusatzaufgaben

5 Wie groû ist nach den Messungen des Aristarch dasVolumen der Erde im Vergleich zu dem der Sonnebzw. dem des Mondes?

Lösung:Aus dem gemessenen Winkel von 87 des Dreiecksnach Abb. 81.2 schlieût man mit Aristarch auf dasVerhältnis der Entfernung von der Erde zur Sonneund zum Mond eS=eM 1=cos 87 19,1. Wegender Gleichheit der scheinbaren Durchmesser vonSonne und Mond gilt dieses Verhältnis auch fürdie Durchmesser und Radien dieser beiden Him-melskörper: dS=dM rS=rM eS=eM.Schlieûlich schätzte Aristarch das Verhältnis vonMondradius zu Erdradius mit rM=rE 0,36 ab.Damit lässt sich auch das Verhältnis des Sonnen-radius zum Erdradius bestimmen:rS rE 0,36 rS=rM 19,1 ´ 0,36 6,88.Daraus bestimmen sich die VolumenverhältnisseVS=VE (rS=rE)3 326,VM=VE (rM=rE)3 0,0467 1

21.

2 Gravitation54

Page 55: Metzler Physik

2.1.3 Newtons Gravitationsgesetz

1 Wie groû ist die Gravitationskraft zwischena) zwei Schiffen von je 100 000 t, die sich mit dem

Schwerpunktabstand d 200 m begegnen;b) zwei Autos von je 900 kg, die im (Schwer-

punkt-)Abstand von 5 m aneinander vorbeifah-ren;

c) zwei WasserstoffatomenmH 1,6734 10ÿ27 kg im Abstand vond 10ÿ8 cm?

Lösung:a) F 1,186 N;b) F 2,162 ´ 10ÿ10 N;c) F 1,869 ´ 10ÿ48 N.

2 Mit welcher Kraft ziehen sich die kleinem1 20 g und die groûe m2 1,46 kg Blei-kugel der Gravitationsdrehwaage im Abstandr 4,5 cm an?

Lösung:F 9,622 ´ 10ÿ10 N.

*3 Bei einem Versuch nach Abb. 85.2 misst man denWeg x und die Zeit t, den der Lichtzeiger aufder e 8,90 m entfernten Skala zurücklegt. Diehalbe Länge des Querarms ist d 5,0 cm, dermittlere Abstand zwischen groûer und kleiner Ku-

gel beträgt r 4,5 cm, jede groûe Kugel hat dieMasse M 1,46 kg. Werten Sie die Messreiheaus; vergleichen Sie das Ergebnis mit dem Litera-turwert.

x in cm 0 1,3 2,8 4,9 7,4 11,3 15,0 19,2

t in s 0 30 45 60 75 90 105 120

Lösung:a) Eine mit Skizzen belegte Darstellung, die

im einzelnen die Figur Abb. 85.2 (mit der hal-ben Länge des Querarms d) wiedergibt, führtüber a 2 s=t2 mit s d x=2 e und m a 2 Fg

zur experimentellen Bestimmung von g zug ar2=(2 M).

b) Die Tabelle in Aufgabe 3 liefert:

x in cm 0 1,3 2,8 4,9 7,4 11,3 15,0 19,2

t in s 0 30 45 60 75 90 105 120

a in 10ÿ5

m=s2

± 2,89 2,77 2,72 2,63 2,79 2,72 2,67

Daraus erhält man als Mittelwert a 2,74 ´ 10±5

m=s2 und über s 2,809 ´ 10ÿ3 x zur Beschleuni-gung des Querarmes a 7,70 ´ 10ÿ8 m=s2,schlieûlich g 5,34 ´ 10ÿ11 m3=(s2 kg) mit 18%Fehler zum Literaturwert.

2.1.4 Anwendungen des Gravitationsgesetzes

1 Berechnen Sie die Masse der Erde jeweils über denmittleren Abstand des Satelliten vom Erdmittel-punkt:

Satellit Abstand zur Erdoberfläche Umlaufzeit

im Perigäum Apogäum

Nimbus 1 095 km 1 100 km 107,3 min

Skynet 2 270 km 36 041 km 636,5 min

Nato 1 34 429 km 35 786 km 1 401,6 min

Lösung:Bezeichnet rmin den Abstand von der Erde imPerigäum und rmax den Abstand im Apogäum, soist der Schwerpunkt a vom Erdmittelpunkt zum Sa-tellit a 1

2 (rmin rmax) rE (genannt sind nämlichdie Abstände zur Erdoberfläche) mit rE 6,731 ´106 m. Die Masse der Erde ergibt sich dann ausmE 4 p2 a3=(g T2) zu:

mE in 1024 kg

Nimbus 5,946

Skynet 2 5,985

Nato 1 5,970

im Mittel 5,967

Genaue Werte für die Erdmasse lassen sich aus Sa-tellitenmessungen nicht gewinnen, weil einmalStörungen durch andere Himmelskörper, z. B.Mond, Sterne usw., die Bewegung des Satellitenbeeinflussen; die Bewegung eines Satelliten umdie Erde ist kein reines Zweikörperproblem. Zumanderen ist die Erde keine Kugel, und sie ist in ih-rem Aufbau auch nicht homogen.Genauere Werte als z. B. der Mittelwert, der sichaus obigen Berechnungen ergibt mit mE 5,967 ´ 1024 kg und der immerhin nur 0,2% Fehleraufweist, erhält man durch kompliziertere Metho-den.

Seite

85

Seite

88

2.1 Das Gravitationsgesetz 55

Page 56: Metzler Physik

2 Berechnen Sie aus den Daten der vier von GALILEI

entdeckten Jupitermonde die Masse des Jupiters:

Name mittlerer Abstand siderische Umlaufzeit

Jo 412 000 km 1,769 d

Europa 670 900 km 3,551 d

Ganymed 1 070 000 km 7,155 d

Callisto 1 880 000 km 16,689 d

Lösung:Die Jupitermasse ergibt sich aus Gleichsetzenvon Gravitations- und Radialkraft zu mJ 4 p2 a3=(g T2). (Im Gegensatz zu Aufgabe 1 Seite88 ist hier der mittlere Abstand als Entfernung derbeiden Schwerpunkte zu nehmen.)

3 Wie groû wäre die (siderische) Umlaufzeit desMondes,a) wenn der Mond die doppelte Masse besäûe und

sich auf der gleichen Umlaufbahn bewegte;b) wenn sich seine Bahngeschwindigkeit verdop-

pelte?

Lösung:Aus dem Ansatz, dass die Gravitationskraft die fürdie Bahnbewegung (Kreisbewegung) erforderlicheZentripetalkraft liefertmM(2 p=TM)2 rME g mM mE=(r2

ME),sieht man, dass die Umlaufzeit TM des Mondes nurvon der Masse mE der Erde und der Entfernung rME

Erde±Mond, nicht aber von der Masse mM desMondes abhängt.Aus dem 3. Keplerschen Gesetz r3=T2 konstantund v r 2 p=T ergibt sich T C=v3.Mit v2 2 v1 ist dann T2 C=v3

2 18 C=v3

1 18 T1.

Legt man die siderische Umlaufzeit zugrunde(T1 2,36 ´ 106 s 27,32 d), so heiût die neue Um-laufzeit T2 2,95 ´ 105 s 3,415 d.

*4 Berechnen Sie die Lage des gemeinsamen Schwer-punktes von Erde und Mond.

Lösung:Der Abstand Erde (Masse mE, Radius R) ± Mond(Masse mM) sei r (r 3,844 ´ 108 m). Weiter seiendie Entfernungen Erdmittelpunkt-Schwerpunktvon Erde und Mond r1 und Mondmittelpunkt±Schwerpunkt r2. Dann gilt mE r1 mM r2 ausmE w2 r1 mM w2 r2 und r1 r2 r. Daraus folgt

r1 mM=(mMmE) r 4,672 106 m 0,733 R,r2 mE=(mMmE) r 3,797 108 m.

Der gemeinsame Schwerpunkt liegt also noch in-nerhalb des Erdkörpers.

*5 Wie groû sind die Gravitationskraft des Mondesund die Zentrifugalkraft infolge der Drehung umden gemeinsamen Schwerpunkt von Erde undMond (Aufgabe 4), die auf eine Wassermengeder Masse m 1 kg auf der mondzugewandtenund auf der mondabgewandten Seite der Erdober-fläche wirken? Wie groû ist die resultierende Kraftin beiden Fällen?

Lösung:Die überall gleich groûe Zentrifugalkraft ist

FZ m w2 r1 g mM m=r2 3,319 ´ 10ÿ5 N

mit r1, der Entfernung Erdmittelpunkt-Schwer-punkt, m, der Masse des Probekörpers von 1 kg,mM, der Mondmasse, und r, dem Abstand derSchwerpunkte von Mond und Erde.Für die Gravitationskräfte auf den Probekörper er-gibt sich auf der mondzugewandten Seite

FgA g mM m=(rÿR)2 3,432 ´ 10 ÿ5 N

und auf der mondabgewandten Seite

FgB g mM m=(rR)2 3,212 ´ 10ÿ5 N

mit dem Erdradius R.Die resultierende Kraft ist auf der mondzugewand-ten Seite

FA FZÿFgA ÿ1,13 ´ 10ÿ6 N ÿ1,1 ´ 10ÿ6 N

und auf der mondabgewandten Seite

FB FZÿFg B 1,07 ´ 10ÿ6 N 1,1 ´ 10ÿ6 N.

*6 a) Geben Sie die Gezeitenbeschleunigungen durchdie Sonne formelmäûig an und berechnen Siesie.

b) Vergleichen Sie sie mit der des Mondes.

Lösung:a) Mit dem (mittleren) Radius der Erde R, der

Masse m8 der Sonne und der (mittleren) Ent-fernung r zur Sonne wird die Gezeitenbeschleu-nigung

DgA ÿDgB 2 gm8 R=r3

5,05 ´ 10ÿ7 m=s2.

b) Die Gezeitenbeschleunigung des Mondes

DgA ÿDgB 2gmM R=r3 1,10 ´ 10ÿ6 m=s2

ist doppelt so groû wie die Sonne.

2 Gravitation56

Page 57: Metzler Physik

2.2.1 Feldbegriff und Feldstärke

1 Berechnen Sie jeweils die Gravitationsfeldstärkeauf der Oberfläche der Erde am Pol und am ¾qua-tor, auf der Mondoberfläche, auf der Oberflächeder Sonne.

Lösung:Am ¾quator gÈA 9,7982 m=s2,am Pol gP 9,8643 m=s2,auf der Mondoberfläche gM 1,6237 m=s2

0,166 gE,auf der Sonnenoberfläche gS 2,7399 ´ 102 m=s2

*2 Für einen Punkt im Erdinnern zählt für die Gravita-tionskraft nur die Masse der Kugel, deren Radiusseinem Abstand vom Mittelpunkt entspricht. Ho-mogene Massenverteilung vorausgesetzt berechnemana) die Gravitationsfeldstärke in 1000 km Tiefe,b) die Abhängigkeit der Gravitationsfeldstärke im

Innern der Erde vom Abstand vom Erdmittel-punkt.

Lösung:a) Aus mr (r=rE)3mE und g g mr=r2 ergibt

sich gr gE r=rE.Mit gE 9,814 m=s2 ist in 1000 km Tiefegr 8,274 m=s2.

b) Die Feldstärke im Abstand r vom Erdmittel-punkt wächst dem Betrag nach linear mit demAbstand vom Erdmittelpunkt:

gr g(mE=r3E) r, vektoriell~gr ÿg(mE=r3

E)~r.

*3 Erde (Masse M) und Mond (m) haben den Schwer-punktsabstand r. Gibt es einen Punkt, an dem dieFeldstärke null ist (allgemein und betragsmäûig)?

Lösung:Aus der Gleichsetzung der Feldstärkeng mE=r2

1 g mM=r22 folgt als erste Lösung

r1 mE=mM

pr2, wenn r1 die Entfernung Erde zu

diesem Punkt und r2 diejenige vom Mond zu ihmist, und damit über r1 r2 e

folgt r1 e=

mM

mE

r 1

! 3,460 ´ 108 m

und r2 e=

mE

mM

r 1

! 0,384 ´ 108 m

Die zweite Lösung r1 ÿmE=mM

pr2 ergibt mit

r1 e=ÿ1ÿ

mM=mE

p 4,323 ´ 108 m und

r2 e=ÿ1ÿ

mE=mM

p ÿ 0,480 ´ 108 m

den Punkt jenseits des Mondes auf der Verbin-dungslinie Erde±Mond, an dem die Feldstärkenzwar betragsmäûig gleich sind, sich aber nicht auf-heben.

Zusatzaufgaben

4 Berechnen Sie die Gravitationsfeldstärke auf denPlaneten, der Sonne und dem Mond nach Tab. Seite576.

Lösung:Die Gravitationsbeschleunigung ist G g M=R2.

Planet G in m=s2

Merkur 3,53

Venus 8,80

Erde 9,809

Mars 3,68

Jupiter 24,9

Saturn 10,4

Uranus 9,1

Neptun 11,6

Pluto 36

Sonne 2,74 ´ 102

Mond 1,624

*5 a) Wie muss man die Gravitationsfeldstärke am¾quator unter Berücksichtigung der Lösungin Aufgabe 1 (Seite 90) korrigiert werden, wennman dort die Erdbeschleunigung angeben will?

b) Wie groû wäre an den genannten Stellen dieGravitationskraft auf einen Körper der Masse80 kg?

Lösung:a) Die Korrektur muss die Zentrifugalbeschleuni-

gung am ¾quator berücksichtigen, also

g0ÈA g ÈAÿw2 r ÈA; mit w2 r ÈA 3,373 ´ 10ÿ2 m=s2

ist g0ÈA 9,7688 m=s2.

b) Es istFÈA 784,2 N und F0ÈA 781,5 N.FP 789,5 N,FM 129,9 N,FS 2,192 ´ 104 N.

*6 Berechnen Sie die Feldstärke des Erdfeldes imAbstand r1 R, r2 2R, . . . r10 10R (R : Erdra-dius) vom Erdmittelpunkt in Einheiten der Feld-stärke g auf der Erdoberfläche und stellen Sie dieAbhängigkeit graphisch dar.

Seite

90

2.2 Das Gravitationsfeld 57

Page 58: Metzler Physik

Lösung:Mit r nR (R Erdradius) ist g g mE=r2 gE=n2:

r R 2 R 3 R 4 R 5 R 6 R 7 R 8 R 9 R 10 R

g

ggE

ge

1

14 gE

0,25

19 gE

0,111

116 gE

0,0625

125 gE

0,040

136 gE

0,028

149 gE

0,020

164 gE

0,016

181 gE

0,012

1100 gE

0,010

2.2.2 Potentielle Energie im Gravitationsfeld

1 Ein Satellit m 1,5 twird von der Erdoberflächeaus auf die Höhe 25 000 km gebracht. BerechnenSie die erforderliche Energiea) mit der Näherung, dass die Gravitationsfeld-

stärke konstant (wie auf der Erdoberfläche) ist,b) im (inhomogenen) Radialfeld der Erde.

Lösung:a) Em g h 3,68 ´ 1011 Jb) E gm M (1=r0ÿ 1=r) 7,48 ´ 1010 J

mit r0 6,731 ´ 106 m und r1 r0 2,5 107 m.

2 Berechnen Sie die potentielle Energie der Sonnen-sonde Helios m 370,5 kg bezüglich der Son-nenoberfläche für den sonnenfernsten Punkt derBahn (Entfernung zum Sonnenmittelpunkt147,5 Mio. km) und den sonnennächsten Punkt(Entfernung 46,5 Mio. km).

Lösung:Die Entfernung der Helios-Sonde im sonnennächs-ten Punkt ist 46,5 Millionen Kilometer. Mit derSonnenmasse M, dem Sonnenradius R, der Entfer-nung rE der Erde von der Sonne und der Entfernungr der Sonde von der Sonne ist die potentielle Ener-gie bezogen auf die Sonnenoberfläche (R)

Epot(r) gmm8 (1=R8ÿ1=r), also im

Aphel Epot 1 7,032 ´ 1013 J und imPerihel Epot 2 6,959 ´ 1013 J und damit

DEpot Epot 1ÿEpot 2 7,3 ´ 1011 J.

Bezieht man die potentielle Energie auf den Start-punkt der Sonde, die Erde (rE), so wird mit

E0pot(r) gmM (1=rEÿ 1=r) im

Aphel E0pot 1 ÿ 5,132 ´ 199 J und diePerihel E0pot 2 ÿ 7,334 ´ 1011 J und damit

DE0P E0pot 1ÿE0pot 2 7,283 ´ 1011 J 7,3 ´ 1011 J.

*3 Berechnen Sie für die Daten der Aufgabe 2 das Po-tential im Aphel und Perihel der Sonnensonde He-lios bezüglich der Sonnenoberfläche.

Lösung:Mit dem Abstand r1 147,5 ´ 109 m der Sonde vonSonnenmittelpunkt in Aphel und r2 46,5 ´ 109 min Perihel sowie Sonnenradius R8 und Sonnen-masse m8 wird für das Potential

V(r) gm8 (1=R8ÿ1=r)

und im Aphel

VAphel 1,898 ´ 1011 J=kg

und im Perihel

VPerihel 1,878 ´ 1011 J=kg.

Sind die Lösungen von Aufgabe 2 bekannt, berech-net man das Potential einfacher mithilfe der Defini-tion: V (r) = EPot (r)/m.

*4 Berechnen Sie allgemein und zahlenmäûig für dasGravitationsfeld der Erde (Masse M)a) das Potential (Bezugspunkt im Unendlichen)

auf der Erdoberfläche und im Abstand desein-, zwei- und dreifachen Erdradius über ihr(Abstand r vom Erdmittelpunkt);

b) die Potentialdifferenzen für die genannten Ab-stände.

Lösung:a) Allgemein gilt V(r)ÿg M=r, auf der

Erdoberfläche V(R)ÿg M=R und im Ab-stand des ein-, zwei-, dreifachen Erdradius da-von mit n 1, 2, 3

V( (n 1)R)ÿg M=( (n 1)R).

ZahlenmäûigV(R) ÿ 6,26 ´ 107 J=m,V(2R) ÿ 3,13 ´ 107 J=m,V(3R) ÿ 2,09 ´ 107 J=m,V(4R) ÿ 1,56 ´ 107 J=m.

Seite

92

2 Gravitation58

Page 59: Metzler Physik

b) Die Potentialdifferenz vom n-fachen Abstandzu Erdoberfläche istDVn V( (n 1)R)ÿV(R)ÿg m=((n 1)R)g M=R,

in Zahlen

DV1 3,13 ´ 107 J=m, DV2 4,17 ´ 107 J=m,DV3 4,69 ´ 107 J=m.

*5 Stellen Sie für das Gravitationsfeld der Erde(Masse M) das Potential bezüglich der Erdoberflä-che als Funktion des Abstands r vom Erdmittel-punkt aufa) für den Bereich des Erdinneren (! Aufgabe 2,

S. 90);b) für den Bereich auûerhalb der Erde.c) Zeichnen Sie den Verlauf der Funktion in ihrer

allgemeinen Form für den Bereich von r 0 bisr 3R.

Lösung:a) Nach Aufgabe 2, Seite 90, ist die Feldstärke im

Erdinnern g (r)ÿg (M=R3) r, also die äuûereaufzuwendende Kraft F mg (M=R3) r, undfolglich berechnet sich das Potential einesPunktes P (r) im Erdinnern gegenüber einemPunkt auf der Erdoberfläche P (R) zu

Vi (r) g M=R3

RrR

r dr

12 g(M=R3) (r2ÿR2) < 0 für r < R.

b) Auûerhalb der Erde gilt (r > R)Va (r) gM (1=Rÿ 1=r) > 0.

c) Vi (r) stellt eine nach oben geöffnete Parabel darmit dem ScheitelpunktVi (0)ÿ 1

2 g (M=R3)R2 ÿ 12 gM=R

Va (r) verläuft nach einer Hyperbel mit derAsymptote Va (1) g M=R:

Asymptote

R

V ( )r

γ12– M

R

γ MR

aV ( )r

r

iV ( )r

Zusatzaufgaben

6 a) Wie groû sind die potentielle, die kinetischeund die gesamte Energie eines Satelliten aufeiner stabilen, kreisförmigen Umlaufbahn?Drücken Sie diese Energien ausschlieûlichdurch die Erdmasse M, die Satellitenmasse m,den Radius r der Umlaufbahn und die Gravita-tionskonstante g aus.

b) Begründen Sie, ob und weshalb der Radius derUmlaufbahn zu- oder abnimmt, wenn die Rei-bungskraft der äuûeren Lufthülle der Erde dieGesamtenergie vermindert.

Lösung:a) Aus mv2=r g mM=r2 folgt

Ekin 12 m v2 1

2 gmM=r, und mitEpot ÿg mM=r dannES EkinEpot ÿ 1

2 g m M=r.b) Wenn die Gesamtenergie ES abnimmt, folgt aus

ES2 < ES1 oder ÿ 12 gm M=r2 < ÿ 1

2 gmM=r1,dass r2 < r1 ist.Der Satellit nähert sich der Erde.

*7 Berechnen Sie das Potential des Erdfeldes imInnern der Erde (unter Berücksichtigung der Auf-gabe 2, Seite 90) mit dem Potential-Null auf derErdoberflächea) in 1000 km Tiefe,b) in Abhängigkeit vom Abstand zum Erdmittel-

punkt.c) Stellen Sie den Verlauf graphisch dar.

Lösung:Mit der Feldstärke für r R giltgr g M=R3 r und der Kraft Fr g M m=R3 r andieser Stelle auf einen Körper der Masse m erhältman als Energie, um den Körper von der Erdober-fläche auf diesen Abstand zu bringen,

Eÿ 12 g mM=r3 (R2ÿ r2)

und damit als Potential eines Punktes im Erdinnerngegenüber der Erdoberfläche

Vÿ 12 g M=R3 (R2ÿ r2)ÿ 1

2 g M=R 1ÿ (r=R)2:a) In 1000 km Tiefe ist

V ÿ 3,130 ´ 107 J=kg

1ÿ

5,371 106 m6,371 106 m

2

ÿ 9,055 ´ 106 J=kg.

b) V ÿ 3,130 ´ 107 J=kg

1ÿ

r6,371 106 m

2.

c) Der Verlauf wird dargestellt von einer nachoben geöffneten Parabel 2. Grades vom Typ

V V0

1ÿ (r=R)2

2.2 Das Gravitationsfeld 59

Page 60: Metzler Physik

im Bereich 0 [ r [ R mitV0 ÿ 3,130 ´ 107 J=kg, dem Potential im Erd-mittelpunkt, und VR 0, dem Potential auf derErdoberfläche.

8 Berechnen Sie das Potential des Erdfeldes im Ab-stand r1 R, r2 R, . . . r10 10R (R Erdradius)vom Erdmittelpunkt in Einheiten des Potentials aufder Erdoberfläche, wenn man die Potentiale auf ei-nen unendlich entfernten Punkt bezieht, und stellenSie die Abhängigkeit graphisch dar.

Lösung:Es ist V (r)ÿgM=r oder V (n)ÿgM=(nR)ÿ 6,259 ´ 107 1

n J/kg.

n r V in 107 J/kg

1 R ± 6,257

2 2 R ± 3,130

3 3 R ± 2,086

4 4 R ± 1,565

5 5 R ± 1,252

6 6 R ± 1,043

7 7 R ± 0,894

8 8 R ± 0,782

9 9 R ± 0,695

10 10 R ± 0,626

*9 Berechnen Sie auf der Verbindungslinie Erde-Mond das Potential beider Körpera) auf der Erdoberfläche,b) auf der erdab- und erdzugewandten Seite der

Mondoberfläche,c) 5 ´ 107 m oberhalb der Erde undd) 5 ´ 107 m vor und hinter dem Mond.

Lösung:Ist M die Masse der Erde, m die des Mondes, R derRadius der Erde, r der des Mondes, d der Abstand(der Mittelpunkte) von Erde und Mond und e die inc) und d) genannte Entfernung, so wird:a) auf der Erde: V ÿg M=Rÿg m=(dÿR)

ÿ 6,261 ´ 107 J=kg;b) erdabgewandt: V ÿg M=(d r)ÿg m=r

ÿ 1,046 ´ 106 J=kg,erdzugewandt: V ÿg M=(dÿ r)ÿg m=r

ÿ 1,055 ´ 106 J=kg;c) oberhalb:

V ÿg M=(R e)ÿg m=(dÿ (R e)) ÿ 7,089 ´ 106 J=kg

d) vor dem Mond:V ÿg M=(dÿ (r e))ÿg m=(r e)

ÿ 1,294 ´ 106 J=kghinter dem Mond:

V ÿg M=(d (r e))ÿg m=(r e) ÿ 1,009 ´ 106 J=kg.

2.2.3 Schwere und träge Masse

Zusatzaufgabe:

1 Schwere (ms) und träge (mt) Masse. Läût man aneinem Faden der Länge l eine Kugel aus Blei, da-nach eine Kugel aus Eisen einen Kreis mit gleicherWinkelgeschwindigkeit w beschreiben, so dasssich die Kugeln ähnlich wie der Körper eines Flieh-kraftreglers bewegen und der Faden mit der Senk-rechten einen Winkel a bildet, so ist der Quotientaus den Beiträgen der Gewichtskraft G und Zentri-petalkraft Z

G=Z ms g=(mt w2 r) cot a.a) Weisen Sie die Gültigkeit der Beziehungen

nach. Dabei ist zu beachten, dass die Körper-eigenschaften träge Masse und schwere Massezu unterscheiden sind.

b) Versuche zeigen, dass unabhängig vom Mate-rial (Blei oder Eisen usw.) der Winkel a kon-stant bleibt, wenn die gleiche Winkelgeschwin-digkeit gewählt wird. ± Wie kann man daraus

auf die ¾quivalenz von schwerer und trägerMasse schlieûen?

Lösung:a) Nach der Abbildung Seite 55 unten ist

cot aG=Z mS g=(mt w2 r):Die Gewichtskraft ist auf die Eigenschaft derschweren, die Zentriputalkraft auf die der trä-gen Masse zurückzuführen.

b) Ist bei gleicher Fadenlänge l und der gleichenWinkelgeschwindigkeit w unabhängig von derGröûe der Masse und vom Material der Winkela konstant, so bedeutet das, dass der Quotientaus ms und mt unabhängig von der Stoffmengeund vom Material ist: Man kann also dieschwere und die träge Masse mit der gleichenMaûzahl erfassen, wenn man auûerdem zeigt,dass der Quotient ms : mt 1 ist.

Seite

93

2 Gravitation60

Page 61: Metzler Physik

2.3.1 Zentralkraft; Kepler'sche Gesetze

1 Bestimmen Sie die Umlaufzeit des Uranus ausder mittleren Entfernung Erde±Sonne r1 1,496 1011 m und der mittleren Entfernung Ura-nus±Sonne r2 2,87 1012 m.

Lösung:Für Uranus (U) und Erde (E) lautet das 3. Kepler'-sche Gesetz

T3U : T3

E r3U : r3

E oder

TU 2,65 ´ 109 s 3,07 ´ 104 d 84 a.

2 Der Saturnmond Mimas umkreist seinen Planetenin der Umlaufzeit T 0,94221 d mit der groûenHalbachse a 1,856 105 km. Wie groû ist dieMasse des Saturns?

Lösung:Aus dem Ansatz Gravitationskraft gleich Radial-kraft für Saturn (S) und seinen Begleiter Mimas(M)

g mS mM=r2 mM(2p=T)2r

gewinnt man

mS (4p2=T2) (r3=g) 5,71 ´ 1025 kg

3 Die Masse des Jupiters ist aus seiner UmlaufzeitT 4332,60 d und der groûen Halbachse seinerBahn a 5,2028 AE mithilfe der Daten von Sonneund Erde zu berechnen (Zwei-Körper-Problem).

Lösung:Für Jupiter (J), Erde (E) und Sonne (S) gilt nachdem 3. Kepler'schen Gesetz in seiner genauerenFassung

a3J=a3

E T2J (MSMJ)=(T2

E (MSME))

und daraus

MJ T2E a3

J=(T2J a3

E) (MSME)ÿMS

oder

MJ 1,9 ´ 1027 kg

*4 Die Möglichkeit, Sternmassen direkt zu bestim-men, bieten so genannte Doppelsterne, die sichum ihren gemeinsamen Schwerpunkt bewegen.Vom Doppelsternsystem Sirius (S) und seinem Be-gleiter (B) sind die gemeinsame UmlaufzeitT 49,9 a und die beiden groûen HalbachsenaS 6,8 AE und aB 13,7 AE bekannt (aus Mes-sungen vor den Hintergrundsternen des Doppel-sternsystems).

a) Bestimmen Sie zuerst die Massen des Doppel-sternsystems und danach über die Halbachsendie Masse des Sirius und seines Begleiters.

b) Geben Sie beide Massen in Sonnenmassen an.

Lösung:a) Zur Bestimmung der Gesamtmasse des Dop-

pelsternsystems betrachtet man im System desSirius (S) die Kreisbewegung seines Begleiters(B) um den Sirius

T2B=r3

B 4p2=(g (mSmB))

und erhält mit TB T und rB aS aB

mSmB 6,88 ´ 1030 kg.

Aus mS aS mB aB erhält man die einzelnenMassen, wenn man beide Gleichungen kombi-niert:

mS 4,60 ´ 1030 kg und mB 2,28 ´ 1030 kg.

b) Aus dem Vergleich mit der Masse der Sonnem8 1,989 ´ 1030 kg zeigt sich, dass die Massevon Sirius und die von seinem Begleiter in derGröûenordnung der Sonnenmasse liegen

mS 2,31 m8 und mB 1,15 m8.

*5Führen Sie das Iterationsverfahren nach Abb. 96.1bis zum 20. Schritt weiter aus und zeichnen Sie.

Lösung:Mit den Anfangsbedingungen

x (0) 0,500; y (0) 0,000;vx 0,000; vy 1,630

ergeben sich die Ausgangswerte für die Geschwin-digkeiten

vx ( 12 D t) vx(0) ( 1

2 D t)ÿx (0)r3 (0)

,

vy ( 12 D t) vy(0) ( 1

2 D t)ÿx (0)r3 (0)

,

und mit r (0)x2 (0) y2 (0)

p

0,5002 0,0002p

0,500.

vx(0,05) 0,000 0,05ÿ0,5000,5003

ÿ0,200,

vy(0,05) 1,630 0,05ÿ0,0000,5003

ÿ1,630.

DarausberechnensichdieKoordinatenfür t0,1zu

x (0,1) x (0) 0,1 vx (0,05) 0,500 0,1 (ÿ 0,200) 0,480,

Seite

96

2.3 Bewegungen im Gravitationsfeld 61

Page 62: Metzler Physik

y (0,1) y (0) 0,1 vy (0,05) 0,000 0,1 (1,630) 0,163.

Nun berechnet man weiter mit dem System der Ite-rationsgleichungen

vx (t 0,05) vx tÿ 0,05) 0,1ÿx (t)r3 (t)

,

vy (t 0,05) vy tÿ 0,05) 0,1ÿx (t)r3 (t)

mit r (t)x2 (t) y2 (t)

pund weiter

x (t 0,1) x (t) 0,1vx(t 0,05),

y (t 0,1) y (t) 0,1vy(t 0,05)

nacheinander für t 0,2; 0,3; . . . bis 2,0 die Koor-dinaten der Bildpunkte, die insgesamt eine Ellipsemit den Achsen 2a 1,5 und 2 b 1,4 bilden.

t vx(tÿ 12 Dt) vy(tÿ 1

2 Dt) x (t) y (t)

0,0 0,000 1,630 0,500 0,000

0,1 ± 0,200 1,630 0,480 0,163

0,2 ± 0,568 1,505 0,423 0,313

0,3 ± 0,858 1,290 0,337 0,442

0,4 ± 1,054 1,033 0,232 0,545

0,5 ± 1,166 0,771 0,115 0,622

0,6 ± 1,211 0,525 ± 0,006 0,675

0,7 ± 1,209 0,306 ± 0,127 0,706

0,8 ± 1,175 0,115 ± 0,244 0,717

0,9 ± 1,119 ± 0,050 ± 0,356 0,712

1,0 ± 1,048 ± 0,191 ± 0,461 0,693

1,1 ± 0,968 ± 0,311 ± 0,558 0,662

1,2 ± 0,882 ± 0,413 ± 0,646 0,621

1,3 ± 0,792 ± 0,499 ± 0,725 0,571

1,4 ± 0,700 ± 0,572 ± 0,795 0,514

1,5 ± 0,606 ± 0,633 ± 0,856 0,451

1,6 ± 0,511 ± 0,682 ± 0,907 0,383

1,7 ± 0,416 ± 0,723 ± 0,949 0,311

1,8 ± 0,321 ± 0,754 ± 0,981 0,236

1,9 ± 0,225 ± 0,777 ± 1,004 0,158

2,0 ± 0,129 ± 0,792 ± 1,017 0,079

Zeichnung siehe Abb. 96.1

*6 Senkrecht über Quito wird ein Satellit in 400 kmHöhe über NN mit der (horizontalen) Geschwin-digkeit v0 7,800 km=s geortet, Kurs genau Ost.Berechnen Sie iterativ in Abständen Dt 1 min die

nächsten Bahnpunkte. Wo (geografische Längeund Breite (Atlas!), Höhe über NN) befindet sichder Satellit nach 7 min?

Lösung:Die Aufgabe wird als ebenes Problem behandelt: xAbstand ¹parallelª zu Erdoberfläche, y Höhe überdem Erdmittelpunkt.

Startwerte: x (0) 0 (Quito);y (0) 6,3782 ´ 106 m 0,4 ´ 106 m,

vx (0) 7,8 ´ 103 m=s; vy (0) 0;D t 1 min 60 s.

Iterationsvorschrift nach Abb. 24.1 auf Seite 24 mit

ax (t)ÿgmE x (t)=x2 (t) y2(t)

p 3;

ay (t)ÿgmE y (t)=x2 (t) y2(t)

p 3.

Damit ergeben sich die Werte:

n t in s x (t) in 106 m y (t) in 106 m

0 0 0 6,7782

1 60 0,468000 6,7782

2 120 0,936000 6,74695

3 180 1,40186 6,68468

4 240 1,86346 6,59176

5 300 2,31875 6,46872

6 360 2,76572 6,31624

7 420 3,20243 6,13515

Die Längendifferenz zwischen Quito und demZielort (nach 7 min) ergibt sich aus

Dl 180

parc tan

x (420)y (420)

27,56 27,6.

Die neue Länge des Zielortes ist, wenn man (ausdem Atlas) als Länge von Quito 78,6 (westlicheLänge) nimmt:

le 51,0.

Die Höhe des Zielortes ist h x (420)ÿR542,47 km. Nach 7 min befindet sich der Satellitüber dem ¾quator in 542,47 km Höhe auf derLänge le 51,0 West etwa über der Mündungdes Amazonas.

2 Gravitation62

Page 63: Metzler Physik

2.3.2 Bahnform und Energie der Satelliten

1 Wie groû sind Geschwindigkeit, Radius und Höheeines Satelliten über der Erdoberfläche, der überdemselben Punkt der Erde stehen bleibt (geostatio-näre Bahn)?

Lösung:Aus m w2 r g mM=r2 mit der Masse M derErde und der Umlaufzeit T 1 d erhält man denBahnradius des Satelliten zu r 4,224 ´ 107 m unddamit die Höhe h 3,586 ´ 107 m 35 860 km überdem ¾quator, seine Geschwindigkeit liefert derAnsatz v2 g M=r2 zu v 3,072 ´ 103 m=s 3,072 km=s.

*2 Ein Satellit bewege sich in 600 km Höhe über dem¾quator auf einem Orbit (Umlaufbahn) und startevon dort mit einem neuen Schub senkrecht zur Ver-bindungslinie Erde±Startpunkt mit einer (zusätzli-chen) Geschwindigkeit Dv0 2,0 103 m=s.Wie weit sind Perigäum und Apogäum der ellipti-schen Bahn vom Erdmittelpunkt entfernt?

Lösung:In 600 km Höhe hat der Satellit den AbstandrP R 600 km 6,978 ´ 106 m vom Erdmittel-punkt. Dieser Startpunkt ist das Perigäum der El-lipsenbahn, die der Satellit nach dem Geschwindig-keitsschub Dv beschreibt.Seine neue Geschwindigkeit ist

vP vKreisbahnDvg M=rP

pDv

9,558 ´ 103 m=s.

Aus der für das Perigäum berechneten Gesamt-energie

ÿ 12 g mM=a 1

2 m v2Pÿg mM=rP

berechnet man die groûe Halbachse der Ellipse

a gM rP=(2gMÿ v2 rP) zu a 1,741 ´ 107 m

und daraus den Abstand des Apogäum von Erdmit-telpunkt.

rA 2aÿ rP 2,784 ´ 107 m.

*3 Ein Satellit befindet sich auf einer Parkbahn in400 km Höhe (in Bezug auf den ¾quatorradius).a) Wie groû muss der tangentiale Geschwindig-

keitsschub sein, der den Satelliten auf eine El-lipse mit dem Apogäum in 35 786 km Höheüber der Erde bringt?

b) Mit welchem zweiten (tangentialen) Ge-schwindigkeitsschub im Apogäum bringt manden Satelliten auf eine geostationäre (Kreis-)Bahn in dieser Höhe?

Anmerkung: Die Manöver in a) bzw. b) bezeichnetman als Hohmann-Transfer (! S. 99). Sie sindvom Energieaufwand besonders günstig gegenüberanderen möglichen Übergängen von einem Orbitauf einen höheren.

Lösung:a) Im Perigäum 400 km über dem ¾quator im Ab-

stand rP R 400 km 6,778 ´ 106 m hat derSatellit die Kreisbahngeschwindigkeit

vP gM=rP

poder vP 7,669 ´ 103 m=s.

ErsolldurchdenGeschwindigkeitsschubDvdasApogäum erreichen, das rA R 35 786 kmvon Erdmittelpunkt entfernt ist.Die Halbachse der (neuen) Ellipsenbahn ist

a 12 (rP rA) 2,447 ´ 107 m.

Dazu muss der Satellit im Perigäum auf die Ge-schwindigkeit

vPneu gM (2=rPÿ 1=a

p) oder

vP 1,0066 ´ 104 m=s

kommen.Der Geschwindigkeitsschub ist folglich

Dv vPneuÿ vP 2,397 ´ 103 m=s.

b) Für die Geschwindigkeiten im Perigäum undApogäum gilt nach dem 2. Kepler'schen Gesetz

vA rA vP rP und daraus vA 1,618 ´ 103 m=s.

Der Satellit soll nun im Apogäum auf eineKreisbahn gebracht werden, die eine Ge-schwindigkeit

vA Kreis gM=rA

poder

vA Kreis 3,075 ´ 103 m=s

erforderlich macht.Er hat also seine Geschwindigkeit im Apogäumum

Dv vA Kreisÿ vA oder Dv 1,456 ´ 103 m=s

zu steigern.

*4 Die Fluchtgeschwindigkeit von der Erde aus demAnziehungsbereich der Sonne (3. kosmische Ge-schwindigkeit) beträgt v3 16,7 km=s, falls manbeim Start die Geschwindigkeit vE der Erde umdie Sonne ausnutzt.a) Wie groû ist die Geschwindigkeit vE?

Seite

98

2.3 Bewegungen im Gravitationsfeld 63

Page 64: Metzler Physik

b) Berechnen Sie die Fluchtgeschwindigkeit v03aus dem Gravitationsfeld der Sonne ohne Aus-nutzung der Bewegung der Erde um die Sonne.

c) Zeigen Sie über eine Energiebetrachtung, dassfür die Fluchtgeschwindigkeit v3 gilt:v2

3 v22v03ÿ vE2 und berechnen Sie v3 (v2

ist die 2. kosmische Geschwindigkeit).

Lösung:a) Die Bahngeschwindigkeit vE der Erde um die

Sonne berechnet sich aus

mE v2E=rS g mE mS=r2

S oder

vE 29,79 ´ 104 m=s 29,8 km=h.

b) Ohne Berücksichtigung der Bewegung derErde um die Sonne beträgt die Fluchtgeschwin-digkeit von der Erde aus dem Bereich der Sonnemit rS 1 AE

v023 2g mS=rS oder

v03 4,212 ´ 104 m=s 42,2 km=h.

c) DieEnergieE3 12 mv2

3 beim StartvonderErde,die sich gegenüber der Sonne mit der Geschwin-digkeit vE bewegt, muss der Energie zum Aus-tritt aus dem Erdfeld EE 1

2 mv22 mit der 2. kos-

mischen Geschwindigkeit v2 und der zum Ver-lassen des Sonnenfeldes ES 1

2 m (v03ÿ vE)2

gleich sein: 12 mv2

3 12 m v2

2 12 m (v03ÿ vE) oder

v23 v2

2 (v03ÿ vE)2, also ist die 3. kos-mische Geschwindigkeit v3 1,665 ´ 104 m=s 16,7 km=h.

Zusatzaufgaben

*5 Die Fluchtgeschwindigkeit vf aus dem Bereich desErdfeldes ist abhängig von der Entfernung r vomErdmittelpunkt.a) Wie stellt sich die Fluchtgeschwindigkeit vf als

Funktion vor r dar?b) Es gilt vf v2

R=r

p, wenn v2 die Fluchtge-

schwindigkeit von der Erde und R der Erdradiusist.

c) Berechnen Sie die Fluchtgeschwindigkeit vf imAbstand r R, 2 R . . .5 R, 10 R und stellen Sievf f (r) graphisch dar.

Lösung:a) Für die Fluchtgeschwindigkeit gilt, wenn man

R r setzt, vf 2 g M=r

p.

b) Mit v2 2g M=R

pwird vf

2g M=r

p

2 g (M=R) (R=r)

p v2

R=r

p.

c) Für r nR, n 1, 2, . . ., 10 ist vf v2=np

:

r n R R 2 R 3 R 4 R 5 R

vf inkm=s

11,2 7,91 6,46 5,59 5,00

r n R 6 R 7 R 8 R 9 R 10 R

vf inkm=s

4,57 4,23 3,95 3,73 3,54

*6 Setzt man für vf in Aufgabe 5 v ds=dt, so kannman die Zeit t vom Start von der Erdoberfläche biszum Erreichen des Abstandes r vom Erdmittel-punkt durch Integration ermitteln.a) Bestätigen Sie durch Herleitung des Integrals,

daû sich die Zeit zu t 23 r

32ÿR

32=

2 gMp

dar-stellen läût.

b) Berechnen Sie die Zeit t vom Verlassen derErdoberfläche bis zum Erreichen der Entfer-nung r 2R, 5R, 10 R.

c) Wie lange braucht danach eine Rakete bis zumschwerefreien Punkt zwischen Erde und Mond?

Lösung:a) Mit vf

2g M=r

pund vf dr=dt erhält man

d t (1=2 gMp

)rp

d r und durch

Integration t (1=2 gMp

)RrR

rp

d r

(1=2 gMp

) 23 (r

23ÿR

23).

b) Für r n R mit n 2, 5, 10 ergibt sicht (1=

2 gMp

) 23 R

32 (n

32ÿ 1), also für

r 2 R t 6,941 ´ 102 s 11,6 min,r 5 R t 3,865 ´ 103 s 1 h 4,4 min,r 10 R t 1,162 ´ 104 s 3 h 13,7 min.

c) Für den schwerefreien Punkt zwischen Erdeund Mond sind die Feldstärken von Erde undMond gleich:

g mE=r21 g mM=r2

2,

wobei r1 die Entfernung von der Erde zumschwerefreien Punkt, r2 die vom Mond dort-hin und e r1 r2 die Entfernung Erde±Mondist.

Aus r21 mE=mM r2

2 und r2 eÿ r1 erhält man

r21ÿ2mE=(mEÿmM)er1mE=(mEÿmM)e2 0

und daraus r11;12e (mEmE mMp

)=(mEÿmM):

Da r1 < e für den schwerefreien Punkt zwischenErde und Mond (in einem zweiten Punkt hinterdem Mond ± von der Erde aus gesehen ± sinddie Feldstärken auch gleich: 2. Lösung für r1),gilt mit

mEÿmM (mEp

mMp

) (mEp ÿ

mMp

)

2 Gravitation64

Page 65: Metzler Physik

für den Abstand des schwerefreien Punktes vonder Erde

r1 mEp

=(mEp

mMp

) e 3,460 ´ 108 m

und entsprechend für den Abstand diesesPunktes vom Mond

r2 mMp

=(mEp

mMp

) e 3,837 ´ 107 m.

DerschwerefreiePunkt liegtr1(r1=e) e 0,90evon der Erde entfernt.

Für r1 3,640 ´ 108 m ergibt sich aus a)t 1,515 ´ 105 s 42 h 5,8 min 42,1 h.

*7 Ein Körper startet in 2000 km Höhe über der Erd-oberfläche senkrecht zur Verbindungslinie Erde±Startpunkt mit einer Geschwindigkeit v0.a) Bei welcher Startgeschwindigkeit v0 umfliegt

er die Erde auf einem Kreis?b) Bei welcher Startgeschwindigkeit v0 verlässt er

das Gravitationsfeld der Erde auf einer Parabel-bahn?

c) Wie groû sind die gröûte (rmax) und die kleinste(rmin) Entfernung vom Erdmittelpunkt auf einerBahn, die der Körper bei einer Startgeschwin-digkeit v0 4,0 ´ 103 m=s bzw.v0 8,0 ´ 103 m=s beschreibt?

Lösung:a) Damit der Körper eine Kreisbahn mit dem Ra-

dius r0 2000 km R (R Erdradius) um dieErde beschreibt, muss für seine Geschwindig-keit im Startpunkt gelten:

v1 gM=r0

p 6,902 ´ 103 m=s.

b) Beim Start aus dieser Höhe beschreibt er eineParabel, wenn er nach Abb. 98.1 mit der Ge-schwindigkeit

v2 2g M=r0

p 9,761 ´ 103 m=s

den Startpunkt verlässt.c) Ist die Startgeschwindigkeit v0 in diesem Punkt

kleiner als die Grenzgeschwindigkeit v2, bei derder Körper das Gravitationsfeld der Erde aufeiner Parabel verlässt, ergibt sich eine Ellipse,deren einer Brennpunkt der Erdmittelpunkt ist.Nach 2.3.1 lässt sich aus der Gesamtenergie desKörpers in einem beliebigen Punkt seiner Bahndie groûe Halbachse der Bahnellipse nachEgesamt ÿg m M=(2a) bestimmen. Mit der po-tentiellen Energie, die der Körper im Startpunktbesitzt, und der vorgegebenen Geschwindigkeitv0 8 103 m=s lässt sich die Gesamtenergieund daraus die Halbachse a berechnen:

ÿg mM=2a 12 m v2

0ÿg mM=r0

und daraus

a 12,75 ´ 106 m > r0 8,371 ´ 106 m.

Damit ist der Erdmittelpunkt der dem Start-punkt nächstgelegene Brennpunkt der Ellipse.Der Startpunkt ist auf der neuen Ellipbahn auchder erdnächste Punkt, das Perigäum. DasApogäum liegt auf der anderen Seite der Erdeim Abstand (! Abb. 97.2) rA 2 aÿ r0 17,13 ´ 106 m vom Erdmittelpunkt.Für die Startgeschwindigkeit v0 4 103 m=serhält man nach gleicher Rechnung als Halb-achse a 5,030 ´ 106 m. Da der Durchmesserdieser Bahnellipse 2 a kleiner als r0R ist, ver-liefe diese Bahn teilweise in der Erde: Die Bahnist nicht realisierbar.

*8 a) In welcher Entfernung von der Erde (Erdmittel-punkt) liegt der schwerefreie Punkt zwischenMond und Erde?

b) Wie lange braucht eine Rakete bis zu diesemschwerefreien Punkt?

c) Eine Rakete soll von der Erdoberfläche so ab-geschossen werden, dass sie den schwerefreienPunkt (fast) mit der Fluchtgeschwindigkeit ausdem Gravitationsfeld des Mondes erreicht, weilsie dann (gerade noch) vom Gravitationsfelddes Mondes eingefangen wird. ± Wie groû darfhöchstens die Startgeschwindigkeit von derErde sein?

Lösung:a) Wie in Aufgabe 6 c ergibt sich r1 3,460 ´ 108 m.b) Wie in Aufgabe 6 c erhält man t1 1,515 ´ 105 s 42,1 h.

c) Für die Fluchtgeschwindigkeit aus dem Schwe-refeld des Mondes gilt im schwerefreien Punktzwischen Erde und Mond (Abstand r2 vomMond siehe Aufgabe 6 c)

v2f 2g mM=r2 oder vf 5,056 ´ 102 m=s.

Beim Start von der Erdoberfläche aus muss diePotentialdifferenz zum schwerefreien Punktzwischen Erde und Mond überwunden werden(d Abstand Erde ± Mond):

DV gmE (1=Rÿ 1=r1)gmM (1=(dÿR)ÿ1=r2) 6,133 ´ 107 J=kg.

Daraus ergibt sich als Mindestgeschwindig-keit v0

2D Vp

11,07 ´ 103 m=s und alsHöchstgeschwindigkeit beim Start v v0 vf

11,58 ´ 103 m=s.

2.3 Bewegungen im Gravitationsfeld 65

Page 66: Metzler Physik

2.3.3 Rakete und Raketengleichung

1 Berechnen Sie die Endgeschwindigkeit einer ein-stufigen Rakete (Gesamtmasse m 280 t, Treib-stoff 230 t, Ausströmgeschwindigkeit 2500 m/s,Brennzeit te 60 s).

Lösung:Es ist ve c ln (m0=mE) 4306,9 m=s.

*2 a) Welche Geschwindigkeit erreicht die Raketenach Aufgabe 1 unter Berücksichtigung derFallbewegung im Erdfeld? (g werde als kon-stant angenommen.)

b) Welche Höhe erreicht die Rakete?

Lösung:a) Hier gilt ve c ln (m0=me)ÿ g te 3718,3 m=s.b) Setzt man gleichmäûig Abbrand des Treib-

stoffes voraus, dass also die Abnahme desTreibstoffes proportional der Zeit ist ± mitm (t) Raketenmasse zur Zeit t, te Brennzeit, m0

Anfangsmasse und me Masse der Rakete beiBrennschluss ±

m (t)m0ÿ (m0ÿme)=te tm0ÿm t,

so gilt nach der Reaktengleichung

v (t) c ln (m0=m (t)) c ln (m0=(m0ÿm t)).

Mit ds=dt v (t) oder ds v (t) dt ist dann

se Zte

0

c lnm0

m0ÿm tdt.

Das Integral löst sich durch Substitution, z. B.mit

Rln x dx x (ln xÿ 1) c zu

se c teÿ c me=(m0ÿme) ln (m0=me) te mit

ve c ln (m0=me) zu

se (cÿme=(m0ÿme) ve) te.

Für die Aufgabe mit ve aus Aufgabe 1 wird se 9,382 ´ 104 m4 94 km.Berücksichtigt man wie in b) die Fallbewegung,so ist 1

2 g t22 von se abzuziehen. Dann wird

die Steighöhe der Rakete nach Brennschlussse 7,616 ´ 104 m der se 76 km.

*3 Zeigen Sie für eine zweistufige Rakete, dass sichdie Brennschlussgeschwindigkeiten addieren unddie Nutzlastverhältnisse multiplizieren. Zur Ver-einfachung soll die Austrittsgeschwindigkeit bei-der Stufen als gleich angenommen werden.

Lösung:Die Rakete mit der Startmasse m0 hat nach Brenn-schluss die Masse m1 und die Geschwindigkeit v1

v1 vT ln (m0=m1).

Nach Brennschluss wird die erste Stufe abgetrennt.Die zweite Stufe hat dann die Masse m2 zu Beginnund die Masse me nach Brennschluss. Sie erfährtden Geschwindigkeitsschub Dv v2 mit

v2 vT ln (m2=me).

Die Endgeschwindigkeit ist folglich

ve v1 v2 vT ln (m0=m1) vT ln (me=me) vT ln (m0=m1) (m2=me).

Die Geschwindigkeiten addieren sich, wie unmit-telbar einsichtig, die Massenverhältnisse

V1 m0=m1 und V2 m2=me

multiplizierten sich

ve v1 v2 vT ln (V1 V2).

unter der vereinfachenden Annahme, daû die Aus-trittsgeschwindigkeiten beider Stufen gleich sind.

*4 Eine dreistufige Version der europäischen Träger-rakete Ariane 4 startet vom europäischen Welt-raumflughafen Kourou in Französisch Guayanamit einer Nutzlast von 1,98 t mit Behälter für Nutz-last von 440 kg und für Ausrüstung von 527 kg,Leermasse m0, Treibstoffmasse mT, Ausströmge-schwindigkeit vT der Stufen. Berechnen Sie dieBrennschlussgeschwindigkeiten der einzelnen Stu-fen und die Gesamtendgeschwindigkeit. (Gravita-tionskräfte sollen nicht berücksichtigt werden.)

Stufe m0 mT vT

1. 17,7 t 229,0 t 2436 m/s

2. 3,7 t 34,9 t 2729 m/s

3. 1,25 t 10,5 t 4281 m/s

Lösung:Die Nutzlast von 1,98 t mit den Behältern und derAusrüstung zusammen machen rund 3,0 t aus.Die Rakete startet mit der gesamten LeermassemL 22,56 t, mit dem gesamten TreibstoffmT 274,4 t und der gesamten Nutzlast vonmN 3,0 t, also mit der Masse m10 300 t, wovonnach Brennschluss noch verbleiben m1e 71 t.Die Leermasse der ersten Stufe wird abgestoûen, 2.Start mit m20 53,3 t, nach Brennschluss 18,4 t.Nach Abstoûen der Leermasse 3. Start mitm30 14,7 t, nach Brennschluss m3e 4,2 t.

Seite

101

2 Gravitation66

Page 67: Metzler Physik

Damit gewinnt die Rakete die Endgeschwindigkeit

ve vT1 ln (300=71) vT2 ln (53,3=18,4) vT3 ln (14,7=4,2)

oder

ve 11,78 ´ 103 m=s 11,8 km=s.

*5 Wie groû müsste das Massenverhältnis einer ein-stufigen Rakete sein, die das Gravitationsfeld derErde verlassen soll, wenn die Ausströmgeschwin-digkeit des Treibstoffes a) vT 2500 m=s, b)vT 4600 m=s betrüge.

Lösung:Mit ve c ln (m0=me) wird m0=me e

vec , wobei

ve 2 g M=R

p.

(Dabei wird dann vereinfachend vorausgesetzt, daûdie Rakete ihre Endgeschwindigkeit schon ebenüber der Erde (r R!) erreicht.)

a) Für c 2500 m=s ist m0 : me 87,8 : 1.b) Für c 4600 m=s ist m0 : me 11,4 : 1.

Beide Massenverhältnisse sind für die beiden mög-lichen Austrittsgeschwindigkeiten technisch nichtmachbar.

Zusatzaufgaben

*6 Eine zweistufige Rakete (Gesamtmasse m2400 t)hat als 1. Stufe eine Kerosin-Sauerstoff-Rakete(Treibstoffmasse m1 1800 t, Ausströmgeschwin-digkeit c1 2500 m=s, Leergewicht m10 200 t)und als 2. Stufe eine Wasserstoff-Sauerstoff-Ra-kete (m2 320 t, m20 79,95 t, c2 4500 m=s)mit einer Nutzlast von 50 kg. Die Brenndauer der1. Stufe sei t1 150 s, die der 2. Stufe t2 400 s.(Die Abnahme des Treibstoffes werde proportionalder Zeit angenommen.) ± Sie werde im Erdfeld(g konstant) senkrecht gestartet.a) Welcher Geschwindigkeitszuwachs ergibt sich

nach Brennschluss jeweils der 1. bzw. der 2.Stufe?

b) Welche Endgeschwindigkeit erreicht die Ra-kete?

c) Welche Höhe erreicht die Rakete?

Lösung:a) Für die 1. Stufe mit m0 2400 t und me 600 t

ist ve1 3466 m=s und für die 2. Stufe ± nachAbstoûen der Leermasse von 200 t ± mitme 80 t die Geschwindigkeitszunahmeve2 7242 m=s.

b) Die Endgeschwindigkeit ist ve ve1 ve2

10 708 m=s, und unter Berücksichtigungdes Erdfeldes v0e ve1 ve2 ÿ g (t1 t2) 5,31 ´ 103 m=s.Da g mit wachsender Höhe abnimmt, ist v0e einunterer Wert für die Endgeschwindigkeit.

c) Nach Aufgabe 2, Seite 66, wirdse1 (2500ÿ (600=1800) 3466) 150 m 2,02 ´ 105 m und

se2 (4500ÿ (80=320) 7242) 400 m 1,076 ´ 106 m,

also se 1,278 ´ 106 m

*7 Eine dreistufige Rakete hat die Startmassem 800 t, die Nutzlast m0 100 kg und die 1. Stufe(Treibstoff m1 = 600 t, Leermasse m10 160 t), die2. Stufe (m2 30 t, m20 8 t), die 3. Stufe (m3

1,5 t, m30 400 kg). Die Brenngeschwindigkeitbetrage für alle Stufen c 2800 m=s.a) Berechnen sie jeweils für Start und Brenn-

schluss jeder Stufe die Start- und die Leermasse.b) Berechnen Sie den Geschwindigkeitszuwachs,

den jede Stufe erzeugt.c) Berechnen Sie die Endgeschwindigkeit der

Rakete.d) Wie kann man die Endgeschwindigkeit auf-

grund des gleichen Massenverhältnisses jedeStufe angeben?

Lösung:a) Es ist für die 1. Stufe m0 800 t, me 200 t,

davon werden am Brennschluss 160 t abgesto-ûen.Für die 2. Stufe gilt m0 40 t, me 10 t, davonwerden 8 t abgestoûen.Für die 3. Stufe gilt m0 2 t, me 0,5 t, eswerden 0,4 t abgestoûen.

b) Die Geschwindigkeitszunahme lauten1. Stufe ve1 (2800 ln 4) m=s 3,88 ´ 103 m=s,ebenso 2. und 3. Stufe ve2 ve3 ve1 .

c) Die Endgeschwindigkeit istve 1,164 ´ 104 m=s.

d) Es gilt allgemeinve c1 ln (m01=me1 ) c2 ln (m02=me2 )

c3 ln (m03=me3 ) und mitc1 c2 c3 c sowie

m01

me1

m02

me2

m03

me3

m0

me4

dann ve 3c ln (m0=me) oderve c ln (m0=me)

3, also in der Aufgabeve 2800 m=s ln 43 1,164 ´ 104 m=s.

2.3 Bewegungen im Gravitationsfeld 67

Page 68: Metzler Physik

3.1.1 Schwingungsvorgänge und -gröûen

1 Wie kann man die Periodendauer T eines Oszilla-tors (Abb. 106.2) möglichst genau bestimmen? Inwelchem Punkt des Bewegungsablaufs kann dieZeitmessung beginnen, wann muss sie gestopptwerden?

Lösung:Man misst mehrmals die Zeit für möglichst vieleSchwingungen (n 10) und berechnet T ausT t(n)=n. Die Zeitmessung beginnt, wenn sichdas Pendel in einem Umkehrpunkt befindet undwird dann auch wieder gestoppt.

2 Das Pendel einer Wanduhr macht in 2 Minuten 150Schwingungen. Berechnen Sie Periodendauer undFrequenz des Pendels. Wie viele Schwingungenmacht das Pendel in einem Tag, in einem Jahr?

Lösung:T 120 s=150 0,8 s;f 1=T 1,25 s±1;nTag 24 ´ 60 ´ 60 s=0,8 s 108 000;nJahr 39 420 000

3 Zwei Pendel mit den SchwingungsdauernT1 1,5 s und T2 1,6 s starten gleichzeitig ausder Ruhelage. Nach welcher Zeit gehen beide wie-der genau gleichzeitig durch die Ruhelage? Wieviele Schwingungen hat jedes Pendel in dieser Zeitgemacht?

Lösung:Es muss die Bedingung n1T1 n2T2 erfüllt sein.Das kleinste gemeinsame Vielfache von T1 und T2

ist 24, also n1 16 und n2 15.

3.1.3 Gesetze der harmonischen Schwingung

1 Eine harmonische Schwingung hat die Amplitudey 10 cm und die Periodendauer T 2,0 s.a) Stellen Sie die Werte der Elongation, Ge-

schwindigkeit und Beschleunigung für die Zei-ten t n T=8 (für n 0, 1, . . . , 8) in einer Ta-belle zusammen.

b) Zeichnen Sie die Graphen der drei Gröûen inAbhängigkeit von der Zeit (MaûstabT 12 cm).

Lösung:Es ist y y sin w t, vw y cos w t und

aÿw2 y sin w t mit w 2p=T

a) t in s y in cm v in cm=s a in cm=s2

0 0 31.4 0

0.25 7.07 22.2 ± 69.8

0.5 10. 0 ± 98.7

0.75 7.7 ± 22.2 ± 69.8

1. 0 ± 31.4 0

1.25 ± 7.07 ± 22.2 69.8

1.5 ± 10. 0 98.7

1.75 ± 7.07 22.2 69.8

2. 0 31.4 0

b)

–100

–50

50

100

t21,510,5

υy, ,a

2 Zeichnen Sie das Zeit-Weg-Diagramm eines har-monischen Oszillators mitD 0,5 N=m,m2,0 kgund y 4,0 cm und tragen Sie maûstäblich die Ge-schwindigkeits- und die Beschleunigungsvektorenfür die Zeiten t n T=8 (für n 0, 1, . . . , 8) ein(Maûstab T 12 cm).

Lösung:Aus w2D=m folgt w 0,5 s±1.Mit t n T=8 und T 2 p=w folgtw t n ´ p=4.Daraus ergeben sich folgende Funktionen:y 4 cm ´ sin (0,5 t), v 2 cm=s ´ cos (0,5 t) unda ± 1 cm=s2 ´ sin (0,5 t).

3 Mechanische Schwingungen und Wellen68

Seite

107

Seite

110

Page 69: Metzler Physik

t2

υy, ,a

2

4

–2

–4

4 6 8 10 12 14

3 Die Elongation eines harmonischen Oszillators be-trägt 0,2 s nach dem Nulldurchgang y 4 cm. DieAmplitude ist 6 cm. Berechnen Sie Frequenz undPeriodendauer.

Lösung:Aus y y sinw t folgt4 cm 6 cm sin (w ´ 0,2 s), alsow 5 s±1 arc sin (4=6) 3,64 s±1.Daraus ergeben sichT 1,72 s und f 0,58 s±1.

4 Zu welchen Zeiten nach dem Nulldurchgang er-reicht die Elongation einer harmonischen Schwin-gung mit y 5 cm und f 0,4 Hz die Wertea) y1 8 mm, b) y2 2 cm, c) y3 4 cm?

Lösung:a) Aus y y sinw t folgt

0,8 cm 5 cm ´ sin (2 p ´ 0,4 s±1 t), alsotÿ1=2p 0;4 sÿ1

arc sin0;8=5 0,064 s

b) t 0,164 s c) t 0,369 s

*5 Zeigen Sie, dass auch die Zeit-Elongation-Funk-tion y y cos w ta die DifferentialgleichungmyÿD y erfüllt.

Lösung:Aus y y cos w ta folgt

_yÿw y sin w ta undyÿw2 y cos (w ta ÿw2 y.

Einsetzen in die Differentialgleichung ergibtÿmw2 yÿDy,d. h. mit w2D=m ist die Differentialgleichungdurch y y cos w ta zu jedem Zeitpunkt t er-füllt.

*6 Ein harmonischer Oszillator mit T 2 s erreichtzur Zeit t 0,4 s die Amplitude y 5 cm. Wie groûist die Phase ò0 und die Elongation y 0 für t 0?

Lösung:Aus y y sin w ta folgt5 cm 5 cm ´ sin 2 p t=T ò0, also2p t=T ò0 p=2 oder ò0 0,314.Damit ergibt sich y (0) 1,55 cm.

3.1.4 Die gedämpfte harmonische Schwingung

*1 Im Schattenwurf misst man die Amplitude y der 1.,50., 100., . . . Schwingung mit der PeriodendauerT 0,8 s:

n 1 50 100 150 200 250 300

y in cm 5,0 4,0 3,2 2,6 2,2 1,7 1,4

a) Ermitteln Sie die Dämpfungskonstante k ausder Darstellung des natürlichen Logarithmusder Amplitude.

b) Berechnen Sie aus den Versuchsdaten die Halb-wertszeit der Schwingung.

Lösung:a) Für gedämpfte Schwingungen gilt:

y yeÿkt bzw. lnfyg lnfygÿ k t

0,250,500,751,001,251,501,752,00ln ( )y

50 100 150 200 250 300 t00

Aus der graphischen Darstellung ergibt sichk 5;28 10ÿ3 sÿ1.b) Die Halbwertzeit ergibt sich aus dem Ansatz

0;5 y yeÿktH oder tH ln2=k 131 s.

Seite

111

3.1 Schwingungen 69

Page 70: Metzler Physik

2 Die Amplituden der 3. und 4. Schwingung einesPendels betragen 8 cm bzw. 7 cm. Wie groû istdie Amplitude der 1. Schwingung?

Lösung:Für die Amplitude der n. Schwingung giltyn y0 eÿk nÿ1T oder yn y0 qnÿ1 mit q eÿk T .

3 Die Amplitude der 10. Schwingung eines gedämpf-ten Oszillators ist halb so groû wie die Amplitudeder 1. Schwingung. Bei der wievielten Schwingungbeträgt die Amplitude ein Zehntel des Anfangswer-tes?

Lösung:Nach dem Ansatz aus Aufgabe 2 folgt y10 y0 q9

oder 0,5 q9 und q 0,926.yn 1

10 y0 y0 qnÿ1; log0,1 nÿ 1 log q;n 1 29,94 30, n 31.Es ist y4 : y3 q 7 : 8 0,875;aus y3 y0 q2 folgt y0 10;45 cm.

*4 Die Differentialgleichung einer gedämpftenSchwingung lautet my t k _y tDy t 0.Schreiben Sie für die Werte m 0,05 kg, D 50 kg=s2, k 1,3 kg=s ein Programm, das iterativdie Zeit-Elongation-Funktion berechnet und zeich-net. Variieren Sie k.

Lösung:

y (t)ÿD y (t) k _y (t)m

,

_y (t) _y (tÿDt)y (tÿDt) Dt,y (t) y (tÿDt) _y (tÿDt)Dt

Startwerte mit D 50 N=m; k 1,3 kg=s;m 0,05 kg und Dt 0,01 s: y (0) 0,1 m;_y (0) 0; y (0)ÿ 100 sÿ2 ´ y (0)

–0,04–0,02

0,020,040,06

0,080,10

0,1 0,2 0,3 0,4 0,5 t

k = 1,3 kg/s

y t( )

0,02

0,04

0,06

0,08

0,10

0,1 0,2 0,3 0,4 0,5 t

k = 2,8 kg/s

y t( )

3.1.5 Beispiele harmonischer Schwingungen

1 Eine Kugel der Masse m 2,0 kg hängt an einemleichten Faden der Länge l 2,40 m (Schwerepen-del).a) Berechnen Sie die Periodendauer T für ein-

en Ort, an dem die Erdbeschleunigungg 9,81 m=s2 beträgt.

b) An einem anderen Ort misst man mit demsel-ben Pendel die Schwingungsdauer T 3,12 s.Wie groû ist dort die Erdbeschleunigung?

Lösung:a) T 2p

l=g

p 3,108 s.

b) g 4 p2

T2l liefert g 9,73 m=s2

2 Die Länge eines Sekundenpendels ± das ist einPendel, das für eine Halbschwingung eine Sekundebraucht ± beträgt am ¾quator l1 99,09 cm, am Pol

l2 99,61 cm und auf 45 Breite l3 99,35 cm.Berechnen Sie die zugehörigen Erdbeschleuni-gungen.

Lösung:Mit T 2 s und g 4 p2 l=T folgt:g ÈAquator 9,780 m=s2; gPol 9,831 m=s2;g45 9,805 m=s2.

3 Bei einem Federpendel sind f 8 Hz undD 380 N/m. Wie groû ist m?

Lösung:Mit T 1=f und T 2 p

m=D

pergibt sich

mD=(4p f 2) 150 g.

4 Bei einem Federpendel wird die Periodendauer Tdreimal so groû, wenn die angehängte Masse m umDm 50 g vergröûert wird. Wie groû ist die ur-sprüngliche Masse m?

Seite

113

3 Mechanische Schwingungen und Wellen70

Page 71: Metzler Physik

Lösung:Aus T 2 p

m=D

pund 3 T 2p

(mDm)=D

p

folgt 3mDm

m

r, mDm

8 6,25 g.

5 Wie groû wird die Periodendauer, wenn bei glei-cher Masse m zwei Federn mit den Konstanten D1

und D2 aneinander gehängt werden? (BerechnenSie zuerst die Federkonstante der Kombination.)

Lösung:Bewirkt die Kraft F bei den Federn mit den Feder-konstanten D1 und D2 die Verlängerungen y1 und y2,so werden die aneinander gehängten Federn mit denneuen Federkonstanten D um y1 y2 verlängert(Gewichtskraft der unteren Feder vernachlässigt).Aus F D (y1 y2), y1 F=D1 und y2 F=D2,folgt für die Federkombination 1=D1=D11=D2.

Einsetzen in T 2p

mD

rergibt T 2p

D1D2

D1 D2

r.

6 Eine an einer Feder hängende Kugel m 2,0 kg,die um 2,0 cm nach unten ausgelenkt und dann sichselbst überlassen wurde, schwingt mit der Fre-quenz f 4 Hz.a) Wie groû ist die Richtgröûe D der Feder?b) Wie weit dehnt sich die Feder, wenn die Kugel

vor Beginn der Schwingung angehängt wird?c) Wie groû ist die auf die Kugel wirkende Kraft in

den Umkehrpunkten der Schwingung?

Lösung:a) Es ist D 4p2 f 2 1,263 N=m.b) Aus y0 F=D folgt mit F mg eine Ausdeh-

nung von y0 1,55 cm (siehe Abb. 113.1).c) In den Umkehrpunkten wirkt die Kraft

F ÿDs 25,26 N; mit F m a und aw2 yund m 2 kg: F 25,26 kg.

*7 In einem U-Rohr konstanten Querschnitts befindetsich eine Flüssigkeitssäule der Gesamtlänge l. Be-rechnen Sie die Periodendauer der Schwingung,die entsteht, wenn man kurz in das eine Rohrendebläst. Zeigen Sie, dass die Periodendauer nur vonder Länge der Flüssigkeitssäule abhängt. (Hinweis:Betrachten Sie den Schwingungszustand, in demdie Flüssigkeitssäule in dem einen Rohr z. B. umdie Strecke y0 gestiegen, in dem anderen dann umdie Strecke y0 gesunken ist, und bestimmen Sie dierücktreibende Kraft.)

Lösung:Steigt die Flüssigkeitssäule auf der einen Seite umy0, so sinkt sie auf der anderen Seite um die gleiche

Strecke, und es wirkt als rücktreibende Kraft dieGewichtskraft der h 2y0 FlüssigkeitssäuleF Gmg rV g r g 2 A y0.Es ist D F=y0 2 A r g und man erhält mitm lAr (Masse der gesamten Flüssigkeitssäulemit der Länge l) die Periodendauer T 2 p

m=D

p

2 pl=(2g)

p.

Zusatzaufgaben

8 Ein mit Bleikugeln beschwertes Reagenzglas derGesamtmasse m mit einer Querschnittsfläche Aschwimmt in einer Flüssigkeit der Dichte r. Drücktman es tiefer in die Flüssigkeit und lässt es los, sovollführt es eine stark gedämpfte Schwingung.Zeigen Sie, dass es sich ohne Berücksichtigung derDämpfung um eine harmonische Schwingung han-delt, und berechnen Sie die Periodendauer. (Hin-weis: Die rücktreibende Kraft ergibt sich aus derDifferenz von Auftriebskraft und Gewichtskraft.)

Lösung:Beim schwimmenden Reagenzglas halten sichAuftriebskraft und Gewichtskraft das Gleichge-wicht. Verändert sich die Eintauchtiefe um y, soändert sich die Auftriebskraft um F y A r g. Alsoist die rücktreibende Kraft F der Auslenkung y pro-portional.

Aus D F=y und T 2 pm=D

pfolgt

T 2 pm=(Arg)

p

9 Man denke sich einen geraden, quer durch die Erdeund den Erdmittelpunkt verlaufenden Tunnel, inden man einen Körper der Masse m fallen lässt.Auf diesen Körper wirkt dann im Abstande r vomMittelpunkt der Erde stets nur die Gravitationskraftder Teilkugel vom Radius r.Zeigen Sie, dass unter der Annahme einer konstan-ten Dichte der Erde r 5,5 ´ 103 kg=m3 der Körperim Tunnel eine harmonische Schwingung ausführtund berechnen Sie die Schwingungsdauer.

Lösung:Die Masse der Teilkugel der Erde istm T 4

3 p r3 r. Die Gravitationsbeschleunigungdurch dies Teilkugel ist g gmT=r2 4

3 prg r.Damit ist die rücktreibende Kraft F mg= 4

3 prgmr proportional zu r, also zum Abstandvon der Ruhelage.Aus T 2p

m=D

pergibt sich

T 3p=rg

p1,41 h.

3.1 Schwingungen 71

Page 72: Metzler Physik

3.2.1 Überlagerung zweier harmonischer Schwingungen

Zusatzaufgaben

1 Von zwei Stimmgabeln der Frequenz 1700 Hz wirdeine durch Erwärmung verstimmt, sodass sich 10Schwebungen innerhalb von 8 Sekunden ergeben.Welche Frequenz hat die verstimmte Stimmgabel?Welche Frequenz hat der an- und abschwellendeTon?

Lösung:Die Frequenz einer Stimmgabel nimmt bei Erwär-mung ab. Die Schwebungsfrequenz ist fS f1ÿ f2,also f2 f1ÿ fS 1698,75 Hz.Die Frequenz des an- und abschwellenden Tonesbeträgt f 1699,375 Hz.

3.2.3 Die Energie des harmonischen Oszillators

1 a) Berechnen Sie die kinetische, die potentielleund die gesamte Energie der harmonischenSchwingung nach Aufgabe 1, S. 110 für die an-gegebenen Zeiten unter der Annahme, dass derschwingende Körper eine Masse von m 0,5 kgbesitzt.

b) Fertigen Sie eine Zeichnung der kinetischenund der potentiellen Energie als Funktion derZeit bzw. als Funktion der Auslenkung an.

Lösung:

a) n t in s Ekin in s Epot in s

012345678

00,250,50,7511,251,51,752

0,0246740,01233700,0123370,0246740,01233700,0123370,024674

00,0123370,0246740,01233700,0123370,0246740,0123370

b)

0,005

0,010

0,015

0,020

0,025Ekin potE

2 4 6 8 t in T/8

0,005

0,010

0,015

0,020

0,025Ekin potE

–0,10 –0,05 0,05 0,10 y

*2 Berechnen Sie die maximale kinetische und diemaximale potentielle Energie des Pendels für Auf-gabe 2, S. 110.

Lösung:Ekin 1

2 m v2 12 m (w y)2 1

2 Dy2 Epot 4 10ÿ4 J

3.2.4 Erzwungene Schwingungen

1 Geben Sie weitere Beispiele für Resonanzphäno-mene an.

Lösung:Mitschwingen von Teilen von Lautsprecherver-kleidungen bei bestimmten Frequenzen, starkesMitschwingen von Teilen von Schiffen bei be-stimmten Drehzahlen des Schiffsmotors usw.

2 Warum ist bei der Resonanzfrequenz und Phasen-differenz von Dò p

2 die Energieübertragung zwi-schen Erreger und Resonator besonders groû?

Lösung:Bei einer Phasendifferenz von p

2 wirkt vom Erregerauf den Resonator ständig eine Kraft, die die Be-wegung unterstützt. Kraft des Erregers und Ge-schwindigkeit des Resonators sind in Phase.

Seite

115

Seite

119

Seite

121

3 Mechanische Schwingungen und Wellen72

Page 73: Metzler Physik

3 Welche Bedingungen sollte bei Musikinstrumen-ten ein guter ¹Resonanzkörperª (z. B. Geige,Cello) hinsichtlich der Eigenfrequenzen erfüllen?

Lösung:Eigenfrequenzen des Resonanzkörpers sollten imhörbaren Bereich möglichst vermieden oder stark

gedämpft werden, damit nicht einzelne Töne vomInstrument besonders laut abgestrahlt werden. DerResonanzkörper soll möglichst alle Schwingungendes Instrumentes möglichst in gleicher Weise aufdie umgebende Luft übertragen.

3.2 Überlagerung von Schwingungen 73

Page 74: Metzler Physik

3.3.1 Lineare Wellen; die Wellengleichung

1 Während 12 Schwingungen innerhalb von 3 Se-kunden ablaufen, breitet sich eine Störung um3,6 m aus. Berechnen Sie Wellenlänge, Frequenzund Ausbreitungsgeschwindigkeit der Welle.

Lösung:Es ist T 0,25 s; c 1,2 m=s; f 4,0 Hz;l c=f 0,3 m.

2 Gleiche Pendel sind in einer Reihe im Abstand von0,4 m aufgestellt. Sie werden nacheinander im zeit-lichen Abstand von 0,5 s angestoûen, sodass das 1.und 5., das 2. und 6. usw. Pendel phasengleichschwingen. Mit welcher Geschwindigkeit, Wellen-länge und Frequenz läuft die Welle über die Pen-delkette?

Lösung:Umformungen liefernEs ist l 1,6 m; f 0,5 Hz; T 2 s; l f 0,8 m=s.

*3 Zeigen Sie an der Wellengleichung, dass sich für tund tT bzw. für x und x l gleiche Schwin-gungszustände ergeben.

Lösung:y (x, t T) y sin

2 p

tT

Tÿ x

l

y sin

2 p

tTÿ x

l

2 p

y sin

2 p

tTÿ x

l

.

y (x l, t) y sin

2p

tTÿ x l

l

y sin

2 p

tTÿ x

l

ÿ 2 p

y sin

2 p

tTÿ x

l

.

*4 Weisen Sie nach, dass sich aus der Wellenglei-chung deduktiv die Phasengeschwindigkeit, mitder sich ein Schwingungszustand bewegt, zuvPh l=T ergibt.

Lösung:Die Gleichung einer linearen Welle heiût y (x,t) y sin [2 p (t=T ÿ x=l). Zum Zeitpunkt t 0 istam Ort x 0 die Phase der Schwingung ò 0.Dieselbe Phase ist am Ort x nach der Zeit t ange-kommen, wenn gilt:

tTÿ x

l

0: Daraus folgt

xt l

T c.

*5 Eine harmonische Schwingung y t y sin w tbreite sich vom Nullpunkt als transversale Störunglängs der x-Achse mit der GeschwindigkeitvPh 7,5 mm=s aus. Es sei weiter y10 cm,w p

2 Hz.a) Berechnen Sie die Periodendauer T, die Fre-

quenz f und die Wellenlänge l.b) Wie heiût die Wellengleichung?c) Zeichnen Sie maûstäblich das Momentanbild

der Störung nach t1 4 s, t2 6 s und t3 9 s.d) Wie heiûen die Schwingungsgleichungen für

die Oszillatoren, die an den Orten x1 5,25 cmbzw. x2 7,5 cm von der Störung erfasst wer-den?

Lösung:a) Aus w 2p f folgt T 2p=w 4,0 s und

f 0,25 Hz und daraus l cT 0,03 m.b) Die Wellengleichung lautet

y (x, t) 0,01 m ´ sin

2p

t

4 sÿ x

0,03 m

c) Den Zeichnungen liegen die Funktionen

y (x; t1) 0,01 m ´ sin

2p

4 s4 sÿ x

0,03 m

ÿ 0,01 m ´ sin

2 p

x0,03m

y (x; t2) 0,01 m ´ sin

2p

6 s4 sÿ x

0,03 m

0,01 m ´ sin

2 p

x0,03 m

y (x; t3) 0,01 m ´ sin

2 p

9 s4ÿ x

0,03 m

0,01 m ´ cos

2 p

x0,03 m

.

–0,005

–0,010

0,005

0,010

0,01 0,02 0,03 0,04 0,05 0,06

y ( )t, x

x

t12 3t t

Seite

123

3 Mechanische Schwingungen und Wellen74

Page 75: Metzler Physik

d) An der Stelle x1 5,25 cm beginnt die Störungnach der Zeit t1 x1=c 7,0 s mit der Schwin-gung; oder aus der Wellengleichung

y (x1, t) 0,01 m ´ sin

2p

t

4 sÿ 0,0525 m

0,03 m

0,01 m ´ sin

p

2

tsÿ 7

und für die Stelle x2

y (x2, t) 0,01 m ´ sin

2 p

t

4 sÿ 0,075 m

0,03 m

0,01 m ´ sin

p

2

tsÿ 10

.

3.3.3 Zwei- und dreidimensionale Wellen

1 Bei stroboskopischer Beleuchtung beobachtet mandie scheinbar stillstehenden kreisförmigen Wellenin der Wellenwanne und misst zwischen dem ers-ten und sechsten Wellenberg einen Abstand von10 cm.a) Wie groû ist die Wellenlänge?b) Wie groû ist die Wellengeschwindigkeit, wenn

die Schwingungsdauer des Erregers T 0,25 sbeträgt?

Lösung:a) l 2 cm b) c l=T 8 cm=s

*2 Die Amplitude p des Schalldrucks hängt mit derDichte r der Luft, der Schallgeschwindigkeit v,der Frequenz f und der Amplitude y der Schwin-gung eines Luftmoleküls, das von der Schallwelleerfasst wird, über die Beziehung p 2p f v r y zu-sammen. Ein Ton der Frequenz f 400 Hz ist (bei20 C) eben noch zu hören, wenn seine Schall-druckamplitude p 8 10ÿ5 N=m2 80 mPa be-trägt.a) Wie groû ist die Amplitude der Schwingung?b) Bestimmen Sie mithilfe der ersten Ableitung

der Wellengleichung die maximale Geschwin-

digkeit des Teilchens, die Amplitude derSchallschnelle.

Lösung:a) Mit v 344 m=s und rLuft 1,29 kg=m3 wird

y p= (2 p f v r) 7,17 ´ 10ÿ11 m 72 pm.Diese Länge liegt in der Gröûenordnung derKernabstände in einem Molekül.

b) Die (partielle) Ableitung vony y sin [2 p (t=T ÿ x=l)]nach der Zeit ergibt die Schallschnelle vs

vS 2 p

Ty cos

2p

tTÿ x

l

, also vS

2 p

Ty.

Für p 8 10ÿ5 N=m2 ist dannvs 1,80 10ÿ7 m=s.

Zum Vergleich: Bei Schalldrücken vonp 101 N=m2 nähert man sich der anderenGrenze des Hörbereichs, der Schmerzgrenze.Für diesen Schalldruck ergeben sich um denFaktor 106 gröûere Amplituden und Schall-schnellen, nämlich

y 10ÿ4 m und vS 110 m=s.

3.3.4 Der Doppler-Effekt

1 Welchen Ton hört ein Beobachter, an dem einepfeifende Lokomotive (1500 Hz) mit einer Ge-schwindigkeit von 120 km/h vorbeifährt, vorherund nachher? Die Schallgeschwindigkeit betrage340 m/s.

Lösung:Beim Nähern des Zuges ist

fE f

1ÿ u=vPh 1663 Hz, beim Entfernen

fE f

1 u=vPh 1366 Hz.

2 Die Hupe eines stehenden Autos besitze eine Fre-quenz von 440 Hz. Welche Frequenz nimmt ein Au-tofahrer wahr, der sich mit 100 km/h nähert (ent-fernt)? Die Schallgeschwindigkeit betrage 340 m/s.

Lösung:Beim Nähern des Empfängers istfE f (1 u=vPh) 476 Hz, beim EntfernenfE f (1ÿ u=vPh) 404 Hz.

3 Eine Pfeife mit der Frequenz 400 Hz wird mit3 Umdrehungen je Sekunde auf einer Kreisbahnmit dem Radius 1 m herumgeschleudert. Zwischen

Seite

127

Seite

129

3.3 Entstehung und Ausbreitung von Wellen 75

Page 76: Metzler Physik

welchen Werten schwankt die Frequenz des Tones,den ein ruhender Beobachter registriert?

Lösung:Die maximale Geschwindigkeit auf den Beobach-ter zu bzw. von ihm weg beträgt v 2p r=T 18,85 m=s. Mit den Formeln aus der Aufgabe 1ergibt sich fzu 423 Hz bzw. fweg 379 Hz.

*4 Leiten Sie eine Formel für die Frequenzänderungfür den Fall her, dass sich Sender und Empfängerbewegen.

Lösung:a) Sender und Empfänger bewegen sich in entge-

gengesetzten Richtungen aufeinander zu: DieBewegung des Senders führt zu einer Verkür-zung der Wellenlänge auf lM l (1ÿ uS=vPh),die Bewegung des Empfängers führt zu einerErhöhung der Frequenz auf f M (vPh uE)=lM.Einsetzen des Terms für lM in den Term für f Mergibt mit

l vPh=f : f M fvPh uE

vPhÿ uS:

b) Sender und Empfänger bewegen sich in entge-gengesetzten Richtungen voneinander weg:Rechnung wie unter a), jedoch mit entgegenge-setzten Rechenzeichen.

c) Der Empfänger bewegt sich hinter dem Sen-der her: Die Bewegung des Senders führt zueiner Vergröûerung der Wellenlänge auf

lM l(1 uS=vPh), die Bewegung des Empfän-gers führt zu einer Erniedrigung der Frequenzauf f M (vPhÿ uE)=lM. Einsetzen des Termsfür lM in den tern für f M ergibt mit

l vPh=f : f M fvPh uE

vPh uS:

d) Der Sender bewegt sich hinter dem Empfängerher: Rechnung wie unter c) jedoch mit entge-gengesetzten Rechenzeichen.

Zusatzaufgabe

5 EinRennwagenfährtaneinemZuschauervorbei, istdie Frequenz des Motors f1 288 Hz; beim Entfer-nen hört der Zuschauer den Motor mit der Frequenzf2 178 Hz.Berechnen Sie bei einer Schallgeschwindigkeit von340 m=s die Geschwindigkeit des Rennwagens unddie Drehzahl des Motors.

Lösung:Aus den Gleichungen fE1

f1ÿ u=vPh

und

fE2 f

1ÿ u=vPhfolgen die Gleichungen

fE1 vPhÿ fE1 u f und fE2 vPhÿ fE2 u f , aus denen

sich ergibt: u f1ÿ f2

f1 f2v 289 km=h und

f 220 Hz 13 200 min±1.

3.3.5 Phasen- und Gruppengeschwindigkeit; Dispersion

1 Eine Stimmgabel der Frequenz f beginne zur Zeitt 0 zu schwingen und werde nach einem Zeitin-tervall 2 Dt angehalten. Die entstehende Wellen-gruppe der Länge 2Dx enthält ca. N mittlere Wel-lenlängen. Da man am Anfang und am Ende derWellengruppe nicht genau entscheiden kann, obnoch eine halbe Wellenlänge vorhanden ist odernicht, beträgt die Ungenauigkeit jDNj 1.a) Wie hängen N, f und Dt zusammen?b) Drücken Sie die Wellenlänge durch 2Dx und N

aus.c) Zeigen Sie, dass sich aus diesem Sachverhalt

die Formel Df Dt 12 für die akustische Un-

schärfe ergibt.

Lösung:

a) f N2Dt

b) l 2 DxN

c) f Df N 12Dt

also Df 12Dt

*2 Die Dispersionsrelation für Wasserwellen heiûtw2 g k mit k 2 p=l. Bestimmen Sie mithilfeder Beziehung vGr dw=dk für Wellen der Fre-quenz 1 Hz die Gruppengeschwindigkeit.

Lösung:

vGr dwdk d(

g kp

)dk

gp

2kp mit

kp w

gp folgt

vGr g

2w 0,78 m=s.

Seite

130

3 Mechanische Schwingungen und Wellen76

Page 77: Metzler Physik

3.4.1 Interferenz zweier Kreiswellen

1 Zeichnen Sie eine Interferenzfigur ähnlichAbb. 133.1, die entsteht, wenn der Erreger L2 ge-genüber L1 mit der Phasenverschiebung Dò p

schwingt. (Zeichnung L1L2 7,0 cm; l 2,0 cm;Wellenberge durchgezogen, Wellentäler gestri-chelt; Interferenzstreifen durch den Phasenunter-schied kennzeichnen.)

Lösung:In Abb. 133.1 werden die Hyperbeln der Maximamit denen der Minima vertauscht.

*2 Zwei phasengleich schwingende Wellenerreger er-zeugen Kreiswellen der Wellenlänge l. Ihr Ab-stand beträgt die fünffache Wellenlänge.a) Welchen Winkel bildet der ¹gerade Teilª des

Interferenzmaximums 1. Ordnung mit derSymmetrieachse?

b) Wie viele Interferenzhyperbeln (Interferenz-maxima) werden erzeugt?

c) Lässt sich eine Gesetzmäûigkeit zwischen derAnzahl der Interferenzhyperbeln und dem Ab-stand der Wellenerreger bei vorgegebener Wel-lenlänge aufstellen?

Lösung:a) Für das erste Interferenzmaximum ist Ds l

und mit d 5l wird tan a a=eDs=d sin a 1=5, a 11,3. (Für kleine Winkel isttan a sin a).

b) Auûer den Interfe-renzmaxima aufder Mittelsenkrech-ten zu L1 L2 durchL0 mit Ds 0 erhältman 4 Interferenz-hyperbeln (mit jezwei Hyperbe-lästen links undrechts der Mittel-senkrechten). Dennfür einen beliebigen Punkt auûerhalb der Sym-metrieachsen kann die Differenz der Entfer-nung zu den Erregerzentren nur l, 2 l, 3 l,und 4 l betragen.

c) Zählt man die Mittelsenkrechte als entarteteHyperbel hinzu, so ergeben sich soviele Inter-ferenzhyperbeln, wie der auf eine ganze Zahlabgerundete Quotient d=l angibt.

a

e

P0 P

α∆

αs

d

L1 0 2L L

*3 Ist d L1L2 der Abstand der beiden phasengleichschwingenden Erreger, Ds der Gangunterschied ei-

nes weit von L1 und L2 entfernten Punktes und ader Winkel zwischen der Symmetrieachse von L1

und L2und der Richtung von der Mitte L0 zwischenden Erregern zum Punkt P1, so gilt entsprechendAbb. 133.1:a) sin aDs=d und tan a PP0=L0 P0 an=e

oder für kleine Winkel Ds=d an=e;b) für Interferenzmaxima sin a nl=d oder

n l an d=e;c) für Interferenzminima sin a 2nÿ 1 l=2d

oderÿnÿ 1

2

l an d=e.

Begründen Sie die Beziehungen anhand der Zeich-nung.

Lösung:Siehe die Zeichnung zur Lösung von Aufgabe 2.

Zusatzaufgaben

4 Berechnen Sie die Gleichung der entstehendenWelle, die sich bei der Interferenz zweier gleich-laufender linearer Wellen gleicher Frequenz mitden Amplituden y1 y und y2 1

2 ya) bei einem Gangunterschied von Ds 0,b) bei einem Gangunterschied von Ds l=2 ergibt.c) Konstruieren Sie beide Überlagerungen mit

y 2 und y 6 cm zur Zeit t 5T (Moment-aufnahme).

Lösung:Aus y1 y sin [2 p (t=T ÿ x=l)] undy2 1

2 y sin [2 p [(t=Tÿ x=l)ÿDs=l]] ergibt sich füra) Ds 0: y 3

2 y sin [2 p (t=T ÿ x=l)]b) Ds l=2: y 1

2 y sin [2 p (t=T ÿ x=l)]c)

–3

–2

–1

1

2

3

5 10 15 20 25 30 x

y1 2( )x +y ( )x ∆s = 0 ∆s = λ2

5 Untersuchen Sie mathematisch die Interferenzzweier gleichlaufender linearer Wellen gleicherFrequenz und Amplitude mit dem Gangunter-schied DsDx. (Man setze in die Wellenglei-chung einmal x und ein zweites Mal xDx ein.)Für welchen Gangunterschiede ergibt sich maxi-male Verstärkung bzw. völlige Auslöschung?

Seite

133

3.4 Wechselwirkungen von Wellen 77

Page 78: Metzler Physik

Lösung:Aus y y sin

2p

tTÿ x

l

y sin

2p

tTÿ xDx

l

folgt y1 2 cos pDxl

sin

2 p

tTÿ x

l

.

Die Cosinusfunktion gibt die Amplitude der Über-lagerungswelle an. Die Amplitude ist maximal fürpDx=l np, also für Dx nl; sie ist null fürpDx=l (2n 1)p=2, also fürDx (2n 1)l=2, n 0, 1.

3.4.2 Das Huygens'sche Prinzip

Zusatzaufgabe

1 Konstruieren Sie für 8 Erregerpunkte, die gleich-mäûig auf einer Kreislinie angeordnet sind, dieÜberlagerung der Elementarwellen zu neuen Wel-lenfronten.

Lösung:Zeichnung in Anlehnung an Abb. 134.1 d).

3.4.3 Reflexion und Berechnung ebener Wellen

1 In einer Wellenwanne läuft eine Welle von einemseichten Bereich in ein Gebiet mit tieferem Wasserunter dem Einfallswinkel von 45 und dem Bre-chungswinkel von 60.a) Bestimmen Sie das Verhältnis der Geschwin-

digkeiten in beiden Teilen der Wellenwanne.b) Bestimmen Sie die Geschwindigkeit im flachen

Teil, wenn sie im tiefen 25 cm/s ist.

Lösung:

a)vPh1

vPh2 sin 45

sin 60 0,816

b) vPh1 0,816 ´ vPh2 20,4 cm=s

2 Wasserwellen bewegen sich in tiefem Wasser mitder Geschwindigkeit v1 34 cm=s. Sie treffen un-ter dem Winkel a 60 auf die Grenzlinie zu ei-nem flacheren Teil, wo sie sich mit v2 24 cm=sbewegen. Erhöht man die Frequenz ein wenig, sosinkt die Geschwindigkeit im tieferen Teil aufv1 32 cm=s.a) Berechnen Sie in beiden Fällen den Brechungs-

winkel.b) Die Wellenlänge im tieferen Teil beträgt im er-

sten Versuch l 1,7 cm. Wie groû ist die Wel-lenlänge im flacheren Teil und welche Fre-quenz hatte die Welle?

Lösung:

a) AusvPh1

vPh2 sin a1

sin a2folgt a2 37,7.

Im flachen Wasser ändert sich die Geschwindigkeit

nicht, daher folgt ausvPh1

vPh2 sin a1

sin a2: a2 40,5.

b) Die Frequenz ist in beiden Bereichen gleich,also f2 f1 v1=l2 20 Hz und l2 1,2 cm.

3 Konstruieren Sie nach dem Huygens'schen Prinzipa) die Reflexion von Kreiswellen, die von einem

Erregerzentrum Z ausgehen, an einem geradenHindernis (Reflexionsgerade). Zeichnen Siedazu mehrere Kreisbögen um Z mit Abstandvon 1 cm, die die Reflexionsgerade schneiden.Zeigen Sie, dass der Mittelpunkt Z0 der reflek-tierten Wellenfronten das Spiegelbild von Z ander Geraden ist.

b) Führen Sie die Konstruktion auch für ebeneWellen aus, die an einem Hohlspiegel (Kreis-linie mit r 8 cm) reflektiert werden.

Lösung:

a)

+

+

Z

Z′

Seite

134

Seite

136

3 Mechanische Schwingungen und Wellen78

Page 79: Metzler Physik

b)

M

3.4.5 Stehende Wellen; Eigenschwingungen

1 Auf einer einseitig eingespannten Blattfeder wirddurch einen Elektromagneten, der von Wechsel-strom durchflossen wird, die 1. Oberschwingungerzeugt. Wie groû ist die Phasengeschwindigkeit,wenn die Feder l 9 cm lang ist?

Lösung:Die eiserne Blattfeder wird vom Elektromagnetenmit der Frequenz f 100 Hz in Schwingungen ver-setzt. Bei der ersten Oberschwingung ist die Längel 0,09 m 3 l=4. Daraus ergibt sich vPh 12m=s.

2 Zeichnen Sie vier Momentbilder im AbstandDt T=8 von einer Welle und von ihrer am freienEnde reflektierten Welle sowie der sich ergeben-den stehenden Welle.

Lösung:Zeichnung entsprechend Abb. 139.1 a).

3.4.7 Stehende Schallwellen

1 Ein teilweise mit Wasser gefülltes Rohr(Abb. 142.2) zeigt Resonanz bei einer Längel1 24,9 cm der Luftsäule, dann bei l2 41,5 cm.Die Schallgeschwindigkeit in Luft beträgtvPh 340 m=s.a) Zeichnen Sie die stehende Geschwindigkeits-

welle und Druckwelle für die Länge l2 ein.b) Welche Wellenlänge und Frequenz hat der aus-

gesandte Ton? Welche (Ober-)Schwingungenwurden gemessen?

Lösung:a) Die stehende Geschwindigkeitswelle hat eine

Länge von 5l=4, am festen Ende einen Knoten,am offenen Ende einen Bauch; bei der stehen-den Welle für den Schalldruck ist es umgekehrt(s. Abb. 141.2).

b) Aus 41,5 cm ÿ 24,9 cm l=2 folgtl 33,2 cm und f 1024 Hz.

2 Ineinem 40cmlangen,beiderseitsoffenen Glasrohrbilden sich bei der Frequenz f 1 1222 Hz an vier

Stellen (einschlieûlich beider Enden) und beif 2 1634 Hz an fünf Stellen jeweils in gleichemAbstand Bäuche der Kundt'schen Staubfiguren aus.a) Zeichnen Sie für beide Fälle die stehende Ge-

schwindigkeits- und Druckwelle.b) Berechnen Sie die Schallgeschwindigkeit als

Mittelwert aus beiden Messungen.

Lösung:a) Die Kundt'schen Staubfiguren bestehen aus

zwei ganzen Bäuchen und je einem halbenBauch an den offenen Enden, sodass die Längel des Rohres gleich 3l=2 ist (s. Abb. 142.1). DieDarstellung der Geschwindigkeitswelle ent-spricht der Darstellung der 2. Oberschwingungfür einen Wellenträger mit zwei freien Enden(s. Abb. 140.1). Die stehende Druckwelle hatgegenüber der Geschwindigkeit Knoten undBäuche vertauscht.

b) Mit l1 2 l=3 ist vPh1 325,9 m=s bzw. mitl2 l=2 ist vPh2 326,8 m=s, also vPh 326,3 m=s.

Seite

140

Seite

143

3.4 Wechselwirkungen von Wellen 79

Page 80: Metzler Physik

3 Ein Messingstab der Länge l 30 cm, dessen Endemit Stempel in eine Kundt'sche Röhre gestecktist, wird zu Schwingungen in der Grundfrequenzerregt. Es ergeben sich in der Röhre Kundt'-sche Staubfiguren im Abstand von Ds 3,0 cm.Die Schallgeschwindigkeit in Luft beträgtvPh 340 m=s.a) Berechnen Sie die Schallgeschwindigkeit in

Messing.b) Zeichnen Sie die stehende Welle des Messing-

stabes und die der Geschwindigkeitswelle inder mit Luft gefüllten Röhre.

Lösung:a) Mit l=2 0,03 m ergibt sich f 5,7 kHz. Wenn

der an beiden Seiten freie Messingstab in derGrundschwingung schwingt, ist seine Längel l=2. Also vPh;Messing 3,4 km=s.

b) Auf dem Messingstab hat die stehende Welle inder Mitte einen Knoten, an den Enden je einenBauch (s. Abb. 140.1). Die stehende Geschwin-digkeitswelle in der Kundt'schen Röhre ent-spricht je nach deren Länge und dem anderenEnde den Darstellungen in Abb. 140.1.

4 Eine offene Pfeife der Länge l1 und eine gedacktePfeife (gedackt: ein geschlossenes Ende) seien aufgleichen Grundton abgestimmt. (Zungen- und Lip-penpfeifen besitzen an dem Ende, an dem sich dieMetallzungen oder die Schneide befindet, ein offe-nes Ende.)a) Welcher Zusammenhang besteht zwischen den

Längen beider Pfeifen?b) Welche Töne kann man zu dem Grundton, der

Prime, durch so genanntes ¹Überblasenª (soheiût das Erzeugen der Oberschwingungen)mit der offenen und welche mit der gedacktenPfeife erzeugen?

Lösung:a) Bei der offenen Pfeife liegt an beiden Enden

ein Bauch der Geschwindigkeitswelle, alsol=2 l1, bei der gedackten Pfeife liegt amfesten Ende ein Knoten, also l=4 l2. Darausfolgt l2 l1=2.

b) Für die erste Oberschwingung gilt l1 l1, alsof1 2 f0 bzw. bei der gedackten Pfeifel2 4 l2=3, also f2 3 f0.

3 Mechanische Schwingungen und Wellen80

Page 81: Metzler Physik

4.1.1 Die Temperatur und die Gasgesetze

1 Ein Gas nimmt bei 20 C ein Volumen von 3 l ein.Auf welche Temperatur muss das Gas bei gleichemDruck erwärmt werden, damit es 4 l, das doppelte,das dreifache Volumen einnimmt?

Lösung:isobar, also V2=T2 V1=T1; T2 T1 V2=V1 293 K 4 (6; 9)

3 390,7 K (586 K; 879 K)

2 Während einer Unterrichtsstunde steigt im Physik-raum mit den Abmessungen 12 m, 5 m und 4 m dieTemperatur von 18 C auf 20 C. Wie viel Luftentweicht?

Lösung:DV V2ÿV1 T2=T1ÿ 1V1

293=291ÿ 1 12 5 4 m3 1,649 m3

3 Ein abgeschlossenes Luftvolumen steht bei 20 Cunter einem Druck von 1 bar. Bei welcher Tempe-ratur beträgt der Druck 0,5 bar (1,3 bar, 2 bar, 3 bar)?

Lösung:isochor, also p2=T2 p1=T1; T2 T1 p2=p1 0;5 bar

1 bar 293 K 146,5 K 586 K, 879 K

4.1.2 Der atomistische Aufbau der Stoffe

1 Zum Ölfleckversuch: Verdampft man 1 g Triolein,so stellt man fest, dass der Dampf unter Normalbe-dingungen ein Volumen von 78,5 cm3 einnimmt.17 Tropfen einer Ölsäure-Benzinlösung (0,1Vol.-%) ergeben 1 cm3. Lässt man davon einenTropfen auf die mit Bärlappsamen bestreute Was-seroberfläche fallen, so entsteht ein Kreis mit 31 cmDurchmesser. Berechnen Sie aus diesen Angabendie Avogadro-Zahl (Dichte von Triolein: r0,89 g=cm3). Welches mittlere Volumen entfälltauf ein einzelnes Atom des Ölmoleküls (Triolein:C18H34O2)?

Lösung:V 1

17 cm3 0,001 15,5 cm2 p dd 7,79 10ÿ8 cm; V1 d3 4,734 10ÿ22 cm3.N V=V1 1,243 1017.Masse m V r 5,235 10ÿ5 g1 g Triolein:Stoffmenge n1 78,5 cm3 mol=22,4 103 cm3 3,504 ´ 10ÿ3 molHier vorliegende Stoffmenge:n3,504 10ÿ3 5,23510ÿ5 mol1,835 10ÿ7 mol

NA Nn 6,78 1023

mittl. Volumen pro Atom: VA V1=18 34 2 8,77 10ÿ24 cm3 8,77 10ÿ30 m3

) mittl. Kantenlänge a VA

3p 2,06 10ÿ10 m).

2 Berechnen Sie die absolute Masse eines Sauer-stoff-, Eisen-, Uranatoms.

Lösung:

m ANA 16,0 g (55,8 g; 238,0 g)

NA

2,66 10ÿ23g 9,27 10ÿ23g; 3,95 10ÿ22g

3 Berechnen Sie mithilfe der Avogadro-Zahl dasVolumen eines Wassermoleküls. (m(H2O) 18;r 1 g=cm3.)Vergleichen Sie dieses Volumen mit dem Raum-anteil, der auf ein einzelnes Gasmolekül bei Nor-malbedingungen entfällt.

Lösung:V 1 cm3=N 1 cm3 18=NA 2,99 10ÿ23cm3

(Flüssigkeit)V 22,4 103cm3=N 22,4 103cm3 18=NA

6,70 10ÿ19 cm3 (Gas)

4 Wie viele Atome enthalten 12 g Kohlenstoff, 18 gSauerstoff, 27 g Aluminium? Welche Masse haben3 ´ 1012 Uranatome?

Lösung:Da es immer 1 mol ist, folgt N NA;

mu 3 1012 238 g=NA 1,185 10ÿ9g

5 Welche Masse haben die Stoffmengen der folgen-den Teilchenarten: 0,3 mol CO2-Moleküle; 0,2 molO2-Moleküle; 1 mol N-Atome; 1 mol C6H12O6-Mo-leküle?

Lösung:m 0,3 12 2 16g 13,2 g;

0,2 2 16 g 6,4 g; 14 g;6 12 12 6 16 g 180 g.

Zusatzaufgaben

6 Wie viel Gramm Argongas enthält ein 300 cm3 gro-ûer Glühlampenkolben, wenn der Innendruck bei15 C 2,6 mbar beträgt?

4.1 Grundlagen

Seite

148

Seite

151

4.1 Grundlagen 81

Page 82: Metzler Physik

Lösung:

Es ist n p VR T

; mit V 3 ´ 10±4 m3, p 2,6 ´ 102 Pa

und T 288,15 K wird n 3,26 ´ 10±5 mol und mitMr 39,95 folgtm n Mr kg1,30 10ÿ6 kg 1,3 mg.

7 In einer Stahlflasche von 10 l Inhalt steht ein Sauer-stoffgas unter einem Druck von 200 bar. Wie vielLiter Sauerstoff kann man der Flasche bei gleicherTemperatur unter normalem Luftdruck entneh-men?

Lösung:Nach Boyle-Mariotte V2 p2 V1 p1 wird mitV1 10 l, p1 200 bar, p2 1 bar V2 2000 l; eskönnen 1990 l entnommen werden.

8 Wieviel Teilchen enthält 1 cm3 des idealen Gasesbei der Temperatur 20 C und dem Druck 10±8 bar?

Lösung:Es ist n p V=RT; mit p 10ÿ3 Pa, V 10ÿ6m3

und T 293,15 K wird n 4,103 10ÿ13mol undN nNA 2,47 1011.

9 Welches Volumen nehmen 7 kg Stickstoffgas (N2)bei Zimmertemperatur (20 C) unter einem Druckvon 100 bar ein?

Lösung:Es ist V n R T=p; mit T 293,15 K, n 250 molund p 107 Pa wird V 6,09 ´ 10±2 m3 60,9 l.

4 Wärmelehre82

Page 83: Metzler Physik

4.2.2 Kinetische Gastheorie und Molekülbewegung

1 Ein Wasserstoffgas hat die Temperatur qÿ100 C.a) Wie groû ist die mittlere Geschwindigkeit eines

H2-Moleküls (Vereinfachung v2 v2)?b) Wie groû sind die mittlere kinetische Energie

und der mittlere Impuls eines H2-Moleküls?c) Welcher Impuls wird von einem Teilchen beim

senkrechten elastischen Stoû auf die Wandübertragen?

d) Welche Energie steckt in 1 mol des Gases?e) Berechnen Sie die verschiedenen Gröûen in den

Teilaufgaben (a) bis (d) für die Temperaturq 1000 C

Lösung:

a) 12 m v2 3

2 k T ; v2 3 k Tm 3 k T NA

2 10ÿ3 kg;

v1470 m=s

b) Ekin 32 k T 3,586 10ÿ21 J

pmv 210ÿ3kg=NA v 4,88 ´ 10±24 kg m=s

c) Dp 2 p 9,76 10ÿ24N sd) E NA E NA 52 k T 3,60 103 J

e) v 3986ms

; Ekin 2,637 10ÿ20 J;

p 1,32 10ÿ23 kg m=s; Dp 2,65 10ÿ23;

E 2,65 104 J.

2 Ein Volumen von 1000 cm3 enthält 3,24 ´ 1020 Teil-chen eines einatomigen idealen Gases mit derEnergie 6 J. Berechnen Sie Druck und Temperaturdes Gases.

Lösung:E 6 J=3,24 1020 3

2 k T ; T 894,2 K

pV 23 N Ekin; p 2

3 N Ekin=V 23 6 J=10ÿ3 m3

4 103pa 0,04 bar

3 Welche Temperatur hat ein Gas, das beim Druckvon 10ÿ8 mbar 108 Teilchen pro cm3 enthält?

Lösung:p V=T n R; T p V=n R p V NA=N NA k p V=N k 10ÿ8 10ÿ3 10ÿ5 Pa 10ÿ6m3=108 kT 724,3 K

4 Welche Temperatur hat ein Sauerstoffgas, wenndie mittlere Geschwindigkeit der O2-Molekülev 540 m=s beträgt?

Lösung:32 k T 1

2 m v2;

Tm v2=3 k32 10ÿ3kg v2=3k NA374,1 K

5 Unter Normbedingung ist in einem Stickstoffgasdie mittlere freie Weglänge l 58 nm. BerechnenSie die mittlere Zeit zwischen zwei aufeinanderfolgenden Stöûen eines N2-Moleküls.

Lösung:v2 3 k T=m 3 k T NA=2 14 10ÿ3 kg;v 493,3 m=s; Dt l=v 1,18 10ÿ10 s

Seite

155

4.2 Die kinetische Gastheorie 83

Page 84: Metzler Physik

4.3.1 Wärmeenergie und innere Energie

1 200 g Aluminiumschrot (cAl 0,9 J/(g K)) wer-den in kochendem Wasser erhitzt und anschlie-ûend in ein Thermogefäû mit 300 g Wasser(cWasser 4,18 J/(g K)) von 20 C gegeben. WelcheTemperaturerhöhung stellt sich ein?

Lösung:0,9 J=(g K) ´ 200 g (100 Kÿ qm) 4,18 J=(g K) ´ 300 g (qmÿ 20 K)qm 30C

2 a) Wie ändert sich die thermische Energie einesWasserbeckens mit den Abmessungen 10 m;4 m; 2 m, wenn die Temperatur um 1 C sinkt?

b) In welche Höhe könnte man mit dieser Energieeinen Körper der Masse 5000 kg auf der Erdeheben?

Lösung:a) V 80 m3; m 8 104 kg; DE c m Dq

4,18 J=(g K) ´ 8 ´ 107 g ´ 1 K 3,344 ´ 108 Jb) DEm g h; h 6818 m

3 Welche Energie wird zum Verdampfen von 1 kgEis der Temperatur qÿ10 C benötigt? (Spezifi-sche Wärmekapazität von Eis: cEis 2,05 J=(g K),von Wasser cWasser 4,18 J=(g K), Schmelzwärmevon Eis: QS 333,5 J=g, VerdampfungswärmeQV 2257 J=g)

Lösung:E1 2,05 J=(g K) ´ 1000 g ´ 10 K 20 500 J;ES 333,5 J=g ´ 1000 g 333 500 J;E2 4,18 J=(g K) ´ 1000 g ´ 100 K 418 000 K;EV 2257 J=g ´ 1000 g 2 257 000 J;E 3,029 ´ 106 J

4.3.2 Der erste Hauptsatz der Wärmelehre

1 In einer beidseitig mit einem Stopfen verschlosse-nen Pappröhre befinden sich 0,8 kg Bleischrot. Aneinem Stopfen kann die Temperatur des Bleisgemessen werden. Wird die aufrecht gehalteneRöhre schnell um 180 gedreht, so fällt das Bleiim Innern der Röhre herunter. Bei 25-maligem Um-drehen stellt man eine Temperaturerhöhung vonDT 1,8 K fest. Berechnen Sie die zugeführte me-

chanische Energie bei einer Fallhöhe von 1 m so-wie die thermische Energie, wenn die spezifischeWärmekapazität von Blei cBl 0,128 J=g Kbeträgt.

Lösung:Emech 25 ´ 0,8 kg ´ 9,81 m=sÿ2 ´ 1 m 196,2 J;Etherm 0,128 J=(g K) ´ 800 g ´ 1,8 K 184,3 J

4.3.6 Wärmekraftmaschine, Wärmepumpe und Kältemaschine

1 Ein Kühlschrank von 150 l Inhalt nimmt eine elek-trische Leistung von 150 W auf. Nach einer Fül-lung mit 75 kg Lebensmitteln mit einer Temperaturvon 20 C, die im Wesentlichen aus Wasser(cW 4,18 kJ/(kg K)) bestehen, wird er auf eineTemperatur von 5 C eingestellt.a) Mit welchem technischen Wirkungsgrad arbei-

tet der Kühlschrank, wenn die Kühlplatten imInnern 0 C und der Wärmetauscher auûen50 C haben?

b) Welche Wärmeenergie muss den Lebensmit-teln entzogen werden?

c) Wie lange würde bei einer Zufuhr dieserWärmeenergie mit einer Heizung von 150 Wdie Temperaturerhöhung von 5 C auf 20 Cdauern?

d) Woran liegt es, dass das Herunterkühlen miteinem Kühlschrank derselben Leistung nur ca.1,5 h dauert?

Lösung:a) h T2=T1ÿT2 273=323ÿ 273 5,46b) Q 75 103 kg 4,18 J=(g K) ´ 15 K 4,7 ´ 106 Jc) tQ=P 4,7 ´ 106 J=150 W 522,5 mind) Der theoretische Wirkungsgrad der Kältema-

schine ist h 5,46. Um die Energie Q4,7 ´ 106 J bei der Temperatur T1 abzugeben,wird die Energie EQ=h 861 kWs benötigt.Diese Energie kann bei einer Leistung von150 W in 1 h 36 min zugeführt werden.

2 Bei einer Auûentemperatur von ÿ5 C soll derHeizkessel in einem Haus mit einer Wärmepumpeauf einer Temperatur von 40 C gehalten werden.

Seite

157

Seite

158

Seite

163

4 Wärmelehre84

Page 85: Metzler Physik

Welche elektrische Energie muss der Pumpe zuge-führt werden, um dem Heizkessel eine Wärme-energie von 1 kJ zuzuführen?

Lösung:hWP jQj=E und hWP T1=T1ÿT2 313=313ÿ2686,96; E 1 kJ=6,96 144 J

4.3.7 Der Viertaktmotor

1 Berechnen Sie die mechanische Energie E, die beider adiabatischen Expansion eines idealen Gasesmit p p1 und V V1 auf V V2 abgeführt wird.

Lösung:

EZV2

V1

pvdV ZV2

V1

p1Vk1

VkdV p1Vk

1

ZV2

V1

dVVk

p1Vk1

1ÿkV1ÿk V2

V1 p1Vk

1

1ÿk1

Vkÿ12

ÿ 1Vkÿ1

1

p1V1Vkÿ11

kÿ 11

Vkÿ12

ÿ 1Vkÿ1

1

p1V1

kÿ 11ÿ V1

V2

kÿ1( )

*2 Wie verändert sich der Wirkungsgrad eines Vier-taktmotors, wenn durch die Verwendung von Su-perbenzin das Kompressionsverhältnis von 1 : 7auf 1 : 11 erhöht wurde (k1,4)? VergleichenSie die aktuellen Preise für Normal- und Superben-zin und schätzen Sie den Nutzen ab.

Lösung:h 1ÿ v1=v2 kÿ1; h1 0,541; h2 0,617Der Wirkungsgrad erhöht sich um ca. 14%. Super-benzin kostet aber nur ca. 3% mehr als Normalben-zin. Der Motor fährt mit Superbenzin also etwa10% wirtschaftlicher.

Seite

165

4.3 Energieumwandlungen 85

Page 86: Metzler Physik

1 Die Sonne hat einen Radius von 696 000 km und ist149,5 Mio. km entfernt. Wir empfangen von ihreine Strahlungsleistung von 1,353 kW/m2. Berech-nen Sie die Oberflächentemperatur der Sonne.

Lösung:P8 149,5 109 m2 p 1,353 103 W

m2

9,50 1025 W A8 Is

T

P8

A8 s4

s

9,50 1025 W m2 K4

696000 103 m2 p 5,6703 10ÿ8W

4

s

5760 K

2 Der Wolframfaden (a 0,25) einer Glühlampe(100 W/220 V) wird auf q 2300 C erhitzt undgibt dabei die gesamte elektrische Leistung alsStrahlung wieder ab.a) Welche Fläche muss der Faden haben?b) Bei welcher Wellenlänge strahlt der Faden ma-

ximal?c) Wie viel Prozent der Strahlungsleistung

werden im sichtbaren Bereich (400 nm l 800 nm) abgegeben?

d) Wie lang muss der Faden sein, wenn sein spe-zifischer Widerstand r 77,5 mW cm beträgt?

Lösung:a) P A I A a Is A a s T4

A pa s T4

100 W m2 K4

0,25 5,6703 10ÿ8 W2300 273,154 K4

1,609 10ÿ4 m2 1,609 cm2

b) lmax 2,898 10ÿ3 m=T 1,13 mm

c) Il1;l2 Zl2

l1

ISl;Tdl 2 h c2

Zl2

l1

dlÿe

hckTlÿ 1

l5

I 1,191 10ÿ16

Zl2

l1

dl

l5 e5;591 10ÿ6m

l ÿ 1 ;

0,4 ´ 10ÿ6 m

(nummerische Integration)ergibt:I 1,191 10ÿ16 5,029 1020 5,989 104 W

m2

IIg 5,989 104 W

m2

sT4 0,0241 2,41 %.

d) R r lAqr spez. Widerstand, Aq Quer-schnittsfläche des Glühfadens)

R r l=r2 p; PU2=R 100 W;

R (220 V)2=100 W 484 WA 2p r l Oberfläche des Glühfadens

) r A2p l

R rl4 p2 l2

p A2 r 4p l3

A2

l R A2

4 pr3

s

484W(1,609 10ÿ4 m2)2

4p 77,510ÿ6 W10ÿ2 m

s 1,09 m

Seite

169

4 Wärmelehre86

Page 87: Metzler Physik

4.5.1 Irreversible Vorgänge

1 Nennen Sie je ein Beispiel für irreversible Vor-gänge aus Mechanik, Wärmelehre und Elektrizi-tätslehre und beschreiben Sie die energetischenVeränderungen.

Lösung:Herabrutschen eines Körpers auf einer schiefenEbene. Potentielle Energie des Systems Körper-Gravitationsfeld wird in thermische Energie umge-wandelt, die in die Umgebung und alle an derWechselwirkung beteiligten Körper abflieût.

Plötzliche Kompression eines Gases, z. B. Luft.Mechanische Energie wird dem Gas zugeführt, so-dass sich die Temperatur des Gases erhöht. Diethermische Energie des Gases flieût in die Umge-bung ab.Entladung eines Kondensators über einen Wider-stand. Die im Feld des Kondensators gespeicherteEnergie wird am Widerstand in thermische Energieumgewandelt und flieût in die Umgebung ab.

4.5.2 Definition der Entropie

1 Wie groû ist die Entropieänderung bei der isother-men Expansion eines idealen Gases vom VolumenV1 auf V2?

Lösung:Bei einer isothermen Expansion ist die innere Ener-gie U konstant, sodass QÿE ist. Die Entropie-änderung ergibt sich zu DSQ=T .

EÿZy2

Y1

p dV, p n RTV

Eÿn RTZy2

y1

dVVÿnRTln vV2

V1ÿnRT ln

V2

V1

Q nRT ln (V2=V1)

DSQ=T nR ln (V2=V1)

2 Berechnen Sie die Entropieänderung für den Kühl-prozess mit den Werten aus Aufgabe 1, S. 163.

Lösung:DSDS1DS2;DS2 Q2=T2 4,7 ´ 106 J/273 K 17 216 J/K;DS1 ÿ(QE)=T1 ÿ (4,7 ´ 106 J + 150 W ´ 1,5 ´3600 s)=293 Kÿ 18 805 J=K; DSÿ1589 J=K,es wurde Entropie erzeugt.

3 Die Dampftemperatur eines 1000 MW-Kraftwerksist 350 C, die des kühlenden Wassers 20 C. Be-rechnen Sie die pro Stunde als Abwärme abgege-bene Energie.

Lösung:Unter der (idealisierenden) Annahme der Reversi-bilität gilt Q1 DS T1 und Q2 DS T2, sodassEQ1Q2 (Q2 < 0) folgt: EDS(T1ÿT2).Daraus ergibt sichQ2 E T2=(T1ÿT2) 1000 MWh ´ 293 K= (623 K ÿ 293 K 888 MWh.

4.5.3 Entropie, Irreversibilität und Energieentwertung

1 Mit welcher Begründung sind mechanische, elekt-rische und chemische Energie als besonders wert-voll, thermische Energie jedoch als weniger wert-voll anzusehen.

Lösung:Thermische Energie ist um so wertvoller je höherdie Temperatur ist, bei der sie vorliegt. Mit mecha-nischer, elektrischer und chemischer Energie kannman thermische Energie beliebig hoher Tempera-tur herstellen.

Seite

172

Seite

174

Seite

175

4.5 Die Entropie 87

Page 88: Metzler Physik

5.1.1 Trennung und Nachweis von Ladungen

1 Erklären Sie die Wirkungsweise eines Elektro-skops.

Lösung:Abstoûung gleichnamig geladener Körper; rück-treibende Kraft ist die Schwerkraft.

2 Wie kann man bei einem elektrisch geladenenKörper feststellen, ob seine Ladung positiv odernegativ ist?

Lösung:Man lädt ein beweglich aufgehängtes Probekügel-chen mit einem geriebenen Glasstab auf; wird esbeim Annähern an den Körper angezogen (abgesto-ûen), so ist der Körper negativ (positiv) geladen.Oder: Man entlädt den Körper über eine Glimm-lampe; glimmt der dem Körper zugewandte (ab-gewandte) Pol, so war der Körper negativ (positiv)geladen (s. Kap. 6.3.3, Versuch 2).

3 Können Nichtleiter genauso wie im Versuch 2geladen werden? Begründen Sie Ihre Antwort.

Lösung:Nein. Auf Nichtleitern gibt es keine frei beweg-lichen Ladungen.

4 Begründen Sie, weshalb sich die Ladungen einesgeladenen metallischen Körpers auf der Auûen-fläche verteilen.

Lösung:In Metallen sind überschüssige Ladungen frei be-weglich; gleichnamige Ladungen stoûen sich ab.

5 Beschreiben Sie einfache Experimente, die für dieErhaltung der elektrischen Ladung sprechen.

Lösung:Reibungsversuche (Stab und Reibzeug haben ge-gengleiche elektrische Ladung); Influenz- undAuf- bzw. Entladeversuche mit Konduktorkugel,Elektroskop und Faraday-Becher; experimentellerNachweis des 1. Kirchhoffschen Gesetzes; Ver-zweigung und Vereinigung von Strömen, d. h. flie-ûenden Ladung (s. Kap. 5.3.2).

5.1.2 Zusammenhang von Ladung und Strömstärke

1 Welche Ladung ist durch den Leiterquerschnittgeflossen, wenn eine konstante Stromstärke I1,8 mA 5 min und 12 s lang gemessen wird?

Lösung:DQ I D t 1,8 ´ 10ÿ3 A ´ 312 s 0,56 C.

2 Eine Autobatterie kann eine Ladung von 88 Ahabgeben. Wie lange kann man ihr einen Stromvon 0,5 A entnehmen?

Lösung:

D tDQ=I 88 Ah0,5 A

176 h.

3 Bei der Entladung eines Kondensators erhält manfolgende Messwerte:

t in s 0 4 8 12 16 20 24

I in mA 50 43 35 29 24 20 17

t in s 28 32 36 40 44 48 52

I in mA 14 12 10 9 7,5 6 5

Stellen Sie die Messwerte grafisch dar (mm-Pa-pier) und bestimmen Sie die Ladung durch Auszäh-len der Flächeneinheiten.

Lösung:Auszählen in mm-Papier ergibt Q 9,6 ´ 10ÿ4 C.Zur Integration wird der Ansatz I I0 eÿa t

(Kap. 5.3.3) gemacht. Die Messreihe ergibt

a ln (In=In1)= D t 4,54 ´ 10ÿ2 sÿ1 und

QZ1

0

I d t 1a

I0 1,1 ´ 10ÿ3 C.

4 Wie lässt sich im t-Q-Diagramm die Stromstärkezu einer bestimmten Zeit t ablesen?

Lösung:Die Stromstärke zur Zeit t1 ist gleich der Steigungdes Graphen der Funktion Q (t) an der Stelle t1.

Zusatzaufgaben

5 Durch einen Leiterquerschnitt flieût in 20 s dieLadung 5,6 C. Wie groû ist die Stromstärke?

Lösung:

I DQ=D t 5,6 C20 s

0,28 A.

5 Ladung und elektrisches Feld

Seite

179

Seite

181

5 Ladung und elektrisches Feld88

Page 89: Metzler Physik

6 Zehn kleine geladene Körper sind gleichmäûig aufeinem Kreis mit dem Radius 80 cm verteilt. JederKörper besitzt eine Ladung von 90 nC. Wie groû istdie Stromstärke auf dem Kreisbogen, wenn derKreis mit 1200 Umdrehungen pro Minute rotiert?

Lösung:f 1=T 20 sÿ1; Q 9 ´ 10ÿ7 C;I Q=T 1,8 ´ 10ÿ5 A 18 mA.

*7 Gegeben ist folgende Zeit-Strom-Funktion: I (t) 0,2 t 4 A=s4. Berechnen Sie die zwischen der 2. und3. Sekunde geflossene elektrische Ladung.

Lösung:

QZ3 s

2s

0,2As4

t4 d t 8,44 C.

5.1 Elektrische Ladung und elektrischer Strom 89

Page 90: Metzler Physik

5.2.2 Elektrische Feldstärke

1 Begründen Sie: Das Innere eines Leiters enthält imelektrostatischen Gleichgewicht weder ein elektri-sches Feld noch überschüssige Ladungen.

Lösung:Bestünde im Leiter ein elektrisches Feld, so würdendie im Leiter frei beweglichen Ladungen beschleu-nigt werden. Es herrschte dann kein elektrostati-sches Gleichgewicht.Die auf einen Leiter gebrachten Ladungen gleichenVorzeichens stoûen sich gegenseitig ab und vertei-len sich daher auf der Leiteroberfläche. Bei einerBewegung von Ladungen ins Innere des Leitersentstünde eine rücktreibende Kraft, da eine solcheBewegung eine Annäherung gegenüber gleichna-migen Ladungen bedeutet.

2 Erklären Sie die Abschirmwirkung der Drahthaubein Abb. 183.3.

Lösung:Durch Influenz des äuûeren Feldes werden diebeweglichen negativen Ladungen (je nach Feld-richtung) auf die Auûen- bzw. Innenfläche derDrahthaube bewegt. Auf der Auûenfäche sind dieüberschüssigen Ladungen durch das äuûere Feldgebunden. Auf der gesamten Innenfläche ist derLadungsüberschuss gleichnamig; der Innenraumist feldfrei, da zwischen gleichnamigen Ladungenkeine Feldlinien verlaufen.

3 Welche Unterschiede bestehen zwischen elektri-schen Feldern und Gravitationsfeldern?

Lösung:In elektrostatischen Feldern gibt es ¹Quellenª und¹Senkenª, in Gravitationsfeldern nur ¹Senkenª.

4 Berechnen Sie die elektrische Feldstärke an einemOrt, in dem auf einen Körper mit der Ladung Q 26 nC die Kraft F 37 mN wirkt.

Lösung:E F=Q 1,4 ´ 103 N=C 1,4 kN=C.

5 Berechnen Sie die Kraft, die ein Körper mit derLadung Q 78 nC in einem Feldpunkt mit derFeldstärke E 810 kN=C erfährt.

Lösung:F Q E 6,3 ´ 10ÿ2 N 63 mN.

6 Die Feldlinien eines Plattenkondensators verlaufenvertikal von unten nach oben. Ein in den Platten-raum eingebrachtes positiv geladenes Öltröpfchen,dessen Masse m 4,7 ´ 10ÿ10 kg ist, schwebt

gerade. Messungen ergaben eine FeldstärkeE 7,2 GN=C. Berechnen Sie die Ladung desÖltröpfchens.

Lösung:Q F=Em g=E 6,4 ´ 10ÿ19 C.

7 Ein elektrisches Feld der Stärke 180 N=C sei senk-recht zur Erdoberfläche nach unten gerichtet. Ver-gleichen Sie die nach oben gerichtete elektrostati-sche Kraft auf ein Elektron (q 1,6 ´ 10ÿ19 C,m 9,1 ´ 10ÿ31 kg) mit der nach unten gerichtetenGravitationskraft.Wie stark müsste eine 3 g schwere Münze geladensein, damit die durch dieses Feld bewirkte elek-trostatische Kraft die Gravitationskraft (g 9,81 m=s2) ausgleicht?

Lösung:Fel=FG QE=(mg) 3,2 ´ 1012,QM ÈumM Èu g=E 1,6 ´ 10ÿ4 C.

*8 Warum stehen elektrische Feldlinien im statischenFall immer senkrecht auf Oberflächen von Leitern?

Lösung:An einer Leiteroberfläche muss die Tangential-komponente der von auûen wirkenden Feldkraftüberall gleich null sein; das ist der Fall, wenn jedeFeldlinie auf der Oberfläche senkrecht steht.Wäre nämlich die Tangentialkomponente nichtgleich null, so würde diese eine Verschiebung derLadung auf der Leiteroberfläche bewirken, daselektrostatische Gleichgewicht wäre also nochnicht vorhanden.

*9 In einem isoliert aufgestellten Faraday-Käfig hängteine mit der Ladung Q aufgeladene Kugel, ohne ihnzu berühren. Wie kann man Q messen, ohne Zu-gang in das Innere des Käfigs zu haben? BegründenSie das Verfahren.

Lösung:Man lässt über einen Messverstärker die Ladungvon der Auûenseite des Faraday-Käfigs abflieûen.Begründung: Influenz.

*10 Zwei negativ geladene Körper können einanderanziehen. Unter welcher Voraussetzung ist dasmöglich (Anleitung: Influenz im inhomogenenFeld)?

Lösung:Wenn die Körper sehr unterschiedlich stark gela-den sind, kann die bei dem weniger geladenen

Seite

185

5 Ladung und elektrisches Feld90

Page 91: Metzler Physik

Körper durch Influenz bewirkte Anziehungskraftungleichnamiger Ladungen gröûer sein als dieAbstoûungskraft der gleichnamigen.

*11 Warum beginnen elektrische Feldlinien nicht imleeren Raum?

Lösung:Der leere Raum kann nicht Träger von elektri-scher Ladung sein.

*12 Eine Pendelkugel sei mit der Ladung Q 52 nCgeladen und habe die Masse m 0,40 g. Siehänge an einem Faden der Länge l 1,80 m ineinem horizontal gerichteten homogenen elektri-schen Feld. Durch die Kraft des Feldes wird sieum d 15 mm ausgelenkt. Wie groû ist die Feld-stärke E des homogenen Feldes?

Lösung:F mg sin amg d=l;E F=Qm gd=lQ 630 N=C.

Zusatzaufgabe

*13 Wie groû ist der Ausschlag d einer Probekugel derMasse m 0,25 g, die an einem Faden der Längel 1,5 m in einem elektrischen Feld der StärkeE 560 N=C hängt, wenn sie die Ladung Q 60 nC trägt?

Lösung:F mg sin amg d=l;E F=Qm gd=(lQ);d E lQ=(mg) 2,1 cm.

5.2.3 Energieumwandlung im elektrischen Feld

1 Zwischen zwei parallelen Leiterplatten mit demAbstand d 5 cm, einem so genannten Plattenkon-densator, besteht ein elektrisches Feld der StärkeE 9,4 kN=C. Welche Energie ist erforderlich,um die Ladung q 5,5 pC von der einen Plattezur anderen zu transportieren?

Lösung:W QE d 2,6 ´ 10ÿ9 Nm 2,6 nJ.

2 Im homogenen Feld der Feldstärke E 85 kN=Cwird ein geladenes Teilchen (q 25 nC) a) parallelzu den Feldlinien und b) unter einem Winkel von30 zu den Feldlinien 1,2 cm weit gegen das Feldtransportiert. Berechnen Sie die dafür erforderlicheEnergie.

Lösung:a) W QE d 2,6 ´ 10ÿ5 Nm 26 mJ,b) W QE d cos 30 2,2 ´ 10ÿ5 Nm 22 mJ.

3 Welche Energie ist erforderlich, um im elektrosta-tischen Feld einen geladenen Körper auf einer ge-schlossenen Kurve einmal herum zu bewegen?

Lösung:W 0.

4 Weshalb gibt es im elektrostatischen Feld keinegeschlossenen Feldlinen?

Lösung:Transportiert man einen geladener Körper längseiner geschlossenen Feldlinie von P1 nach P1, so

würde die potentielle Energie des Systems Körper-Feld beim Transport in der einen Richtung zuneh-men und beim Transport in der anderen Richtungabnehmen. Das widerspräche jedoch dem Satz vonder Unabhängigkeit vom Weg.

*5 Ein Elektron werde mit einer Anfangsgeschwin-digkeit von v 3000 km=s in Richtung eines ho-mogenen elektrischen Feldes mit der FeldstärkeE 1 kN=C geschossen. Wie weit bewegt sichdas Elektron, bevor es vollständig abgebremst istund ruht (me 9,1 ´ 10ÿ31 kg, qe 1,6 ´ 10ÿ19 C)?

Lösung:Wel Fel s E qe s, Wkin 1

2 me v2,E qe s 1

2 me v2,s 1

2 me v2=(E qe) 2,6 cm.

*6 Ein Proton werde in ein homogenes elektrischesFeld mit der Feldstärke E 5 N=C gebracht undlosgelassen. Mit welcher Geschwindigkeit bewegtes sich, nachdem es 4 cm zurückgelegt hat (mP 1,67 ´ 10ÿ27 kg, qP 1,6 ´ 10ÿ19 C)?

Lösung:Gleichmäûig beschleunigte Bewegung:Aus v a t, s 1

2 a t2 folgt v2asp

.Aus der wirkenden Feldkraft mp a qp E ergibtsich die Beschleunigung a qp E=mp.Also ist v

2 qp E S=mp

p 6,2 km=s.

Seite

187

5.2 Das elektrische Feld 91

Page 92: Metzler Physik

5.2.4 Elektrisches Potential und elektrische Spannung

1 Zwischen zwei parallelen Platten liegt die Span-nung U 1,5 kV. Welche Energie ist erforderlich,um die Ladung Q 8,2 nC von einer Platte zuranderen zu transportieren?

Lösung:W QU 1,2 ´ 10ÿ5 Nm = 12,3 mJ.

2 Wie groû ist nach Abb. 188.2 das Potential der ne-gativ geladenen Platte, wenn der Bezugspunkt P0

im Abstand 3 cm von ihr entfernt im Feld liegt?Geben Sie die s-ò-Funktion als Gleichung an.

Lösung:ò1 ÿ150 V; ò (s) 50 (V=cm) s ÿ 150 V.

3 Warum sinkt das Potential im Feld eines positivgeladenen Körpers mit wachsendem Abstand?

Lösung:Das Potential ist über die potentielle Energie defi-niert, die das System Feld-Körper gewinnt, wennder im Feld bewegte Körper positiv geladen ist.Entfernt sich ein positiv geladener Probekörpervon einem positiv geladenen Körper, so verliertdas System potentielle Energie.

Oder: Das Potential im Radialfeld beträgt

ò (r) 14p e0

Qr

, wird also mit wachsendem r klei-

ner (s. Kap. 5.2.6).

4 Zwischen zwei Platten mit einem Abstand von d 1,8 cm besteht ein elektrisches Feld der StärkeE 85 kN=C. Die negative Platte ist geerdet.Welches Potential hat die andere Platte gegenüberErde (òErde 0)?

Lösung:ò01 E d 1,53 kV.

5.2.5 Feldstärke und felderzeugende Ladung

1 Zwei parallel aufgestellte Platten mit der FlächeA 1,2 ´ 10ÿ2 m2 und dem Abstand d 0,5 mmsind mit Q1 0,26 mC und Q2 ÿ 0,26 mC ge-laden. Berechnen Sie die Flächenladungsdichte,die Feldstärke E und die Spannung U.

Lösung:Q=A 22 mC=m2,EQ=(A e0) 24 kV=cm,U E d 1,2 kV.

2 Zwei Platten (A 4,8 ´ 10ÿ2 m2, d 2 mm) werdenmit einer Spannung U 1,5 kV aufgeladen. Be-rechnen Sie die Feldstärke E, die Flächenla-dungsdichte und die auf jeder Platte befindlicheLadung Q.

Lösung:EU=d 7,5 kV=cm,Q=A e0 E 6,64 mC=m2,Q e0 E A 319 nC.

3 Zwei parallele Platten (A7,9 ´ 10ÿ2 m2, d2 mm)werden mit einer Spannung U 32 V aufgeladen.Die dabei aufgenommene Ladung beträgt Q 11,2 nC. Bestimmen Sie aus diesen Messwertenden Wert der elektrischen Feldkonstanten e0.

Lösung:e0 Qd=(AU) 8,86 ´ 10ÿ12 C=(Vm).

5.2.6 Coulomb'sches Gesetz

1 Berechnen Sie die Kraft, mit der sich zwei gleichgeladene Körper mit der Ladung a) Q 35 mC imAbstand r 12 cm, b) Q 1 C im Abstand r 1 mabstoûen.

Lösung:

F 14 pe0

Q1 Q2

r2 Q2

4 pe0 r2;

a) F 7,6 ´ 102 N 760 N;b) F 9,0 ´ 109 N 9,0 GN

2 Der Abstand zwischen Proton und Elektron imWasserstoffatom sei d 10ÿ10 m. Das Proton trägtdie Ladung Q 1,6 ´ 10ÿ19 C, das Elektron einegleich groûe negative.

Seite

189

Seite

195

Seite

197

5 Ladung und elektrisches Feld92

Page 93: Metzler Physik

a) Wie groû ist die Coulomb-Kraft, mit der sichdie beiden Teilchen anziehen?

b) Wie groû ist die Gravitationskraft zwischenden beiden Teilchen? (mP 1,7 ´ 10ÿ27 kg,me 9,1 ´ 10ÿ31 kg)

c) In welchem Verhältnis stehen elektrostatischeAnziehungskraft und Gravitationskraft? Hängtdas Verhältnis vom Abstand der Teilchen ab?

Lösung:a) Fel Q2=(4pe0 r2) 2,3 ´ 10ÿ8 N,b) FG g mp me=r2 1,0 ´ 10ÿ47 N,c) Fel=FG Q2=(4pe0 gmp me) 2,3 ´ 1039; das

Verhältnis ist vom Abstand der Teilchen unab-hängig.

*3 Zwei Punktladungen Q1 2 C und Q2 8 Chaben den Abstand d 1 m. In welchem Punkt istdie Feldstärke Null?

Lösung:

Aus E E1E2 1

4pe0(Q1=r2

1 ÿQ2=r22) 0

folgt Q1=r21 Q2=r2

2 ; mit r1 r2 1 m ergibt sich

r1 13 m, r2 2

3 m.

*4 Auf einem geradlinigen dünnen Draht der Länge lsei die Ladung Q gleichmäûig verteilt. BestimmenSie die Feldstärke E im Abstand r vom Draht fürr l.(Anleitung: Betrachten Sie die Ladungsdichte aufder Oberfläche eines Zylinders mit dem Radius r.)

Lösung:EQ=(e0 A)Q=(2p e0 r l).

Zusatzaufgaben

5 Zwei gleich geladene Körper stoûen sich imAbstand d 20 cm mit der Kraft F 0,015 Nab. Wie groû sind die Ladungen?

Lösung:F Q2=(4p e0 r2);Q

4 p e0 r2 Fp

2,6 ´ 10ÿ7 C 260 nC.

6 2 m von einem mit 3 nC geladenen Körper entferntbefindet sich ein Körper mit der Ladung ± 2 nC.Wie groû ist die elektrische Feldstärke im Mittel-punkt der Verbindungslinie zwischen beiden Kör-pern?

Lösung:E E1E2 (jQ1 j=r2

1 jQ2 j =r22)=(4p e0)

45 N=C.

*7 Begründen Sie die Festigkeit des abgebildetenIonengitters.

+ – + – + – + – + –

+– + – + – + – + –

+ – + – + – + – + –

+– + – + – + – + –

+ – + – + – + – + –

+– + – + – + – + –

Lösung:Ungleichnamige Ladung sind sich näher als gleich-namige, die anziehenden Kräfte sind gröûer als dieabstoûenden (Coulombsches Gesetz).

*8 Zwei Punktladungen Q1 1 C und Q2ÿ 1 C sindim Abstand 2 cm starr miteinander verbunden; siebilden einen elektrischen ¹Dipolª. Der Dipol liegein einem rechtwinkligen Koordinatensystem aufder y-Achse punktsymmetrisch zum Ursprung; Q1

liege im Punkt (0 j 1 cm).Bestimmen Sie die Koordinaten und den Betragdes elektrischen Feldvektors in den PunktenP0 (0 j 0), P1 (1 cm j 0), P2 (ÿ 1 cm j 0),P3 (0 j 2 cm) und P4 (1 cm j 1 cm).

Lösung:E0x 0; E0y ÿ0,5 104=(p e0) C=m2;jE0 j 0,5 104=(p e0) C=m2 180 ´ 1012 N=C.

E1x 0; E1y ÿ2p 104=(16 p e0) C=m2

ÿ0,088 104=(p e0) C=m2;jE1 j 0,088 104=(p e0) C=m2 31,8 ´ 1012 N=C.

E2x E1x; E2y E1y; jE2 j jE1 j.E3x 0; E3y 2 104=(9p e0) C=m2

0,22 104=(p e0) C=m2;jE3 j 0,22 104=(p e0) C=m2 79,9 ´ 1012 N=C.

E4x ( 14ÿ

5p

100 ) 104=(p e0) C=m2

0,228 104=(p e0) C=m2;E4y ÿ

5p

50 104=(p e0) C=m2

ÿ0,045 104=(p e0) C=m2;jE4j 0,232104=(pe0) C=m2 83,5 ´ 1012 N=C.

*9 In der Anordnung von Aufgabe 8 befinde sich einzweiter gleicher Dipol Q3 Q4, und zwar liege diepositive Ladung Q3 im Punkt (10 cm j ÿ1 cm) unddie negative Ladung Q4 im Punkt (10 cm j 1 cm).a) Ziehen sich die Dipole an? Begründen Sie Ihre

Antwort.

5.2 Das elektrische Feld 93

Page 94: Metzler Physik

b) Berechnen Sie die wirkenden Kräfte und ver-gleichen Sie die Kräfte zwischen den Dipolenmit den Kräften zwischen den Punktladungen.

c) Die Kraft zwischen den Dipolen nennt man»kurzreichweitig«. Begründen Sie diese Be-zeichnung.

Lösung:a) Ja; die anziehende Kraft ist stärker als die ab-

stoûende, da sich die ungleichnamigen Ladun-gen näher sind als die gleichnamigen.

b) Auf jede Punktladung wirken die KräfteFanz Q2=(4p e0 0,01) mÿ2

Q2=(4p e0) 100 mÿ2,

Fabst Q2=(4p e0 0,0104) mÿ2

Q2=(4p e0) 96,2 mÿ2,

Fabstx Fabst 5=26p

Q2=(4p e0) 94,3 mÿ2,

und damit die Gesamtkraft

F FanzÿFabstx Q2=(4p e0) ´ 5,7 mÿ2.

Die Kraft zwischen den Dipolen beträgt nur5,7% der Kraft zwischen zwei Punktladungen.

c) Liegen die Dipole näher beieinander, so ist dieanziehende Kraft erheblich gröûer als die absto-ûende. Die Kraft nimmt mit der Entfernungstark ab. (Mitteilung ohne Nachweis: F 1=r4).

*10 An einer Ecke eines Rechtecks mit den Kanten-längen a 2 cm und b 4 cm befindet sich eineelektrische Punktladung Q1 30 nC, an der dia-gonal gegenüberliegenden Ecke eine Punkt-ladung Q2 ÿ 4 nC.Wie groû ist die elektrische Feldstärke E an denbeiden anderen Ecken des Rechtecks?

Lösung:

E1x 1

4 p e0

Q1

b2; E1y

14p e0

Q2

a2;

E1 E2

1xE21y

q9,1 ´ 105 N=C ;

E2x 1

4 p e0

Q2

b2; E2y

14p e0

Q1

a2;

E2 E2

2xE22y

q7,1 ´ 105 N=C .

*11 Berechnen Sie die Feldstärke in der Umgebungeiner positiv geladenen Kugel (r 3,6 cm, Q 84 nC) in den Punkten P1 und P2, die r1 5,3 cmund r2 12 cm von ihrem Mittelpunkt entferntsind. Berechnen Sie die Spannung zwischenihnen.

Lösung:

E 14p e0

jQ jr2

; U V1ÿV2 Q

4 p e0

1r2ÿ 1

r1

.

E1 2,7 ´ 105 N=C, E2 5,2 ´ 104 N=C,U 80 V.

*12 Im Wasserstoffatom hat das Elektron (Qeÿ1,6 ´ 10ÿ19 C) einen Abstand von etwa 10ÿ10 mvom Proton (QP 1,6 ´ 10ÿ19 C). Wird dasElektron aus dem Anziehungsbereich des Protons(ins Unendliche) entfernt, so sagt man, das Was-serstoffatom wird ionisiert. Berechnen Sie diedafür erforderliche Ionisierungsenergie.

Lösung:

W Qe (V0ÿV1)QeQp

4p e0

0ÿ 1

r1

2,3 ´ 10ÿ18 J.

*13 Zwei Punktladungen Q1 Q und Q2 3 Q stehensich im Abstand d gegenüber. Ein Punkt P1 habevon Q1 die Entfernung 1

2 d und von Q2 die Entfer-nung 4

5 d. P2 sei der an der Mittelsenkrechten vonQ1 Q2 gespiegelte Punkt P1.a) Welche Spannung misst man zwischen P1und

P2? (Anleitung: Wie die Feldstärken so über-lagern sich auch die Potentiale zweier Felderungestört).

b) Welches Messergebnis erhält man für einenPunkt P02, der Spiegelpunkt von P1 bezüglichder Achse Q1 Q2 ist?

Lösung:a) V1

14p e0

Qÿ12

d 3Qÿ

45

d 5,75Q

4p e0 d,

V2 1

4p e0

Qÿ45

d 3Qÿ

12

d 7,25Q

4p e0 d,

U V2ÿV1 1,5Q

4 p e0 d.

b) U V 02ÿV1 0.

5 Ladung und elektrisches Feld94

Page 95: Metzler Physik

5.2.7 Kapazität von Kondensatoren

1 Ein Kondensator nimmt bei der Spannung U3 kVdie Ladung Q 24 nC auf. Berechnen Sie die Ka-pazität.

Lösung:C Q=U 8,0 ´ 10ÿ12 F 8,0 pF.

2 Ein Plattenkondensator wird aufgeladen und dannvon der Spannungsquelle getrennt. Wie ändernsich die Feldstärke E und die Spannung U, wennman den Plattenabstand halbiert?

Lösung:E bleibt unverändert, U halbiert sich.

3 Ein Plattenkondensator (d 2 mm, A 314 cm2)wirda) bei konstanter Spannung U 180 V,b) bei konstanter Ladung Q 0,37 mCmit Glimmer (er 7) ausgefüllt.Untersuchen Sie das Verhalten von E und Q (bzw.U).

Lösung:a) EU=d 9 ´ 104 V=m 90 kV=m;

wegen U1 U0 gilt E1 E0;die Feldstärke bleibt konstant.

Q0 C0 U e0 (A=d) U 2,5 ´ 10ÿ8 C;Q1 e0 er (A=d) U 7 Q0;

der Kondensator nimmt die 7fache Ladung auf.

b) U0 Q=C0 Qd=(e0 A) 2,7 ´ 106 V,U1 Q=C1 Qd=(e0 er A) 1

7 U0;

E0 U0=d 1,3 ´ 109 V=m,E1 U1=d 1

7 U0=d 17 E0;

die Feldstärke und die Spannung sinken auf den7. Teil.

4 Auf einem Plattenkondensator (d 4 mm,A 400 cm2) wird bei einer Spannung U 200 V ohne Glasfüllung Q0 20 nC und mit Glas-füllung Q1 110 nC gemessen. Bestimmen Sie er

für Glas.

Lösung:Q0=U e0 A=d, Q1=U e0 er A=d,er Q1=Q0 5,5.Die Abmessungen des Kondensators spielen keineRolle.

*5 Zeigen Sie: CK 4 p e0 r ist die Kapazität einer freistehenden Leiterkugel mit dem Radius r. Anlei-tung: Betrachten Sie die Potentialdifferenz zwi-

schen Kugeloberfläche und Erde (unendlich fernerPunkt).

Lösung:Die Spannung der Kugeloberfläche gegenüber ei-nem unendlich ferner Punkt ist

U Q4 p e0

1r. Daraus ergibt sich CK

QU 4 p e0 r.

*6 Ein Kugelkondensator besteht aus zwei konzentri-schen leitenden Kugelflächen mit den Radien ra

und ri. Berechnen Sie seine Kapazität. (Anleitung:Siehe Aufgabe 5.)

Lösung:

U Zra

ri

E dr Q4 p e0

Zra

ri

drr2 Q

4 p e0

1r1ÿ 1

r2

,

C QU 4 p e0

ri ra

raÿ ri:

*7 Berechnen Sie die Kapazität der Erdkugel. (Mitt-lerer Radius R 6370 km.)

Lösung:CE 4 p e0 R 7,09 ´ 10ÿ4 F = 709 mF.

*8 Eine Metallkugel mit dem Radius r1 5,5 cm undder Ladung Q1 0,8 nC wird von einer neutralenzweiten Metallkugel von doppeltem Radius be-rührt. Wie verteilt sich die Ladung? (VerwendenSie CK aus Aufgabe 5.)

Lösung:Nach Berührung ist U1 U2; also mit U Q=Cergibt sich

Q01=Q02 C1=C2 4pe0 r1=(4pe0 r2) 12 ;

Q01 12 Q02;

Q01 13 Q1 2,7 ´ 10ÿ10 C 0,27 nC,

Q02 23 Q2 5,3 ´ 10ÿ10 C 0,53 nC.

Zusatzaufgaben

9 Berechnen Sie die Folienfläche, die mit einerGlimmerschicht (d 0,2 mm, er 6) aufgewickeltwerden muss, um eine Kapazität C 1 nF zu er-halten.

Lösung:AC d=(er e0) 37,7 cm2.

Seite

199

5.2 Das elektrische Feld 95

Page 96: Metzler Physik

*10 Eine in der Technik häufig vorkommende Kon-densatorform ist der Zylinderkondensator, deraus zwei koaxialen Zylindern besteht (ohneDeckflächen). Berechnen Sie die Kapazität einesZylinderkondensators mit der Höhe h und den Ra-dien ra und ri für den Fall, dass raÿ ri viel kleinerals ra ist. Wegen h raÿ ri sollen Randstörungen

vernachlässigt werden. (Anleitung: Benutzen Siedie Formel für den Plattenkondensator.)

Lösung:CZ e0 er A=d e0 er 2

ra ri

2h=(raÿ ri)

e0 er ph (ra ri)=(raÿ ri).

5.2.8 Energie des elektrischen Feldes

1 Welche Energie speichert ein Plattenkondensator(A 314 cm2, d 0,5 mm, Dielektrikum: er 7)bei einer Spannung U 220 V?

Lösung:

Wel 12 C U2 1

2 er e0 (A=d) U2

9,4 ´ 10ÿ5 J 94 mJ.

2 Zeigen Sie, dass im radialsymmetrischen Feld dieEnergiedichte mit der 4. Potenz des Abstandes ab-nimmt.

Lösung:

E 14 p er e0

Qr2

,

rel 12 er e0 E2 1

32 p2er e0

Q2

r4, rel

1r4

.

*3 Welche Energiedichte kann ein elektrisches Feld infeuchter Luft höchstens haben, wenn es bei einerFeldstärke E 2 MV=m zum Funkenüberschlagkommt? Berechnen Sie die Spannung, die man un-ter dieser Voraussetzung an einen luftgefülltenPlattenkondensator mit einem Plattenabstand vond 4 mm höchstens legen kann.

Lösung:relmax

12 e0 E2

max 18 J=m3,

Umax Emax d 8 ´ 103 V 8 kV.

*4 Wie groû müsste die Plattenfläche eines luftgefüll-ten Plattenkondensators sein, der bei einem Plat-tenabstand von d 1 mm und einer Spannungvon U 220 V die gleiche Energie speichert wieeine Autobatterie von 12 V und 88 Ah?

Lösung:W 88 Ah ´ 12 V = 3,8 ´ 106 J;

W 12 C U2 1

2 e0 (A=d) U2,

A 2W d=(e0 U2) 1,8 ´ 1010 m2 1,8 ´ 104 km2.

*5 Begründen Sie: Verdoppelt man den Plattenab-stand eines von der Spannungsquelle getrenntenPlattenkondensators, so verdoppelt sich auch derEnergieinhalt des Feldes. Woher kommt die ge-wonnene elektrische Energie?

Lösung:Wel 1

2 Q2=C 12 Q2 d=(e0 A); d. h.

Wel d für Q, A konstant.

Beim Vergröûern des Plattenabstandes wird La-dung gegen das Feld bewegt, also Energie zuge-führt.

Seite

201

5 Ladung und elektrisches Feld96

Page 97: Metzler Physik

5.3.1 Widerstände im Stromkreis

1 Erklären Sie die Arbeitsweise eines Drehspulmess-gerätes und eines Weicheisenmessgerätes. Erin-nern Sie sich dabei an das, was Sie in der Sekun-darstufe I über Wirkungen des elektrischen Stromsgelernt haben!

Lösung:Drehspulmessgerät: Der zu messende Strom flieûtdurch eine Spule, die sich in einem Magnetfeld be-findet. Das Magnetfeld der Spule richtet sich imäuûeren Magnetfeld aus. Eine mechanische Gegen-kraft (z. B. Spiralfeder) sorgt dafür, dass der Aus-richtungswinkel von der Stromstärke abhängt.Weicheisenmessgerät: Ein in einer Spule befindli-cher Weicheisenkern wird umso stärker magneti-siert, je gröûer der Spulenstrom ist (wenn man vonder Sättigung hinreichend weit entfernt ist ± s. Kap.6.2.3). Die antreibende Kraft für den Zeiger ent-steht dadurch, dass bei Stromfluss durch die Spuleder Weicheisenkern am Spulenende in die Spulehineingezogen oder gedreht wird. Die Rückstell-kraft kann durch eine elastische Feder realisiertwerden.

2 Wie müssen Strommessgeräte in einen Stromkreiseingeschaltet werden? Wie kann man den Messbe-reich eines Strommessers ändern?

Lösung:Amperemeter werden in Reihe geschaltet. Durcheinen parallelgeschalteten Widerstand wird derMessbereich erweitert; durch einen in Reihe ge-schalteten Widerstand (Vorwiderstand) wird derMessbereich verkleinert.

3 Wie sind im Haushalt alle Elektrogeräte geschal-tet? Weshalb sind sie so geschaltet?

Lösung:Parallel. Die Geräte können unabhängig voneinan-der ein- und ausgeschaltet werden; sie sind für einebestimmte Leistung und damit für eine bestimmteSpannung (220 V) konstruiert, bei Parallelschal-tung liegen sie alle an gleicher Spannung.

4 Aus vier Widerständen R1 10 W, R2 20 W,R3 50 W, R4 100 W, werden die abgebildetenSchaltungen aufgebaut. An jede Schaltung wirdeine Spannung von 24 V gelegt. Berechnen Siefür jede Schaltunga) den Gesamtwiderstand;b) die an jedem Widerstand liegende Spannung;c) den durch jeden Widerstand flieûenden Strom.

Lösung:a) A: 180 W, B: 25 W, C: 5,56 W,

D: 22,5 W, E: 40 W, F: 124,29 W:

b) A: U1 1,33 V, U2 2,67 V,U3 6,67 V, U4 13,33 V;

B: U1 U3 8 V, U2 U4 16 V;C: U1 U2 U3 U4 24 V;D: U1 10,67 V, U2 U3 U4 13,33 V;E: U1 U2 4 V, U3 U4 20 V;F: U1 1,93 V; U2 U3 2,76 V,

U4 19,31 V.c) A: I1 I2 I3 I4 0,13 A;

B: I1 I2 0,8 A, I3 I4 0,16 A;C: I1 2,4 A, I2 1,2 A,

I3 0,48 A, I4 0,24 A;D: I1 1,07 A, I2 0,67 A,

I3 0,27 A, I4 0,13 A;E: I1 I3 0,4 A, I2 I4 0,2 A;F: I1 I4 0,913 A, I2 0,138 A,

I3 0,055 A.

*5 Eine elektrische Kochplatte wird von zwei Spiralengeheizt, die einzeln, hintereinander oder parallelgeschaltet werden können (4 Stufen). Bei einerSpannung von 220 V ist die niedrigste Heizleistung200 W und die höchste 2000 W.Welche Widerstände haben die Heizspiralen? Wiegroû ist die Leistung bei den beiden anderenSchaltstufen?

Lösung:Die Leistung PU2=R ist maximal bei Parallel-schaltung:

Pmax U2=

R1 R2

R1R2

2000 W 2 kW

und maximal bei Hintereinanderschaltung:

Pmin U2=(R1R2) 200 W.

Mit U 220 V ergibt sich aus beiden Gleichungenfür die Widerstände:

Seite

203

5.3 Elektrische Stromkreise 97

Page 98: Metzler Physik

R1 214,7 W, R2 27,27 W.

Die Leistungen der beiden anderen Stufen sind

P1 U2=R1 225,4 W undP2 U2=R2 1775 W.

*6 Bei der Sparschaltung eines Heizofens wird dieHeizspannung um 15 % gesenkt. Um wie vielProzent sinkt die elektrische Leistung (Annahme:R konstant)?

Lösung:P1 U2

1=R;P2 (U1ÿ 15

100 U1)2=R U21=Rÿ 27;75

100 U21=R.

Die Leistung sinkt um 27,75%.

*7 Wie ändert sich bei einem Heizofen die aufgenom-mene Leistung, wenn die angelegte Spannung hal-biert wird (Annahme: R konstant)?

Lösung:P1 U2=R; P2 ( 1

2 U)2=RU2=(4 R) 14 P1:

*8 Aus Konstantandraht wurde ein Kantenmodelleines Würfels hergestellt. Jede Kante hat einen Wi-derstand von 1 W. An zwei gegenüberliegendeEcken des Würfels wird eine Spannung von 2 Vgelegt. Welchen Strom muss die Spannungsquelleliefern?(Anleitung: Der Strom verteilt sich an den Eck-punkten auf 3 bzw. 2 gleiche Wege.)

Lösung:Ersatzschaltbild:

R 56 W; I U=R 2,4 A.

Zusatzaufgaben

*9 Welchen Strom liefert die Batterie in der abgebil-deten Schaltung? Welcher Strom flieût durch den20 W-Widerstand?

24 V

10 Ω

4 Ω5 Ω

20 Ω

Lösung:Ersatzschaltbild:

24 V

20 Ω10 Ω

4 Ω

5 Ω

I U=R 24 V=10 W 2,4 A; durch den 20 W-Widerstand flieût, ebenso wie durch den 4 W- undden 5 W-Widerstand, kein Strom.

*10 a) Durch welchen Widerstand wird in dem abge-bildeten Netzwerk die 18 V-Batterie insge-samt belastet?

b) Wie groû ist das Potential im Punkt A?c) Der Draht wird an den mit Pfeilen markierten

Stellen unterbrochen. Wie groû ist dann dasPotential im Punkt A?

18 V

18 Ω

10 Ω8 Ω

6 Ω

A

12 Ω

9 Ω

6 Ω

3 Ω

5 Ladung und elektrisches Feld98

Page 99: Metzler Physik

Lösung:Das Netzwerk wird wie in der Abbildung angege-ben reduziert. Der Widerstand von 10 W ist kurz-geschlossen und entfällt daher:

12 Ω

6 Ω

3 Ω

A A A

R

= 0

1

23 4

0

R

R

V

R

1R

6 Ω

8 Ω

18 Ω

9 Ω

= 18 V2V

1V

a) R1 1=(1=18 W 1=9 W) 3 W 9 W;R2 1=(1=6 W 1=12 W) 8 W 12 W;R4 1=(1=R3 1=R2) 4 W;Rx R1R4 13 W.

b) V1ÿV0 U1 (R4=Rx)Uges 5,54 V;V1 U1V0 5,54 V.

c) V1 18 V.

5.3.2 Kirchhoff'sche Gesetze

*1 In dem abgebildeten verzweigten Stromkreis sinddie Klemmspannungen U01 und U02 der Batterienund die Widerstände R1, R2 und R3 bekannt. Dieohmschen Widerstände (Innenwiderstände) derBatterien können Sie zur Vereinfachung vernach-lässigen.a) Berechnen Sie die Stromstärken I1, I2 und I3 der

durch die Widerstände flieûenden Ströme fürU01 24 V, U02 6 V, R1 20 W, R2 30 Wund R3 8 W. Wie sind die Stromrichtungen(Flieûrichtung positiver Ladung)?

b) Drücken Sie I1, I2 und I3 allgemein durch diegegebenen Gröûen aus.

c) Weisen Sie nach, dass je nach Dimensionierungder Bauelemente die Ströme I1 und I3 ihre Rich-tung verändern, dass der Strom I2 jedoch immervon B nach A flieût.

d) Welche Ströme ergeben sich für den Fall, dassR2 unendlich groû wird?

e) Geben Sie jeweils eine Bedingung dafür an,dass I1 bzw. I3 gleich Null sind, ohne dass R1

bzw. R3 unendlich groû werden.

Lösung:Festlegung der Messrichtungen:

24 V

U

C B D

1 2

A

1

01 02

3

+ – – +

R

U

U

I

2U3U

3I2I

2I1I 3I

1R R

6 V

a) Aufgrund der Festlegung istU01 ÿ 24 V und U02 ÿ6 V

I. U01U1U2 0II. U02U3U2 0III. I1 (ÿ I2) I3 0 j 30 W

I. ÿ 24 V 20 W I1 30 W I2 0II. ÿ 6 V 8 W I3 30 W I2 0III. 30 W I1ÿ 30 W I2 30 W I3 0

IIIII. ÿ 6 V 30 W I1 38 W I3 0 j 2I ÿ II. ÿ 18 V 20 W I1 8 W I3 0 j 3 jÿ

42 V 100 W I3 0

Seite

205

5.3 Elektrische Stromkreise 99

Page 100: Metzler Physik

I3 ÿ0,42 A

I3 in II: I2 0,312 A

I2, I3 in III: I1 0,732 A

I1 flieût von C nach B,I2 flieût von B nach A,I3 flieût von B nach D.

b)I. U01R1 I1R2 I2 0II. U02R2 I2R3 I3 0III. R2 I1ÿR2 I2R2 I3 0

IIIII. U02R2 I1 (R2R3) I3 0 j R1

I ÿ II. U01ÿU02R1 I1ÿR3 I3 0 j R2 jÿ

U02 R1ÿ (U01ÿU02)R2

(R1 R2R2 R3R1 R3) I3 0

I3 U01 R2ÿU02 (R1R2)R1 R2R2 R3R1 R3

IIII. U01 (R1R2) I1R2 I3 0 j R3

I ÿ II. U01ÿU02R1 I1ÿR3 I3 0 j R2 j

U01 R3ÿ (U01ÿU02)R2

(R1 R2R2 R3R1 R3) I1 0

I1 U02 R2ÿU01 (R2R3)R1 R2R2 R3R1 R3

I2 I1 I3 ÿU01 R3ÿU02 R1

R1 R2R2 R3R1 R3

c) I1 < 0 für U02 R2 < U01 (R2R3)I1 > 0 für U02 R2 > U01 (R2R3)

I3 < 0 für U01 R2 < U02 (R1R2)I3 > 0 für U01 R2 > U02 (R1R2)

I2 ist immer positiv, da für U01, U02 < 0 stetsÿU01 R3ÿU02 R1 > 0

d)

limR2!1

I1 limR2!1

R2

U02ÿU01ÿ

R3

R2U01

R2

R1R3

R1 R3

R2

U02ÿU01

R1R3,

limR2!1

I3 U01ÿU02

R1R3,

also: limR2!1

I1 ÿ limR2!1

I3,

limR2!1

I2 limR2!1

ÿU02 R3ÿU02 R1

R2

R1R3

R1 R3

R2

0

e) I1 0, wenn U01 R2

R2R3U02,

I3 0, wenn U01 R1R2

R2U02.

5.3.3 Auf- und Entladung eines Kondensators

1 Welche Gesamtkapazität erhält man, wenn manKondensatoren mit C1 0,5 mF und C2 8 mFhintereinander schaltet?

Lösung:Cges 1=(1=C1 1=C2) 0,47 mF.

2 Berechnen Sie alle möglichen Kapazitäten, dieman aus Kondensatoren mit den Kapazitäten C1 300 pF, C2 500 pF und C3 1 nF schalten kann.

Lösung:Kleinste Kapazität bei Serienschaltung:Cges1 0,16 nF.

Zwei Kondensatoren parallel und der dritte in Serie:Cges2 0,25 nF, Cges3 0,36 nF, Cges4 0,44 nF.

Zwei Kondensatoren in Serie und der dritte parallel:Cges5 0,63 nF, Cges6 0,73 nF, Cges7 1,19 nF.

Gröûte Kapazität bei Parallelschaltung:Cges8 1,8 nF.

3 Berechnen Sie die gröûte und die kleinste Gesamt-kapazität aus C1 1 mF, C2 1 mF, C3 2 mF undC4 4 mF.

Lösung:Gröûte Kapazität bei Parallelschaltung:Cmax C1C2C3C4 8 mF.

Kleinste Kapazität bei Serienschaltung:Cmin 1=(1=C1 1=C2 1=C3 1=C4) 0,36 mF.

Seite

207

5 Ladung und elektrisches Feld100

Page 101: Metzler Physik

*4 Die Zeit, in der bei der Entladung eines Kondensa-tors mit der Kapazität C über einen Widerstand Rdie Spannung auf die Hälfte ihres Ausgangswertesabsinkt, heiût Halbwertszeit tH. Leiten Sie die For-mel tH RC ln 2 zur Bestimmung der Halbwerts-zeit her.

Lösung:Uc (t tH) 1

2 Uc (t)

ÿU0 eÿ1

RC (t tH) ÿ 12 U0 eÿ

1RC t

eÿ1

RC tH 12

e1

RC tH 2

tH RC ln 2

5 In Aufgabe 3, S. 181 wurden Messwerte fürdie Entladung eines Kondensators gegeben. Zeich-nen Sie den Graphen der t-I-Funktion und be-stimmen Sie daraus die Halbwertszeit tH desEntladevorgangs. Wie groû war der Vorwider-stand, wenn die Kapazität des KondensatorsC 8 mF betrug?

Lösung:Ansatz: I I0 eÿa t, In1=In eÿa D t.

Die Messreihe ergibta ln (In=In1)=D t 4,54 ´ 10ÿ2 sÿ1;

tH (1=a) ln 2 12 s; a 1=(RC),R 1=(aC) 2,8 ´ 106 W 2,8 M W.

*6 Zeichnen Sie mit den Daten von Aufgabe 9 das t-U-Diagramm, das t-I-Diagramm und das t-Q-Dia-gramm.Wie ändern sich die Diagramme, wenn a) R hal-biert wird und wenn b) C halbiert wird?

Lösung:Ausgehend von (0 jU0) bzw. (0 j I0 ÿU0=R)werden weitere Punkte über die Halbwertzeit er-mittelt:

(tH jU0=2) bzw. (tH j I0=2),(2 tH jU0=4) bzw. (2 tH j I0=4) etc.

a) Halbierung von R:Halbierung von tH; U0 unverändert; steiler Ab-fall des t-U-Diagramms; I0 ist wegen I0 U0=Rdoppelt so groû; steilerer Abfall des t-I-Diagramms; es schneidet das ursprünglichet-I-Diagramm bei (tH j I0=2).

b) Halbierung von C:Halbierung von tH; U0 und I0 unverändert; stei-ler Abfall beider Kurven.

7 Beschreiben Sie alle Ihnen bekannten Möglichkei-ten, die Kapazität eines Plattenkondensators expe-rimentell zu bestimmen.

Lösung:1. Berechnung aus den Kondensator-Daten nach

C e0 eR A=d.2. Aufladung mit der Spannung U, Entladung und

Messung von Q über einen Messverstärker:CQ=U.

3. Aufladung mit der Spannung U0, Entladungüber einen bekannten Hochohmwiderstand Rund ein Mikroamperemeter; Bestimmung derKonstanten a aus dem t-I-Diagramm der Funk-tion I (t) I0 eÿa t;

Berechnung von Q0 nach Q0 R10

I(t) dt; dann:Q0=U0.

(Oder: Bestimmung von Q0 durch Auszählender Fläche im t-I-Diagramm.)

4. Experimenteller Ablauf wie bei 3; Beobach-tung der Zeit tH, in der I auf I=2 abgesunkenist; dann:C tH=(R ln 2).

5. (Erst nach Behandlung der Wechselstromlehremöglich.)Man legt eine bekannte Wechselspannung Umit bekannter Frequenz f an den Kondensatorund misst die Stromstärke I (Effektivwert). Un-ter Vernachlässigung des Ohm'schen Wider-standes gilt:I U=RC U 2p f C, C I=(2 p f U).

8 Weisen Sie nach, dass die Zeitkonstante t RC imInternationalen Einheitsystem (SI) die Einheit Se-kunde hat.

Lösung:RCRCU=IQ=U1V=A 1 As=V 1 s.

*9 Ein Kondensator wird über einen Widerstand R 50 kW entladen. Zur Zeit t0 0 liegt an ihm dieSpannung U0 200 V. Nach 10 s ist die Spannungauf U 180 V gesunken. Berechnen Sie Kapazität,Zeitkonstante, Halbwertszeit und die zur Zeit t0 aufeiner Platte befindliche Ladung Q0.

Lösung:U1 U0 eÿ

1RC t1 ,

C t1=(R ln U0=U1) 1,9 ´ 10ÿ3 F 1,9 mF;t RC 95 s, tH t ln 2 66 s;Q0 C U0 0,38 C.

5.3 Elektrische Stromkreise 101

Page 102: Metzler Physik

*10 Ein Kondensator der Kapazität 2 mF werde auf48 V aufgeladen und anschlieûend über einen200 W-Widerstand entladen. Wie groû ista) die am Ende des Aufladevorgangs im Kon-

densator vorhandene Ladung,b) die Anfangsstromstärke im 200 W-Wider-

stand zu Beginn der Entladung,c) die Zeitkonstante,d) die Ladung im Kondensator nach 2 ms unde) die Stromstärke im 200 W-Widerstand zum

Zeitpunkt 2 ms?Berechnen Sief) die Energie, die nach der Aufladung undg) nach 0,4 ms im Kondensator gespeichert ist,

undh) die Leistung, die zu Beginn des Entladevor-

gangs dem Widerstand zugeführt wird.

Lösung:a) Q0 C U0 96 mC ;b) I0 U0=R 0,24 A ;c) t RC 0,4 ms ;d) Q (t)Q0 eÿ

1RC t 0,65 mC ;

e) I(t) I0 eÿ1

RC t 1,6 mA ;f) W 1

2 C U20 2,3 mJ ;

g) W(t) 12 C (U0 eÿ

1RC t)2 4,7 pJ ;

h) P0 U0 I0 11,5 W.

Zusatzaufgabe

*11 Ein auf die Spannung U1 100 V aufgeladenerKondensator der Kapazität C1 20 mF wirdeinem ungeladenen Kondensator der KapazitätC2 10 mF parallelgeschaltet. Berechnen Siedie Spannung U, die an beiden Kondensatorenanliegt.Untersuchen Sie, ob bei diesem Vorgang a) dieLadung und b) die elektrische Energie erhaltenbleibt. Begründen Sie Ihre Aussage, gegebenen-falls auch mit quantitativen Angaben.

Lösung:Q C1 U1, Q (C1C2)U;U C U1=(C1C2) 66,7 V.

a) Die Ladung bleibt erhalten, da keine Ladungnach auûen abflieût oder von auûen zuflieût.

b) Die elektrische Energie verringert sich um0,033 J:

W1 12 C1 U2

1 0,100 J,

W2 12 (C1C2) U2 0,067 J.

Die Energie wandelt sich in einem Funken oderin einem dazwischenliegenden Widerstand inWärme um.

5 Ladung und elektrisches Feld102

Page 103: Metzler Physik

5.4.1 Ionenleitung in Flüssigkeiten

1 Berechnen Sie das elektrochemische ¾quivalentvon einwertigem Kupfer.

Lösung:

ÈAmQMr kg

F z 63,5

9,65 107 1kg kmolkmol C

6,58 ´ 10ÿ7 kg=C 0,658 mg=C.

2 Wie lange muss ein Strom der Stärke I 0,45 Aflieûen, damit aus einer CuCl2-Lösung 3,5 gKupfer abgeschieden werden?

Lösung:Für zweiwertiges Kupfer gilt ÈA 0,329 mg=C;

ÈAmQ m

I t,

t mÈAI 3,5 103

0,329 0,45s 2,36 ´ 104 s 6,6 h.

3 Eine Schale (A 120 cm2) soll durch Elektrolyseeinen d 0,2 mm dicken Silberüberzug (rAg 10,5 ´ 103 kg=m3) erhalten. Berechnen Sie die dazuerforderliche Zeit, wenn die Stromstärke I 10 Abeträgt (Tab. 209.1).

Lösung:tQ=I, Qm=ÈA, m rAg V rAg A d;t rAg A d=( ÈAI) 2,25 ´ 103 s 37,6 min.

*4 Berechnen Sie die Dicke der Kupferschicht(rCu 8,9 ´ 103 kg=m3), die ein Strom der StärkeI 5,8 A in der Zeit t 3 h auf der Oberfläche einerKugel (r 8,3 cm) gleichmäûig abscheidet(Tab. 209.1).

Lösung:d V=A V=(4p r2)m=(rCu 4p r2),m ÈAQ ÈAI t;d ÈAI t=(rCu 4p r2) 2,7 ´ 10ÿ5 m 0,027 mm.

*5 Zur Abscheidung von 12,5 mg eines zweiwertigenStoffes ist ein Ladungsfluss von 150,7 C erforder-lich. Berechnen Sie die relative Molekülmasse Mr

des Stoffes. Um welches Element handelt es sich?

Lösung:ÈAm=QMr kg=zF;

Mr kg m z F=Q 12,5 10ÿ6 2 9,65 107=150,7 kg=kmol 16,0 kg=kmol;

es handelt sich um Sauerstoff.

5.4.2 Die Elementarladung

1 Ein Öltröpfchen (m 3,5 ´ 10ÿ9 mg, r 0,950 g=cm3) schwebt zwischen den Platten einesKondensators mit dem Plattenabstand d 0,50 cmbei einer Spannung U 214 V.a) Wie viel Elementarladungen trägt es?b) Mit welcher Geschwindigkeit würde es in

Luft ohne elektrisches Feld sinken (hLuft 1,828 ´ 10ÿ5 Ns=m2)?

c) Bei welcher Spannung würde es mit der in b)errechneten Geschwindigkeit steigen?

Lösung:a) mgQ U=d,

Qm g d=U 8,0 ´ 10ÿ19 C 5 e.

b) rm=( 43 p r3),

r3 m=(4p r)3

p 9,6 ´ 10ÿ7 m;

m g 6 p h r v,vm g=(6ph r) 1,04 ´ 10ÿ4 m=s.

c) mg=(6ph r) (QEÿmg)=(6ph r),QE 2mg, Q U=d 2 mg;U 2 mgd=Q 430 V.

2 Berechnen Sie die Anzahl der überschüssigen Ele-mentarladungen auf einer metallischen Kugel(r 6,5 cm), deren Spannung gegenüber ErdeU 4,5 kV beträgt. Warum wurde bei den Ver-suchen mit flieûender Ladung und bei den elektro-statischen Versuchen die Ladungsquantelung nichtbeobachtet?

Lösung:Q CK U 4p e0 r U 3,25 ´10ÿ8 C2,0 ´1011 e.Die Genauigkeit der Strom- und Ladungsmessge-räte ist bei weitem geringer als DQ e.

3 Der Kern eines Sauerstoffatoms enthält 8 Protonen,die Schale 8 Elektronen.a) Wie groû ist die negative Ladung von 10 mg

Sauerstoff? (Benutzen Sie Tab. 209.1.)b) Man stelle sich die Ladung der Elektronen und

die Ladung der Protonen von 10 mg Sauerstoffjeweils punktförmig konzentriert vor. WelcheKraft würden sie im Abstand von 2 cm aufein-ander ausüben?

Seite

209

Seite

211

5.4 Bewegung geladener Teilchen im elektrischen Feld 103

Page 104: Metzler Physik

Lösung:a) 10 mg Sauerstoff entsprechen 10

16 ´ 10ÿ6 kmolAtome;

Q 1016 10ÿ6 kmol NA 8 e 5 ´ 10ÿ6 ´ F kmol

5 ´ 10ÿ6 ´ 9,649 ´ 107 C 482 C.(F Faraday-Konstante)

b) F 14p e0

Q2

r2 5,2 ´ 1018 N

(F Kraft).

5.4.3 Elektronenleitung in Metallen

*1 Berechnen Sie mit der in der Tabelle 213.2 gege-benen Beweglichkeit die Dichte der Elektronen inSilber und in Bismut. (Spezifische WiderständerAl 0,016 Wmm2=m, rBi 1,1 Wmm2=m).

Lösung:n 1=(u re).

nAg 7,0 ´ 1028 mÿ3,wenn jedes Atom ein Elektron liefert.

nBi 1,4 ´ 1025 mÿ3.Die Elektronendichte ist also weit geringer als beiSilber.

*2 Berechnen Sie die Driftgeschwindigkeit der Elek-tronen in einem Silberdraht der Länge d 3,5 mbei einer Spannung U 1 V.

Lösung:v uE u U=l 1,6 ´ 10ÿ3 m=s.Die Elektronen legen in Richtung des Drahtes ineiner Sekunde 1,6 mm zurück.

*3 Eine Spule (n 5000, r 12 cm) wird aus derAchsendrehung ( f 20 sÿ1) abgestoppt. Dabeiwird eine Ladung DQ 0,6 nC gemessen. Spuleund Messgerät haben den GesamtwiderstandR 504 W. Berechnen Sie daraus die spezifische

Ladung eines Elektrons (Tolman-Versuch!).(Anleitung: Die Kräfte Fmech m a m Dv=Dtund FelQE sind im Gleichgewicht; der messbareSpannungsstoû ist U Dt R I Dt R DQ.)

Lösung:Fmech Fel, maQ U=l;

Q=m a l=U (Dv=Dt) (2p r n)=U 2p f r 2 p r n=(UD t) 4 p2 r2 f n=(RDQ) 1,9 ´ 1011 C=kg.

Zusatzaufgabe

*4 Obwohl die Driftgeschwindigkeit von Elektronenin einem metallischen Leiter sehr klein ist, leuchteteine elektrische Lampe fast sofort nach demSchlieûen des Schalters auf. Erklären Sie dies.

Lösung:Elektronen bewegen sich, weil im Leiter ein elekt-risches Feld vorhanden ist; das Feld breitet sich mitLichtgeschwindigkeit aus.Auch im Wassermodell ist eine Erklärung möglich:obwohl sich die einzelnen Wasserteilchen langsambewegen, strömt in einem Rohr bei Entstehen einerDruckdifferenz ¹sofortª das gesamte Wasser.

5.4.4 Austritt von Elektronen aus Leiteroberflächen

1 Nennen Sie Unterschiede und Gemeinsamkeitenbeim Vergleich des Elektronenaustritts aus Metall-oberflächen mit dem Austritt von Wassermole-külen aus der Wasseroberfläche.

Lösung:Gemeinsam: Der Austritt ist nur für energiereicheTeilchen möglich.Verschieden: Auch bei niedrigen Temperaturenkönnen einzelne Wassermoleküle austreten (Ge-schwindigkeitsverteilung).

2 Durch eine Elektronenröhre flieût ein Strom von8,5 mA. Wie viele Elektronen treffen pro Sekundeauf die Anode?

Lösung:I 8,5 mA 8,5 ´ 10ÿ6 C=s 8,5 ´ 10ÿ6=(1,6 ´ 10ÿ19) e 1=s 5,3 ´ 1013 e 1=s

3 Erklären Sie, weshalb in einer Elektronenröhre beikonstanter Anodenspannung die Anodenstrom-stärke gröûer wird, wenn die Heizspannung erhöhtwird.

Seite

213

Seite

215

5 Ladung und elektrisches Feld104

Page 105: Metzler Physik

Lösung:Bei höherer Temperatur der Kathode treten mehrElektronen aus; es steht also mehr Ladung für denLeitungsvorgang in der Röhre zur Verfügung.

5.4.6 Ablenkung eines Elektronenstrahls im elektrischen Feld

1 Berechnen Sie die Geschwindigkeit und die kine-tische Energie von Elektronen, die eine Beschleu-nigungsspannung U 300 V im Vakuum durch-laufen haben.

Lösung:

v2(e=m) U

p 1,03 ´ 107 m=s,

Wkin eU 4,8 ´ 10ÿ7 J 0,48 mJ.

2 Berechnen Sie, ohne die Formeln der klassischenPhysik in Frage zu stellen, die Spannung, die einElektron aus der Ruhelage durchlaufen müsste, umLichtgeschwindigkeit (3 ´ 108 m=s) zu erreichen.

Lösung:

U v2

2em

256 kV.

Die Spannung reicht jedoch wegen der relativisti-schen Massenveränderlichkeit

m m01ÿ v2

c2

r

(siehe Kapitel 9.3.1) nicht aus, Elektronen fast aufLichtgeschwindigkeit zu beschleunigen.

3 Geben sie die Geschwindigkeit eines Elektrons mitder kinetischen Energie Ekin 220 eV in km=s an.

Lösung:12 me v2 Wkin;

v 2Wkin=me

p

2 220 1,602 10ÿ19=(9,11 10ÿ31) J=kg

p

8,80 ´ 106 m=s 8800 km=s.

*4 Wie ändert sich die Bahn geladener Teilchen ineiner Elektronenstrahl-Ablenkröhre, wenn sichstatt der Elektronen

a) Teilchen mit der dreifachen Masse,b) Teilchen mit der dreifachen Ladung oderc) Teilchen mit dreifacher Masse und dreifacher La-

dung bewegten?d) Kann man mit einer Ablenkröhre die Masse oder

die Ladung der Elektronen bestimmen?

Lösung:Gleichung der Bahnkurve

y eUy

2m d v20

; wegen v20

2 eUx

mgilt: y Uy

4 d Uxx2.

Die Bahnkurve ist von der Ladung und von denMassen der Teilchen unabhängig; sie ändert sichalso in den drei Fällen a, b und c nicht.

d) Eine Bestimmung von Masse oder Ladung derElektronen mit der Elektronenstrahl-Ablenk-röhre allein ist daher nicht möglich.

*5 Die Beschleunigungsspannung und die Ablenk-spannung an einer Elektronenstrahl-Ablenkröhreändern sich im gleichen Verhältnis. Wie ändertsich die Elektronenbahn?

Lösung:Aus der Bahngleichung (siehe Lösung zuAufgabe 4) ergibt sich, dass sich die Elektronen-bahn nicht verändert.

*6 Ein Elektron, das die BeschleunigungsspannungUa 150 V durchlaufen hat, fliegt senkrecht zumelektrischen Feld in die Mitte zwischen zwei pa-rallele geladene Platten mit dem Abstandd 15 cm. Zwischen den Platten liegt die Span-nung U 250 V.

a) Wie lange dauert es, bis das Elektron auf eine Platteaufschlägt?

b) Wie weit ist der Auftreffpunkt vom Plattenrandentfernt?

Lösung:

a)d2 1

2a t2

1, a e Em

;

t1da

r

d meE

r

d2

em

U

vuut 2,3 ´ 10ÿ9s2,3 ns.

b) x1 v0 t12

em

Ua

r d2

em

U

vuut 2Ua d2

U

r 1,6 cm.

Seite

219

5.4 Bewegung geladener Teilchen im elektrischen Feld 105

Page 106: Metzler Physik

Zusatzaufgabe

*7 Ein Elektronenstrahl wird mit der Anodenspan-nung Ua 500 V beschleunigt und durch den Ab-lenkkondensator nach Abbildung 218.1 geschickt(d 2 cm, s 10 cm, b 20 cm). Während dieAblenkspannung von U1 ÿ24 V bis U2 40 Vlinear mit der Zeit steigt, wird der Elektronenstrahldurch die Sägezahnspannung vertikal gerade vonx1 ÿ5 cm bis x2 5 cm über den Leuchtschirmabgelenkt.Berechnen Sie die Bahnpunkte und zeichnen Siedie Bahn des Leuchtflecks auf dem Leuchtschirmin ein x-y-Koordinatensystem ein. (Der Nullpunktdes Systems ist durch den unabgelenkten Elektro-nenstrahl gekennzeichnet.)

Lösung:

y1 U1 s

b s

2

2 d Uaÿ 3 cm,

y2 U2 s

b s

2

2 d Ua 5 cm.

y in cm

5

5

–5

–3

x in cm

5.4.7 Leitungsvorgänge in Gasen

1 Die bekannteste Funkenentladung ist der Blitz.Geben Sie die Wirkungsweise eines Blitzableitersan.

Lösung:In der Spitze des Blitzableiters wird durch diegeladene Gewitterwolke die entgegengesetzteLadung influenziert. An der Spitze bildet sich einstarkes Feld aus, das eine selbstständige Entladungbewirkt (Spitzeneffekt). Dadurch wird die Luftleitend und dem Blitz ein ungefährlicher Weg überden Blitzableiter zur Erde gebahnt.

2 Warum verlöscht ein Lichtbogen, wenn seineLänge einen bestimmten Wert überschreitet?

Lösung:Die elektrische Feldstärke reicht dann nicht mehraus, die zur Stoûionisation erforderliche Energie inder freien Weglänge auf die Ladungsträger zu über-tragen.

Zusatzaufgaben

*3 Ein Blitz hat bei einer Spannung U 100 MV zwi-schen Wolke und Erde eine Stromstärke I 10 kA.Die Dauer einer Entladung ist t 10ÿ4 s. Berech-nen Sie die bewegte Ladung, die elektrische Ener-gie und die ¹Kostenª eines Gewitters, das 100Blitze entlädt. (1 kWh kostet 0,30 DM.)

Lösung:Q I t 1C;WelU I t 108 J 28 kWh, (1 kWh 3,6 ´ 106 J);Preis: 840 DM.

*4 Geben Sie Beispiele für die Verwendung von Fun-kenstrecken zum Schutz gegen Überspannung an.

Lösung:Z. B. bei Transformatoren und Schaltern.

Seite

221

5 Ladung und elektrisches Feld106

Page 107: Metzler Physik

6.1.2 Magnetische Feldstärke

l Die Horizontalkomponente der Feldstärke B desmagnetischen Erdfeldes beträgt ungefähr BH 19 mT. Berechnen Sie die Kraft auf eine in Ost-West-Richtung verlaufende Freileitung (I 100 A, Abstand zwischen zwei Masten a 150 m).

Lösung:~I ?~BH, Fm BH I a 0,29 N.Da die Horizontalkomponente von ~B betrachtetwurde, ist Fm die Kraft in vertikaler Richtung.Die Kraft ist wesentlich kleiner als das Gewichtder Leitung.

2 Ein gerader Draht von 0,5 m Länge verläuft lot-recht und wird von einem Strom von 6 A von untennach oben durchflossen. Er ist von einem Magnet-feld der Stärke B 70 mT umgeben, das horizontalnach Norden gerichtet ist. Geben Sie Betrag undRichtung der auf ihn wirkenden magnetischenKraft an.

Lösung:~I ?~B, Fm BI l 2,1 ´ 10ÿ4 N 0,21 mN.Die Kraft ist nach Westen gerichtet.

3 Ein waagerechter Draht von 15 cm Länge wird voneinem Strom von 5 A durchflossen. Geben Sie Be-trag und Richtung der magnetischen Feldstärke deskleinsten Magnetfeldes an, das den Draht mit derMasse 4 g in der Schwebe hält.

Lösung:B F=(I l)mg=(I l) 5,2 ´ 10ÿ2 N=(Am).~B ist horizontal, senkrecht zum Draht gerichtet. DieOrientierung von~B ist so zu wählen, dass~I,~B und

die von unten nach oben gerichtete Vertikale in die-ser Reihenfolge ein Rechtssystem bilden.

4 Mit welcher Kraft wirkt ein homogenes Magnet-feld auf einen stromdurchflossenen Draht, der pa-rallel zu den Feldlinien liegt?

Lösung:F 0, da û (~I,~B) 0 oder 180.

*5 Ein von einem Strom I 4 A durchflossener Leiterder Länge l 5 cm erfährt in einem homogenenMagnetfeld der Feldstärke B 0,3 T die Kraft F0,04 N. Welchen Winkel bildet der Leiter mit denmagnetischen Feldlinien?

Lösung:F BI l sin a, sin a F=(BI l) 0,6667,a 41,8.

Zusatzaufgabe

*6 In einem horizontalen Magnetfeld der Stärke B 0,2 Vs=m2 befindet sich unter einem Winkel von30 ein ebenfalls horizontal verlaufender geraderDraht von 75 cm Länge, der von einem Strom von10Adurchflossenwird.GebenSieBetragundRich-tung der auf ihn wirkenden magnetischen Kraft an.

Lösung:F BI l sin a 0,75 N.Die Kraftrichtung ist vertikal. Die Orientierung derKraft kann nicht angegeben werden, da über dieOrientierung des Stromes nichts ausgesagt ist.

6.1.3 Lorentz-Kraft

1 Ein Körper mit der Ladung Q ± 0,2 nC bewegtsich in einem waagerecht nach Süden gerichtetenMagnetfeld der Stärke B 50 mT mit der Ge-schwindigkeit v 3 km=s in Richtung Westen. Ge-ben Sie Betrag und Richtung der magnetischenKraft auf den geladenen Körper an.

Lösung:F jQvBj 3 10ÿ8 N. Wegen ~F Q~v~B undQ < 0 ist die Kraft senkrecht nach unten gerichtet.

2 Geben Sie Betrag und Richtung der Stärke B desMagnetfeldes an, das das Gewicht eines Elektronsmit einer waagerecht nach Westen gerichtetenGeschwindigkeit von 2 cm=s kompensieren kann.

Lösung:me g evB, B g=( e

mev)2,79 ´10ÿ9T2,79nT.

~B ist horizontal nach Norden gerichtet.

*3 Ein a-Teilchen (Q 2 e) durchläuft eine Beschleu-nigungsspannung U 200 V und tritt dann in einMagnetfeld der Stärke B 0,12 T ein. BerechnenSie die magnetische Kraft für die Fälle, dass dieGeschwindigkeit mit B einen Winkel von a) ò1 90, b) ò2 60, c) ò3 30 und d) ò4 0 ein-schlieût (ma 6,64 ´ 10±27 kg).

Lösung:Q=ma 2 e=(4u) 4,82 ´ 107 C=kg,

Seite

227

Seite

229

6.1 Kräfte im magnetischen Feld 107

Page 108: Metzler Physik

v2(Q=ma) U

p 1,39 ´ 105 m=s;

F Q vB sin ò;

F1 5,34 ´ 10ÿ15 N, F2 4,62 ´ 10ÿ15 N,F3 2,67 ´ 10ÿ15 N, F4 0.

Zusatzaufgaben

4 Wie ist die magnetische Kraft auf ein Elektron ge-richtet, das sich in einem waagerecht nach Nordengerichteten Magnetfeld waagerecht nach Osten be-wegt?

Lösung:Wegen ~F Q~v~B und Q < 0 ist die Kraft senk-recht nach unten gerichtet.

5 Ein Proton besitzt in einem senkrecht nach untengerichteten Magnetfeld der Stärke B 40 Vs=m2

eine nach Westen gerichtete Geschwindigkeit von7500 km=s. Geben Sie Betrag und Richtung derwirkenden Kraft an.

Lösung:Q e, F QvB 4,8 ´ 10ÿ11 N.Die Kraft ist horizontal nach Süden gerichtet.

6.1.4 Der Hall-Effekt

1 Diskutieren Sie die Polung der Hall-Spannung,wenn a) nur positive und b) positive und negativeLadungsträger gleichermaûen frei beweglich wä-ren.

Lösung:a) Wenn nur positive Ladungsträger frei beweg-

lich wären, würde eine Hall-Spannung entgege-gesetzter Polung entstehen (! in Abb. 230.1:oben eine positive Aufladung).

b) Wären positive und negative Ladungsträger freibeweglich, könnte keine Hall-Spannung nach-gewiesen werden.

2 Was können Sie über die Hall-Spannung aussagen,wenn in Abbildung 230.1 die B-Linien parallel zurFläche der Folie verlaufen?

Lösung:UH 0. Begründung: Die Komponente von ~B inRichtung von ~I erzeugt keine Lorentzkraft; dieKomponente von~B in Richtung P1 P2 erzeugt eineLorentzkraft senkrecht zur Folie.

3 Wie kann man experimentell mithilfe einer Hall-Sonde die Richtung der Feldlinien eines Magnet-feldes bestimmen?

Lösung:Die Hall-Sonde wird so lange gedreht, bis die Hall-Spannung maximal ist. Die magnetischen Feld-linien durchsetzen dann die Folie der Sonde senk-recht.

*4 Zwischen den Rändern einer 2,5 cm breiten Metall-folie misst man in einem Magnetfeld B 0,28 T dieSpannung UH 12 mV. Wie schnell sind die Elekt-ronen?

Lösung:vUH=(b B) 1,7 mm=s.

*5 Eine Kupferfolie (d 10 mm) wird von einemStrom der Stärke 10 A durchflossen. Im Magnet-feld B 0,43 T wird die Hall-Spannung UH 22 mV gemessen. Berechnen Sie die Hall-Kon-stante von Kupfer und die Dichte n der Elektronen.Berechnen Sie daraus die Anzahl der freien Elekt-ronen in 1 mol Kupfer und vergleichen Sie sie mitder Avogadro-Konstante.

Lösung:RH UH d=(I B) 5,1 ´ 10ÿ11 m=C,n 1=(RH e) 1,2 ´ 1029 mÿ3.Mit den Molvolumen Vm 7,1 ´ 10ÿ3 m3=kmol er-hält man als Anzahl der Elektronen in 1 kmolN nVm 8,5 ´ 1023 kmolÿ1 (in 1 mol 8,5 ´ 1023).Die Anzahl der freien Elektronen ist gröûer als dieder Atome.

6.1.5 Teilchen auf Kreisbahnen

1 Ein Proton bewegt sich in einem homogenen Mag-netfeld der Stärke B 2 T mit einer Geschwindig-keit v 750 km=s senkrecht zu den Feldlinien.Berechnen Sie den Radius seiner Kreisbahn.

Lösung:evBmp v2=r, r mP v=(eB) 3,9 ´ 10ÿ3 m.

Seite

231

Seite

232

6 Bewegte Ladungsträger und magnetisches Feld108

Page 109: Metzler Physik

2 Die magnetische Feldstärke im homogenen Teildes Helmholtz-Spulenfeldes wird mit einer Hall-Sonde zu B 965 mT bestimmt. Bei einer Be-schleunigungsspannung von U 210 V wird imFadenstrahlrohr der Durchmesser der Kreisbahnd 10,2 cm gemessen. Berechnen Sie die spezifi-sche Ladung e=m der Elektronen.

Lösung:e=m 2 U=(B2 r2) 1,73 ´ 1011 C=kg (vgl. S. 213,Aufgabe 3).

3 Wie kann sich ein geladener Körper durch einMagnetfeld bewegen, ohne dass das Feld eine Kraftauf ihn ausübt?

Lösung:In Richtung der Feldlinien.

4 In welcher Masse Wasserstoff ist 1 g Elektronenenthalten?

Lösung:mH 1 g (mpme)=me 1837g

*5 In einem bestimmten Gebiet des interstellarenRaumes gibt es freie Elektronen mit der kineti-schen Energie 1 meV, die sich auf Kreisbahnenvom Radius 25 km bewegen. Wie groû ist die mag-netische Feldstärke, die diese Bahn verursacht?

Lösung:

v2Wkin=me

p, evBme v2=r;

B me v=(er) 2Wkin mep

=(er) 4,27 ´ 10ÿ12 T 4,27 pT.

6.1.7 Teilchenbeschleuniger

1 Ein a-Teilchen besitzt eine Masse von 6,64 ´ 10±27

kg. Berechnen Sie den Radius der Kreisbahn, dieein a-Teilchen beschreibt, das sich nach Aufgabe 3a) (S. 229) bewegt.

Lösung:Qv Bma v2=r, v

2 (Q=ma)U

p;

r ma v=(QB)

ma U=(eB2)

p 2,4 cm.

2 In einem Zyklotron ist der maximale Krümmungs-radius der Bahnkurve von geladenen TeilchenR 0,8 m. Die magnetische Feldstärke beträgtB 1,5 T. Welche Potentialdifferenz müssten Pro-tonen in einem elektrischen Feld durchlaufen, da-mit sie dieselbe Endgeschwindigkeit wie in demZyklotron erhalten?

Lösung:U 1

2 (e=mp)B2 R2 6,9 ´ 107 V 69 MV.

3 Ein Zyklotron gibt a-Teilchen mit einer Energievon 2,5 ´ 10±12 J ab. Die magnetische Feldstärkebeträgt 2 T. Berechnen Sie den gröûten Krüm-mungsradius der Bahnkurven dieser a-Teilchen.

Lösung:

v2Wkin=ma

p, Q vBma v2=r;

r ma v=(QB)

2 ma Wkinp

=(QB) 0,28 m.

4 In einem Bereich des Strahlungsgürtels der Erdeist die magnetische Feldstärke des ErdfeldesB 100 mT. Mit welcher Frequenz umlaufen dorta-Teilchen die magnetischen Feldlinien?

Lösung:Qv Bm v2=r, Qv Bm wv, w (Q=m)B;f QB=(2 pm) 2eB=(2pm) 760 sÿ1.

Zusatzaufgaben

*5 Das elektrische Feld zwischen den Platten einesGeschwindigkeitsfilters in einem Massenspektro-skop nach Abbildung 233.1 hat die Stärke E 0,12 kV=m. Die beiden Magnetfelder haben dieStärke B 0,6 T.a) Ein Strahl einfach ionisierter Neonatome be-

wegt sich auf einer Kreisbahn mit einem Radiusvon 7,28 cm. Bestimmen Sie die Masse desNeonisotops.

b) Ein anderes (häufigeres) Neonisotop besitzt dieMasse m 3,32 ´ 10±26 kg. Welchen Radiusbeschreibt ein Strahl aus solchen Neonisoto-pen?

Lösung:v E=B 2 ´ 105 m=s;

a) evBmNe v2=r,mNe eBr=v 3,50 ´ 10ÿ26 kg.

b) rmNe v=(eB) 6,91 cm.

*6 Ist ein Massenspektroskop besser geeignet, dieIsotope leichter oder die Isotope schwerer Ele-mente zu trennen?

Seite

235

6.1 Kräfte im magnetischen Feld 109

Page 110: Metzler Physik

Lösung:r m v=(eB), D r (v=eB)Dm;bei unverändertem Geschwindigkeitsfilter und un-verändertem Ablenkfeld gilt für Ionen gleicher La-dung: D r Dm.Daher ist die Auflösung eines Massenspektrogra-phen von der Gesamtmasse m unabhängig.

*7 In einem Zyklotron wird bei einem Magnetfeld derStärke B zur Beschleunigung von Protonen eineHochfrequenz von 10 Mhz benötigt. Welche Fre-quenz ist erforderlich, um a) Deuteronen (mD 2 mP, Q e), b) einfach ionisierte Heliumatome

He+ (mHe 4 mP) und c) zweifach ionisierte He-liumatome He2+ (a-Teilchen) bei gleich starkemMagnetfeld zu beschleunigen?

Lösung:Qv Bm w r, f w=(2p)QB=(2pm);B (mp=e) 2 p f 6,5 T.

a) f1 e=(2 mp) B=(2 p) f =2 5 MHz,

b) f2 e=(4 mp) B=(2 p) f =4 2,5 MHz,

c) f3 2e=(4 mp) B=(2 p) f=2 5 MHz.

6 Bewegte Ladungsträger und magnetisches Feld110

Page 111: Metzler Physik

6.2.1 Magnetfelder eines langen Leiters und einer langen Spule

1 Zwei geradlinige lange Leiter verlaufen in einemAbstand von 10 cm parallel zu einander. Sie wer-den in entgegengesetzter Richtung von den Strö-men I1 15 A und I2 25 A durchflossen. Berech-nen Sie die magnetische Feldstärke in einem Punktin der von den Leitern aufgespannten Ebene, dera) von beiden Leitern gleich weit entfernt ist; b)2 cm von Leiter 1 und 8 cm von Leiter 2 entferntist; c) 2 cm von Leiter 1 und 12 cm von Leiter 2entfernt ist. d) In welchen Punkten ist die magneti-sche Feldstärke gleich null?

Lösung:a) r1 r2 5 cm;

B (m0=(2p)) (I1=r1 I2=r2) 1,6 ´ 10ÿ4 T 160 mT;

b) r1 2 cm, r2 8 cm;B (m0=(2p)) (I1=r1 I2=r2) 2,13 ´ 10ÿ4 T 213 mT;

c) r1 2 cm, r2 12 cm;B (m0=(2p)) (I1=r1 I2=r2) 1,08 ´ 10ÿ4 T 108 mT;

d) B ist gleich null für I1=r1 I2=r2 undr2 r1 10 cm, also r1 15 cm, r2 25 cm.

2 Lösen Sie Aufgabe 1 für den Fall, dass die beidenStröme gleich gerichtet sind.

Lösung:a) r1 r2 5 cm;

B j(m0=(2p)) (I1=r1ÿ I2=r2) j 4,0 ´ 10ÿ5 T 40 mT;

b) r1 2 cm, r2 8 cm;B (m0=(2 p)) (I1=r1ÿ I2=r2) 8,75 ´ 10ÿ5 T 87,5 mT

c) r1 2 cm, r2 12 cm;B (m0=(2 p)) (I1=r1ÿ I2=r2) 1,92 ´ 10ÿ4 T 192 mT

d) B ist gleich Null für I1=r1 I2=r2 undr1 r2 10cm,also r1 3,75cm,r2 6,25cm.

3 In einer Spule (l 70 cm, n 300) wird bei derStromstärke I 1,5 A die magnetische FeldstärkeB 840 mT gemessen. Berechnen Sie daraus diemagnetische Feldkonstante m0.

Lösung:m0exp Bl=(I n) 1,31 ´ 10ÿ6 Vs=(Am).

*4 Durch einen langen geraden senkrechten und einenlangen geraden waagerechten Draht flieût der glei-che Strom I. Der Abstand beider Drähte beträgt 2 r.Geben Sie Betrag und Richtung der magnetischen

Feldstärke für den Mittelpunkt ihres Verbindungs-lotes an.

Lösung:B1 B2 m0 I=(2p r); û(~B1,~B2) 90;B

2B2

1

p

2p

m0 I=(2p r).~B liegt in der zum Verbindungslot senkrechtenEbene und ist um 45 gegen die Horizontale ge-neigt.

*5 Durch Kompensation mit einer langen Spule solldie Horizontalkomponente BH der magnetischenFeldstärke des Erdfeldes bestimmt werden. Miteiner in Nord-Süd-Richtung aufgestellten Spule(n 340, l 60 cm) wird mit einer StromstärkeI 31 mA Feldfreiheit erreicht.a) Bestimmen Sie BH.b) Berechnen Sie daraus mit dem Inklinationswin-

kel ò 67 den Betrag der gesamten magneti-schen Feldstärke des Erdfeldes.

Lösung:a) BH m0 I n=l 2,21 ´ 10ÿ5 T 22,1 mT.b) Bges BH=cos ò 5,65 ´ 10ÿ5 T 56,5 mT.

Zusatzaufgaben

6 Durch einen langen Leiter flieût ein Strom von 6 A.Wie groû ist die magnetische Feldstärke in einemPunkt, der 2,5 cm von ihm entfernt ist?

Lösung:B m0 I=(2p r) 4,8 ´ 10ÿ5 T 48 mT.

7 Zwei geradlinige unendlich lange Leiter verlaufenin einem Abstand von 20 cm parallel zueinander.Sie werden von den Strömen I1 I2 10 A in ent-gegengesetzter Richtung durchflossen. Geben SieBetrag und Richtung der magnetischen Feldstärkein einem Punkt an, der von jedem Leiter den Ab-stand 20 cm besitzt.

Lösung:B1 B2 m0 I=(2p r) 1,0 ´ 10ÿ5 T 10 mT.û (~B1,~B2) 120;B 2 B1 cos 60 10 mT.~B zeigt senkrecht auf die Ebene, die durch die bei-den Leiter aufgespannt wird.

*8 Weshalb ziehen sich zwei parallele stromdurch-flossene Leiter bei gleicher Stromrichtung an undstoûen sich bei entgegengesetzter Stromrichtungab?

Seite

241

6.2 Ströme als Ursache von Magnetfeldern 111

Page 112: Metzler Physik

Lösung:Man betrachte zur Begründung jeweils den einenLeiter als stromdurchflossenen Leiter in dem vomanderen Leiter erzeugten Magnetfeld (s. Abb.227.2).

*9 Mit einer Geschwindigkeit von 5450 km=s bewegtsich ein Elektron parallel zu einem langen geradenDraht, der von einem Strom von 35 A durchflossenwird. Wie groû ist die Kraft auf das Elektron, wennes 12 cm vom Draht entfernt ist?

Lösung:~v?~B, B m0 I=(2p r) 5,8 ´ 10ÿ5 T 58 mT,F ev B 5,09 ´ 10ÿ17 N 50,9 aN.

*10 Eine senkrecht aufgehängte Spiralfeder ist Teileines Stromkreises. Das am Ende der Feder hän-

gende metallische Gewichtsstück berührt eineKontaktstelle, über die der Stromkreis geschlos-sen ist. Was ist beim Einschalten des Stromes zubeobachten? Welche Probleme ergeben sich dar-aus für den Bau von Spulen zur Erzeugung starkerMagnetfelder?

Lösung:Die vom Strom durchflossenen Spulenwindun-gen ziehen einander an; der Stromkreis wird un-terbrochen.Spulen zur Erzeugung starker Magnetfelder müs-sen fest gewickelt sein, um mechanische Auswir-kungen dieser Anziehungskräfte zu vermeiden. InSpulen, die mit Wechselspannung betrieben wer-den, wirken die Kräfte rhythmisch mit einer Fre-quenz von 100 Hz.

6.2.2 Das Linienintegral der magnetischen Feldstärke

1 Ein langer gerader Draht werde von einem Stromder Stärke 10 A durchflossen. Bestimmen Sie denBetrag der magnetischen Feldstärke B im Abstanda) 10 cm, b) 30 cm und c) 3 m.

Lösung:B m0 I=(2p r) 2 10ÿ7 (Vs=A m) (I=r)a) B 20 mT;b) B 6,7 mT;c) B 0,67 mT.

2 Ein gerader von einem Strom I durchflossener Lei-ter ist an einer Stelle zu einer Kreisschleife aufge-spalten. Welches Magnetfeld herrscht im Mittel-punkt der Schleife?

Lösung:

12 I

I

12 I

I

B 0:

*3 Ein langes gerades Koaxialkabel besteht aus einemInnenleiter und einer konzentrischen zylindrischenAbschirmung mit dem Radius R. Das Innenkabelsei an einem Ende mit der Abschirmung verbun-den. Am anderen Ende seien Innenkabel und Ab-schirmung mit den Polen einer Spannungsquelleverbunden. Bestimmen Sie B a) im Raum zwischenAbschirmung und Innenkabel; b) auûerhalb derAbschirmung.

Lösung:In beiden Leitern flieûen gleich starke, aber entge-gengerichtete Ströme I.Anwendung des AmpeÁre'sche Gesetzesb~B d~s m0 I:a) B m0 I=(2p r);b) B 0.

Seite

243

6 Bewegte Ladungsträger und magnetisches Feld112

Page 113: Metzler Physik

6.2.3 Materie im magnetischen Feld

1 Im Feld einer Spule (n 500, l 0,60 m) wird beieiner Stromstärke I 1,2 A mit Eisenfüllung dieFeldstärke B 0,75T gemessen. Berechnen Sie diePermeabilitätszahl mr für Eisen.

Lösung:B0 m0 I n=l 1,26 ´ 10ÿ3 T; mr B=B0 600.

2 Ferromagnetische und paramagnetische Stoffewerden von einem Magneten angezogen, diamag-netische Stoffe werden abgestoûen. Versuchen Sie,diese Beobachtung aufgrund der in diesem Kapitelbeschriebenen Eigenschaften dieser Stoffe zu er-klären.

Lösung:Die Stoffe werden im Magnetfeld magnetisiert,ferro- und paramagnetische Stoffe in Richtungdes Feldes, diamagnetische Stoffe dem Feld entge-gengerichtet. Das wird durch die Modellvorstel-lung elementarer Kreisströme von Elektronen imAtomverband deutlich: Beim Ferro- und Paramag-netismus handelt es sich um die Ausrichtung vor-handener Kreisströme, beim Diamagnetismus umeinen Induktionsvorgang (s. Kap. 6.3.).

Seite

245

6.2 Ströme als Ursache von Magnetfeldern 113

Page 114: Metzler Physik

6.3.1 Das Induktionsgesetz

1 In einem homogenen magnetischen Feld der Stärke0,2 T befindet sich senkrecht zu den Feldlinien einekreisförmige Leiterschleife mit dem Radius 4,5 cmund einem Widerstand von 0,32 W. Die magneti-sche Feldstärke nimmt linear in 3 ms auf null ab.Welcher Strom flieût während dieses Vorgangsdurch die Schleife?

Lösung:

U A _B p r2DB=D t 0,42 V, I Ui=R1,3 A.

2 In einem Magnetfeld der Stärke B 0,58 T wird inder Zeit Dt 0,10 s die Fläche einer kreisförmigenLeiterschleife (d 10,5 cm) halbiert. BerechnenSie die induzierte Spannung, wenn die Fläche Aa) senkrecht zu ~B steht, b) mit ~B den Winkela 30 einschlieût und c) parallel zu ~B liegt.

Lösung:a) U B _A Bp r2=(2D t) 25 mV,b) U B _A sin ò 12,5 mV,c) U 0.

3 In einer Spule (n 2000, l 3,1 cm, d 4,8 cm)wird die magnetische Feldstärke B 27 mT in 2 sgleichmäûig auf null geregelt. Berechnen Sie dieinduzierte Spannung.

Lösung:U nA _B np r2 DB=Dt 49 mV.

*4 Eine kreisförmige Leiterschleife (d 6,5 cm) liegtin einem homogenen Magnetfeld der Stärke B 33 mT. Berechnen Sie den magnetischen Fluss fürdie Fälle, dass der Winkel zwischen der Leiter-schleife und der Feldstärke ~B a) ò1 90,b) ò2 60, c) ò3 45, d) ò4 0 unde) ò5 180 beträgt.

Lösung:F AB cos ò p r2 B cos ò;a) F1 0, b) F2 5,5 ´ 10ÿ5 Tm2,c) F3 7,7 ´ 10ÿ5 Tm2, d) F4 1,1 ´ 10ÿ4 Tm2,e) F5 ÿ 1,1 ´ 10ÿ4 Tm2.

6.3.2 Energieerhaltung und das Vorzeichen im Induktionsgesetz

1 Ein horizontal gerichtetes homogenes Magnetfeldzeigt nach Norden. Senkrecht zu seinen Feldlinienbefindet sich in waagerechter Lage ein Metallstab.Welches Ende des frei fallenden Stabes wird nega-tiv aufgeladen?

Lösung:Das nach Westen gerichtete Ende des Stabes wirdnegativ aufgeladen.

*2 Die 6,0 m langen Rotorblätter eines Hubschraubersdrehen sich horizontal mit 9 Umdrehungen je Se-kunde an einem Ort, an dem die senkrecht nachunten gerichtete Komponente der Feldstärke desmagnetischen Erdfeldes 58 mT beträgt. Wie groûist die zwischen Drehachse und Flügelspitzen in-duzierte Spannung?

Lösung:U _AB (DA=D t) B (p r2=T)B p r2 f B 59 mV.

3 Ein Stabmagnet wird in eine Spule hineingeführt.Bestimmen Sie (für alle Möglichkeiten) die Rich-tung des Induktionsstromes a) nach der Lenz'schenRegel und b) mithilfe der Lorentz-Kraft.

Lösung:Ansatz: Einem Heranführen des Stabmagneten ent-sprichta) eine Vergröûerung des magnetischen Flusses in

der Spule undb) eine Bewegung der freien Elektronen in den

Spulenwindungen relativ zum Magnetfeld.

4 Eine Spule wird in axialer Richtung auf eine strom-durchflossene Spule gleicher Bauart zu bewegt.Flieût der in ihr induzierte Strom in gleicher oderin entgegengesetzter Richtung wie der Strom in derzweiten Spule?In welcher Richtung flieût der Induktionsstrom,wenn die stromdurchflossene Spule von der erstenentfernt wird?

Lösung:Annäherung der Spulen: Der Induktionsstromflieût in entgegengesetzter Richtung. Entfernender Spulen voneinander: Der Induktionsstromflieût in der gleichen Richtung.

*5 An einem Pendel schwingt eine senkrecht zu denFeldlinien gerichtete Aluminiumscheibe durch einbegrenztes Magnetfeld hindurch. Beschreiben Sieden Vorgang a) mithilfe der Lenz'schen Regel und

Seite

249

Seite

251

6 Bewegte Ladungsträger und magnetisches Feld114

Page 115: Metzler Physik

b) mit der Lorentz-Kraft. (In der Scheibe treten so-genannte Wirbelströme auf.)

Lösung:a) Es wird ein Strom induziert (Wirbelströme in

der Scheibe), der so gerichtet ist, dass er derUrsache seiner Entstehung (Pendelbewegung)entgegenwirkt: Das Pedal wird im Magnetfeldgebremst. Das geschieht dadurch, dass dasdurch die induzierten Kreisströme erzeugteMagnetfeld beim Hineinpendeln den äuûerenMagnetfeld entgegengerichtet und beim Her-auspendeln ihm gleichgerichtet ist.

b) Man betrachte jeweils die Richtung der Lo-rentzkraft, die auf die freien Elektronen amRande der Scheibe beim Eintreten in das Mag-

netfeld bzw. beim Verlassen des Magnetfeldeswirkt.

*6 Durch eine waagerechte Leiterschleife fällt einStabmagnet. Beschreiben Sie qualitativ den Ver-lauf der induzierten Spannung. Wie wirkt der In-duktionsstrom auf die Bewegung des Stabmagne-ten?

Lösung:Die Induktionsspannung wächst zunächst an, sinktdann auf Null, wächst mit entgegengesetztem Vor-zeichen wieder an und sinkt erneut auf Null.Die Beschleunigung des fallenden Magneten wird(geringfügig) vermindert.

6.3.3 Selbstinduktion

1 In einer luftgefüllten Spule (l 70 cm, n 500,d 12 cm) wird die Stromstärke in der Zeit Dt 2 svon I1 1 A auf I2 8 A gesteigert. Berechnen Siedie Induktivität der Spule und die Selbstinduk-tionsspannung.

Lösung:L p0 n2 A=l p0 n2 p r2=l 5,1 ´ 10 ÿ3 H 5,1 mH,

UL L _I L DI=D t 1,8 ´ 10ÿ2 V 18 mV.

2 In einer Spule (n 700, l 30 cm, d 4 cm,mr 200) beträgt die Stromstärke I 5 A. Berech-nen sie die Induktivität und die beim Ausschalten(Dt 0,02 s) induzierte Spannung.

Lösung:L mr m0 n2 A=l mr m0 n2 p r2=l 0,52 H,

UL L _I L D I=D t 130 V.

3 Eine Spule (Querschnittsfläche A 20 cm2, n 600, l 40 cm) hat mit Eisenkern bei I 6 A eineInduktivität L 2 H. Wie groû ist die Permeabili-tätszahl des Eiskerns unter diesen Bedingungen?

Lösung:L mr m0 n2 A=l, mr Ll=(m0 n2 A) 880.

4 Ein starker mit Gleichstrom betriebener Elektro-magnet wird abgeschaltet. Was könnte passieren?

Lösung:Durch hohe Induktionsspannung kann es zum Fun-kenüberschlag kommen.

5 Wie muss man eine Spule wickeln, damit sie zwareinen Widerstand R, aber möglichst keine Indukti-vität L besitzt?

Lösung:Gegensinnig. Darunter versteht man, dass mannach Wickeln der ersten Hälfte des Spulendrahtesdie zweite Hälfte rückläufig parallel zur ersten wi-ckelt, sodass die Ströme gegenläufig sind und sichdie von der ersten und zweiten Hälfte erzeugtenMagnetfelder kompensieren.

6* In welcher Zeit nach dem Einschalten erreicht einStrom I in einem RL-Kreis die Hälfte seines Maxi-malwertes? Hängt diese Zeit von der angelegten(konstanten) Spannung ab?

Lösung:

ÿU0

R(1ÿ eÿ

RL tH )ÿ1

2U0

R; eÿ

RL tH 1

2; tH

LR

ln 2

Die Zeit hängt nicht von der Spannung U0 ab.

Seite

253

6.3 Elektromagnetische Induktion 115

Page 116: Metzler Physik

6.3.4 Energie des magnetischen Feldes

1 Eine Spule (n 230, l 20 cm, A 15 cm2) wirdvon einem Strom der Stärke I 5 A durchflossen.Berechnen Sie die magnetische Energie ihres Fel-des, wenn a) Luft und b) ein Eisenkern mit mr 200in der Spule ist.

Lösung:L mr m0 n2 A=l, Em 1

2 LI2 mr m0 n2 AI2=(2 l);

a) Em 6,2 ´ 10ÿ3 J = 6,2 mJ,b) Em 1,25 J.

2 Beschreiben Sie ein homogenes elektrisches Feld,das jeweils die gleiche Energie wie das in Aufgabe 1gegebene magnetische Feld speichert.

Lösung:Für das Feld eines Plattenkondensators gilt:Eel 1

2 C U2 12 e0 (A=d)U2 Em,

(A=d)U2 2Em=e0, A (2Em=e0) (d=U2);

a) (A=d) U2 1,4 ´ 109 V2 m; für z. B. U 220 Vund d 1 mm ist A 29 m2.

b) (A=d) U2 2,8 ´ 1011 V2 m; für z. B. U 220 Vund d 1 mm ist A 5800 m2.

3 Was kann man über die Energiedichte an den En-den einer langen Spule gegenüber der im Innerensagen, wenn die Feldstärke B dort auf die Hälfte desInnenwertes zurückgegangen ist?

Lösung:B 1

2 Bi; wegen rm 12 B2=(mr m0) ist rm 1

4 rmi.

*4 Bestimmen Sie a) die magnetische Energie, b) dieelektrische Energie und c) die Gesamtenergie in

einem Volumen von 1 m3, in dem ein elektrischesFeld von 105 V=m und ein Magnetfeld von 1,5 Therrschen.

Lösung:a) Wmag rmag V 1

2 B2 V=m0 9,0 ´ 105 J 0,9 kJ,

b) Wel rel V 12 e0 E2 V 0,044 J 4,4 mJ,

c) Wges WmagWel 0,9 kJ.

*5 Man kann in Luft elektrische Felder mit Feldstär-ken bis zu 107 V=m und magnetische Felder mitmagnetischen Feldstärken bis zu 3 T erzeugen. Be-rechnen Sie die Energiedichten dieser Felder.

Lösung:rel 1

2 e0 E2 4,4 ´ 102 J=m3,

rmag 12 B2 =m0 3,5 ´ 106 J=m3.

*6 Der Heizwert von 1 kg Steinkohle beträgt 29 230kJ. Berechnen Sie die Energiedichte von Stein-kohle.

Lösung:Dichte von Steinkohle: r 1300 kg/m3.rE E=V E r=m 3,8 ´ 1010 J=m3.

*7 Eine 12 V-Autobatterie speichert eine Ladung von88 Ah. Die Zellen haben ein Volumen von 7,8 dm3.Berechnen Sie die Energiedichte.

Lösung:rE E=V 88 3600 12=(7,8 10ÿ3) AsV=m3

4,9 ´ 108 J=m3.

6.3.5 Kopplung von elektronischem und magnetischem Feld

*1 Ein ebener Kreisring wird von einem homogenenelektrischen Feld durchdrungen, das zeitlich ab-nimmt. Weist das magnetische Feld in Richtungdes Uhrzeigersinns oder in die entgegengesetzteRichtung, wenn man in die Richtung des elektri-schen Feldes blickt?

Lösung:Gegen den Uhrzeigersinn.(Vgl. Abb. 257.1 und Bild auf S. 259.)

*2 Zeigen Sie, dass für den Verschiebungsstrom imZwischenraum eines Plattenkondensators IV CUÇ gilt.

Lösung:Da sich ~A und d nicht verändern und ~E jj~A, gilt:

IV e0d (~E ~A)

dt e0

d (E A)dt e0

d ( Ud A)

dt

e0Ad

_U C _U.

*3 Zeigen Sie, dass das elektrische Wechselfeld desPlattenkondensators in Abb. 257.1 von einemMagnetfeld umgeben sein muss. (Anleitung:Zeichnen Sie in Abb. 257.1 einen geeigneten Inte-grationsweg, der sowohl den Leiter als auch denRaum des Plattenkondensators teilweise um-schlieût und für den gilt: b~B d~s 0.)

Seite

255

Seite

257

6 Bewegte Ladungsträger und magnetisches Feld116

Page 117: Metzler Physik

Lösung:

I

E B

Kondensatorplatten

IA

P

1

2 3

4P

P

P

Es gilt (siehe Abbildung)

b~B d~s

RP2

P1

~B d~sRP3

P2

~B d~sRP4

P3

~B d~sRP1

P4

~B d~s 0,

weil durch die geschlossene Schleife kein Strom Iflieût und weil der elektrische Fluû ~E ~A durch die

Schleife stets null ist, alsod(~E ~A)

dt 0

(Erweitertes AmpeÁre'sches Gesetz).Da jedoch das Wegstück P1 P2 für das Linieninteg-ral einen Beitrag ungleich null liefert und die Bei-träge der Wegstücke P2 P3 und P4 P1 gleich null

sind, mussRP4

P3

~B d~s ÿRP2

P1

~B d~s= 0 sein.

Das Wechselfeld des Kondensators ist also voneinem Magnetfeld umgeben.

*4 Zählen Sie alle Analogien auf, die zwischen einerSpule (für magnetische Felder) und einem Platten-kondensator (für elektrische Felder) bestehen.

Lösung:

Spule Kondensator

Träger einesFeldes

magnetisches elektrisches

Feldstärke B mr m0

nIl

E 1er e0

QA

Energie Wmagn: 12

L I2 Wel 12

C U2

Energiedichte rm 12

1mr m0

B2 rel 12

er e0 E2

Eigenschaft Induktivität

L mr m0

n2 Al

Kapazität

C er e0Ad

Verstärkungdurch Medium

Permeabilitätszahlmr

Dielektrizitätszahler

Einschalt-vorgang

I(t)ÿI0(1ÿeÿRLt)

UL(t)ÿU0 eÿRLt

I(t)ÿI0eÿ1

RC t

UC(t)ÿU0 (1ÿ eÿ

1RCt)

Ausschalt-vorgang

I(t)ÿI0 eÿRL t

UL(t)U0 eÿRL t

I(t) I0 eÿ1

RC t

UC(t)ÿU0 eÿ1

RC t

Zeitkonstante t LR

t RC

Halbwertszeit tH LR

ln 2 tH RC ln 2

6.3 Elektromagnetische Induktion 117

Page 118: Metzler Physik

7.1.1 Erzeugung von Wechselspannung

1 Begründen Sie anschaulich sowohl mit der magne-tischen Flussänderung als auch mit der Lorentz-Kraft, bei welchen Stellungen der rotierendenSpule die induzierte Spannung ihren Scheitelwertbzw. ihren Nulldurchgang erreicht.

Lösung:

υ

υ

B

υ υ

Die induzierte Spannung ist . . .. . . am gröûten, . . . Null,

a) weil der die Spule durchsetzende magnetischeFluss . . .sein Vorzeichen ändert, . . . sich nicht ändert

b) weil der Leiter sich . . .. . . senkrecht . . . parallelzum Magnetfeld bewegt.

2 Wie könnte man mit der Versuchsanordnung inAbb. 260.1 Betrag und Richtung der Feldstärke Bdes magnetischen Erdfeldes ermitteln?

Lösung:Die Rotationsachse der Spule sei zunächst horizon-tal von West nach Ost ausgerichtet. Man dreht dieSpule um die Vertikale, bis die induzierte Span-nung maximal ist; der Winkel, den die Rotations-achse mit der West-Ost-Richtung einschlieût, istdie Deklination. Mit der Formel uà n B A w kannder Beitrag von B berechnet werden. Um die Inkli-nation zu bestimmen, dreht man die Spule 90 umdie Vertikale, sodass die Rotationsachse in der ma-gnetischen Meridianebene liegt. Nun dreht man dieRotationsachse in dieser Ebene, bis die Induktions-spannung wieder maximal ist. Der Winkel zwi-schen der Spulenachse und der Horizontalen istdie Inklination. (Das Vorzeichen des Feldes ist da-bei allerdings noch nicht ermittelt, demzufolge dasmagnetische Erdfeld insgesamt von Süden nachNorden zeigt.)

3 Im homogenen Feld eines Magneten dreht sich eineSpule mit 300 Umdrehungen pro min. Wie groû istdie induzierte Scheitelspannung ß, wenn die Win-dungszahl n 550, die Spulenfläche A 20 cm2

und die magnetische Feldstärke B 750 mT be-tragen?

Lösung:uà nBAw nBA 2 p f 550 ´ 750 ´ 10ÿ3 T ´ 20 ´ 10ÿ4 m2 ´ 2 p 300=60 s 25,9 V.

4 Mit einer rotierenden Spule wird die magnetischeFeldstärke eines Magneten gemessen. Die Drehfre-quenz wird zu f 800 Hz bestimmt. Wie groû istB, wenn n 1000, A 4 cm2 und die Scheitelspan-nung u 810 mV betragen?

Lösung:Aus uà nBAw folgtB uÃ=(2p f nA) 810 10ÿ3 V=(2p800 Hz 1000 4 10ÿ4 m2) 0,403 mT.

5 Leiten Sie für den in Abb. 260.1 gezeichneten Auf-bau die induzierte Wechselspannung auch mit derLorentz-Kraft FL evB (! 6.1.2) her.

Lösung:Die Momentanspannung u lässt sich auch mit derLorentz-Kraft FL ev B (! 6.1.3) herleiten. DieLorentz-Kraft FL wirkt auf die durch die Drehungder Spule mit bewegten Leitungselektronen, wobeinur die Geschwindigkeitskompetente v wirksamist, die senkrecht zum Vektor der magnetischenFluûdichte B gerichtet ist. So gesehen, bewegt sichin Abb. 260.1 der vordere Teil der Spule nach hin-ten, der hintere nach vorn. Nach der Schraubenre-gel (! Abb. 226.2) wirkt daher im vorderen Teildie Lorentz-Kraft nach links auf die Leitungselekt-ronen, im hinteren Teil nach rechts. In den Längs-seiten a der Spule, wo Leiter und Kraft parallel ge-richtet sind, kommt es zu einer Verschiebung derElektronen und dem zufolge zum Aufbau eineselektrischen Feldes. Die sich daraus ergebendeelektrische Coulomb-Kraft FC eE (E ist dieelektrische Feldstärke) hält der magnetischen KraftFL evB das Gleichgewicht; aus FC FL folgtE vB. Die induzierte Spannung Uind berechnetsich als Produkt aus der elektrischen Feldstärke Eund dem Weg 2 a, längs dessen das Feld vorhandenund dem Weg gleichgerichtet ist:

Seite

261

7 Elektromagnetische Schwingungen und Wellen118

Page 119: Metzler Physik

Uind 2aE 2a Bv

Die Längsseiten der Spule bewegen sich mit derWinkelgeschwindigkeit w auf einer Kreisbahnmit dem Radius b=2. Die Bahngeschwindigkeitist demnach vwb=2; die Komponente dieser Ge-schwindigkeit senkrecht zum magnetischen Feld-vektor ist

vwb2

sin a,

denn für a 0 bewegt sich der Leiter parallel, füra 90 senkrecht zum Feldvektor (Abb. 260.1).Damit erhält man für die Momentanspannung u,wenn man noch die Windungszahl n der Spuleund aw t berücksichtigt.

u nUind n 2aBwb2

sin a nBAw sin w t.

6 Bei einem Fahrraddynamo dreht sich in der Induk-tionsspule ein mehrpoliger Permanentmagnet. Er-

klären Sie anhand eines Modells aus der Physik-sammlung, wieso hier eine Spannung induziertwird.

Lösung:Bei einem Fahrraddynamo dreht sich ein mehr-poliger Permanentmagnet (mit z. B. abwechselnd6 Nord- und 6 Südpolen) in einer Spule, wobei Spu-lenachse und Drehachse des Magneten identischsind. Die magnetische Flussänderung erreichtman durch eine Umhüllung aus Weicheisen, diemit Zungen im Wechsel einmal von oben und dannvon unten in das Innere der Spule reicht und dortabwechseld den Nord- und Südpolen gegenübersteht. Dreht sich der Magnet, so steht jeder Pol ab-wechselnd einer nach oben und dann nach untenreichenden Zunge gegenüber, sodass der Fluss ab-wechselnd von unten nach oben und dann von obennach unten durch die Spule geleitet wird.

7.1.2 Phasenbeziehungen im Wechselstromkreis

1 Zeichnen Sie den zeitlichen Verlauf eines Wech-selstroms und geben Sie die Stellen mit den gröûtenund kleinsten Stromänderungen pro Zeiteinheit an.Skizzieren Sie damit den Spannungsverlauf an ei-ner Spule.

Lösung:

u

i

i > 0

i < 0t

minimal

maximali = 0i > 0

maximal

i = 0

t

t

u ~ ti∆

∆∆∆

∆∆

2 Zeichnen Sie den zeitlichen Verlauf eines Wech-selstroms durch einen Kondensator. Wann hat der

Kondensator seine gröûte positive Ladung, wanndie kleinste negative? Skizzieren Sie damit denSpannungsverlauf am Kondensator.

Lösung:

t

t

negativ null positiv negativnull

u ~ Q

u

Ein positiver Stromlädt den Kondensatorpositiv auf

Ein negativer Stromlädt den Kondensatornegativ auf

Die Kon-densator-ladung ist

i

7.1.3 Wechselstromwiderstände

1 Die Kapazität eines Kondensators lässt sich be-stimmen, indem man bei bekannter Frequenz feiner angelegten Wechselspannung die Effektiv-werte U und I misst.

Berechnen Sie bei folgenden Werten die KapazitätC:a) f 50 Hz, U 6,3 V, I 2,2 mAb) f 3,5 kHz, U 20 V, I 0,88 A

Seite

263

Seite

265

7.1 Wechselstromtechnik 119

Page 120: Metzler Physik

Lösung:Aus XC U=I 1=(w C) folgt C I=(2p f U).Damit wird

a) C 2,2mA2p 50Hz 6,3 V

1,11 mF,

b) C 0,88A2p 3,5kHz 20 V

2,00 mF.

2 Stellen Sie für einen Kondensator von C 10 mFdie Impedanz Z als Funktion der Frequenz vonf 50 Hz bis f 1 kHz grafisch dar. Machen Siedasselbe für eine Spule der Induktivität L 1 H.

Lösung:

Xin

C

L

X =C 15,9 k Hzf

300

200

100

0300 5001000 700 900

Xin k

f in Hz

500300100 700 9000

0

2

4

6X =C 6,28 f

Hz

f in Hz

Ω

Ω

Ω

Ω

3 Berechnen Sie die Induktivität einer Luftspule mit800 Windungen von 60 cm Länge und einerQuerschnittsfläche von A 12 cm2. Wie groû istder Wechselstromwiderstand XL bei f 50 Hz;3,5 kHz; 100 kHz; 8 MHz?

Lösung:L m0 n2 A=l4p10ÿ7 Vs=Am 8002 12 10ÿ4 m2=0,6m1,61 mH.

f 50 Hz 3,5 kHz 100 kHz 8 MHz

XL 2p f L 0,505 W 35,4 W 1,01 kW 80,9 kW

*4 Eine Luftspule von 50 cm Länge wird auf einenSpulenkörper mit kreisförmigem Querschnitt(d 6,5 cm) gewickelt. Für die 2500 Windungenwird ein Kupferdraht von 0,2 mm Durchmesserverwendet. Bei welcher Frequenz f ist der induk-tive Widerstand der Spule gleich ihrem ohmschenWiderstand? Der spezifische Widerstand von Kup-fer beträgt r 1;55 10ÿ8 Wm.

Lösung:Aus XL R und 2p f L R folgt f R=(2pL).

Mit

R rlA

1,55 ´ 10ÿ8 Wm2p 3,25 10ÿ2 m 2500

p (10ÿ4 m)2 252 W

und

L m0

n2 Al 4p10ÿ7 Vs=Am

´25002 p (3,25 10ÿ2 m)2

0,5 m 52,1 mH

folgt

f R2pL

252W2 p52,1mH

769 Hz.

Zusatzaufgabe

5 Eine 60 W-Lampe für 110 V soll an das 230 V-Netzangeschlossen werden. Wie groû muss die Induk-tivität einer vorgeschalteten Spule sein, damit dieLampe normal hell brennt?(Der ohmsche Widerstand der Spule werde ver-nächlässigt.)

Lösung:An der Spule muss die Spannung UL abfallen:

UL (230 V)2ÿ (110 V)2

p 202 V.

Da ein Strom von I 60 W : 110 V 0,545 A flie-ûen soll, berechnet sich der Wechselstrom-widerstand XL der Spule zu

XL 202 V

0,545 A 371 W 2 p 50 Hz L.

Damit wird L 1,18 H.

6 Bei einer Gleichspannung von 12 V wird in einerDrosselspule die Stromstärke 0,6 A gemessen.Legt man an die Spule die Netzspannung(230 V=50 Hz), so misst man einen Strom von1,5 A. Berechnen Sie die Induktivität und denPhasenwinkel.

Lösung:

Es giltR2X2

L

p 230 V=1,5 A;

mit R 12 V=0,6 A 20 W

folgt XL (230 V=1,5 A)2ÿ (20 W)2

p

152 W 2p 50 Hz ´ L

und daraus L 0,439 H 439 mH

Für den Phasenwinkel folgttan ò XL=R 152 W=20 W 7,6 oder ò 82,5.

7 Elektromagnetische Schwingungen und Wellen120

Page 121: Metzler Physik

7.1.4 Die Leistung im Wechselstromkreis

1 Eine 100 W-Glühlampe wird an die Netzspannungvon 230 V angeschlossen. Berechnen Sie Effektiv-und Scheitelwert der Stromstärke sowie die elek-trische Energie, die die Lampe in 3 Stunden demNetz entnimmt.

Lösung:Effektivwert I P=U 100W=230 V 0,435 A,

Scheitelwert iÃ2p

I 2p 0,435 A 0,615 A,

El. Energie Eel P D t 100 W ´ 3 h 0,3 kWh.

2 Durch einen Kondensator flieût ein Wechselstromvon I 7 mA. Der Scheitelwert der Wechselspan-nung beträgt ß 9 V. Zeichnen Sie für eineinhalbPerioden den Strom-, Spannungs- und Leistungs-verlauf.

Lösung:

i in mA U in V

024

68

10

–2–4–6–8

–10–12

12

–20–40–60–80–100

100

80604020

–120

90° 180° 270° 360° 450° 540° ω t

P (t)in mWi = 9,90 mA sin ω t

u = 9V sin ( – 90°)ω t

P (t) = – 44,55 W sin 2ω t

*3 An einer Spule wird ein Wechselstrom mitià 10 mA und eine Wechselspannung mit ß 6 Vbei einer Phasendifferenz von ò 55 gemessen.Berechnen Sie anhand des Phasendiagramms dieSchein-, Wirk-, und Blindleistung.

Lösung:Scheinleistung

S Ãu iÃ=2 6 V ´ 10 mA= 2 30 mW

WirkleistungP S cos ò 30 mW cos 55 17,2 mW

BlindleistungQ S sin ò 30 mW sin 55 24,6 mW

*4 Lösen Sie Aufgabe 3 für die Phasendifferenzòÿ75. Welche Schaltung kann diese Phasen-differenz erzeugen?

Lösung:Scheinleistung

S Ãu iÃ=2 6 V ´ 10 mA=2 30 mW

WirkleistungP S cos ò 30 mW cos (ÿ 75) 7,76 mW

BlindleistungQ S sin ò 30 mW sin (ÿ 75)ÿ 30 mW

Die negative Phasendifferenz kann an einer Rei-hen- oder Parallelschaltung eines ohmschenWiderstandes und eines Kondensators gemessenwerden.

*5 Ein elektrisches Gerät gibt bei Anschluss an eine110 V-Wechselspannung eine Wärmeleistung von450 W ab. Dabei flieût ein Strom von I 5,3 A.Berechnen Sie die Schein-, Wirk- und Blind-leistung.

Lösung:Scheinleistung

SU I 110 V ´ 5,3 A 583 W,

WirkleistungP 450 W,

BlindleistungQ

S2ÿQ2p

5832ÿ 4502p

W 371 W,

Zusatzaufgabe

6 Eine Drosselspule hat einen ohmschen Widerstandvon 4,5 W und eine Induktivität von 0,3 H.Wie groû ist der Leistungsfaktor cos ò bei 50 Hz?

Lösung:Für den Phasenwinkel ò folgt

tan ò XL=RwL=R 2 p 50 Hz 0,3 H=4,5 W 20,9.

Damit ist ò 87,3, und man erhält für den Leis-tungsfaktor cos ò cos 87,3 0,0477 4,77%.

Seite

267

7.1 Wechselstromtechnik 121

Page 122: Metzler Physik

7.1.5 Wechselstromschaltungen

1 Berechnen Sie die Teilspannungen über dem ohm-schen Widerstand R 250 W, dem KondensatorC 12 mF und der Spule L 1,8 H, die in Seriegeschaltet von einem Wechselstrom I 120 mA( f 50 Hz) durchflossen werden. Zeichnen Sieein maûstabgerechtes Phasendiagramm und be-rechnen Sie die Impedanz Z, die Generatorspan-nung UG und die Phasendifferenz ò.

Lösung:Es istR 250 W,UR 250 W 0,12 A 30 V,

XL 2 p 50 Hz 1,8 H 565 W,UL 565 W 0,12 A 67,9 V,

XC (2 p 50 Hz 12 mF)ÿ1 265 W,UC 265 W 0,12 A 31,8 V.

ϕ

U = 30 V

U = 67,9 V

U = 31,8 V

U

IR

C

L

Z R2 (XLÿXC)2

p

2502 (565ÿ 265)2

pW 391 W.

Damit ergibt sichU Z I 391 W 0,12 A 46,9 V

undtan ò (XLÿXC)=R (565ÿ 265)=250 1,20oder ò 50,2.

2 Ein ohmscher Widerstand R 1,5 kW, eine SpuleL 75 mH und ein Kondensator C 12 nF werdenparallel an eine Wechselspannung UG 24 V( f 4,5 kHz) angeschlossen. Berechnen Sie dieStröme in den einzelnen Bauteilen und zeichnenSie ein maûstabgerechtes Phasendiagramm. Be-rechnen Sie die Impedanz Z, den Gesamtstrom Iund die Phasendifferenz ò.

Lösung:

I = 8,14 mA

I = 16 mA

I = 11,3 mA

U

I

ϕ

C

R

L

Rechnerisch ergibt sich

Z 1

1

R2

1XLÿ 1

XC

2s

1

1

1,5

2

12,12ÿ 1

2,95

2s

kW 1,47 kW,

I U=Z 24 V=1,47 kW = 16,3 mA und

tan ò

1RLÿ 1

RC

1R

12,12ÿ 1

2,95

´1,5 0,198)ò 11,2.

3 Ein ohmscher Widerstand R 150 W und eineSpule L 2,1 H werden hintereinander geschaltetund an eine Wechselspannung U 12 V( f 650 Hz) angeschlossen. Wie groû sind dieStromstärke und der Phasenwinkel? Welche Leis-tung wird im Widerstand umgesetzt? Wie groûmuss die Kapazität eines in Serie geschalteten Kon-densators sein, damit der Strom maximal wird?

Lösung:Mit RL w L 2 p 650 Hz 2,1 H 8,58 kW wird

I U=R2R2

L

p

12 V=(150 W)2 (8,58 kW)2

p 1,40 mA

und tan òwL=R 8,58 kW=150 W 57,2oder ò 89.

Die Leistung berechnet sich zu

P U I cos ò 12 V 1,40 mA cos 89 0,294 mW I2 R (1,40 mA)2 ´ 150 W 0,294 mW.

Der Strom wird maximal für RL RC oderwL 1=w C .

Seite

269

7 Elektromagnetische Schwingungen und Wellen122

Page 123: Metzler Physik

Mit C 1

(2 p f )2 L

1

(2 p 650 Hz)2 2,1 H 28,5 nF

ergibt sich: Imax U=R 12 V=150 W 80 mA.

*4 Legt man an eine Reihenschaltung aus ohmschemWiderstand R und Kondensator C eine Eingangs-spannung U1 und greift die Ausgangsspannung U2

entweder über dem Kondensator oder über demWiderstand ab, so hat man einen frequenzabhängi-gen Spannungsteiler. Der Abgriff über dem Kon-densator heiût RC-Tiefpass, der über dem Wider-stand RC-Hochpass, weil im ersten Fall nur die tie-fen Frequenzen, im zweiten Fall nur die hohen Fre-quenzen übertragen werden.a) Geben Sie für Hoch- und Tiefpass das Verhält-

nis von Ausgangsspannung zu Eingangsspan-nung U2=U1 als Funktion der Frequenz f an.Zeichnen Sie für C 10 nF und R 120 kWdie Frequenzgangkurven, d. h. die Abhängig-keit des Verhältnisses U2=U1 von der Fre-quenz f.

b) Bei der so genannten Grenzfrequenz fg sindohmscher und kapazitiver Widerstand gleichgroû. Wie groû ist dann die Phasendifferenzzwischen Ausgangs- und Eingangsspannung?Wie berechnet sich die Grenzfrequenz fg beiHoch- und Tiefpass aus R und C?Berechnen Sie fg für C 10 nF undR 120 kW.

c) Welchen Wert hat das SpannungsverhältnisU2=U1 bei der Grenzfrequenz fg für Hoch-und Tiefpass?

Lösung:

R

CU1 2

Tiefpass

U R

C

U1

Hochpass

2U

Für beide Fällefolgt aus demPhasen-diagrammU2

1 U2RU2

C.

U

G

I R

C1U

U

U

a) Tiefpass: U2 UC

U21 U2

RU22

U2=U1 1=1 (UR=UC)2

p

1

1

I R

I=(2 p f C)

2s

1=1 (2 p f R C)2

p

Hochpass: U2 UR

U21 U2

2 U2C

U2=U1 1=

1

UC

UR

2s

1=

1

I=2 p f R C

I R

2s

1=

1

1

2 p f R C

2s

U /U2 1

0

0,1

0,2

0,3

0,4

0,5

0,6

0,7

0,8

0,9

1,0

100 200 300 400 500

f in Hz

0 fg

TiefpassHochpass

R = 120 kC = 10 nF

Ω

b) Für den Phasenwinkel òg gilt: jòg j 45.

Aus R Xc 1=(2p fg C) folgtfg 1=(2 p R C) 1=(2 p 120 kW 10 nF) 133 Hz.

c) Aus beiden Gleichungen in a) folgt für R Xc

U2=U1 1=1 1p

0,707.

7.1 Wechselstromtechnik 123

Page 124: Metzler Physik

5 Ein Tiefpass soll die Grenzfrequenz fg 10 kHzhaben.a) Berechnen Sie die zu R 47 kW gehörige Ka-

pazität.b) Bei welcher Frequenz beträgt die Ausgangs-

spannung U2 nur noch 10% der Eingangsspan-nung U1?

Lösung:a) Aus Aufgabe 4 folgt mit fg 1=(2 p R C) für

die Grenzfrequenz die KapazitätC 1=(2 p fg R)2

1=(2 p 10 kHz 47 kW) 339 pF.

b)U /U2 1

0

0,1

0,2

0,3

0,4

0,5

0,6

0,7

0,8

0,9

1,0

f in kHz

0 10 20 30 40 50 60 70 80 90

fg

c) Aus U2=U1 11 (2 p f R C)2

p

1

1

f

10 kHz

2s

110

folgt 1 ( f =10 kHz)2 100 oder f 99,5 kHz.

Zusatzaufgaben

6 Bei Leuchtstofflampen wird zur Strombegrenzungder Röhre ein Vorschaltgerät (Drosselspule) in Se-rie geschaltet und an das Netz (230 V/50 Hz) ange-schlossen. Über der Röhre wird eine Spannung von60 V, über der Drossel von 215 V gemessen, wobeiein Strom von 0,38 A flieût.Zeichnen Sie ein maûstabsgerechtes Phasendia-gramm. Wie groû ist der Phasenwinkel ò?(Die Röhre sei ein reiner Wirkwiderstand.) Wiegroû sind die in der Röhre und im Vorschaltgerätumgesetzten Leistungen? Welche Werte haben In-duktivität und ohmscher Widerstand des Vor-schaltgeräts? Wie groû ist die Blindstromkompo-nente?

Lösung:

20 V

UNetz Drossel

= 230 V = 215 V

96,8° I = 0,35 A

I = 0,38 ARöhre = 60 V

bϕU

U

Es ist cos ò (602 2302ÿ 2152)=(2 60 230) 0,372 oder ò 68,1.

Damit wirdPRÈohre U I 60 V ´ 0,38 A 22,8 W

PDrossel 230 V ´ 0,38 A ´ cos 68,1 ÿ 22,8 W 9,8 W.

Für die Drossel gilt

R (230 V cos òÿ 60 V)0,38 A

25,8 V0,38 A

67,9 W

wL 230 V sin ò=0,38 A 562 W

L 536 W=(2 p 50 Hz) 1,79 H

mit der Blindstromkomponente

IQ I sin ò 0,38 ´ sin 68,1 0,353 A.

7 In Gebäuden mit vielen Leuchtstofflampen schrei-ben die Elektrizitätswerke eine Blindstrom-kompensation vor, um die Verluste in den Zu-leitungen möglichst gering zu halten. Dazu wer-den jeweils zwei Lampen zu einer DUO-Schaltungzusammengefasst. Einer Lampe mit Vorschalt-gerät wird eine zweite parallel geschaltet, diezusätzlich zum Vorschaltgerät mit einem Kon-densator in Serie geschaltet ist. Dieser Kondensa-tor ist so bemessen, daû der zweite Zweig die glei-che Blindstromkomponente wie die erste hat,jedoch nicht induktiv sondern kapazitiv, sodassbeide Lampen zusammen einen reinen Wirk-strom haben. Wie groû muss die Kapazität sein?Gehen sie von den Daten der vorhergehendenAufgabe 6 aus. (Der Kondensator kann als ideal

7 Elektromagnetische Schwingungen und Wellen124

Page 125: Metzler Physik

angesehen werden.) Zeichnen Sie ein Phasendia-gramm.Wie groû ist die Spannung am Kondensator?Wie groû ist der (Wirk-)Strom in der Zuleitung?

Lösung:Nach Aufgabe 6 beträgt der induktive Widerstanddes Zweiges der ersten Lampe XL 562 W. Derkapazitive Widerstand XLC im parallelen Zweigder zweiten Lampe muss dann ebenfalls den Wert562 W haben, sodass sich in diesem Zweig dergleich Strom I2 0,38 A ergibt, jedoch mit derPhasendifferenz ò ÿ 68,1. Der kapazitive Wi-derstand des Kondensators XC muû demnach dop-pelt so groû wie der des Vorschaltgeräts XL (Dros-selspule) sein:

XC 2 562 W 1=(w C) und daraus C 2,8 mF.

Damit beträgt die Spannung am KondensatorUC I2 XC 0,38 A ´ 2 ´ 563 W 427 V.Der Strom I in der Zuleitung ist gleich dem doppel-ten Wirkstrom in einem Zweig:

I 2 I1 cos ò 2 ´ 0,38 A cos 68,1 0,283 A

(statt 2 0,38 A 0,76 A ohne DUO-Schaltung).

L

C

U

I = 0,287 A

Röhre

Netz

Starter

12I I

L

20 V

0,1 A

–+ϕ ϕ 60 V

215

V

60 V

215 V1I

2IU Netz=

427

V

UN

etz

= 23

0 V

1I

I

7.1.6 Der Transformator

1 Wie groû ist die Ausgangsspannung eines Hoch-spannungstransformators mit n1 250 und n2 23 000 Windungen im Leerlauf, wenn primärseitig230 V anliegen?

Lösung:U2 U1 n2=n1 230 V 23 000=250 21,2 kV.

2 Man möchte einen Klingeltrafo für 16 V herstellen.Wie viele Sekundärwicklungen muss man auf ei-nen Eisenkern aufbringen bei U1 230 V undn1 1000 Windungen?

Lösung:n2 n1 (U2=U1) 1000 16 V=230 V 69;6

Also müssen 70 Windungen aufgebracht werden.

*3 Ein Hochstromtransformator hat sekundärseitigeine einzige Windung aus sehr dickem Kupfer-draht. Wie groû ist der Strom darin, wenn die Pri-märspule (n1 500) bei 230 V eine Blindleistungvon 60 W aufnimmt?

Lösung:Aus P1 U1 I1 und I2 : I1 500 : 1 folgtI2 500 P1=U1 500 60 W=230V 130 A.

*4 Geben Sie die Phasenbeziehungen zwischen denStrömen und den Spannungen (i1; u1), (u1; u2),(i1; i2) an, wenn die Last vom Leerlauf bis zumKurzschluss zunimmt.

Lösung:Der Phasenwinkel ò zwischen den primärseitigenGröûen i1 und u1 bestimmt die Aufnahme derWirkleistung des Transformators P1U1 I1 cos ò1.Da weder im Leerlauf noch beim sekundärseitigenKurzschluss Leistung abgegeben wird, wird dannbei einem idealen Transformator primärseitig auchkeine Leistung aufgenommen. Der Phasenwinkelò1 ist demnach im Leerlauf 90, nimmt mit zuneh-mender Belastung ab, geht dann aber wieder gegen90, wenn bei Annäherung an den sekundärseitigenKurzschluû die Leistungsabgabe geringer undschlieûlich null wird.Für die Phasendifferenzen der Spannungen u1 undu2 gilt:0 (im Leerlauf) Phasendifferenz (u1, u2) 90(im Kurzschluss)

Für die Phasendifferenzen der Spannungen i1 undi2 gilt:

Seite

271

7.1 Wechselstromtechnik 125

Page 126: Metzler Physik

90 (im Leerlauf) Phasendifferenz (i1, i2) 180(im Kurzschluss)

*5 Der Thomson'sche Ringversuch (! 6.3.2) lässtsich auch mit Wechselstrom durchführen: Dabeisteht das Joch eines Trafokerns senkrecht in einerSpule. Die Spule hat 500 Windungen und kann andas Netz angeschlossen werden. Legt man einenRing aus dickem Kupferdraht über das Joch, soschwebt der Ring. Ein Ring aus dünnem Kupfer-draht fällt hingegen auf die Spule herunter. Wielässt sich das erklären?

Lösung:Die in der Sekundärspule induzierten Spannungenu21 und u22 eilen den Strömen i1 und i2 jeweils um90 voraus. Im Kurzschluû ist u2 0 und damitsind u21 und u22 in Gegenphase. Folglich gilt dasauch für die Ströme i1 und i2.Der dicke Kupferring lässt sich als kurzgeschlos-sene Sekundärspule eines Transformators auffas-

sen. Die Ströme i1 und i2 sind also in Gegenphase,stoûen sich ab und der Ring schwebt. Beim dünnenRing nimmt antiproportional zur kleineren Quer-schnittfläche der Wirkwiderstand zu. Man könntemeinen, auch der dünne Ring sollte schweben,denn in gleichem Maûe, wie sich proportional zurkleineren Querschnittfläche der Strom i2 verklei-nert, verkleinert sich auch die Gewichtskraft. Ent-scheidend ist, dass der dünne Ring nicht mehr alskurzgeschlossener Transformator aufgefasst wer-den kann. Der gegenüber dem induktiven Wider-stand des Rings nicht mehr vernachlässigbareWirkwiderstand führt zu einer Phasenverschie-bung, sodass die Ströme i1 und i2 nicht mehr gegen-phasig sind. Damit verkleinert sich die zu i1 gegen-phasige Komponente des Stroms i2: Die absto-ûende Kraft wird kleiner und der dünne Ring kannnicht mehr schweben.

7 Elektromagnetische Schwingungen und Wellen126

Page 127: Metzler Physik

7.2.1 Der elektrische Schwingkreis

1 Geben Sie die analogen Gröûen und Gleichungenfür elektrische und mechanische Schwingungenan.

Lösung:Der Spannung u beim elektrischen Schwingkreisentspricht die Auslenkung s beim Federpendel.Dem Strom i entspricht die Geschwindigkeit v(oder der Impuls p).Der maximalen elektrischen Feldenergie Eel ent-spricht die maximale Spannenergie ESp:

Eel 12 C uà 2 ESp 1

2 D sà 2

Der maximalen magnetischen Feldenergie Emag

entspricht die maximale kinetische Energie Ekin:

Emag 12 L ià 2 Ekin 1

2 m vÃ2

*2 Wiederholen Sie die Herleitung der Thomson'-schen Gleichung und stellen Sie eine analoge Glei-chung für das Federpendel auf.

Lösung:Für ein Federpendel mit der Masse m und der Fe-derkonstanten D folgt aus F m amsÈ undF ÿDs die Differentialgleichung

m d2s=d t2D s 0 analog zuL d2 i=d t2 i=C 0.

Mit dem Ansatzx xà sin w t analog zu i ià sin w t

folgtw 2p f

D=m

panalog zu

w 2p f 1=(LC)

p.

7.2.2 Elektrische Schwingungen

1 Ein Kondensator mit C 0,1 mF und eine Spulemit L 44 mH bilden einen Schwingkreis. Berech-nen Sie die Eigenfrequenz. Durch Einschieben ei-nes Eisenkerns in die Spule vergröûert sich derenInduktivität um den Faktor 23. Wie verändert sichdadurch die Eigenfrequenz?

Lösung:

f 1

2pLCp 1

2p44 mH 0,1 mF

p 2,40 kHz,

fEisen f=23p

500 Hz.

2 Eine lange Spule (n 340, l 60 cm, d 8 cm)wird mit einem Kondensator der KapazitätC 0,1 mF und einem Widerstand R 200 W inSerie geschaltet. Berechnen Sie die Resonanzfre-quenz.

Lösung:L m0 n2 A=l 4 p 10ÿ7 Vs/Am ´ 3402 ´ p (0,04 m)2=0,6 m 1,22 mH,

f 1

2pLCp 1

2p1,22 mH 0,1mF

p 14,4 kHz.

Die Rechnung zeigt, dass man für die gedämpfteSchwingung eine kleinere Frequenz erhält.Es gilt:

f0 1

2p

1

LCÿR2 C

4L2

r f

1ÿR2 C

4L

r 6,09 kHz.

3 Ein Schwingkreis mit einer Kapazität vonC 47 nF schwingt bei einer Frequenz vonf 3,7 kHz. Wie groû ist die Induktivität?

Lösung:Aus f 1=2 p

LCp

folgt L 1=(2p f )2 C

1=(2 p 3,7 kHz)2 47 nF 39,4 mH.

4 Die Zeit-Strom-Funktion eines elektrischenSchwingkreises ist durchit 0,03 A ´ sin (350p t=s)gegeben. Die Induktivität des Schwingkreises be-trägt L 0,5 H.Berechnen Sie die Schwingungsdauer T, die Ei-genfrequenz f0, die Kapazität C des Kondensators,den Effektivwert UG der Generatorspannung unddie maximalen Energiewerte des elektrischen undmagnetischen Feldes.

Lösung:Mit w 350p=s 2 p=T wird T 1

175 s 5,71 ms,

C 1=(w2 L) T2=(4p2 L) (5,71ms)2=(4p2 0,5 H) 1,65 mF,

uà XL ià w L ià (350 p=s) ´ 0,5 H ´ 0,03 A 16,5 V; UG 16,5 V=

2p 11,7 V

EÃ m 12 L iÃ2 225 mJ; EÃ el 1

2 C uÃ2 225 mJ.

*5 Leiten Sie die Thomson'sche Gleichung aus derResonanzbedingung für den Serienschwingkreisher.

Seite

275

Seite

277

7.2 Elektrische Schwingungen und elektromagnetische Wellen 127

Page 128: Metzler Physik

Lösung:Die Impedanz einer Reihenschaltung aus Wider-stand, Spule und Kondensator

Z R2 (wLÿ 1=wC)2

p

wird minimal (Resonanz), wenn wL 1=wC.Daraus folgt w 1=

LCp

.

*6 Stellen Sie die Resonanzkurve eines Serien-schwingkreises ± Stromstärke I als Funktion derFrequenz f ± grafisch dar. Es seien L 0,44 Hund C 5 mF. Der ohmsche Widerstand habe denWert R 50 W und die angelegte Spannung seiUG 24 V. Wie groû sind bei Resonanz die Span-nungen an der Spule und am Kondensator?

Lösung:Aus der Impedanz Z

R2 (wLÿ 1=wC)2

p

folgt mit w0 2p f0 1=LCp

I UZ U

R2 LC

ff0ÿ f0

f

2s

U=R1 L

R2 C

ff0ÿ f0

f

2s .

Mit Imax U=R 24 V=50 W 480 mA,f0 107,3 Hz und L=(R2 C) 35,2 ergibt sich diedargestellte Resonanzkurve.

500

100

400

300

200

100

050 1500

I in mA

f in Hz

7.2.3 Ungedämpfte elektrische Schwingungen

1 Berechnen Sie die Induktivität L eines Schwing-kreises, der mit 1,5 MHz schwingt; die Kapazitätsei C 50 pF. Wie viele Windungen müsste dieSpule haben, wenn sie auf einen Stab (l 4 cm,d 6 mm) gewickelt wäre?

Lösung:

Aus w 1LCp wird

L 1

(2p f )2 C 1

(2p 1,5 MHz)2 50 pF 225 mH,

so dass sich nach

L m0 n2 Al

ergibt n

Llm0 p r2

s

225 mH 4 cm

4 p10ÿ7 Vs=Am p (3 mm)2

s 503.

2 Erklären Sie, wie in der Dreipunktschaltung inAbb. 279.1 die Rückkopplung funktioniert.

Lösung:In der Rückkopplungsschaltung nach Meissnerwird mit der in Abb. 278.1 eingezeichnete Span-nung uL auch die Spannung uind in einer Sekundär-spule induziert. Dort liegt sie in einem Kreis an derBasis-Emitter-Strecke des Transistors T, wo sie ei-

nen Basistrom flieûen lässt, der momentan phasen-richtig den Transistor öffnet. Die Aufgabe der Se-kundärspule übernimmt in der Dreipunktschaltungder obere Teil der Schwingkreisspule. Denkt mansich in Abb. 279.1 mit gleicher Phase wie inAbb. 278.1 eine Spannung uL eingezeichnet, soliegt der obere Teil dieser Wechselspannung eben-falls an der Basis-Emitter-Strecke des TransistorsT1 und öffnet ihn momentan. Man beachte dabei,dass die Batterie für die hochfrequente Wechsel-spannung einen Kurzschluss darstellt und an demWechselstromwiderstand des 250 pF-Kondensa-tors nur ein Teil der Spannung abfällt.

*3 Wiederholen Sie die Überlegungen zur Rückkopp-lung in Abb. 279.2. Zeichnen Sie ein Phasendia-gramm, in dem neben der Spule auch im Konden-sator Verluste auftreten.

Lösung:Verluste in einem Kondensator können z. B. durcheinen Leckstrom auftreten, der den Kondensatorerwärmt. Im Phasendiagramm eines realenSchwingkreises mit Verlusten in der Spule undim Kondensator ist ohne Rückkopplung dieSumme der drei Ströme iL durch die Spule, iC durchden Kondensator und iR iR(C) IR(C) durch denWiderstand R null. R erfasst als Ersatzwiderstand

Seite

279

7 Elektromagnetische Schwingungen und Wellen128

Page 129: Metzler Physik

die Verluste in Spule und Kondensator; Spule undKondensator wirken als Generator, die die Verlust-energie liefern. Flieût der Strom iR durch eineRückkopplung von auûen über den Widerstand R,so wird die Energie, die der Kreis sonst verlierenwürde von auûen durch P u iR ausgeglichen.

u

iL

R (C)

R (L)

C

i

i

i

7.2.4 Mikrowellen

1 Bei einer an einer Metallwand reflektierten Wellebeträgt der Abstand der Intensitätsmaxima 1,24 cm.Wie groû sind Wellenlänge und Frequenz derMikrowelle ?

Lösung:Mit l 2 1,24 m folgtf c=l 3 108 m=s=2,48 m 121 MHz.

2 In Abb. 281.1 b) sei ò der Drehwinkel des Gitters.Geben Sie den Feldvektor E3 parallel zur HF-Diodeals Funktion des Drehwinkels ò und des Vektors E1

der einfallenden Welle an und stellen Sie die Ab-hängigkeit des Feldvektors E3 vom Drehwinkel ò

in einem Schaubild dar.

Lösung:Es gilt Esenk E sin ò

E3 Esenk cos ò

und damitE3 E1 sin ò cos ò

ϕϕ

Epar

1

3

senk

E

E

E

0,6

0,5

0,4

0,3

0,2

0,1

00,50 1,0 1,5 2,0

Drehwinkel (in rad)ϕ

E = E sin cos mit E = 1ϕ3 1 ϕ 1

45° 90°

3 Von Metallstäben weiû man, dass sie von einerelektromagnetischen Welle zu elektrischenSchwingungen angeregt werden und dann selbsteine Welle aussenden. Wie ist es zu erklären, dassdie Stäbe in Abb. 281.1 a) nur eine Welle nachvorn, nicht aber nach hinten aussenden?(Siehe hierzu auch 7.2.7 ¹Der Hertz'sche Dipolª.)

Lösung:Die einfallende Mikrowelle regt die Metallstäbe zuerzwungenen Schwingungen an, sodass dieseselbst zu Sendern werden und Wellen nach vornund hinten ausstrahlen (! 7.2.7). Die nach vornausgesandte Welle ist die reflektierte Welle, diemit der einfallenden stehende Wellen bildet. Beider Reflexion tritt ein Phasensprung von p aufund am Stab bildet sich ein Knoten. Ein Teil dereinfallenden Welle tritt durch die Stäbe hindurch,löscht sich aber hinter den Stäben mit der von die-sen ausgesandten Welle wegen des Phasensprungsvon p aus. Die Stäbe haben gerade einen solchen

Seite

281

7.2 Elektrische Schwingungen und elektromagnetische Wellen 129

Page 130: Metzler Physik

Abstand, dass dies eintritt. Würde man den Ab-stand vergröûern, könnte man mit zunehmender In-tensität hinter den Stäben die hindurch tretendeWelle nachweisen.

7.2.5 Elektromagnetische Wellen

1 Welche Analogie besteht zwischen den stehendenelektromagnetischen Wellen auf der Lecher-Lei-tung und den Eigenschwingungen einer Pfeife(! 3.4.7)?

Lösung:Die Lecher-Leitung mit einem geschlossenen undeinem offenen Ende ist analog zur gedeckten Pfeifein Abb. 142.2 b). Am geschlossenen Ende der Le-cher-Leitung wird mit dem Induktionsstrom einSchwingungsbauch der magnetischen Feldstärkeangeregt; die elektrische Feldstärke hat hier einenKnoten. Bei der Pfeife regt an der Schneide derLuftstrom einen Schwingungsbauch der Bewegungder Luftsäule an, der Luftdruck hat hier einenSchwingungsknoten. Am jeweils anderen Endeist es umgekehrt: Dort hat die elektrische Feld-stärke einen Schwingungsbauch ebenso wie derLuftdruck.

2 Zeichnen Sie die stehende elektromagnetischeWelle in Abb. 282.2 b) zum Zeitpunkt t T 8.

Lösung:Zu den in Abb. 282.2 gezeichneten Zeitpunkten hatentweder die elektrische Feldstärke E ihre maxima-len Auslenkungen (t 0 und 2

4 T) wobei die magne-tische Feldstärke B hier gerade null ist oder umge-kehrt die magnetische Feldstärke B hat ihre maxi-malen Auslenkungen (t 1

4 T und 34 T) und die elekt-

rische Feldstärke E ist gerade null. Bei t 18 T gibt

es demnach sowohl eine elektrische als auch einemagnetische Auslenkung: Es sind die durchgezo-gene blaue Kurve (für das elektrische Feld) und diedurchgezogene rote Kurve (für das magnetischeFeld). Beide Kurven haben an jeder Stelle gerade70,7% ihres dortigen maximalen Wertes.

7.2.6 Maxwells elektromagnetische Wellen

1 Berechnen Sie die Lichtgeschwindigkeit c aus denFeldkonstanten e0 und 0. Wie genau ist dies mög-lich?

Lösung:c 1=

e0 m0p

1=8,854187 817 As=(Vm) 1,256637 061 10ÿ6 As=(Vm)

p

2,99792458 108m=s

Die Berechnung der Lichtgeschwindigkeit ist ex-akt:Wären die Feldkonstanten e0 und m0 experimentellbestimmte Werte, so könnte die Lichtgeschwindig-keit auf diese Weise nur im Rahmen der Messun-genauigkeit berechnet werden.Tatsächlich ist die magnetische Feldkonstantezu m0 4p 10ÿ7 Vs=(Am) definiert und die Licht-geschwindigkeit auf c 2,997 924 58 ´ 108 m=sfestgesetzt. Die elektrische Feldkonstante ist mitobiger Gleichung zu e0 1=(m0 c2) festgelegt.

2 Warum fehlt in der 2. Maxwell'schen Gleichungein Term, der dem Term S In in der 1. Gleichungentspricht?

Lösung:Der Term S In in der 1. Maxwell'schen Gleichungerfasst die Bewegung elektrischer Ladungen durchdie Integrationsfläche. Analog dazu müsste in der2. Maxwell'schen Gleichung ein Term stehen, derdie Bewegung magnetischer Ladungen berück-sichtigt. Es gibt aber nur magnetische Dipole (mag-netische Momente) und keine magnetischen Mo-nopole; trotz intensiver Suche hat man sie nichtgefunden. Daher fehlt der entsprechende Term.

3 Zeigen Sie, dass sich in den Abb. 285.1 und 285.2aus den Maxwell'schen Gleichungen die gleichenFeldrichtungen ergeben, wenn der Wellenberg dieIntegrationsfläche rechts verlässt.

Lösung:Verlässt der Wellenberg in Abb. 285.1 die Integra-tionsfläche, so sind rechts elektrische Feldstärke E

Seite

283

Seite

285

7 Elektromagnetische Schwingungen und Wellen130

Page 131: Metzler Physik

und Weg a gleichgerichtet. Daher gilt jetztREs dsE a. Der magnetische Fluss

RBn dA,

der die Fläche durchsetzt, nimmt nun aber ab, daher

istddt

RBn dA < 0. Zusammen mit dem Minuszei-

chen in der 2. Maxwell'schen Gleichung ergibt sichauch hier E aÿ(ÿBca)) E Bc. Entspre-chendes gilt für die 1. Maxwell'sche Gleichung.

4 Erklären Sie mithilfe der 1. Maxwell'schen Glei-chung, wieso ein zeitlich veränderlicher Strom eineelektromagnetische Welle auslöst. Geben Sie denzeitlichen Verlauf eines Stroms an, der den obigenWellenberg erzeugt.

Lösung:Betrachten wir eine aus-gedehnte Metallplatte,durch die ein Strom Ipro Länge a von obennach unten flieût. DerStrom erzeugt vor undhinter der Platte ein ho-mogenes Magnetfeldder Stärke B mit deneingezeichneten Rich-tungen. Mit der 2. Max-well'schen Gleichungfolgt

B

I

a

B

bBs ds m0

PIn ) B 2 a m0 I

Bei konstantem Strom I folgt ein konstantes Mag-netfeld B m0 I=2a. Für die Ausbreitung einerWelle ist aber ein zeitlich veränderliches Feld not-wendig. Dies ergibt sich, wenn sich der Strom zeit-lich ändert, die elektrischen Ladungen also be-schleunigt werden. ¾ndert sich der Strom währendeiner Zeit t linear, also z. B. nach der Gleichung

I(t) I0

tt, so ändert sich während dieser Zeit das

Magnetfeld vor der Platte linear mit der Zeit undIntegrationen nach den Abb. 285.1 und 2 liefernden dargestellten Wellenberg.

5 Ein Mikrowellensender strahlt über die FlächeA 25 cm2 einer Hornantenne eine Leistung von600 W ab. Wie groû ist die elektrische Feldstärkeam Antennenausgang?

Lösung:Mit der Gleichung S

(e0=m0)

pE2 folgt

ES

m0

e0

rs

600 W

25 10ÿ4 m2

4p 10ÿ7 Vs=(Am)

8,854 10ÿ12 As=(Vs)

svuut 9,5kVm

.

7.2.7 Der Hertz'sche Dipol

1 Dreht man den elektrischen Feldvektor E in Rich-tung des magnetischen Feldvektors B, so erhältman nach einer Rechtsschraube fortschreitend dieAusbreitungsrichtung der elektromagnetischenWelle (Korkenzieher-Regel). Überprüfen Sie diesin Abb. 287.2 an mehreren Stellen.

Lösung:

+

E

B

E

B

c

E

B

c

2 Bei Fernseh- und UKW-Antennen werden Hertz'-sche Dipole als Direktoren und Reflektoren ver-wendet. Wie lang muss ein Hertz'scher Dipol sein,der auf den UKW-Kanal 11 ( f 90,3 MHz) abge-stimmt ist?

Lösung:Aus c f l erhält man l=2 c=(2 f )3 108 m=s=(2 90,3 MHz) 1,66 m.Der abgestimmte Dipol ist ca. 1,50 m lang.

Seite

287

7.2 Elektrische Schwingungen und elektromagnetische Wellen 131

Page 132: Metzler Physik

7.2.8 Rundfunktechnik

1 Der CB-Sprechfunk, der den Frequenzbereich27,005 MHz bis 27,125 MHz umfasst, ist in 12 Ka-näle unterteilt. Wie groû ist die Bandbreite einesKanals?

Lösung:Für den CB-Sprechfunk ist das Frequenzband von27,005 MHz bis 27,125 MHz reserviert. Damit hatjeder Kanal eine Breite von

27,125 MHzÿ 27,005 MHz12

10 kHzzur Verfügung.

2 Durch Reflexionen der Fernsehwellen können¹Geisterbilderª auftreten. Warum gibt es beimHörfunk keine derartige Beeinträchtigung?

Lösung:Wegen der gröûeren Wellenlängen sind die durchReflexionen entstandenen Gangunterschiede nor-malerweise klein im Vergleich zur Wellenlängeund wirken sich nicht aus.

3 Wie erklärt es sich, dass die Frequenzmodulationvon Störsignalen weniger beeinträchtigt wird alsdie Amplitudenmodulation?

Lösung:Vereinfacht kann man sagen, dass eine Amplitu-denschwankung aufgrund eines Störeinflusses beider Amplitudenmodulation direkt zu hören ist,während die Frequenzmodulation davon nicht be-troffen ist, da ihre Information im Frequenzhubenthalten ist. Tatsächlich ist zu beachten, dass dieFourier-Transformation der modulierten Träger-schwingung bei der AM endliche, durch die Breitedes NF-Bandes bestimmte Seitenbänder ergibt,während sie bei der FM unendlich viele Seitenfre-quenzen liefert, deren Amplitude allerdings mit zu-nehmender Modulationsfrequenz kleiner wird. DieFM benötigt daher ein breiteres Frequenzband alsdie AM. Mit dieser Verbreiterung des Frequenz-bandes geht nicht nur eine Verbesserung der Über-tragungsqualität einher, sondern auch eine bessereStörbefreiung.

Seite

291

7 Elektromagnetische Schwingungen und Wellen132

Page 133: Metzler Physik

7.3.1 Die Lichtgeschwindigkeit

1 In Versuch 1 misst man bei einem Spiegelabstandvon 6,35 m eine Zeitdifferenz von 42 ns zwischenden beiden Signalen. Berechnen Sie die Lichtge-schwindigkeit c.

Lösung:cDs=Dt2 6,35 m=42 10ÿ9s 3,02 ´ 108 m=s

2 In der Astronomie verwendet man die Strecken, diedas Licht in bestimmten Zeiten zurücklegt, als Län-genmaûe. Berechnen Sie die Strecken ¹Lichtse-kundeª, ¹Lichtstundeª und ¹Lichtjahrª. GebenSie in diesen Einheiten die Entfernungen von derErde zur Sonne, zum Jupiter, zum Pluto und zumnächsten Fixstern (a Centauri; 40 1012 km) an.

Lösung:1 Ls 300 000 km; 1 Lh 1,08 ´ 109 km;1 Lj 9,46 ´ 1012 km.Sonne: 8,5 Lichtminuten,Jupiter(mittl. Abstand 5,2023 AE): 43,2 Lichtminuten,Pluto (mittl. Abstand 78,9 AE): 10,9 Lichtstunden,a Centauri: 4,3 Lichtjahre.

3 Fizeau benutzte für sein Experiment ein Zahnradmit z 720 Zähnen, das sich mit der Umdrehungs-zahl n 12,6 s±1 drehte. Welche Strecke legt dasLicht zurück, während sich das Rad um genau ei-nen Zahn weiter dreht? Wie musste das Experimentfolglich durchgeführt werden?

Lösung:Ds c Dt 3 108 m=s s=(12,6 720) 33,07 km.Der zweite Spiegel muûte also 15±20 km entferntstehen.

Zusatzaufgabe

4 Ein Lichtsignal wird auf den Mond geschickt (mitt-lere Entfernung Erde±Mond e 384 000 km). Wiegroû ist seine Laufzeit?

Lösung:Für den Lichtweg zum Mond hin und zurück giltfür den mittleren Abstande 3,844 ´ 108 m zum Mond: Dt 2e=c 2,56 s.

7.3.2 Beugung und Interferenz am Doppelspalt

1 Ein Doppelspalt mit dem Spaltabstand 1,2 mmwird mit einer Quecksilberlampe beleuchtet. Aufdem 2,73 m entfernten Schirm beobachtet manfür jeweils 5 Streifenabstände im grünen Licht6,2 mm und im blauen Licht 4,9 mm. BerechnenSie die Wellenlängen.

Lösung:

lgrÈun d sina d ae

1,2 10ÿ3 m 6,2 mm=(2,73 m 5) 5,47 ´ 10ÿ7 m.

lblau 4,31 ´ 10ÿ7 m.

2 Im Doppelspaltversuch misst man für die Abständevon einem hellen Streifen zum übernächsten hellenbei rotem Licht 3,5 cm, bei gelbem Licht 3,1 cmund bei grünem Licht 2,5 cm. Die übrigen Maûesind e 4,95 m und d 0,2 mm. Berechnen Sie dieWellenlängen.

Lösung:707 nm; 626 nm; 505 nm.

*3 Berechnen Sie den Zusammenhang zwischen aund dem Gangunterschied Ds der beiden Wellen-züge aus Abb. 295.1 ohne Verwendung der Nähe-rung e d.

Lösung:

d

a

e

l1

2l

Ds l2ÿ l1 a2 (d=2)2

pÿ

a2ÿ (d=2)2

p

*4 Berechnen Sie den Gangunterschied der beidenvon S1 und S2 ausgehenden Wellenzüge für denPunkt P(a) mit a a1=2, zeichnen Sie das dazuge-hörige Zeigerdiagramm und geben Sie die Ampli-tude der Schwingung in P an.

Seite

292

Seite

295

7.3 Licht als klassische Welle 133

Page 134: Metzler Physik

Lösung:Die Phasenverschie-bung d. beiden Wellen-züge ist ò p=2, also

sà (a) sÃ1

2p sÃ2

2p

sÃ(0)2p

=2 sà (0)=2p

.S1

2

S ( )αS

AlsoI(a)=I0 (sà (a)=sà (0))2 = 1

2.

7.3.3 Beugung und Interferenz am Gitter

1 Auf einem Schirm im Abstand e 2,55 m vomGitter (250 Linien pro Zentimeter) wird im mono-chromatischen Licht der Abstand der Maxima 1.Ordnung (links und rechts vom Hauptmaximum0. Ordnung) zu 8,2 cm, der der 2. Ordnung zu16,6 cm und der der 3. Ordnung zu 24,8 cm gemes-sen. Berechnen Sie die Wellenlänge.

Lösung:

ln g an

e2 a2n

(643 nm

651 nm648 nm

); l 647,3 nm.

2 Die beiden Maxima 1. Ordnung der grünen Hg-Linie l 546,1 nm haben auf einem e 3,45 mentfernten Schirm einen Abstand von 18,8 cm. Wiegroû ist die Gitterkonstante? Wie viele Gitterspaltekommen auf 1 cm?

Lösung:g n l

e2 a2

n

p=an

546,1 nm3,452 (0,188=2)2

p=(0,188=2)

2,0 ´ 10ÿ5 m (500 Linien pro cm).

3 Ein Gitter besitzt 20 000 Linien auf 4 cm.a) Berechnen Sie die Winkel, unter denen das

sichtbare Spektrum 1. und 2. Ordnung er-scheint.

b) Wie groû ist der Winkelabstand zwischen bei-den Spektren?

Lösung:1. Ordnung:sin arot lrot=g

780 nm 20000=4 cm 0,39;arot 22,95

sin aviolett lviolett=g 420 nm 20000=4 cm 0,21;

sin aviolett 12,12 Da1 10,83.

2. Ordnung:sin arot 2 lrot=g 0,78; arot 51,26sin aviolett 0,42; aviolett 24,83; Da2 16,47.

WinkelabstandDa aviolett; 2ÿarot; 1 24,83 ÿ 22,95 1,88.

4 Die gelbe Hg-Linie (l 578,0 nm) fällt in der 3.Ordnung fast genau mit der blauen Linie in 4. Ord-nung zusammen. Berechnen Sie daraus die Wel-lenlänge der blauen Linie.

Lösung:3 lgelb=g 4 lblau=g; lblau 3

4 lgelb 433 nm.

*5 Durch ihre feine Rillenstruktur bedingt lässt sicheine CD-ROM als Reflexionsgitter benutzen. Lässtman das rote Licht eines He-Ne-Lasers (l 632,8 nm) von einer solchen CD reflektieren, dannbeobachtet man zwischen dem Hauptmaximum 0.und 1. Ordnung einen Winkel von 22. Wie groû istder Abstand der Rillen? Wie groû ist der Abstandzwischen aufeinander folgenden Bits, wenn auf derCD zwischen r 2,2 cm und r 5,5 cm 600 Mega-byte Daten gespeichert sind?

Lösung:g l=sin a 632,8 nm=sin 22 1,689 mm.

Gesamte Spurzahl:N (5,5 cm ÿ 2,2 cm)=g 19 538.

Gesamte Spurlänge:l N 2p r N 0,242 m 4 726 m.

Ds l8 600 106

0,98 mm.

Seite

297

7 Elektromagnetische Schwingungen und Wellen134

Page 135: Metzler Physik

Zusatzaufgaben

6 Man findet für den Abstand von einem bis zumübernächsten hellen Interferenzstreifen im geo-metrischen Schatten eines Drahtes (Breited 1,5 mm) a 3,2 mm. Der Abstand Draht-Schirm ist e 4,12 m. Wie groû ist die Wellen-länge des Lichtes?

Lösung:Die beiden Ränder des Drahtes können als Licht-quellen angesehen werden mit dem Abstand einesDrahtdurchmessers. Daher wird wie beim Doppel-spalt l d a2=(2e), hier also l 5,825 ´ 10ÿ7 m.

7 Lässt man einen Laserstrahl streifend unter demWinkel a auf die Millimetereinteilung (Strichab-stand d) einer Schieblehre fallen (s. Abbildung),so zeigt die gegenüberliegende Wand die Interfe-renzfigur eines Gitters. Zeigen Sie, dass sich zweibenachbarte Strahlen in der Richtung bn des Maxi-mums n. Ordnung verstärken, wenn gilt

nl d(cos aÿ cos bn),

und geben Sie an, wie man daraus mit Hilfe derVersuchsgröûen der Abbildung die Wellenlängebestimmen kann.

α

β

Laser

n. Maximum

o. Maximum

Wand

Schieblehre

n

0y

α

n

x

y

0

Lösung:Die beiden Parallelstrahlen, die auf zwei benach-barte Strichrillen fallen und von dort aus in alleRichtungen reflektiert werden ± hier sind nur diereflektierten Strahlen in Richtung bn herausgegrif-fen ±, besitzen bis zum Auftreffen auf die Rillenden Gangunterschied Ds1 d cos a und nachdem Auftreffen den von Ds2 d cos bn, insgesamtalso den Gangunterschied DsDs1ÿDs2 (wegena < bn). Damit ergibt sich mit Ds nl als Bedin-gung für ein Maximum in Richtung bn

nl d (cos aÿ cos bn).

Die Winkel a und bn entnimmt man aus der Geo-metrie des Versuchs über tan a y0=x0 undtan bn yn=x0.

7.3.4 Beugung und Interferenz am Spalt

1 An einer Beugungsfigur am Spalt misst man alsAbstand der beiden Beugungsminima 1. Ordnung15 mm, weiter e 212 cm. Eine 10 mm-Messskalawird mit einem Diaprojektor auf der Projektions-wand 45 cm groû abgebildet, der verwendete Spaltunter den gleichen Bedingungen 7 mm groû. Be-stimmen Sie die Wellenlänge.

Lösung:d 10 mm 7 mm=45 cm 0,156 mml d sina d Ds=e 0,156 10ÿ3 m 15 10ÿ3 m=(2,12 m 2) 552 nm.

2 Licht einer Wellenlänge von 550 nm geht durcheinen 0,15 cm breiten Spalt und fällt auf einen2,5 m entfernten Schirm. Wie breit ist der Streifen

des Maximums 0. Ordnung (vom rechten bis zumlinken Minimum 1. Ordnung)?

Lösung:l=d sin aDs=e;2 Ds 2 e l=d

2,5 m 550 10ÿ9 m=0,15 cm 1,83 mm.

*3 Wie breit muss ein Einzelspalt mindestens sein, da-mit man das Minimum n-ter Ordnung (n 1, 2, . . .)beobachten kann? Beschreiben Sie demnach dieVorgänge, die sich abspielen, wenn ein Spalt lang-sam ganz geschlossen wird. Erklären Sie damit denVersuch 1 auf S. 294.

Lösung:1 sin an n l=d; d n l.Schlieût man den Spalt ganz, dann kann manschlieûlich nur noch das zentrale Maximum be-

Seite

299

7.3 Licht als klassische Welle 135

Page 136: Metzler Physik

obachten, das dann den ganzen Winkelbereich hin-ter dem Spalt einnimmt.

*4 Stellen Sie einen Spalt her und verkippen Sie diebeiden Kanten etwas gegeneinander, sodass einkeilförmiger Spalt entsteht. Beobachten Sie durchdiesen Spalt eine möglichst punktförmige Licht-quelle. Beschreiben und erklären Sie Ihre Be-obachtungen.

Lösung:Man beobachtet ein keilförmiges Maximum 0.Ordnung und Beugungsstreifen, die an der breiten

Kante enger stehen als an der Spitze d. Keils, da ander Spitze der Spalt enger ist.

*5 Stellen Sie einen Spalt her. Beobachten Sie durchdiesen Spalt eine möglichst punktförmige Licht-quelle und drehen Sie den Spalt langsam um seineAchse. Beschreiben und erklären Sie Ihre Be-obachtungen.

Lösung:Die Beugungstreifen bilden einen ¹Strahlª, dersenkrecht auf der Spaltrichtung steht und sich beimDrehen des Spaltes mitdreht.

7.3.5 Intensitätsverlauf bei Beugungsfiguren

1 Zeichnen Sie wie in Abb. 300.2 c) den Intensitäts-verlauf I a=I0 hinter einem Gitter mit 7 Spaltenim Bereich 0Ds l und die dazugehörigen Zei-gerdiagramme an den Nullstellen.

Lösung:

I(Ds)I0

sin

7pDsl

7 sin

pDsl

!2

für 0 Dsl 1

0,2

0,4

0,6

0,8

1,0

00,2 0,4 0,6 0,8 1,00

I0I

s∆λ

2 Zeichnen Sie den Intensitätsverlauf I a=I0 hintereinem Einfachspalt zwischen Hauptmaximum unddrittem Nebenmaximum. Berechnen Sie I a=I0

an den (ungefähren) Stellen der MaximaDs=l 1

2 2n 1 mit n 1, 2, 3 , . . . . Wie hängtder Wert I a=I0 an den Maxima von n ab?

Lösung:Zu zeichnen ist f (x) I(a)=I0 (sin px=px)2;

ÿ 4 xDs=l 4

–4

0,2

0,4

0,6

0,8

1,0

0

I0

I

s∆λ

α( )

–3 –2 –1 1 2 3 4

Die Werte in den ¹Maximaª nehmen mit1

(n 1)2ab.

*3 Wie viel Prozent der Gesamtintensität bei der Beu-gung am Einzelspalt entfällt auf das Hauptmaxi-mum, wie viel auf die n-ten Nebenmaximan 1, 2?Lösung:

Die Intensität am Spalt wird beschrieben durch:

I=I0 (sin[p x]=p x)2 (s. Aufgabe 2)

Zur Berechnung der Intensitäten in den Haupt- undNebenmaxima muss man die entsprechenden Flä-chen unter dieser Kurve bestimmen. Dies geschiehtam einfachsten numerisch: Man zerlegt die Flä-chen in kleine Streifen der Breite Dx und summiertihre Flächeninhalte. Ein sehr einfaches Programmin PASCAL könnte so aussehen:

Seite

301

7 Elektromagnetische Schwingungen und Wellen136

Page 137: Metzler Physik

program Spalt;uses crt;

function flaeche(x1,x2:real):real;const pi=3.141592653589;var i,n :Longint;

sum,x,y,dx :real;

begindx:=0.001; ungefähre Schritt-

weiten:=round((x2-x1)/dx);Schrittzahldx:=(x2-x1)/n; exakteSchrittweitesum:=0;x:=x1;for i:=1 to n do

beginif x=0 then y:=1 Division durch

Null vermeidenelse y:=sin(pi*x)/(pi*x);

sum:=sum+y*y*dx;x:=x+dx;

end;flaeche:=sum;

end;

beginclrscr;writeln(flaeche (0,100):10:7);writeln(flaeche (0,1):10:7);writeln(flaeche (1,2):10:7);writeln(flaeche (2,3):10:7);writeln(flaeche (3,4):10:7);readln;

end.

Die Ausgabe ergibt:

0.4999934 (Die Gesamtintensität ist 1);0.4519117 (Die Intensität im

Hauptmaximum ist 90.38%);0.0235580 (Die Intensität im

1. Nebenmaximum(links + rechts) ist 4.71%);

0.0082355 (Die Intensität im2. Nebenmaximum(links + rechts) ist 1.65%);

0.0041690 (Die Intensität im3. Nebenmaximum(links + rechts) ist 0.83%).

7.3.5 Intensitätsverlauf bei Beugungsfiguren

1 Berechnen und zeichnen Sie schematisch den In-tensitätsverlauf der Beugungs- und lnterferenzfi-gura) eines Doppelspalts mit der Spaltbreite

a 0,25 mm und dem Spaltabstandg 0,75 mm;

b) eines Gitters mit 4 Spalten der Breitea 0,25 mm und der Gitterkonstanten (Spalt-abstand) g 0,50 mm.

Lösung:

II0

sin npxnsin px

sin px0px0

!2

;

n Spaltzahl

x Dsl

; Ds Gangunterschied zwischenbenachbarten Spalten.

x0 Ds0

l; Ds0 Gangunterschied zwischen den

Randstrahlen eines Einzelspalts.

Es gilt: x0 ag x

Einsetzen der Werte liefert folgende Kurven:

a)

–7,5 –5 –2,5 0 2,5 5 7,5 10

0,2

0,4

0,6

0,8

1,0 I0I

s∆λ

b)

–7,5 –5 –2,5 0 2,5 5 7,5 10

0,2

0,4

0,6

0,8

1,0 I0I

s∆λ

Seite

302

7.3 Licht als klassische Welle 137

Page 138: Metzler Physik

2 Führen Sie die Rechnungen über den Intensitäts-verlauf des Gitters im Einzelnen aus und berechnenSie für ein Gitter mit N 10 Öffnungen die derAbb. 300.2c) entsprechende Figur für den Intensi-tätsverlauf.

Lösung:

I=I0 = [sin(10 px)=(10 sin (px)]2 ; xDs=l ;Einsetzen der Werte liefert folgende Kurve:

0,2

0,2

0,4

0,6

0,8

1,0

0,4 0,6 0,8 1,0

I0I

s∆λ0

0

*3 a) Schreiben Sie ein Programm zur Berechnungder Intensität hinter der Kreisblende und be-rechnen Sie I(a) für l 500 nm, d 0,2 mm(0 a 4 l=d). Bestätigen Sie damit die obenangegebenen Werte über die Lage der Nullstel-len.

b) ¾ndern Sie das Programm so ab, dass Sie damitdie Beugung hinter einer kreisförmigen Öff-nung mit einer zentralen runden Blende be-rechnen können. Beschreiben Sie die Ergeb-nisse. Diese Anordnung entspricht dem New-tonschen Spiegelfernrohr mit Fangspiegel.

Lösung:a)PROGRAM Kreisblende;

Beugung an der Kreisblende

USES crt,graph;

CONST lambda = 500.0 e-9; Wellenlänge in nm

dy = 0.005; 400 horizontale Streifen VAR phi,dphi,y,alpha,ax,ay,s,ig,i0 :REAL;

i,gm,gd :INTEGER;int :ARRAY[0..200] OF REAL;out :TEXT;

BEGINclrscr;FOR i:=0 TO 200 DO

Intensitätsberechnung für 0 [ alpha [ 2lambda/r

BEGINalpha:=i*0.01*lambda;ax:=0;

x und y-Komponenten des resultierendenE-Vektors

ay:=0;phi:=0;dphi:=2*pi/lambda*dy*sin(alpha);

Phasendifferenz zwischen benachbartenStreifen

Summation ueber alle Streifen von y=-1 bisy=1 :

y:=-1;WHILE y < 1 DO

BEGINs:=sqrt(1-y*y); Breite des Streifens in derHöhe y ax:=ax+s*cos(phi);ay:=ay+s*sin(phi);

Summation der Vektorkomponentenproportional s

phi:=phi+dphi;y:=y+dy; nächster Streifen

END;Int[i]:=2*dy/pi*(ax*ax+ay*ay); Intensitätproportional E^2

END;

Ausdruck

FOR i:= 1 TO 200 DOBEGIN

writeln(0.01*i:4:2,' ',Int[i]:10:6);IF i mod 20 = 0 THEN readln;

END;

7 Elektromagnetische Schwingungen und Wellen138

Page 139: Metzler Physik

readln;

Plot

detectgraph(gm,gd);initgraph(gm,gd,'c:\tp6/ bgi');FOR i:= 1 TO 200 DO

putpixel(2*i,round(getmaxy-getmaxy*Int[i]/Int[0]),white);

readln;closegraph;

END

b)PROGRAM Ringblende;

Beugung an der Ringblende

USES crt,graph;

CONST lambda = 500.0 e-9; Wellenlänge in nm dy = 0.01; 200 horizontale Streifen ri = 0.0; 0 [ ri [ 1 : innerer Radius des

Ringesri = 0 : keine innere Blende

VAR phi,dphi,y,alpha,ax,ay,s,i0,ig : REAL;i,gm,gd : INTEGER;int : ARRAY[0..200] OF REAL;out : TEXT;

BEGINclrscr;FOR i:=0 TO 200 DO

Intensitätsberechnung für 0 [ alpha [ 2lambda/d BEGINalpha:=i*0.01*lambda;ax:=0; x und y-Komponenten des resultierenden

A-Vektors ay:=0;phi:=0;dphi:=2*pi/lambda*dy*sin(alpha); Phasendifferenz zwischen benachbartenStreifen

SummationueberalleStreifenvony=-1bisy=1:y:=-1;WHILE y < 1 DOBEGIN

s:=sqrt(1-y*y);IF abs(y)<ri THEN s:=s-sqrt(ri*ri-y*y);

Breite des Streifens in derH öhe y

ax:=ax+s*cos(phi);ay:=ay+s*sin(phi);SummationderVektorkomponentenprop.sphi:=phi+dphi;y:=y+dy; nächster Streifen END;

Int[i]:=2*dy/pi*(ax*ax+ay*ay); Intensität prop. A^2

END;

Ausdruck

writeln(' innere Blende ri = ',ri:5:3);writeln;writeln('lamda/d Intensitaet ');FOR i:= 1 TO 200 DO

BEGINwriteln(0.01*i:4:2,' ',Int[i]:10:6);IF i mod 20 = 0 THEN readln;

END;readln;

Plot

detectgraph(gm,gd);initgraph(gm,gd,'c:\tp6/bgi');FOR i:= 1 TO 200 DOputpixel(2*i,round(getmaxy-getmaxy*Int[i]/Int[0]),white);readln;closegraph;

END.

7.3 Licht als klassische Welle 139

Page 140: Metzler Physik

7.3.6 Das Auflösungsvermögen optischer Instrumente

1 Der Spiegel des Hubble-Weltraumteleskops hat ei-nen Durchmesser von 2,4 m. Berechnen Sie dennoch auflösbaren Winkelabstand in Bogenmaûund Grad im Licht von 550 nm Wellenlänge. Wel-chen Abstand müssen zwei Punkte a) auf demMond, b) auf dem Mars mindestens haben, damitdas Fernrohr sie gerade noch trennen kann?

Lösung:Da 1,22 l=d 2,29 10ÿ7 1,31 10ÿ5

0,047MM;

Dx (Mond) rMond Da 385 000 km ´ Da 88,2 m;

Dx (Mars) rMars Da 55 ´ 106 km ´ Da 12,6 km.

2 Für das Auflösungsvermögen des Auges entschei-dend ist der Pupillendurchmesser, der bei mittlererBeleuchtung 3 mm beträgt. Welchen Abstand müs-sen zwei Punkte in 5 m Entfernung haben, damitman sie mit bloûem Auge noch trennen kann(l 500 nm)?

Lösung:Da 1,22 l=d 2,03 ´ 10ÿ4;Dx r Da 5 m ´ 2,03 ´ 10ÿ4 1,02 mm.

3 Die Angabe 8 30 auf einem Prismenfernrohr be-deutet, dass das Fernrohr den Sehwinkel achtfachvergröûert, und dass der Objektivdurchmesser30 mm beträgt. Wie groû ist der Abstand zweierPunkte in 500 m Entfernung, die gerade noch un-terschieden werden können (l 500 nm)?

Lösung:Da 1,22 l=d 2,03 ´ 10ÿ5;Dx r Da 1,02 cm.

Der um 8 vergröûerte Sehwinkel beträgt 1,62 ´ 10ÿ4

8 Auflösungsvermögen des Auges (Aufg. 2)

*4 Wie viel Information lässt sich mit einem Mikro-skop von einem Quadratzentimeter Oberfläche mitrotem Licht (l 680 nm) höchstens ablesen? Washat diese Tatsache mit der Speicherdichte vonelektronischen Medien zu tun? Warum versuchtman, die Wellenlänge des Lichtes, das zum Able-sen benutzt wird, immer kleiner zu machen?

Lösung:N (Zahl der Bits) A=(Dx)2

1 cm3=(680 10ÿ9 m)2

2,16 ´ 1012.

Mit den zur Herstellung von Halbleiterstrukturenbenutzten optischen Methoden lassen sich keinekleineren Strukturen erzeugen.

*5 Entwerfen und konstruieren Sie die ¹optimaleªLochkamera, d. h. eine Lochkamera mit einer mög-lichst guten Winkelauflösung. Beachten Sie: Jekleiner die verwendete Lochblende ist, desto klei-ner ist ihr geometrischer Schatten, desto gröûeraber die von ihr hervorgerufene Beugungsfigur.

Lösung:Geometrische Winkelauflösung der Lochkamera:

d

l

1∆α

= d2 · l1∆α

Aus der Abbildung folgt die geometrische Winkel-auflösung: Da1 d=(2 l).

Beugungsbedingte Winkelauflösung:Da2 1,22 l=d.

Optimale Lochkamera:Da1 Da2

d=(2 l) 1,22 l=d ) d 1,56l lp

.

Da d=(2 l) 0,78l=l

p; d. h. man kann im Prin-

zip durch Verlängerung der Kamera ein beliebiggutes Auflösungsvermögen erreichen.

*6 Zwei Radioteleskope in Deutschland und in denUSA werden zu einem einzigen groûen VLBI-In-strument zusammengeschlossen (! S. 305). Be-rechnen Sie den noch auflösbaren Winkelabstandfür Radiowellen im UKW-Bereich.

Lösung:Da 1,22 l=d 1,22 20 cm=6000 km 4,07 ´ 10ÿ8 0,0084MM.

Seite

304

7 Elektromagnetische Schwingungen und Wellen140

Page 141: Metzler Physik

7.3.7 Interferenzen an dünnen Schichten

1 Der Gangunterschied zwischen zwei zur Interfe-renz kommenden Wellenzügen seia) Ds 10 ´ 10ÿ7 m bzw.b) Ds 100 ´ 10ÿ7 m.Welche Wellen aus dem Bereich l 400 nm bisl 750 nm verstärken sich dann gerade maximal?Wie groû sind jeweils die Ordnungszahlen derInterferenzen?

Lösung:Von den Wellenlängen ln Ds=n, die die Interfe-renzbedingung erfüllen, fallen ina) nur l2 500 nm, inb) l14 714,2 nm, . . .

l25 400 nmin den vorgegebenen Bereich.

2 Zu Versuch 1 mit dem Glimmerblatt: Schätzen Siedie Ordnung des Gangunterschiedes für Licht mitt-lerer Wellenlänge ab, das an der Vorder- und an derInnenseite des Glimmerblattes (Dicke d 1,0 mm)reflektiert wird.

Lösung:nDs=l 1 mm=600 nm 1666.

3 Eine 0,75 mm dicke Seifenhaut (n 1,35) wirdsenkrecht bestrahlt. Welches Licht (Wellenlängen)wird bei Reflexion ausgelöscht, welches verstärkt?

Lösung:Verstärkung:ml 2d n l=2(mÿ 1

2 ) l 2d n;l 2d n=(mÿ 1

2 ); m 1, 2, . . .

lm 4,04 mm=(2 mÿ 1) 4,04 mm, 1,35 mm;0,808 mm; 0,577 mm; 0,449 mm; 0,367 mm.

Auslöschung:(2m 1)l=2 2 d n l=2; m 1, 2, . . .m l 2d nl 2,02 mm=m 2,02 mm=1,01 mm=0,674 mm= 0,506 mm=0,404 mm.

4 Newton'sche Ringe beobachtet man häufig auchbei der Projektion von glasgerahmten Diapositi-ven. Wie kommt diese Erscheinung zustande?

Lösung:Die Newton'schen Ringe entstehen, wenn sich dasDiapositiv wellt und so mit dem Glas Luftspaltewie in Abb. 307.1 b bildet.

*5 Die Oberfläche einer Linse (n 1,53) wird mit ei-nem Material (n 1,35) vergütet, sodass die Wel-lenlänge l 550 nm im reflektierten Licht ausge-löscht wird. Wie dick muss die Schicht sein? Wel-che Phasenverschiebung erleiden das reflektierteviolette Licht (l 400 nm) und das rote Licht(l 700 nm)?

Lösung:Wegen nLinse > nM finden beide Reflexionen amdichteren Medium statt.Ds 2 d n l=2d l=4n 550 nm=(4 1,35) 101,8 nm.Dò 2 p Ds=l 2 p 2 101,8nm=l 3,20 [1,82]

*6 Die Radien des 1. und 3. dunklen Ringes einerNewton'schen Plankonvexlinse (Krümmungshalb-messer R 118 cm) sind 0,83 mm bzw. 1,45 mm.Wie groû ist die Wellenlänge des benutzten Lich-tes?

Lösung:

l (r23 ÿ r2

1)(3ÿ 1) R 0,599 mm

7.3.9 Holografie

1 Erläutern Sie das Prinzip der Fresnel'schen Zonen-platte.

Lösung:Ð

2 Wie wirkt sich eine endliche Kohärenzlänge derLichtquelle bei der Herstellung von Hologrammenaus?

Lösung:Sie begrenzt die maximale Ausdehnung, die einGegenstand haben darf, von dem man ein Holo-gramm anfertigen will.

*3 Konstruieren Sie eine Fresnel'sche Zonenplatte fürMikrowellen der Frequenz 9 GHz. Sie soll eineBrennweite von 20 cm haben. Stellen Sie ein Mo-dell aus Karton und Aluminiumfolie her und erpro-ben Sie es.

Seite

307

Seite

311

7.3 Licht als klassische Welle 141

Page 142: Metzler Physik

Lösung:

sn

f+ F

Sie muss Ringöffnungen mit den Radien rn haben,für die gilt:

Dsn snÿ f n ln 0, 1, 2, . . .

sn r2

n f 2p

f 2 r2

n

pÿ f n l

(nl f )2 f 2 r2n

rn 2 nl f n2 l2

q; n 0, 1, 2, . . .

133,2 n 11,08 n2p

cm;

r 0 cm; 11,09 cm; 17,63 cm; . . .

Zusatzaufgaben

4 Linear polarisiertes Licht wird im sogenanntenl=4-Blättchen in zwei Wellen mit dem Gangunter-schied von l=4, zerlegt, die senkrecht zueinanderpolarisiert sind. Überlegen Sie sich, dass ± ähnlichwie die Schwingungen zweier senkrecht zueinan-der stehender Schwingungsvektoren in der Mecha-nik ± beide Wellen sich hinter dem Blättchen zueiner elliptisch polarisierten Welle zusammen,

und diskutieren Sie die verschiedenen Möglichkei-ten, die sich bei der Drehung des l=4-Blättchensergeben.

Lösung:Lässt man linear polarisiertes Licht senkrecht aufl=4-Blättchen unter dem Winkel a zur ausgezeich-neten Richtung des Blättchens fallen, so wird dasLicht in zwei senkrecht zueinander polarisierteKomponenten zerlegt, die das Blättchen mit demPhasenunterschied p=2 verlassen und sich zu einersogenannten elliptisch polarisierten Welle zusam-mensetzen. Wie bei der Überlagerung zweier senk-rechter zueinander stehender Schwingungen ergibtsich in x-Richtung eine Schwingung, deren Ampli-tude sich mit der Frequenz w ändert:x AÃ cos a sin w t,und in y-Richtungy AÃ sin a sin (w tp=2) AÃ sin a cos w t.Beide Schwingungsvektoren setzen sich zu einemzusammen: Durch sin2 w t cos2 w t 1 erhält man

x2

A2 cos2 a y2

A2 sin2 a 1:

Die Spitze des Schwingungsvektors rotiert mit derFrequenz w auf einer Ellipse.Hält man das l=4-Blättchen in 45-Stellung, so istcos2 a sin2 a 1

2 , und die Amplitude beschreibteinen Kreis: Man erhält zirkular polarisiertes Licht.

x2 y2 12 A2.

Für a 0 und a 90 ergibt sich linear polari-siertes Licht, für 0 < a < 45 und 45 < a < 90elliptisch polarisiertes Licht.

7.3.10 Polarisiertes Licht

1 Worauf beruht die Wirkungsweise von Polarisa-tionsfiltern?

Lösung:(vgl. Exkurs).

2 Berechnen Sie allgemein die Amplitude A, die voneiner linear polarisierten Welle (Amplitude A0)durch ein Polarisationsfilter in beliebiger Stellunghindurchgelassen wird.

Lösung:Die durchgelasseneAmplitude der Welle istE(a) E cos a;alsoI(a) E(a)2 I0 cos2 a.

α

αE ( )

E

Seite

312

7 Elektromagnetische Schwingungen und Wellen142

Page 143: Metzler Physik

*3 Berechnen Sie die Intensität einer durch ein Pola-risationsfilter hindurchgehenden linear polarisier-ten Welle und zeichnen Sie den Verlauf der Inten-sität in Abhängigkeit vom Winkel zwischenSchwingungsebene und Stellung des Polarisators.

Lösung:

90° 180°α

I0

I α( )

1 I0I α( ) = · cos2α

Zusatzaufgabe

4 Worauf beruht die Wirkungsweise eines Polarisa-tionsfilters eines Photoapparats?

Lösung:Polarisationsfilter sollen das Licht, das bei Spiege-lung auf Wasser, Fensterscheiben, Glanzpapier,auch auf Farbe, Firnis, Holz meistens polarisiertist, löschen, indem man das Filter geeignet dreht.Metallische Oberfläche polarisieren dagegen dasvon ihnen reflektierte Licht nicht linear, und daherwerden Reflexe auf metallischen Oberflächen vonPolarisationsfiltern nicht beeinflusst.Bei Farbaufnahmen kann man mit einem Polarisa-tionsfilter einen blassblauen Himmel in einen dun-kelblauen verwandeln, da das Himmelslicht durch-weg polarisiert ist.

7.3.10 Polarisiertes Licht

1 Wie groû ist der Brewster-Winkel für Wasser,Schwerflint und Diamant (! Tab. 318.2)?

Lösung:tan aB n;

aB arc tan n(arc tan 1,33 58,95

arc tan 1,76 67,11

arc tan 2,42 75,05

2 Wo liegen die Orte mit dem höchsten Polarisa-tionsgrad des Himmelslichtes a) bei Sonnenauf-gang, b) bei Sonnenuntergang, c) beim höchstenSonnenstand mittags?

Lösung:Sie liegen immer dort, wo der Winkelabstand zurSonne 90 beträgt, also a): auf einer Linie (Groû-kreis) vom Nordpunkt über den Zenit zum Süd-punkt, b): wie a), c): vom Ostpunkt nördlich überden Meridian zum Westpunkt.

*3 Unpolarisiertes Licht trifft unter dem Brewster-Winkel auf einen Glasplattensatz von 12 Platten(n = 1,5).a) Wie hoch ist der Polarisationsgrad des durch-

gelassenen Lichts?b) Wie viele Platten benötigt man, um zu 95% po-

larisiertes Licht herzustellen?(Hinweis: Von Licht, das senkrecht zur Refle-xionsebene polarisiert ist, werden jeweils 4%reflektiert.)

Lösung:a) Vom Licht, das senkrecht zur Reflexionsebene

polarisiert ist, wird insgesamt ein Anteil von

0,04 0,04 ´ 0,96 0,04 ´ 0,962 . . . 0,04 ´ 0,9611

0,04 ´ (1 0,962 . . . 0,9611)

0,041ÿ 0,9612

1ÿ 0,96 1 ÿ 0,9612 0,387

38,7%

reflektiert, d. h. 61,3% durchgelassen.

Licht, das parallel zur Reflexionsebene polari-siert ist, wird nicht reflektiert (Brewsterwin-kel!), also vollständig durchgelassen.

Da vor der Reflexion I? Ijj 0,5 I0 war,wird der Polarisationsgrad p des durchgelasse-nen Lichtes

p IjjI? Ijj

0,5 I0

0,613 0,5 I0 0,5 I0

11 0,613

62,0%

b) Man entnimmt Aufg. a): p 11 0,96n

nlog

1pÿ 1

log 0,96 72,12.

Man benötigt also 73 Platten.

Seite

314

7.3 Licht als klassische Welle 143

Page 144: Metzler Physik

*4 SchicktmaneinenunpolarisiertenLichtstrahldurcheine trübe Flüssigkeit und betrachtet die Anordnungseitlich durch ein Polarisationsfilter, dann ändertsich beim Drehen des Filters die Intensität (¹Tyn-dall-Effektª). Erklären Sie diese Erscheinung.

Lösung:Wie beim Sonnenlicht erläutert, ist jedes Streulichtpolarisiert.

7.3.12 Strahlenoptik

1 Zeigen Sie, dass sich bei der Drehung eines Spie-gels um eine in der Spiegelebene gelegene Achseum den Winkel a der reflektierte Strahl um dendoppelten Winkel dreht.

Lösung:Bei der Drehung eines Spiegels um den Winkel adreht sich das Einfallslot um a. Daher ändert sichsowohl der Einfallswinkel als auch der Reflexions-winkel um a, ihre Differenz also um 2 a.

2 Licht der Fraunhofer-Linie A bzw. F fällt unterdem Einfallswinkel a 30 von Luft auf a) Flint-glas, b) Schwefelkohlenstoff. Berechnen Sie dieBrechungswinkel und die Lichtgeschwindigkeitenin dem neuen Medium.

Lösung:sin a=sin b n; sin b (sin a)=n

c in km=s

a) A (n 1,603) : b 18,17F (n 1,625) : b 17,92

187 019184 487

b) A (n 1,609) : b 18,10F (n 1,652) : b 17,62

186 322181 472

3 Wie groû ist der absolute Brechungsindex von Öl-säure, wenn der relative Brechungsindex für denÜbergang von Wasser in Ölsäure 0,91 und der ab-solute Brechungsindex des Wassers 1,33 betragen?

Lösung:

n12 n2

n1

0,91 1,33n1

; n1 1,46.

4 Welches ist der Winkel der Totalreflexion, wennLichta) von Glas in Wasser,b) von schwerstem Flint in Schwerkron übergeht?

Lösung:

sin bT n2

n1

a) bT 62,5b) bT 57,9

5 In optischen Geräten verwendet man zur Reflexionvorwiegend die Totalreflexion. Welche Vor-, wel-che Nachteile hat die Totalreflexion gegenüber derReflexion an normalen Spiegeloberflächen?

Lösung:Vorteile: s. Text.Nachteile: Sie tritt nur unter bestimmten Winkeln

auf und nur beim Übergang in Richtungvon Glas nach Luft.

6 Wie groû ist die Dielektrizitätskonstante von Was-ser für die Frequenzen des sichtbaren Lichtes?

Lösung:e n2 1,332 1,77.

*7 Vom Boden eines Gefäûes, in dem Schwefelkoh-lenstoff, Wasser und Benzol aufeinander geschich-tet sind, ohne sich zu vermischen, durchläuft einLichtstrahl die erste Schicht (aus Schwefelkohlen-stoff) unter dem Winkel a zur Senkrechten nachoben.a) Unter welchem Winkel tritt der Strahl nach

Durchlaufen der drei Schichten oben aus, wenna 5 war?

b) Für welchen kleinsten Wert a a0 wird derLichtstrahl an der Grenzfläche von Schwefel-kohlenstoff und Wasser bzw. an der von Benzolund Luft totalreflektiert?

Lösung:a) b 8,16

b2 5,43b1 6,13

1,50 = n

β

3

1

β

1,33 = n2

1,63 = n15° = α

C H

S

6

n = 1,128

2

6

H20

Cn = 2

1= 0,816n

n

Seite

320

7 Elektromagnetische Schwingungen und Wellen144

Page 145: Metzler Physik

b) sin bT 0,816 bT 54,7 a0

sin b1 sin a=0,816sin b2 sin b1=1,128

sin a0=(1,128 0,816), a0 67

*8 Unter einem Prisma versteht man in der Optik ei-nen keilförmigen Glaskörper, dessen Flächen umden ¹Prismenwinkelª e gegeneinander geneigtsind.a) Berechnen Sie die Ablenkung d, die ein Licht-

strahl beim Durchgang durch ein Prisma er-fährt, in Abhängigkeit von e und vom Einfalls-winkel. Die Ablenkung d ist minimal, wenn derStrahlengang symmetrisch ist, d. h. wenn derStrahl beim Eintritt in das Prisma ebenso weitvon der Prismenkante entfernt ist wie beimAustritt.

b) Wie groû ist für ein Prisma aus Flintglas F3 miteinem Prismenwinkel von 60 bei symmetri-schem Strahlengang die Breite Dd des Spekt-rums zwischen den Fraunhofer- Linien A undH?

Lösung:

C

AB

ε

α β

δ1

β

a) sin b sin an

in DABC:90 ÿb e (90 ÿa1) 180

, a1 eÿbsin b1 n sin a1;

Ablenkung:d aÿbb1ÿa1

d aÿ arc sin (sin a=n) arc sin (n sin (eÿb)) ÿeb ;

b) symmetrischer Strahlengang:d 2 (aÿ e=2) 2 (arc sin (n sin e=2)ÿ e=2)

Dd d (H)ÿd (A) 60,67 ÿ 46,55 4,12 .

7.4.3 Röntgenstrahlen

1 a) Röntgenlicht der Wellenlänge l 150 pm wirdan einem NaCl-Kristall reflektiert. In welchemBereich muss dazu der Netzebenenabstand d imNaCl-Kristall liegen?

b) Bestimmen Sie für d 278 pm die Glanzwin-kel q, unter denen eine starke Reflexion zu er-warten ist.

Lösung:a) 2d sin q n l sin q 1

2d n l=sin q nlalso d l=2 75 pm.

b) sin qn nl=(2d) n 150 pm=(2 278 pm) n ´ 0,2698

q1 15,65, q2 32,65, q3 54,03.

2 Um Blutbahnen mit Röntgenstrahlen zu erfassen,spritzt man besondere Kontrastmittel ins Blut.Welche Eigenschaften müssen diese Flüssigkeitenhaben?

Lösung:Sie müssen ein hohes Absorptionsvermögen fürRöntgenstrahlen haben und ungiftig sein (z. B.Bariumsulfat BaSO4).

Seite

329

7.4 Das elektromagnetische Spektrum 145

Page 146: Metzler Physik

3 Geben Sie ein Verfahren an, mit dem aus weiûemRöntgenlicht monochromatisches Röntgenlichtausgesondert werden kann.

Lösung:Durch Bragg-Reflexion an einem Einkristall: Nurdie Wellenlängen, die der Bragg-Bedingung genü-gen, werden dabei reflektiert.

4 Welche Eigenschaften machen Röntgenlicht fürLebewesen gefährlich?

Lösung:Die Ionisation: Sie erzeugt somatische und geneti-schen Schäden, s. Text.

5 Berechnen Sie die Abstände der Kristallebenenvon Abb. 329.1b). Der Abstand der Li-Atome imLiF beträgt 208 pm.

Lösung:Man findet elementargeometrisch:

d120 a=5p 89,9 pm

d121 a=6p 82,1 pm.

7 Elektromagnetische Schwingungen und Wellen146

Page 147: Metzler Physik

1 Schreiben Sie ein Computerprogramm, das die ers-ten 50 Werte der Verhulst-Folge berechnet undvergleichen Sie die Ergebnisse mit Abb. 338.1.

Lösung:program verhulst;

Verhulst'scher Wachstumsprozess

USES crt,graph;

type string40 string[40];

var x,Xnext,r: real;i,n,n1,n2,x1,x2,yl,y2: integer;Antw: Char;Nmr: string[4];Titel: string[80];graphdriver,graphmode,errorcode: INTEGER;

BEGINREPEAT

ClrScr;writeln('VERHULST-Prozess:');writeln('x(n1)( r x(n) (1-x(n))');writeln;writeln;write('x0: (0:Ende)');readln(x);IF x0 THEN exit;write('r:');readln(r);write('n1:');readln(n1);graphdriver:detect;InitGraph(GraphDriver,GraphMode,C:\TP6\

BGI';n:0;Titel:' Verhulst'chr(39)'scher

Wachstumsprozess ';str(r:3:1,Nmr);Titel:titel'r'Nmr;str(y:3:1,Nmr);titel:titel' x0'Nmr;SetTextJustify(LeftText, LeftText);SetTextStyle(DefaultFont,Horizdir,1);outTextXY(10,10,titel);rectangle(20,20,getmaxX-20,getmaxY-20);outTextXY(10,getmaxy div 2,'x');outtextxy(10,getmaxy-20-round((getmaxy-40)/

1.5),'1');outTextXY(getmaxx div 2,getmaxy-10,'n');str(n 1:4,Nmr);outTextXY(10,getmaxy-10,Nmr);str(n150:4,Nmr);outTextXY(getmaxx-40,getmaxy-10,Nmr);

IF n1 > 1 THEN

BEGINREPEAT

n:n1;x:r*x*(1ÿx);

UNTIL n >n1;END;

FOR i:0 to 50 DOBEGIN

xnext:r*x*(1ÿx);x1:20round((getmaxx-40)/50*i);y1:getmaxy-20-round((getmaxy-40)/

1.5*y);x2:20round((getmaxx-40)/50*(i1));y2:getmaxy-20-round((getmaxy-40)/

1.5*ynext);IF (x1 >0) and (x1 <getmaxx-20) and

(x2 >0) and (x2 <getmaxx-20)and (y1 >0) and (y1 <getmaxy) and(y2 >0) and (y2 <getmaxy)

THEN line(x1,y1,x2,y2);x:xnext;

END;readln;closegraph;

UNTIL false;END.

2 Zeigen Sie, dass die Fixpunkte der Verhulst-Folgeden Wert x1 (rÿ 1)=r haben. (Hinweis: DerFixpunkt ist erreicht, wenn xn1 xn ist.)

Lösung:x r x (1ÿ x) |x1 r(1ÿ x) rÿ r x, x rÿ 1

r3 Schreiben Sie ein Computerprogramm, das für

2,9 r 4 ein Bifurkationsdiagramm zeichnet.

Lösung:PROGRAM bifurkation;

Bifurkations-Diagramm desVerhulst-Prozesses

USES CRT,GRAPH;

VAR x,r,dr:REAL;t,m,i,k,aa,maxx,maxy:INTEGER;s:STRING;

BEGINDetectGraph(t,m);InitGraph(t,m,'c: \ tp6 \ bgi');maxx:GetMaxX;maxy:GetMaxY;

Seite

339

8.3 Wege ins Chaos ± Verhulst-Dynamik und Feigenbaum-Szenario 147

Page 148: Metzler Physik

s:' ';r:2.5; Anfangsparameter der Verhulst-

Dynamikdr:1.5/maxx;FOR i:1 TO maxx DO den ganzen Bildschirm

füllenBEGIN

x:0.5; Anfangswert der Verhulst-Dynamik

FOR k:1 TO 300 DOBEGIN

x:r*x*(1ÿx); Einschwingen END;

FOR k:1 TO maxy DOBEGIN

x:r*x*(1-x);PutPixel(i,Round(x*200),15);

ENDSetColor(black);OutTextXY(5,5,s);Str(r:10:5,s);SetColor(white);OutTextXY(5,5,s);r:rdr;

END;Readln;CloseGraph;

END.

4 Schreiben Sie ein Programm, das die Bifurkations-punkte der Verhulst-Dynamik möglichst genau be-rechnet und vergleichen Sie die Ergebnisse mitTab. 339.2.

Lösung:PROGRAM feigenbaum;

Halbautomatische Suche nach Bifurkations-punkten im Verhulst-Diagramm. dr, einschwing-zahl und delta muessen sorgfaeltig kontrolliertwerden.

USES crt;VAR x,nextx,delta,r,dr:REAL;

i,k,l,z,astzahl,einschwingzahl :INTEGER;xgrenzwert :ARRAY[1..50]OF REAL;istschondagewesen :BOOLEAN;

BEGINclscr;FOR i:1 TO 50 DO xgrenzwert[i]:100;

r:2.990; Anfangsparameter der Verhulst-Dynamik

dr:0.001; r-Inkrementdelta:1.E-5;einschwingzahl:10 000;

FOR i:1 TO 1000 DO

BEGINx:0.5; Anfangswert der Verhulst-

Dynamik

FOR k:1 TO einschwingzahl DOBEGIN

x:r*x*(1ÿx); EinschwingenEND k;

astzahl :1;xgrenzwert[1]:x;

FOR 1:1 TO 40 DO Bestimme astzahl die Zahl der Werte,die x annimmt

BEGIN

nextx:r*x*(1-x);istschondagewesen :false;

Vergleiche nextx mit allen schon vorgekom-menen x-Werten, um festzustellen, ob nextxein neuer x-Wert ist

z:1;REPEAT

istschondagewesen :(abs(xgrenzwert[z]-nextx) < delta);

inc(z);UNTIL (istschondagewesen OR (z > 50));

IF (NOT istschondagewesen) THENBEGIN

inc(astzahl);xgrenzwert[astzahl]:nextx;

END;

x:nextx;

END 1;

Writeln(r:7:3,' ',astzahl);readln;r:rdr;

END i;END.

8 Chaotische Vorgänge148

Page 149: Metzler Physik

9.1.1 Absoluter Raum und absolute Zeit

1 Nennen Sie Beispiele für Inertialsysteme und fürbeschleunigte Bezugssysteme.

Lösung:Auf der Erde bildet die Horizontale ± und gleich-förmig auf ihr bewegte Körper, wie z. B. ein mitkonstanter Geschwindigkeit v! fahrender ICE-Zug ± ein zweidimensionales Inertialsystem, daman normalerweise die Beschleunigung aufgrundder Erdrotation, der Bewegung um die Sonneoder gar der Bewegung um das Zentrum desMilchstraûensystems vernachlässigen kann.(Die entsprechenden Beschleunigungen sindaErdrot 3,37 10ÿ2 m=s2 cos ò, ò ist der Breiten-grad), aSonnensystem 5,9 10ÿ3 m=s2 und aMilchstr:

1;8 10ÿ10 m=s2).Nimmt man die Vertikale als dritte Dimensionhinzu, so ist der Raum auf der Erde wegen der Gra-vitationskräfte kein Inertialsystem. Erst ein imSchwerefeld frei fallendes Bezugssystem, z. B. ineinem Satelliten, stellt ein lokales Inertialsystemdar (!Allgemeine Relativitätstheorie, S. 367).Fahrstuhl, anfahrende Straûenbahn oder Karussellsind typische Beispiele für beschleunigte Bezugs-systeme.

2 Welche Beobachtungen zeigen, dass die Erde we-gen ihrer Rotationsbewegung kein Inertialsystemist ?

Lösung:Jean Bernard LØon Foucault (1819 ± 1868) führte1850 in der Pariser Sternwarte und 1851 im Pan-theon zu Paris seinen berühmten Pendelversuchdurch, mit dem er die Rotation der Erde nachwies.Der Versuch ist heute nach Foucault benannt, ob-wohl er bereits 1661, also bereits 18 Jahre nach demTode Galileis, erstmals von Viviani durchgeführtwurde.Die Auswirkungen der Corioliskraft (! 1.2.9) unddas Auftreten von Ebbe und Flut weisen ebenfallsauf die Rotation der Erde hin.

3 Zeigen Sie an einigen Beispielen aus der Mechanikdie Gültigkeit des Galilei'schen Relativitätsprin-zips.

Lösung:Berühmt ist das Gedankenexperiment Galileismit dem Stein, der sowohl bei ruhendem als auchbei gleichförmig fahrenden Schiff stets an der sel-ben Stelle des Schiffes niederfällt (aus Dialogo,1632). 1640 führte Pierre Gassendi im Golf vonLyon das Experiment mit dem von Galilei vorher-gesagten Ergebnis durch.Im Speisewagen eines ICE kann man wie in einemüblichen Restaurant problemlos an einer gedecktenTafel speisen; der Zug darf nur seine Geschwindig-keit weder nach Betrag noch nach Richtung ändern.

9.1.2 Das Michelson-Experiment

1 Berechnen Sie die Geschwindigkeit, mit der sichdie Erde um die Sonne bewegt. Die Erde läuft nä-herungsweise auf einem Kreis mit r 149,6 Mio.km.

Lösung:v 2p r=(1a) 2p149,6 106 km=(365,25 24 3600 s) 29,8 km=s

2 In dem von MICHELSON durchgeführten Experi-ment betrug die effektive Länge der Spektromete-rarme d 11 m.a) Berechnen Sie die erwartete Laufzeitdifferenz

tparÿ tsenk; vergleichen Sie sie mit der Schwin-gungsdauer von Licht.

b) Um welchen Phasenwinkel sollten sich dieSchwingungen der beiden Teilbündel bei einer

Drehung der Apparatur um 90 gegeneinanderverschieben?

Lösung:a) tparÿ tsenk

2d=c (1ÿ1ÿ v2=c2

p)=(1ÿ v2=c2)

d v2=c3 3,6 10ÿ16 s

Die Schwingungsdauer von grünem LichtbeträgtT l=c 550 nm=3 108 m=s 1,83 10ÿ15 s.

b) Dò=360 (tparÿ tsenk)=T 3,6 10ÿ16 s=1,83 10ÿ15 s)Dò 71

Seite

343

Seite

344

9.1 Von der klassischen Physik zur Relativitätstheorie 149

Page 150: Metzler Physik

9.1.3 Das Grundprinzip der Relativitätstheorie

1 Wie erklären die Postulate der Relativitätstheorieden negativen Ausgang des Michelson-Experi-ments?

Lösung:Hat das Licht in einem Inertialsystem in jeder Rich-tung die gleiche Ausbreitungsgeschwindigkeit, wiees das zweite Postulat fordert, so hat eine Drehungdes Michelson-Interferometers keine ¾nderung derLaufzeiten zur Folge, und eine Verschiebung derInterferenzstreifen ergibt sich nicht.

2 Was würde man klassisch und was relativistisch ineinem Handspiegel beobachten, wenn man ± wie

Einstein es sich vorstellte ± mit nahezu Lichtge-schwindigkeit läuft?

Lösung:Relativistisch würde sich wegen der Konstanz derLichtgeschwindigkeit das Spiegelbild nicht verän-dern. Nach der klassischen ¾ther-Theorie würdesich das Licht auf dem Weg vom Gesicht zum Spie-gel zunehmend verspäten ± eine Zeit, die auf demRückweg nicht wieder eingeholt werden könnte.Bei Annäherung an die Lichtgeschwindigkeit wür-de man sich zunehmend rückblickend im Spiegelsehen!

Seite

345

9 Relativitätstheorie150

Page 151: Metzler Physik

9.2.1 Die relative Gleichzeitigkeit

1 Der Relativitätsexpress rast mit nahezu Lichtge-schwindigkeit dahin, als ein Blitz in das vordereund einer in das hintere Ende des Zuges einschlägt.Ein Reisender, der sich in der Mitte des Zuges be-findet, und ein Bahnwärter drauûen am Bahndammsehen die Blitze gleichzeitig. Beim Eintreffen dervon den Blitzen ausgesandten Lichtsignale befin-den sich der Reisende und der Bahnwärter auf glei-cher Höhe. Welche Schlüsse ziehen beide darausüber die Zeiten, zu denen die Blitze einschlugen?

Lösung:Anschaulich kann man die Frage anhand vonAbb. 347.1 a) beantworten, wenn man die Abfolgeder Bilder von unten nach oben betrachtet, den zeit-lichen Ablauf also gerade umgekehrt, womit sichalle Bewegungsrichtungen umkehren. Der Rei-sende sitzt in der Mitte der unteren (ruhenden) Ra-kete: Für ihn sind, da ihn die Blitzsignale gleich-zeitig erreichen, die beiden Blitze auch gleichzeitigan den Enden eingeschlagen. Der Bahnwärter stehtdrauûen und beobachtet die obere (bewegte) Ra-kete: Für ihn ist der Blitz eher in die Spitze derRakete (also das Ende des Zuges) eingeschlagenund später in den Anfang des Zuges.

2 Zwei Raumschiffe fliegen mit halber Lichtge-schwindigkeit durch das Sonnensystem. Ihr Ab-stand im Sonnensystem beträgt konstant600 000 km. Geben Sie ein Verfahren an, mitdem die Besatzung der beiden Raumschiffe die Uh-ren an Bord synchronisiert. Um wie viel werden dieUhren im hinteren Raumschiff für einen Beobach-ter auf einem Planeten eher in Gang gesetzt?

Lösung:Die Besatzungen betrachten die beiden Raum-schiffe als Inertialsystem und nehmen die Synchro-nisation wie folgt vor: Vom hinteren Raumschiffwird ein Lichtsignal ausgesandt, gleichzeitig wer-den die Uhren dort in Gang gesetzt. Am vorderenRaumschiff wird das Signal reflektiert und gleich-zeitig werden dort die Uhren in Gang gesetzt. BeimEintreffen des Lichts im hinteren Raumschiff wirddie gesamte Laufzeit gemessen und dem vorderenRaumschiff mitgeteilt, dass dessen Uhren um diehalbe Laufzeit zu spät in Gang gesetzt wurden, umdiese Zeit also vor zu stellen sind.Im Sonnensystem muss das Signal auf dem Hinwegzusätzlich zur Strecke s 600 000 km den WegDs1 T1 c=2 durchlaufen, den das vordere Raum-schiff während der Laufzeit T1 zurücklegt:

sDs1 cT1) s 12 cT1 cT1)

T1 2 s=c 2 600000 km=300000 km=s 4 s.

Auf dem Rückweg braucht das Signal die StreckeDs2 T2 c=2 weniger zu durchlaufen, um die dashintere Raumschiff ihm entgegen kommt:

sDs2 cT2) s 12 cT2 cT2) T2 2 s=3c

2 600000 km=(3 30 000 km=s) 1 13 s.

Die gesamte Laufzeit beträgt also 4 s 1 13 s 5 1

3 s.Für den Beobachter im Sonnensystem müssten dieUhren im vorderen Raumschiff um 4 s vor gestelltwerden, um synchron mit denen im hinteren Raum-schiff zu gehen. Aufgrund des Verfahrens der Be-satzung werden die Uhren nach seinen Berech-nungen im vorderen Raumschiff aber nur um5 1

3 s : 2 2 23 s vorgestellt. Sie gehen also um 1 1

3 snach.

9.2.2 Die Zeitdilatation

1 Berechnen Sie die Geschwindigkeit v, mit der sichin Abb. 349.1 die Uhr C bewegt.

Lösung:Mit DtDtR

1ÿ v2=c2

pfolgt

2 ns 4 ns1ÿ v2=c2

p

) v=c3p

=2 0,866

2 Ein 30-jähriger Weltraumfahrer startet im Jahre1999 zu einer Reise durch das Weltall. Seinedurchschnittliche Reisegeschwindigkeit beträgt re-lativ zur Erde gemessen v 40

41 c. Wie alt ist der

Weltraumfahrer, wenn er im Jahre 2030 zurück-kehrt?

Lösung:Für den Weltraumfahrer vergehen

Dt (2030ÿ 1999a1ÿ (40=41)2

p

31 a 9=41 6,8a

Als Sechsunddreiûigjähriger (30 a 6,8 a) kommter nach 30 Jahren im Jahr 2030 zurück.

Seite

347

Seite

349

9.2 Die Kinematik der speziellen Relativitätstheorie 151

Page 152: Metzler Physik

3 Zwei synchronisierte Uhren A und B haben auf derErde einen Abstand von 600 km. Eine Rakete fliegtmit der Geschwindigkeit v 12

13 c über die Erde hin-weg und kommt erst an Uhr A, dann an Uhr B vor-bei. Bei A zeigt eine Uhr in der Rakete die gleicheZeit wie Uhr A an. Welche Zeit zeigt die Raketen-uhr im Vergleich zur Uhr B an, wenn sie über diesehinwegfliegt?

Lösung:Während des Fluges der Rakete von Uhr A zur UhrB vergeht auf der Erde die ZeitDtR Ds= 12

13 c 13 600 km=(12 300000 km=s) 13

6 ms 2 16 ms.

Währenddessen vergeht in der Rakete nur die Zeit

Dt DtR

1ÿ v2=c2

p 13

6 ms1ÿ ( 12

13 )2q

56 ms.

Die Uhr in der Rakete zeigt daher 2 16 ms ± 5

6ms = 1 1

3 ms weniger an als die Uhr B, wenn sie überdiese hinwegfliegt.

Zusatzaufgaben

4 Im Jahre 1995 startet ein 20-jähriger Astronaut zueiner Weltraumreise. Da seine Rakete mitv (60=61)c fliegt und damit fast Lichtgeschwin-digkeit erreicht, kann er während seiner 33 Jahredauernden Reise auch den 9,7 Lichtjahre entfern-ten Sirius besuchen.a) Welches Jahr schreibt man auf der Erde, wenn

der Astronaut als 53-jähriger zurückkehrt?

b) Wie alt ist der Astronaut, wenn er auf seinemdirekten Flug zu Sirius diesen Stern passiert?

c) Wie schnell hätte der Astronaut fliegen müssen,um während seiner 33 Jahre dauernden Welt-raumreise den 2 Millionen Lichtjahre entfern-ten Andromedanebel zu besuchen?

Lösung:a) Auf der Erde sind 183 Jahre vergangen; man

schreibt das Jahr 2178:

DtR Dt=1ÿ (v=c)2

p

33 a=1ÿ (60=61)2

p 183a

b) In Erdzeit vergehen für die Reise zum SiriusDtR 9,7 Lj : (60c=61) 9,86a; für denAstronauten vergehen jedoch nurDt 9,86a

1ÿ (60=61)2

p 1,78a;

als Einundzwanzigjähriger kommt er am Si-rius vorbei.

c) Aus DtDtR

1ÿ v2=c2

pfolgt mit DtR 33a

und DtR 2e=v (e 2 Mio: Lj 2 106 ca):

vc 1

1

334 106

2s

1ÿ 12

33

4 106

2

1ÿ 34 10ÿ12

Auf 34 Billionstel von c muss sich die Ge-schwindigkeit des Raumschiffs der Lichtge-schwindigkeit nähern.

9.2.3 Myonen im Speicherring

1 Wie groû wäre die Halbwertszeit der Myonen,wenn sie mit derselben Geschwindigkeitv 0,999999 997 c im Ring kreisen würden wieElektronen im Deutschen Elektronensynchrotron(DESY) in Hamburg?

Lösung:Die Halbwertszeit der Myonen wäreDtR 1,52 ms=

1ÿ 0,999 9999972p

19,6ms

2 Angenommen, die Myonen hätten eine Lebenser-wartung von 70 Jahren. Wie lang lebten sie dann imSpeicherring?

Lösung:

70 a=1ÿ 0,99942p

2021 JahreMan stelle sich vor, ein Mensch würde seit demJahre 0 im Speicherring kreisen; er könnte heutenoch leben.

3 Wie schnell muss ein Elektron in einem Beschleu-niger sein, dass dessen Länge für das Elektron aufein Viertel kontrahiert erscheint (! 9.2.4)?

Lösung:Aus lK l

1ÿ v2=c2

pfolgt

v=c 1ÿ (lK=l)2

p

1ÿ ( 1

4 )2q

0,968:

Seite

350

9 Relativitätstheorie152

Page 153: Metzler Physik

9.2.4 Die Längenkontraktion

1 Wie würden Sie die Frage nach Schein oder Wirk-lichkeit der Längenkontraktion beantworten?

Lösung:Die Längenkontraktion ist nicht in dem Sinne real,als dass ein bewegter Körper in seiner Längsrich-tung verformt würde. So ähnlich hatte der nieder-ländische Physiker Hendrik Antoon Lorentz um1900 argumentiert und erklärte damit den negati-ven Ausgang des Michelson-Versuchs: Die Strö-mung des ¾thers durch die Materie sollte die Kräftezwischen den Molekülen vergröûern und dadurchden Körper kontrahieren.Die Relativitätstheorie erklärt dies anders: DieGrundprinzipien haben zur Folge, dass Raum undZeit eine andere Symmetrie besitzen als in der klas-sischen Physik zu Grunde gelegt. Daher habenRaum und Zeit auch andere, unserer täglichenErfahrung scheinbar widersprechende Eigenschaf-ten. Die Längenkontraktion ist eine dieser Eigen-schaften, die wir im täglichen Leben nicht be-obachten können. (Siehe hierzu z. B. R. Sexl undH. K. Schmidt, Raum-Zeit-Relativität, vieweg-stu-dium; 8.2 Schein oder Wirklichkeit?; 8.3 Die Un-sichtbarkeit der Längenkontraktion.)

2 Berechnen Sie die Eigenlänge l und die kontra-hierte Länge lK der Rakete in Abb. 352.1.

Lösung:Die synchronisierten Uhren in dem Raumschiffmessen für das Überfliegen der Uhr C die ZeitDtR 250 ns. Daraus berechnet sich die Eigen-länge zu l 3

5 c 250ns 45m. Mit der Anzeigeder Uhr C Dt 200ns berechnet man die kontra-hierte Länge zu lK 3

5 c 200ns 36m. Dies folgtauch aus der Formel für die Längenkontraktion

lK 45 m1ÿ ( 3

5 )2q

36m.

3 Myonen werden in 20 km Höhe erzeugt und fliegenmit v 0,9998 c auf die Erde zu. Welche Ausdeh-nung hat für die Myonen die Atmosphärenschichtvon 20 km?

Lösung:lK 20 km

1ÿ 0,99982p

340m

4 Warum tritt senkrecht zur Bewegungsrichtungkeine Längenkontraktion auf?

Lösung:Uhren, die senkrecht zur Bewegungsrichtung einesanderen Inertialsystems I0 aufgestellt und in ihrem

Ruhesystem I synchronisiert sind, gehen auch demanderen System I synchron. Daher messen sie keineLängenkontraktion:Auch im bewegten System I0 erreichen die Licht-signale gleichzeitig die Uhren A1 und A2. Hinter-lassen daher die beiden Uhren gleichzeitig Markenin den beiden Bezugssystemen, so wird deren Ab-stand in jedem System gleich gemessen.

C

x x

C

C

C

y ′

υ′ x ′

yyim Ruhesystem und im bewegten System

Synchronisation

Uhr A1

2Uhr A

Uhr A1

2Uhr A

Zusatzaufgaben

5 Ein Raumschiff fliegt mit v 0,6c über eine syn-chronisierte Uhrenkette hinweg. Anfang und Endedes Raumschiffs befinden sich gleichzeitig überzwei Uhren, die einen Abstand von 48 m haben.Wie lang ist das Raumschiff für einen Astronauten,der sich im Raumschiff aufhält? Wie erklärt er seinanderslautendes Ergebnis?

Lösung:Die Eigenlänge der Rakete beträgt lR 60m:

lR 48m=1ÿ 0,62p

60 m.

Für den Astronauten ist die Uhrenkette nicht syn-chronisiert; in Flugrichtung gesehen geht jedenachfolgende Uhr um eine bestimmte Zeitspannegegenüber der davor stehenden Uhr vor! Anfangund Ende des Raumschiffs werden daher für denAstronauten nicht gleichzeitig markiert, sondernder Anfang zu früh, das Ende zu spät. Die Längewird zu klein gemessen (s. umseitige Abb.).

Seite

352

9.2 Die Kinematik der speziellen Relativitätstheorie 153

Page 154: Metzler Physik

l ?

6 In einem Linearbeschleuniger wird ein Elektronauf die Geschwindigkeit v 0,6c beschleunigt.Anschlieûend durchfliegt es mit konstanter Ge-schwindigkeit eine Strecke AB von 9 m Länge.a) Wie lange braucht das Elektron, um diese

Strecke zu durchfliegen?b) Wie lang ist die Strecke im Ruhsystem des

Elektrons?c) Welche Zeit vergeht im Ruhsystem des Elekt-

rons, bis die Strecke durchflogen ist?

Lösung:a) tR s=v 9m=(0,6 3 108 m=s)

5 10ÿ8 s 50ns.b) l lR

1ÿ v2=c2

p 9 m

1ÿ 0,36p

7,2 m.c) t tR

1ÿ v2=c2

p 40ns

odert l=v 7,2m=0,6 c 40ns.

7 Ist es möglich, mit einem 15 m langen Panzer einen10 m breiten Graben mit einer Geschwindigkeitvon v 0,8 c zu überqueren? Aus der Sicht desPanzerfahrers ist der Graben auf 6 m kontrahiert,und die Mitte des Panzers, dort sei der Schwer-punkt, steht noch fest auf der einen Seite, wenndie Vorderkante des Panzers die andere Graben-seite erreicht. Aus der Sicht der Verteidiger istder Panzer auf 9 m kontrahiert. Er schwebt also ei-nen Moment frei in der Luft und müsste in den Gra-ben fallen! Wie löst sich dieser Widerspruch?

Lösung:Dieses Problem wurde von W. Rindler 1961 aufge-worfen (Am. J. Phys. 29, 365 (1961)). Natürlichfällt der Panzer in den Graben, da er aus der Sichtder Verteidiger für einen Moment frei schwebt.Wie erklärt sich dieser Sachverhalt aus der Sichtdes Panzerfahrers? Mit der Lorentz-Transforma-tion zeigt Rindler, dass sich der Panzer in den kon-trahierten Graben biegt. Anschaulich hilft hier einMinkowski-Diagramm. In der Abbildung sind das

Ruhesystem I des Panzers und das relativ dazu nachlinks bewegte System I0 des Grabens gezeichnet.Wir betrachten das Ereignis E (t 0; die Panzer-spitze fährt über den Grabenrand) und nehmen an,die zwischenmolekularen Kräfte im Panzer über-tragen sich mit Lichtgeschwindigkeit. Nach 16 2

3Sekunden hat erst ein Drittel des Panzers Kenntnisvon dem Ereignis E und nur dieses Drittel kanndemnach Kräfte ausüben und den über dem Grabenbefindlichen Teil halten. Das Diagramm zeigt, dasssich bereits 80% dieses Drittels über dem Grabenbefindet. Der Panzer kann demnach nicht starr blei-ben, sondern biegt sich von Anfang an (parabelför-mig) in den Graben.

r

x

10

20

30

40

50

16 23 Lichtsignal

Panzer86 10 12 14 16

Graben

x in m

t in ns

8 Zeigen Sie, dass sich in Abb. 349.1 kein Wider-spruch ergibt, wenn man die Uhr C als ruhendund die Uhren A und B als mit der Geschwindigkeitv nach links bewegt ansieht. Berücksichtigen Siedabei, dass der Abstand der Uhren A und B nunlängenkontrahiert ist und die Uhren A und B nunnicht mehr synchron gehen.

Lösung:Eine sorgfältige Diskussion dieser nicht einfachenAufgabe kann den Abschluss des Themenkreises¹Relative Gleichzeitigkeit, Zeitdilatation und Lo-rentz-Kontraktionª bilden. Geht es doch hier umden von Kritikern erhobenen Vorwurf, dass dieSymmetrie der Zeitdilatation bezüglich zweierInertialsysteme bereits einen inneren Widerspruchdarstellt.In Abb. 349.1 geht die bewegte Uhr nur halb soschnell wie die ruhenden Uhren, woraus sich dieRelativgeschwindigkeit v=c

3p

=2 0,866 er-gibt. Der Abstand der beiden Uhren A und B be-trägt demnach in deren Ruhsystem d 4 ns v;diese Form ist aussagekräftiger als der daraus be-rechnete Wert von 104 cm.

9 Relativitätstheorie154

Page 155: Metzler Physik

Betrachten wir nun die Uhr C als ruhend, so ist derAbstand der bewegten Uhren A und B Lorentz-kontrahiert: dK 2ns v. Die beiden Uhren A undB gehen nicht mehr synchron; welche Zeitdifferenzsie anzeigen, ergibt sich aus der Betrachtung der inihrem Ruhesystem erfolgten Einstein-Synchroni-sation:

tA B

c

Laufzeiten

υ

1 ns · υ

t

c

1 ns · υ

υ

Die Uhr B wird vor der Uhr A in Gang gesetzt;seien tA und tB die Laufzeiten der Synchronisa-tionsblitze (gemessen von der Uhr C), so ergibtdie Rechnung:

Ausc tA 1ns v v tA folgt (cÿ vA) tA 1ns v,und ausc tB 1ns vÿ v tB folgt (cÿ vB) tB 1ns v,sodass sich ergibt

tAÿ tB 1 ns v

1cÿ v

ÿ 1c v

1 ns2

c2=v2ÿ 1 6ns.

Diese von der Uhr C gemessene Zeitdifferenz von6 ns wird jedoch nicht von den bewegten Uhren Aund B angezeigt; sie gehen um den Faktor 1

2 lang-samer, sodass B nur um 3 ns gegenüber A vorgeht.Kommt die Uhr A an der Uhr C vorbei, so zeigt sie100 ns an; Uhr B zeigt demnach (gleichzeitig für C)103 ns. Bis die Uhr B an C vorbeikommt, vergehen2 ns; für die langsamer gehende Uhr B vergeht je-doch nur 1 ns, sodass sie dann 104 ns wie inAbb. 349.1 anzeigt.

9.2.5 Die Zeitdilatation

1 Lesen Sie für das Ereignis ¹Das vordere Ende desBusses befindet sich in der Mitte der Straûenbahnªin Abb. 353.1 die Koordinaten in allen drei Bezugs-systemen ab.

Lösung:Das Ereignis hat die KoordinatenE (t 2 s | x 30 m) E (t0 2 s | x0 20m) E (t00 2 s|x00 40 m).

2 Bestimmen Sie in Abb. 353.1 im Ruhesystem desBusses die Geschwindigkeit der Bahn und im Ruhe-system der Bahn die des Busses.

Lösung:Der Bus hat in I00 die Geschwindigkeitv00 (50mÿ 20m)=3 s 10 m=s.Der Bahn hat in I0 die Geschwindigkeitv0 (0mÿ 30m)=3sÿ10 m=s.

3 Welche Bewegung stellt eine gekrümmte Weltliniedar, deren Steigung zunehmend geringer wird?

Lösung:Im Minkowski-Diagramm stellt dies eine positivbeschleunigte Bewegung dar.

4 Was lässt sich über eine Weltlinie sagen, die dieForm einer nach unten geöffneten Parabel hat?

Lösung:Ein solcher Körper wäre gleichzeitig (z. B. in denSchnittpunkten mit der x-Achse an zwei verschie-denen Orten. Das ist nicht möglich.

Zusatzaufgaben

5 Ein Fluss hat eine Strömungsgeschwindigkeit von50 m/min. Zeichnen Sie ein rechtwinkliges Koor-dinatensystem, das das Inertialsystem ¹Uferª dar-stellt (0 x 400m; 0 t 3min). ZeichnenSie ein schiefwinkliges Koordinatensystem hinein,das das Inertialsystem ¹Flussª darstellt. Lösen Siedamit die folgenden Aufgaben graphisch: Ein Mo-torbootfahrer startet am Bootshaus H (250 m;0 min) zu einer Fahrt stromaufwärts. Sein Bootfährt relativ zum Wasser mit einer Geschwindig-keit von 200 m/min. Nach einer Minute bemerkt er,dass ihm eine halbvolle Whiskyflasche über Bordgefallen ist. Er kehrt um und holt die Flasche 100 munterhalb des Bootshauses ein. Allerdings war ihmkurz nach dem Wenden der Motor für eine halbeMinute ausgefallen, und am Bootshaus musste er38 Sekunden wegen einer Fähre anhalten. Wannund wo fiel die Flasche über Bord? Zeichnen Siein beiden Inertialsystemen ein Netz von Parallelenzu den Achsen und konstruieren Sie die Weltlinienvon Boot und Flasche.

Seite

353

9.2 Die Kinematik der speziellen Relativitätstheorie 155

Page 156: Metzler Physik

Lösung:Aus dem Diagramm liest man ab, dass die Flasche15 Sekunden nach dem Start, etwa 37 m oberhalbdes Bootshauses über Bord gefallen war.

100

t in minUfer

3

2

1

00 200 H 300 400

3

2

1

t in min′Fluß

BootFlasche

x in m

′x in m

6 Tragen Sie in ein rechtwinkliges Koordinatensy-stem die Punkte A (3,5; 1,8), B (5,5; ±5,5) undC (±5; 0) ein. Zeichnen Sie in das erste Koordina-tensystem ein zweites schiefwinkliges System,

dessen Achsen einen Winkel von 120 einschlie-ûen. Beide Koordinatensysteme sollen dabei die-selbe Winkelhalbierende haben. Geben Sie die Ko-ordinaten der Punkte im stumpfwinkligen Systeman, wenn dessen Einheitsstrecken 1,5 mal gröûerals im rechtwinkligen System sind.

Lösung:Im schiefwinkligen Koordinatensystem liest manfolgende Koordinaten ab:A (3; 2), B (3; ±3), C (±3; 7; ±1).

C

y

x

A

B

y

x

–6 –4 –2

–6

–4

–2

–4

–2

–3

–1

–4–2

–3–1

12 3

4

2 4 61

2

3

4

2

4

6

9.2.6 Minkowski-Diagramme

1 Zeichnen Sie in einem Minkowski-Diagramm einrechtwinkliges Koordinatensystem I und einSystem I0, das sich relativ zu I mit v 0,8 c bewegt.Zeigen Sie, dass aus der Sicht eines jeden Systemsdie Uhren im anderen System langsamer gehen.Zeigen Sie, dass auch die Lorentz-Kontraktion indiesem Sinne symmetrisch ist.

Lösung:Wir sehen zunächst das System I als ruhend an, wasbedeutet, dass die Uhren in diesem System syn-chron gehen und Gleichzeitigkeit parallel zur x-Achse abgelesen wird. Dann vergeht z. B. vomZeitnullpunkt an in diesem System die ZeitspanneDtR 5 s, während dessen in I0 nur die ZeitDt0 5 s

1ÿ 0,82p

5s 0,6 3 s vergeht. Sei I0

das ruhende System, so vergehen während D t0R 3 s in I nur Dt 3 s

1ÿ 0,82p

3s 0,6 1,8 s.Zwei Körper, die in I bzw. I0 ruhend dort die Eigen-länge l 1 Ls haben, haben im jeweils anderenrelativ mit v 0,8 c bewegten System die kontra-hierte Länge lK 0,6 Ls.

t in s

2l = 0,6 Ls′ I = 1 Ls x in LsR

0 1 3 4 5

1

2

3

4

5

1

2

3

1

2

3

′t = 5s

t in Ls

R ∆t = 3s

∆t = 3s

R

∆t = 1,8s

x in Ls′

I = 1 Ls

R

I = 0,6 Ls

0

2 Zwei Raumschiffe fliegen in entgegengesetzterRichtung mit den Geschwindigkeiten v1 0,6 cund v2 0,3 c an der Erde vorbei. Zeichnen Sieein Minkowski-Diagramm, in dem das Erdsystem(rechtwinklig) und die Ruhesysteme der beiden

Seite

355

9 Relativitätstheorie156

Page 157: Metzler Physik

Raumschiffe eingetragen sind. Lesen Sie ab, umwelchen Faktor die Uhren des einen Raumschiffslangsamer im Vergleich zu den Uhren des anderenRaumschiffs gehen. Zeigen Sie, dass der Effektsymmetrisch ist. Zeigen Sie, dass die Raumschiffeum den gleichen Faktor längenkontrahiert sind.

Lösung:Das erste Raumschiff (I0) bewegt sich mit 0,6 cnach rechts, das zweite (I00) mit 0,3 c nach links.Vergeht im System I00 die Zeit D t00R 10 s, so liestman in I0 dafür die Zeit Dt0 6,5 s. Daraus ergibtsich der Faktor

1ÿ v2=c2

p 0,65 für die Zeit-

dilatation und die Längenkontraktion.(Diesen Faktor wird man mit dem Additionstheo-rem für Geschwindigkeiten ! 9.2.8 berechnenkönnen: Für die Relativgeschwindigkeit zwischenI00 und I0 ergibt sich und darausv=c (0,6 0,3)=(1 0,6 0;3) 0,763und daraus

1ÿ v2=c2

p

1ÿ 0,7632p

0,647.)

Ein Körper mit der Eigenlänge l 2 Ls wird imjeweils anderen System zu lK 1,3 Ls verkürzt ge-messen.

x in Ls

x in Ls

x in Ls′

′′

∆tR′′ = 10s

∆ t R′= 6s

R

= 6,5s∆t ′

= 3,9s

∆ t ′′

′t ′′ tin s in s

t in s

2 4

6

8

10

2

4 6

8

24

68

2

4

6 8

2

4

6

8

8

6

l

l = 1,3 Ls

= 2 Ls

′′l = 1,3 Ls ′′Rl = 2 Ls

Zusatzaufgaben

3 Um 8.00 Uhr passiert ein Raumschiff mit v 0,8 cdie Erde. Dabei wird auch die Raumschiffsuhr auf8.00 Uhr gestellt. Um 9.30 Uhr fliegt das Raum-schiff an einer Raumstation vorbei, die konstantenAbstand zur Erde hat. Die Uhren in der Station zei-gen Erdzeit an.a) Wie viel Uhr ist es während des Vorbeiflugs im

Raumschiff?

b) Welche Entfernung hat die Raumstation vonder Erde für einen Beobachter auf der Erde?

Zeichnen Sie ein Minkowski-Diagramm.

Lösung:a) Im Raumschiff vergeht die Zeitspanne Dt0

90 min1ÿ 0,82p

54min; beim VorbeifluganderStation ist esalso8.54Uhr imRaumschiff.

b) Die Station hat von der Erde die EntfernungDx 90min 0,8c 72 Lmin:

t

x

x = 72 Lmin

x in Lmin

Erde

Raumschiff

StationErde Raumschifft

0 30 60 90 120

Uhr9.30

9.00

8.30

8.00

Uhr

9.00

8.30

4 Ein Raumschiff der Eigenlänge l 100 m fliegt mitv 0,6 c an einer interplanetaren Station vorbei.Als die Spitze des Raumschiffs einen Sendemastder Raumstation passiert, wird ein Radiosignal aus-gesandt.a) Nach welcher Zeit erreicht das Signal das Heck

des Raumschiffs?b) Nach welcher Zeit passiert das Heck des Raum-

schiffs den Sendemast?Geben Sie die Zeiten jeweils in Raumschiffzeit undStationszeit an. Lösen Sie rechnerisch und zeich-nerisch mit einem Minkowski-Diagramm.

Lösung:Die Spitze des Raumschiffs soll zur Zeit t t0 0den Sendemast passieren, dessen Weltlinie durchdie t-Achse im System I dargestellt sei.a) Das Signal erreicht im Ruhesystem I0 des

Raumschiffs das Heck nach 333 ns:Dt0 100 m : c 333 ns.Im relativ dazu bewegten System der Raumsta-tion bewegen sich das Ende des auf 80 m längen-kontrahierten Raumschiffs und das Signal auf-einander zu; sie treffen sich nach Dt 167 ns:vDt c Dt lK)Dt lK=(v c) l

1ÿ v2=c2=(v c )

p

80m=(0,6 c c) 167ns:

9.2 Die Kinematik der speziellen Relativitätstheorie 157

Page 158: Metzler Physik

b) Im System I der Raumstation kommt das Heckdes Raumschiffs nach 444 ns am Sendemastvorbei: Dt lK=0,6 c 444 ns. Im Raumschiffvergehen Dt0 l=0,6 c 556 ns, bis der Sende-mast am hinteren Ende vorbeifliegt.

30 60 90 120 150

x in m–30–60–90–150 3060

90120

x in m′

t in ns′t in ns

500

400

300

200

100

600

500 400

300

200

100

Sendemast

Radiosignal

t ′a

tb

t ′b

ta

–100

–200

–300–200

–300

l = 80 m

l = 100 m

RRaum-schiff

–120–90

–60

9.2.7 Die Lorentz-Transformation

1 Für Geschwindigkeiten im Alltag ist v c, sodassman Terme mit v=c vernachlässigen kann. ZeigenSie, dass damit die Lorentz-Transformation in dieGalilei-Transformation der (x, t)-Diagramme inAbb. 353.1 übergeht.

Lösung:Für v c wird k 1=

1ÿ v2=c2

p 1. Vernach-

lässigt man Terme mit v=c2, so erhält man:t t0 und x x0 v t0 bzw. x0 x ÿ v t, also dieklassische Galilei-Transformation.Aus x0 xÿ v t folgt z. B. mit x0 0 die Gleichungder roten t0-Achse im x-t-System in Abb. 353.1zu t x=v.

2 Leiten Sie mit den Lorentz-Transformationsglei-chungen die Formel für die Zeitdilatation her. Le-gen Sie zwei Ereignisse für die Ablesung einer in Ibewegten, in I0 ruhenden Uhr fest. Beachten Sie:

Die Ablesung erfolgt in I0 am gleichen Ort(x01 x02), in I an verschiedenen Orten.

Lösung:Eine Uhr bewegt sich im Inertialsystem I mit derGeschwindigkeit v. Zu den Zeiten t1 und t2 befindetsich die Uhr an den Orten x1 bzw. x2. In ihremRuhesystem I0 werden diese beiden Ereignissedurch die Koordinaten (x01=t01) und (x02=t02) beschrie-ben, wobei (x01 x02). Für die Zeitspanne t02ÿ t01 imInertialsystem I ergibt sich:t2ÿ t1 k(t02 (v=c2) x02)ÿ k(t01 (v=c2) x01)

kt02ÿ t01).

Bezeichnen wir die Zeitspanne im System I wieüblich mit DtR t2ÿ t1 und die Eigenzeit mitDt t02ÿ t01, so ergibt sich mit k 1=

1ÿ v2=c2

p

die bekannte Formel für die ZeitdilatationDtDtR

1ÿ v2=c2

p.

9.2.8 Die Addition der Geschwindigkeiten

1 Zeigen Sie, dass Licht, das von einem mit 0,99 cfliegenden Ion ausgesandt wird, sowohl in als auchentgegen der Flugrichtung Lichtgeschwindigkeithat.

Lösung:

Aus u u0 v

1 u0vc2

folgt u c 0,99 c

1 c 0,99cc2

c:

2 Ein radioaktiver Kern fliegt mit v 0,8 c und sen-det in seinem Ruhesystem Elektronen mit einerGeschwindigkeit von 0,6 c aus. Welche Geschwin-digkeiten haben die Elektronen im Laborsystem inder Flugrichtung des Kerns und entgegen der Flug-richtung?

Lösung:Werden die Elektronen in der Flugrichtung desKerns ausgesandt, so fliegen sie mit u1 0,946 c

Seite

356

Seite

357

9 Relativitätstheorie158

Page 159: Metzler Physik

dem Kern voraus. Werden sie entgegen der Flug-richtung ausgesandt, so fliegen sie mit u2 0,385 cdem Kern hinterher:

u1 0,6c 0,8c

1 0,6c 0,8 cc2

0,946c;

u2 ÿ0,6c 0,8 c

1ÿ0,6c 0,8cc2

0,385c;

Zusatzaufgaben

3 Ein Flugzeug fliegt mit einer Geschwindigkeit von1000 km=h und feuert in Vorausrichtung ein Ge-schoss mit ebenfalls 1000 km=h ab. Wie genaumüsste eine Geschwindigkeitsmessung des Ge-schosses sein, wollte man den relativistischenEffekt nachweisen?

Lösung:Mit v 1000 km=h 278 m=s folgt für die rela-tivistische Geschwindigkeit u des Geschosses:

Aus u 2v

1

vc

2 2v

1ÿ

vc

2folgt durch

Umformen der Näherung

2vÿ u2v

vc

2

2783 108

2

8,6 10ÿ13

Die relative Geschwindigkeitsänderung beträgt8,6 10ÿ13; mit dieser Genauigkeit ist eine Ge-schwindigkeitsmessung nicht möglich!

4 Ein Raumschiff fliegt mit 0,2 c durch die Milch-straûe, als es von einer gegnerischen Rakete über-holt wird, die mit 0,8 c die Galaxie durchquert. So-fort löst der Kommandant des Raumschiffs ein0,7 c-Geschoss aus, das die Rakete kurze Zeit spä-ter einholt und zerstört. ± Prüfen Sie im Inertial-system ¹Galaxieª und im System ¹Raumschiffª,ob diese Geschichte wahr sein kann.

Lösung:Das Geschoss holt die Rakete nicht ein. Im galak-tischen System hat das Geschoss die Geschwindig-keit v (0,2c 0,7c)=(1 0,2 0,7)

0,79c< 0,8c;im Ruhesystem des Raumschiffs hat die Rakete dieGeschwindigkeit u (0,8cÿ 0,2c)=(1ÿ 0,8 0,2)

0,71c> 0,7c:

9.2.9 Der optische Doppler-Effekt

1 Eine Wasserstofflinie im Spektrum des Spiralne-bels Hydra hat eine Wellenlänge von 475 nm, wäh-rend man im Labor die Linie mit der Wellenlänge394 nm misst. Wie groû ist die Fluchtgeschwindig-keit des Spiralnebels?

Lösung:Der Spiralnebel Hydra entfernt sich mit 18,5 % derLichtgeschwindigkeit:

vc (lE=ls)

2ÿ 1

(lE=ls)2 1

(475=394)2ÿ 1

(475=394)2 1 0,185.

2 Ein mit der Geschwindigkeit v 0,6 c fliegendesK-Meson zerfällt in zwei p-Mesonen. Im Ruhesy-stem des K-Mesons haben die p-Mesonen eine Ge-schwindigkeit von 0,85 c: Welche Geschwindig-keit haben die p-Mesonen maximal und minimalim Laborsystem?

Lösung:In der Flugrichtung des K-Mesons haben diePi-Mesonen die gröûte Geschwindigkeit mitv (0,6c 0,85c)=(1 0,6 0,85) 0,96c;die entgegen der Flugrichtung ausgesandten Pi-Mesonen haben die Geschwindigkeitu (0,6cÿ 0,85c)=(1ÿ 0,6 0,85)ÿ0,51c;sie fliegen nach hinten mit 0,51 c.

3 Der nächste Fixstern ist Alpha-Centauri am südli-chen Sternenhimmel. Seine Entfernung beträgt 4,5Lichtjahre.a) Wie lange braucht ein Raumschiff mit der Ge-

schwindigkeit v 0,5 c, um zu dem Stern zugelangen?

b) Wie lange dauert der Flug für die Astronauten?c) Welche Geschwindigkeit müsste das Raum-

schiff haben, wenn für die Besatzung nur einJahr verginge?

Seite

359

9.2 Die Kinematik der speziellen Relativitätstheorie 159

Page 160: Metzler Physik

Lösung:a) Das Raumschiff bräuchte 9 Jahre: DtErde

4,5a c=0,5c 9a.b) Für die Astronauten vergingen nur 7,8 Jahre,

was sich allerdings erst bei deren Rückkehr zei-gen würde:DtAstr:DtErde

1ÿ v2=c2

p 9a

1ÿ 0,52p

7,8 ac) Aus DtAstr: DtErde

1ÿ v2=c2

p

folgt 1 a (4,5a c=v)1ÿ (v=c)2

p

und daraus v=c 0,9762.Für die Erdbewohner würde der Flug 4,61 adauern.

4 Werden Uhren bewegt, so gehen sie langsamer.Man könnte daher annehmen, dass der Uhrentrans-port zur Synchronisation entfernter Uhren eine un-geeignete Methode sei. Tatsächlich werden aberAtomuhren zur Synchronisation entfernter Uhrentransportiert. Zeigen Sie, dass die dabei auftretendeZeitabweichung kleiner als jeder vorgegebeneWert gemacht werden kann, wenn der Uhrentrans-port nur genügend langsam erfolgt. Ersetzen Sie inder Rechnung den Wurzelterm durch die Nähe-rung 1 x1=2 1 1

2 x. Mit welcher Geschwin-digkeit darf demnach höchstens eine Uhr vonTokio nach Paris gebracht werden (Entfernungca. 12000 km), wenn der auftretende Zeitfehlerkleiner als 10ÿ8 s sein soll?

Lösung:Der Zeitfehler soll kleiner als 10ÿ8 Sekunden sein!Die Uhr darf dann höchstens mit einer Geschwin-digkeit von 540 km=h transportiert werden:Aus t tR(1ÿ v2=2c2) folgt tRÿ t tR v2=(2c2)

(s=v) v2=2c2 < 10ÿ8 s;damit v[ (2c2=s) 10ÿ8 s

2(3 108 m=s)2=(1,2 107 m) 10ÿ8 s 150m=s 540km=h.

5 Astronauten benötigen für die Hin- und Rückreisezum Mond vier Tage. Berechnen Sie die mittlereGeschwindigkeit und die dadurch auftretende Zeit-dilatation. Könnte man die Zeitdilatation mitAtomuhren messen? Die Ganggenauigkeit neues-ter Atomuhren beträgt Dt=t 10ÿ14; die kleinstemessbare Zeitspanne ist 1 ns.

Lösung:Die mittlere Geschwindigkeit beträgtv 2 384400km=4d 2225m=s.Für die Zeiten tRakete und tErde gilt

tRakete tErde

1ÿ v2=c2

p, oder

tRakete tErde(1ÿ 12 (v=c)2) tErdeÿ 1

2 (v=c)2 tErde.

Die relative Zeitabweichung berechnet sich also zu

(tErdeÿ tRakete)=tErde 12 (v=c)2

12 (2,2km=s=300000km=s)2

2,75 10ÿ11;

dieser Wert ist 2750 mal gröûer als Dt=t 10ÿ14.Die absolute Zeitabweichung beträgt

tErdeÿ tRakete 12 (v=c)2tErde

2,75 10ÿ11 4 d 9,5s

und ist damit 9500-mal gröûer als die kleinste, vonder Atomuhr meûbare Zeitspanne von 1 ns.

6 Ein Raumschiff nähert sich mit v 0,6 c der Erde.Während einer Fernsehübertragung vergeht imRaumschiff eine Stunde. Wie lange dauert die Sen-dung auf der Erde?

Lösung:Auf der Erde dauert die Übertragung eine halbeStunde:Bei Annäherung gilt

TE TS

1ÿ v=c1 v=c

s 1 h

1ÿ 0,61 0,6

r 1

2 h.

∆ ′t = h12

t in h

c

c

t in h′

′x

x

′t = 1

h1

2

Funksignale

Raumschiff

Empfänger

1

2

7 Ein Raumschiff startet am Neujahrstag des Jahres2000 und fliegt mit der Geschwindigkeit v 0,8 czu dem etwa 4 Lichtjahre entfernten Stern Alpha-Centauri. Nach einem Aufenthalt von 2 Jahrenkehrt das Raumschiff mit der Geschwindigkeitv 0,6 c zur Erde zurück. Jeweils zur Jahreswendewerden Neujahrsgrüûe per Funk ausgetauscht.Zeichnen Sie in einem Minkowski-Diagramm denVerlauf der Weltraumreise. Tragen Sie die Funk-sprüche zu Neujahr in das Diagramm ein. In wel-

9 Relativitätstheorie160

Page 161: Metzler Physik

chem Jahr kehrt das Raumschiff zur Erde zurück?Wie viele Botschaften werden ausgetauscht? Deu-ten Sie das Ergebnis mithilfe des Doppler-Effekts.

Lösung:Das Raumschiff kehrt nach 13 2

3 Jahren im Jahre2013 zur Erde zurück. Für die Astronauten sindnur 10 1

3 Jahre vergangen.Für die Neujahrsgrüûe per Funk ergeben sich mitder Formel für den Doppler-Effekt die folgendenZeitabstände TE, die für die Empfänger zwischenzwei aufeinanderfolgenden Funksprüchen verge-hen:

TE 1a

1 v=c1ÿ v=c

s

v=c 0,8 0 ÿ 0,6

TE 3 a 1 a 12 a

Die beiden Minkowski-Diagramme, in die links dieFunksprüche von der Erde zum Raumschiff undrechts vom Raumschiff zur Erde eingezeichnetsind, bestätigen dieses Ergebnis.

2010

t in a

t in a′

′′t in a

2005

2000

1413121110

9876543210

Erdeα-Centauri

43210 x in Lj

109

8

76

543

2

1

t in a

t in a′

′′t in a

1413121110

9876543210

43210 x in Lj

109

8

76

543

2

1Hinreise

Rückreise

Weltlinie derWeltraumreise

Zusatzaufgabe

8 Zeichnen Sie den Graphen für die Rotverschiebungder Lichtwellenlänge v=c ! lE=ls. Bis zu wel-chen Werten v=c darf man die klassische Formelfür den Doppler-Effekt lE ls(1 v=c) verwen-den? Tragen Sie diese Funktion ebenfalls in dasDiagramm ein.

Lösung:Mit der Näherungsformel lässt sich bis 30% derLichtgeschwindigkeit rechnen; dann ist der Fehlerkleiner als 5%.

0,2

λE/ Sλ

0

2

4

6

8

10

0,4 0,6 0,8 1,0 υ /c0

relativistisch

Näherung

9.2 Die Kinematik der speziellen Relativitätstheorie 161

Page 162: Metzler Physik

9.3.1 Die relativistische Massenzunahme

1 Wie groû ist der prozentuale Fehler, wenn man beieiner Geschwindigkeit von 0,1 c die relativistischeMassenzunahme nicht berücksichtigt?

Lösung:Der prozentuale Fehler berechnet sich zu

(mÿm0)=m0 m=m0ÿ 1

1=1ÿ v2=c2

pÿ 1 0,005 0,5%

für v=c 0,1.

Weitere Werte:

v=c 0,2 0,4 0,6 0,8 0,9 0,99

Fehler 2,1% 9,1% 25% 67% 129% 609%

2 Berechnen Sie die Massenzunahme eines Satellitenm0 1000 kg, der auf seiner Erdumlaufbahneine Geschwindigkeit von 28 000 km/h hat.

Lösung:Die Massenzunahme beträgt nur 0,34 Milligramm:Da auch hier noch v c ist, darf man die Näherung1=

1ÿ v2=c2

p 1 v2=2c2 benutzen.

Aus mm0=1ÿ v2=c2

pm0(1 1

2 (v=c)2) folgtDm mÿm0 1

2 (v=c)2 m0

12 (7,8km=s=300000km=s)2 1000 kg 3,4 10ÿ7 kg.

3 Im deutschen Elektronensynchrotron DESY beiHamburg werden Elektronen auf eine Geschwin-digkeit von v 0,999999997 c beschleunigt. Umwelchen Faktor ist die dynamische Masse danngröûer als die Ruhemasse?

Lösung:Die dynamische Masse der Elektronen ist nahezu13 000-mal gröûer als ihre Ruhemasse:Aus: v=c 1ÿ 3 10ÿ9 folgt (v=c)2 1ÿ 6 10ÿ9;also m=m0 1=

6 10ÿ9p

12910:

4 Auf welche Geschwindigkeit muss ein Elementar-teilchen beschleunigt werden, damit sich seineMasse verdoppelt, verzehnfacht, verhundertfacht?

Lösung:Aus m=m0 1=

1ÿ v2=c2

pfolgt

v=c1ÿ (m0=m)2

p:

m=m0 2 10 100

v=c 0,866 0,995 0,999 95

Zusatzaufgaben

5 In einer Röhre werden Elektronen mit einer Span-nung U 20 kV beschleunigt. Wie groû ist der pro-zentuale Fehler bei der Berechnung der Endge-schwindigkeit, wenn man die relativistische Mas-senzunahme nicht berücksichtigt?

Lösung:Mit Ekin e U 20 keV berechnet man klassischaus Ekin 1

2 mv2

v2Ekin=m

p 8,39 107 m=s 0,280c

relativistisch aus der Formel für die kinetischeEnergie

vc

1ÿ 1

1 Ekin

m0 c2

2

vuuut

1ÿ 1

1 20keV

511keV

2

vuuut 0,272.

Der Fehler beträgt 2,6 %.

Weitere Werte:

U in kV 2,5 10 40 100 1000

v=c klassisch 0,099 0,198 0,396 0,626 1,978

relativistisch 0,099 0,195 0,374 0,548 0,941

6 Ein Elektron hat eine Geschwindigkeit von99,997% der Lichtgeschwindigkeit. Wie groû istsein Impuls? Geben Sie den Impuls in kgm=s undin GeV=c an.

Lösung:pm0 v=(

1ÿ v2=c2

p) 3,53 10ÿ20 kg m=s:

Multipliziert man mit

(e=1,60 10ÿ19 As) (3 108 m=s)=c 1;

so erhält man p 66,0MeV=c 0,066GeV=c.

7 Stöût ein Proton, dessen Geschwindigkeit kleinerals ein Zehntel der Lichtgeschwindigkeit ist, gegenein ruhendes Proton, so fliegen die beiden Protonenunter einem Winkel von 90 auseinander. ZeigenSie dies mit den klassischen Erhaltungssätzen fürden Impuls und für die kinetische Energie. Ist dasstoûende Proton relativistisch, so ist der Winkelkleiner als 90. Erklären Sie dies qualitativ.

Seite

361

9 Relativitätstheorie162

Page 163: Metzler Physik

Lösung:Für die Impulse p!1 des stoûenden Protons vor demStoû und p!01 und p!02 der Protonen nach dem Stoûführt das Vektordreieck der Impulserhaltung klas-sisch nur dann auf den Erhaltungssatz der kineti-schen Energie P2

1=(2m p021 =(2m) p022 =(2m),wenn der Winkel a, den die Impulse p!01 und p!02einschlieûen, ein Rechter ist.

αp

1 2

klassisch: relativistisch:

′ p′

p

α2

p′p1′

p

Werden die Geschwindigkeiten relativistisch, sowächst der Impuls p!1 stärker an als die Impulsep!01 und p!02, weil zu ihm die gröûere Geschwindig-keit und damit auch die gröûere Massenzunahmegehört. Das Vektordreieck wird stumpfwinklig undder Winkel a wird kleiner als 90.

8 1963 wurde an der Universität Zürich ein Präzi-sionsexperiment zur relativistischen Massenzu-nahme durchgeführt. Ziel des Experimentes wares, die Formel

mm0=1ÿ v2=c2

p

mit einer Messgenauigkeit von 0,5 Promille zu tes-ten. Die Messungen zeigten, dass im Rahmen die-ser Genauigkeit die experimentellen Ergebnissemit den berechneten Werten übereinstimmen.Der Versuchsaufbau ist in der Abb. skizziert. Miteiner Spannung von maximal 5,5 Millionen Voltwerden Elektronen beschleunigt. Dabei erreichtdie dynamische Masse etwa den sechsfachen Wertder Ruhmasse. Die Elektronen durchfliegen einhomogenes Magnetfeld, das so eingestellt ist, dassdie Elektronen, auf einem genau vorgegebenenSollkreis fliegend, um 180 abgelenkt werden. An-schlieûend gelangen die Elektronen in einen zylin-dersymmetrischen Plattenkondensator, dessenelektrische Feldstärke ebenfalls so eingestellt wird,dass die Elektronen, auf einem Sollkreis fliegend,um 90 abgelenkt werden. Die genaue Einstellungwird mit einem Detektor kontrolliert, in den dieElektronen gelangen.

homogenesMagnetfeld

R = 50 cm

R = 1

00 c

m

e

m

Elektronen-bahn

Detektor

Elektronenquelle

zylindersymmetrischeselektrisches Feld

+

_

Beschleunigungsspannung U = 5,5 · 10 V6

Aus einer genauen Messung der magnetischenFeldstärke B, der elektrischen Feldstärke E undder beiden Kreisradien Rm und Re können die dy-namische Masse m und die Geschwindigkeit v be-stimmt werden, wie folgende Rechnung zeigt:Bei der Bewegung auf einer Kreisbahn (Radius R)wird die ablenkende Zentripetalkraft

Fz m v2=Rim magnetischen Fall durch die Lorentzkraft

Fm e v Bund im elektrischen Fall durch die Coulombkraft

Fe e Eaufgebracht (e ist die Elementarladung). Aus derGleichsetzung der Kräfte

Fz Fm (gemessener Radius Rm)und Fz Fm (gemessener Radius Re)folgen die beiden Gleichungen

m v2=Rm e v Bund m v2=Re e E.Die Division beider Gleichungen liefert

Re=Rm vB=E.Daraus folgt für die Geschwindigkeit

v E Re=(B Rm).Setzt man diese Gleichung in eine der beiden obe-ren Gleichungen ein, so erhält man für die dynami-sche Masse die Beziehung

m e B2 R2m=(E Re).

a) Erklären Sie, warum die Elektronen in beidenFeldern auf Kreisbahnen fliegen.

9.3 Die Dynamik der speziellen Relativitätstheorie 163

Page 164: Metzler Physik

b) Der Versuch wurde so durchgeführt, dass zu-nächst das Magnetfeld auf genau B 20 mTeingestellt wurde. Dann wurde die Beschleuni-gungsspannung so eingeregelt, dass die Elekt-ronen auf dem Sollkreis durch das Magnetfeldflogen, und anschlieûend wurde das elektrischeFeld so eingestellt, dass die Elektronen auch aufihrem Sollkreis im elektrischen Feld flogen;das elektrische Feld hatte dann den Wert E 2,95529 106 v=m. Berechnen Sie die dynami-sche Masse der Elektronen und ihre Geschwin-digkeit. Rechnen Sie mit den genauen Wertenfür die Naturkonstanten.

c) Zeigen Sie mit einer Formel, dass nur Elektro-nen mit einem ganz bestimmten Impuls auf dervorgegebenen Kreisbahn durch das auf B 20 mT eingestellte Magnetfeld fliegen können.Berechnen Sie diesen Impuls.

d) Berechnen Sie mit der dynamischen Masse dieGesamtenergie und mit (c) die Ruheenergie derElektronen. Berechnen Sie die Ruhemasse.

e) Zeigen Sie mit den Ergebnissen aus (b) und (d),dass die Formel für die relativistische Massen-zunahme erfüllt ist.

f) Berechnen Sie die kinetische Energie und dar-aus die Beschleunigungsspannung.

Lösung:a) Wirkt in jedem Moment eine dem Betrage nach

konstante Kraft senkrecht zur Bewegungsrich-tung auf das Elektron, so bewegt es sich aufeiner Kreisbahn. Eine solche Kraft ist im homo-genen magnetischen Feld durch die Lorentz-kraft und im zylindersymmetrischen elektri-schen Feld durch die Coulomb-Kraft gegeben,wenn das Elektron jeweils senkrecht zum Feldeingeschossen wird.

b) Mit der hergeleiteten Formel berechnet man ausden Versuchsdaten die folgende dynamischeMasse:

m B2 R2m

E Ree

(20 10ÿ3T)2 (0,5m)2

2,95529 106 V=m 1,0m 1,602177 10ÿ19 As

5,421386 10ÿ30 kg.

Nimmt man den Wert für die Ruhemasse desElektrons, den allerdings erst die weitere Ver-suchsauswertung liefern soll, vorweg, so be-rechnet man m 5,95 m 0 und daraus v0,985 c.

c) Aus der Gleichsetzung der Lorentzkraft mit derZentripetalkraft ergibt sichevBm v2=Rm und daraus pmv eRm B;

p eRm B 1,602177 10ÿ19 As 0,5m 20 10ÿ3 T 1,602177 10ÿ21 kgm=s 2,997925 MeV=c.

d) Aus der dynamischen Masse m bzw. der Ge-samtenergie E und dem Impuls p lässt sichmit der Impuls-Energie die Ruheenergie be-rechnen:

E0 E2ÿ (cp)2

p

(5,421386 10ÿ30 kg c2)2

pÿ (2,997925MeV)2

(3,041174 MeV)2ÿ (2,997925 MeV)2

p

oder E0 0,511066 MeV.Dieser Wert weicht um weniger als 0,5 Promillevom Literaturwert für die Ruheenergie desElektrons ab.Damit ist die Formel für die relativistische Mas-senzunahme bestätigt, denn dieses Gesetz ist inder soeben benutzten Impuls-Energie enthal-ten:

e) Aus E2ÿ (cp)2 E20 folgt mit Emc2,

E0 m0 c2 und pmv(mc2)2ÿ (cmv)2 (m0 c2)2; dividiertdurch c4 ergibtm2 (1ÿ v2=c2)m2

0; aufgelöst nach m er-hält man die Formel für die relativistischeMassenzunahme.

f) Ekin EÿE0 3,0412MeVÿ 0,5111MeV 2,530MeV eU:

Die Beschleunigungsspannung betrug U 2,530 106 V. (Der Versuch wurde bei verschie-denen Spannungen bis maximal 5,5 Mio. Voltdurchgeführt.)

9 Relativitätstheorie164

Page 165: Metzler Physik

9.3.2 Die Trägheit der Energie

1 Wie groû ist die Ruheenergie eines Elektrons? Aufwelche Geschwindigkeit muss man das Elektronbeschleunigen, um seine Energie zu verdoppeln?

Lösung:Em0 c2 9,109 10ÿ31 kg (2,9979 108 m=s)2

8,187 10ÿ14 J 511 keV 0,511 MeV.

In Aufgabe 361/4 hatten wir gezeigt, dass man einElementarteilchen auf v 0,866 c beschleunigenmuss, damit sich seine Masse verdoppelt; das glei-che gilt für die Energie.

2 Wie groû ist die dynamische Masse der Elektronen,wenn sie im Beschleuniger die Energie 20,5 GeVerhalten?

Lösung:Die Elektronen haben mehr als die 40 000fache Ru-hemasse:m=m0 (E0Ekin)=(m0 c2) 1Ekin=(m0 c2)

1 20,5 GeV=511keV 40118:

3 Um wie viel schwerer wird 1 kg Eis, wenn esschmilzt? KannmandieseMassenzunahmemessenQS 333,5J=g?Lösung:Die Schmelzwärme von Eis beträgt 335 kJ=kg. Da-mit nimmt 1 kg Eis beim Schmelzen um folgendeMasse zu:m 335kJ=(3 108 m=s)2 3,7 10ÿ9g 3,7ng.3,7 Nanogramm lassen sich nicht messen.

4 Zeigen Sie, dass für kleine Geschwindigkeiten dierelativistische Formel für die kinetische Energie indie klassische Formel übergeht.

Lösung:Ekin m0 c2 (1=

1ÿ v2=c2

pÿ 1)

m0 c2 (1 12 (v=c)2ÿ 1) 1

2 m0 v2:

5 Im deutschen Elektronensynchrotron DESY wer-den Elektronen auf eine Energie von 7,5 GeV be-schleunigt.a) Wie schnell sind dann die Elektronen?b) Wie lang ist der Beschleunigungstunnel für dieElektronen? (Der ringförmige Beschleuniger hateinen Durchmesser von 100 m.)

Lösung:a) Aus Em0 c2=

1ÿ v2=c2

p E0Ekin folgt

v2=c2 1ÿ 1=(1Ekin=E0)2

1ÿ (E0=Ekin)2, da Ekin

E0.

v=c 1ÿ 12 (E0=Ekin)2

1ÿ 12 (511keV=7,5GeV)2

0,999999998,

wobei die Wurzel wegen E0 Ekin näherungs-weise berechnet werden kann.

b) l l0

1ÿ v2=c2

p l0

1ÿ (1ÿ (E0=Ekin)2)

p

l0 E0=Ekin 100m p 511keV=7,5GeV 21mm;für die Elektronen ist der Beschleuniger nur21 mm statt 314 m lang.

6 Zwei Teilchen gleicher Ruhemasse m0 und glei-cher kinetischer Energie Ekin 2m0 c2 stoûen zent-ral zusammen und bilden ein neues Teilchen. Wiegroû ist die Ruhemasse M0 des neuen Teilchens?

Lösung:Das neu gebildete Teilchen hat die RuhemasseM0 6m0, da die gesamte kinetische Energie inRuheenergie umgesetzt wird.

Seite

363

9.3 Die Dynamik der speziellen Relativitätstheorie 165

Page 166: Metzler Physik

9.3.3 Die Raum-Zeit und die Impuls-Energie

1 Drei Ereignisse haben im Inertialsystem I dieKoordinaten E1 (1 Lsj2 s), E2 (6 Lsj3 s) undE3 (5 Lsj6 s).a) Transformieren Sie die Koordinaten der drei

Ereignisse in das System IM, das sich mitv 0,6 c bewegt.

b) Bilden Sie in beiden Inertialsystemen die räum-lichen Abstände Dx, die zeitlichen Abstände Dtund die raum-zeitlichen Abstände sRZ der dreiEreignisse.

Lösung:

t in s

0

1

2

3

4

5 6

7

01 2

3

4

5

6

7

x in Ls

t in s′

′x in Ls

3

5

1

2

4

3

5

1

2

4

E1

2

3

E

E

a) Mit xM (xÿ 0,6c t)=0,8 undtM (tÿ 0,6 x=c)=0,8 berechnet man

x in Ls t in s xM in Ls tM in s

E1 1 2 ÿ 0,25 1,75

E2 6 3 5,25 ÿ 0,75

E3 5 6 1,75 3,75

b) Damit berechnet man die räumlichen AbständeDx und DxM, die zeitlichen Abstände D t undD tM und die raum-zeitlichen Abstände D sRZ

und D s 0RZ:

Dtin Ls

Dtin s

DxMin Ls

DtMin s

DsRZ

in LsDs 0RZ

in Ls

E1 E2 5 1 5,5 ÿ 2,5 4,90ÿ1p

4,90ÿ1p

E2 E3 ÿ 1 3 ÿ 3,5 4,5 2,83 2,83

E3 E1 ÿ 4 ÿ 4 ÿ 2 ÿ 2 0 0

Die beiden Ereignisse E1 und E2 können keineWirkung aufeinander ausüben; ihr raum-zeitli-cher Abstand ist in jedem Inertialsystem gleichjedoch imaginär. Die beiden Ereignisse E2 und

E3 haben in jedem System einen raum-zeitli-chen Abstand von 2,83 Ls; E2 findet vor E3 stattund kann dieses bewirken. Die beiden Ereig-nisse E1 und E3 haben den raum-zeitlichenAbstand null, sie können durch die Weltlinieeines Lichtsignals miteinander verbundenwerden.

2 Ein Teilchen hat eine Energie von 5600 MeV undeinen Impuls von 5520,6 MeV=c. Kann es sich beidem Teilchen um ein Neutron handeln?

Lösung:Das Teilchen hat die Ruheenergie des Neutrons:

E0 E2ÿ (pc)2

p

(5600MeV)2ÿ (5520,6MeV)2

p

940MeV.

*3 Im Superprotonensynchrotron des europäischenKernforschungszentrums CERN werden Protonenauf eine maximale kinetische Energie von 400 GeVbeschleunigt. Bei dem Stoû eines beschleunigtenProtons (p) gegen ein ruhendes Proton (p) sollein neues Teilchen X erzeugt werden:p p! p pX. Wie groû kann maximal dieRuheenergie M0 des neuen Teilchens X sein?

Lösung:Die Ruheenergie des Protons sei E0 , die Energiedes stoûenden Protons E1, dessen Impuls p1 unddie Ruheenergie des erzeugten Teilchens sei EX 0.Das erzeugte Teilchen hat dann die gröûte Ruhe-masse, wenn die drei Teilchen nach dem Stoû nichtauseinanderfliegen. Die Impuls-Energie lässt sichdann vor dem Stoû im Laborsystem und nach demStoû im Schwerpunktsystem aufschreiben undgleichsetzen:(E1E0)2ÿ (cp1)2 (2E0EX 0)2:

Mit E1 E0 Ekin, wobei Ekin 400GeV ist, und(cp1)2 E2

1 ÿE20 E2

kin 2Ekin E0 folgt aus obigerGleichung

EX0 2E0(1Ekin=2E0

pÿ 1)

2E0(1 400 GeV=2 938 MeV

pÿ 1)

27,3E0 25,6GeV.

Nur 25,6 GeV können maximal in Ruheenergieumgesetzt werden, der Rest (400 ÿ 25,6) GeV trittals kinetische Energie auf.

Seite

365

9 Relativitätstheorie166

Page 167: Metzler Physik

9.3.4 Eine Anwendung der Impuls-Energie

1 Wie groû ist der Impuls eines Elektrons, dessenEnergie zehnmal so groû wie seine Ruheenergie ist?

Lösung:Aus E2ÿ (pc)2 E2

0 folgt

pE2ÿE2

0

p=c

(10E0)2ÿE2

0

q=c

99p

E0=c.

2 Ein Proton hat einen Impuls von 6 GeV/c. Wie groûist seine Geschwindigkeit?

Lösung:Aus p (m0=

1ÿ v2=c2

p) v folgt

v=c 1=1 (m0 c2=pc)2

p:

Mit der Ruhemasse des Protons von 938 MeV =m0 c2 erhält manv=c 1=

1 (938MeV=6GeV)2

p 0,988.

3 Ein Elementarteilchen hat bei einer Geschwindig-keit von 2,996 ´ 108 m/s eine Masse von 28,1155atomaren Masseeinheiten. Um welches Teilchenhandelt es sich?

Lösung:Die Ruhemasse berechnet sich zu

m0 m1ÿ v2=c2

p

28,1155u1ÿ (2,996=2,997925)2

p

1,0074u.

Das ist die Ruhemasse des Protons.

Zusatzaufgaben

4 Ein Photon der Energie E 2m0 c2 trifft auf einruhendes Teilchen der Ruhmasse m0 und wirdvon ihm absorbiert.Wie groû ist die Geschwindigkeit des Teilchensnachher?

Lösung:Durch die Absorption des Photons entsteht einneues ¹angeregtesª Teilchen, das eine gröûereRuhenergie E00 im Vergleich zur Ruhenergie E0

des Teilchens zuvor hat:

E00 E02ÿ (cp0)2

p.

Für die Energie E0 und den Impuls p0 des angereg-ten Teilchens gilt

E0 EPhE0 3m0 c2; p0 pPh 2m0 c.

Damit wird

E00 (3m0 c2)2ÿ (2m0 c2)2

p

5p

m0 c2.

Für die Geschwindigkeit erhält man:

v=c1ÿ (m00=m0)2

q

1ÿ (

5p

=3)2q

23 .

5 Ein K-Meson zerfällt in zwei Pi-Mesonen. Ein Pi-Meson ist nach dem Zerfall in Ruhe, d. h., das an-dere Pi-Meson übernimmt den gesamten Impulsdes K-Mesons. Berechnen Sie die Energie des K-Mesons und des davonfliegenden Pi-Mesons (Ru-hemasse des K-Mesons 494 MeV; Ruhemassedes Pi-Mesons 137 MeV).

Lösung:Mit EK EK0 EKkin und Ep Ep 0 Ep kin folgtaus dem Energieerhaltungssatz die Gleichung

EK Ep 0 Ep

EKkin EK0 2Ep0 Ep kin

Ep kin ÿEK kin EK0 ÿ 2Ep0 220 MeV (1)

Aus der Gleichheit der Impulse pk pp folgt mitder Impuls-Energie

(cpK)2 (cpp)2

E2KÿE2

K0 E2

pÿE2p 0

2EK0 EK kin E2K kin 2Ep0 Epkin E2

p kin(2)

Aus den beiden Gleichungen (1) und (2) erhält mandie Werte für die kinetische Energie des K-MesonsEKkin 396 MeV und für das Pi-MesonE p kin 616 MeV.

Seite

367

9.3 Die Dynamik der speziellen Relativitätstheorie 167

Page 168: Metzler Physik

10.1.1 Der lichtelektrische Effekt

1 Die Abbildung S. 373 unten zeigt eine mit UV-Strahlung (links) und sichtbarem Licht (rechts)gleicher Intensität beleuchtete Metallplatte, ähn-lich Versuch 2.Welche Ergebnisse wären zu erwarten, wenn dieEnergie des Lichts gleichmäûig an die Elektronender Zinkplatte verteilt würde?

Lösung:Wenn die Energie gleichmäûig an die Elektronender Zinkplatte verteilt wird, erhalten alle Elektro-nen die gleiche Energie. Bei gleicher Intensität giltdies für sichtbares Licht wie für UV-Licht. Nacheiner längeren Zeitspanne müssten alle Elektronengleichzeitig aus der Zinkplatte ausgelöst werdenund einen elektrischen Strom bewirken.

10.1.2 Das Planck'sche Wirkungsquantum

1 Welchen Energiebetrag können Radiostrahlung(l 200 m), Infrarotstrahlung (l 10ÿ6 m) undRöntgenstrahlung (l 10ÿ9 m) an ein Elektronübertragen?

Lösung:Licht einer Radiowelle (l 200 m) überträgtE 9,93 10ÿ28 J, von Infrarotstrahlung (l10ÿ6 m) E 1,99 ´ 10ÿ19 J und von Röntgenstrah-lung (l 10ÿ9 m) E 1,99 ´ 10ÿ16 J.

2 Die Spannung am Kondensator in Abb. 374.1 än-dert sich schrittweise mit jedem Fotoelektron. Wiehängt der zeitliche Verlauf der Kondensatorspan-nung während der Aufladung von der Intensität,wie von der Frequenz des Lichts ab?

Lösung:Mit der Intensität des Lichts erhöht sich die Zahlder Photoelektronen pro Zeiteinheit, also der Auf-ladestrom. Bei einem Kondensator ist die Span-nungsänderung pro Zeit proportional zum Strom.Also ist die Spannungsänderung pro Zeit am Kon-densator ein Maû für die Intensität.Bei gleicher Intensität und groûer Frequenz werdenweniger Photoelektronen ausgelöst als bei kleinerFrequenz. Bei blauem Licht wird der Kondensatoralso langsamer aufgeladen als bei rotem Licht glei-cher Intensität. Der Ladevorgang erfolgt beiblauem Licht (hohe Frequenz) bis zu höherenSpannungen, da die Photoelektronen im Vergleichzur Beleuchtung mit rotem Licht eine höhere kine-tische Energie besitzen.

Zusatzaufgaben

3 Natriumatome emittieren oder absorbieren Strah-lung der Wellenlänge l 5,9 10ÿ7 m. BerechnenSie die Energie der Photonen.

Lösung:EPh h f 2,1 eV.

4 Berechnen Sie Frequenz und Wellenlänge vonPhotonen, die von folgenden Systemen absorbiertwerden:a) Atomkerne absorbieren Energie der Gröûen-

ordnung 103 eV.b) Ein Atom absorbiert etwa 1 eV.c) Ein Molekül absorbiert ca. 10ÿ2 eV.

Lösung:a) f 2,4 ´ 1017 Hz, l 1,24 ´ 10ÿ9 m,b) f 2,4 ´ 1014 Hz, l 1,24 ´ 10ÿ6 m,c) f 2,4 ´ 1012 Hz, l 1,24 ´ 10ÿ4 m,

5 Welche Höhe muû ein Körper der Massem 10ÿ8 kg (Sandkorn) durchfallen (ohne Be-rücksichtigung des Luftwiderstandes), um eine ki-netische Energie zu erhalten, die gleich der Energieeines Photons der Frequenz f 5 1014 Hz (blauesLicht) ist?

Lösung:Für die Höhe H gilt mit mgH h f , H 3,4 10ÿ12 m.(Der Durchmesser eines Atoms beträgt ungefähr10ÿ10 m.)

Seite

373

Seite

375

10 Einführung in die Quantenphysik168

Page 169: Metzler Physik

10.1.4 Umkehrung des lichtelektrischen Effekts mit Leuchtdioden

1 Eine Diode weist eine Durchlassspannung vonU0 0,65 V auf. Welche Wellenlänge hat dasvon der Diode abgestrahlte Licht?

Lösung:Bei einer Einsatzspannung von 0,65 V strahlt dieDiode mit der Wellenlänge l 1,91 mm.

2 Die in Abb. 377.1 gezeigte untere Leuchtdiode für950 nm leuchtet trotz der Stromstärke 10 mA nicht.

Welche Einsatzspannung ist bei dieser Wellen-länge zu erwarten? Warum ist kein Leuchten fest-zustellen?

Lösung:Die Diode für 950 nm hat eine Einsatzspannungvon 1,31 V. Sie sendet Infrarotstrahlung aus, diefür das Auge nicht sichtbar ist.

10.1.5 Die kurzwellige Grenze der Röntgenstrahlung

1 a) In einer Röntgenröhre durchlaufen Elektroneneine Potentialdifferenz UA 40 kV. BerechnenSie die kurzwellige Grenze lmin der Röntgen-strahlung.

b) Zeigen Sie, dass zwischen der kurzwelligenGrenze lmin und der BeschleunigungsspannungUA die Beziehung lmin 1240 V=UA nm gilt.

Lösung:a) Die kurzwellige Grenze der Röntgenstrahlung

beträgt lmin 31,0 pm.b) Setzt man in e U h f für die Frequenz f c=l

ein und löst nach l auf, erhält man l 1239,84V=UA nm.

2 Die vom Zählrohr registrierten Impulse unterstüt-zen die Sichtweise, dass Licht in Form einzelnerPhotonen absorbiert wird. Wie würde sich dasMessergebnis von Versuch 1 ändern, wenn sichder Wert von h halbieren lieûe?

Lösung:Wenn der Wert von h halbiert wird, ändert sich ent-sprechend E h f bei gleicher Frequenz die Ener-gie entsprechend. Bei gleicher Intensität wird danndie gleiche Gesamtenergie übertragen, wenn sichdie Zahl der Photonen verdoppelt. Das Zählrohrzeigt dann die doppelte Zählrate.

3 Mithilfe der Bragg-Reflexion an einem Lithium-fluoridkristall mit der Gitterkonstanten d 201 pmwerden bei verschiedenen fest eingestelltenReflexionswinkeln die Beschleunigungsspannun-gen gemessen, bei denen eine Emission von Rönt-genstrahlung einsetzt:

q 8 9 10 11 12 13

UA in kV 19,8 17,6 15,8 14,6 13,5 12,5

a) Berechnen Sie zunächst mit der Bragg-Glei-chung die Wellenlängen, und stellen Sie danndie Spannung in Abhängigkeit von der Fre-quenz grafisch dar.

b) Bestimmen Sie aus der grafischen Darstellungden Wert von h, dem Planck'schen Wirkungs-quantum.

Lösung:a) Die Wellenlängen betragen 55,9 pm, 62,9 pm,

69,8 pm, 76,7 pm, 83,6 pm und 90,4 pm. DieFrequenzen berechnen sich mit f c=l zu5,36 ´ 1018 Hz; 4,77 ´ 1018 Hz; 4,29 ´ 1018 Hz;3,91 ´ 1018 Hz; 3,59 ´ 1018 Hz; 3,32 ´ 1018 Hz.

17,5

U in kV

15,0

12,5

7,5

5,0

2,5

10,0

20,0

181·10 2·10 3·10 4·10 5·1018 18 18 18

f in Hz

b) Die Steigung des Graphen ist h 6 ´ 10ÿ34 Js.

Zusatzaufgaben

4 Welche Spannung muss wenigstens an eine Rönt-genröhre gelegt werden, um eine Strahlung zu er-zeugen, deren Quanten gleich der Ruhmassenener-gie von Elektronen sind?

Seite

377

Seite

379

10.1 Grundlagen 169

Page 170: Metzler Physik

Lösung:Eruh m0e c2, h f eUa, Ua 511 kV 0,511 MV

5 Warum ist die Existenz einer kurzwelligen Grenzebei der Röntgenbremsstrahlung nicht mit der Wel-lenvorstellung vom Licht vereinbar?

Lösung:Die Röntgenbremsstrahlung besitzt ein kontinuier-liches Spektrum, das bei einer bestimmten klein-sten Wellenlänge in Abhängigkeit von der Anoden-spannung endet. Die Strahlung entsteht beim Ab-bremsen der Elektronen. Wenn also ein beschleu-nigter Körper Energie durch Strahlung verliert,sollte die Wellenlänge der Strahlung keiner Ein-schränkung unterliegen.

10.1.6 Der Compton-Effekt

1 Der Schweif eines Kometen ist stets von der Sonneweg gerichtet. Erklären Sie dieses Verhalten durchdie Wechselwirkung von Photonen mit Materie.

Lösung:Photonen und der Sonnenwind (Protonen) über-tragen Impuls auf die aus dem Kometen strömendeMaterie. Der auf die Elektronen übertragene Im-puls hat die Richtung der Lichtausbreitung.

2 Welche Energie wurde bei einem Compton-Pro-zess an die Elektronen abgegeben, wenn die Fre-quenz der gestreuten Strahlung f 0,990 ´ 1019 Hzund die der ursprünglichen Strahlung f1,000 ´ 1019 Hz beträgt?

Lösung:E h ( f ÿ f M) 414 eV

3 Die Frequenz der einfallenden Strahlung beträgtbei einem Compton-Prozess f 1,2 ´ 1020 Hz. Wiegroû ist dann die Frequenz der gestreuten Strah-lung, wenn die Geschwindigkeit der Elektronennach dem Stoû v 1,5 ´ 108 m=s beträgt?

Lösung:Die abgegebene Energie ist gleich der kinetischenEnergie der Elektronen, die jedoch relativistisch zuberechnen ist.

DE Ekin EgesÿEruh

m0e c2=1ÿ (v=c)2

pÿm0e c2 0,15 m0e c2

7,9 ´ 104 eV,

f M (h f ÿDE)=h 1,01 ´ 1020 Hz.

4 Wie groû ist die maximale Wellenlängenänderungbeim Compton-Prozess? Unter welchem Winkeltritt die Strahlung mit der gröûten Wellenlängen-änderung auf?Warum bemerkt man beim sichtbaren Licht keinenEffekt, der dem Compton-Effekt entspricht?

Lösung:Gröûte Wellenlängenänderung bei ò 180. Siebeträgt l 4,8 pm. Die Masse von Photonen dessichtbaren Lichts ist verglichen mit der Masse vonElektronen zu klein, um eine Übergabe von Ener-gie an die Elektronen im elastischen Stoû zu er-möglichen.

5 g-Quanten radioaktiver Präparate rufen ebenso denCompton-Effekt hervor wie Röntgenquanten.a) Welche Energie geben Photonen von

E 1,92 ´ 10ÿ13 J 1,2 MeV an Elektronenab, wenn sie um 180 zurückgestreut werden?

b) Welche Geschwindigkeit besitzen die gestoûe-nen Elektronen?

Lösung:a) Für den Compton-Effekt gilt:

1f Mÿ 1

f h

m0e c2(1ÿ cos ò),

und für ò 180 ergibt sich:

1f Mÿ 1

f 2h

m0e c2.

Umformungen liefern:

1h f M 1

h f 2

m0e c2,

h f M h f m0e c2

m0e c2 2h f h f

1

1 2h fm0e c2

,

EM 1,2 MeV1

1 2,40,511

0,21 MeV

An die Elektronen wird eine Energie vonDE 0,99 MeV abgegeben.

Seite

381

10 Einführung in die Quantenphysik170

Page 171: Metzler Physik

b) Die Gesamtenergie der Elektronen beträgt dannEges EkinEruh

Eges 0,99 MeV 0,51 MeV.

Aus Eges Eruh=1ÿ (v=c)2

pfolgt

v c1ÿ (Eruh=Eges2

q, also v 0,94 c.

Zusatzaufgaben

6 Die Wirkungsweise von Szintillationszählernberuht auf der Erzeugung von Lichtquanten imsichtbaren Bereich durch schnelle Elektronen inKristallen. Bestrahlt man den Kristall eines Szintil-lationszählers mit monochromatischer g-Strah-lung, so erhält man ein Spektrum der Art inAbb. 485.2.Wie ist dieses Spektrum zu erklären?

Lösung:Die g-Strahlung erzeugt im Photoeffekt Elektro-nen, deren Energie ungefähr gleich der Energieder g-Quanten ist, oder sie gibt ihre Energie inmehreren aufeinanderfolgenden Comptonprozes-sen und einem abschlieûenden Photoeffekt anElektronen ab. Die höchste bei einem Compton-Ef-fekt auf ein Elektron übertragene Energie ergibtsich bei einer Streuung des g-Quants um 180.Also erzeugen monoenergetische g-Quanten Elekt-ronen gleicher Energie und Elektronen geringererEnergie bis zu einem Höchstwert, der kleiner ist alsdie Energie der g-Quanten. Alle diese Elektronengeben ihre Energie im Kristall ab und erzeugen da-bei Photonen im sichtbaren Bereich. Dabei ist dieAnzahl der erzeugten Photonen der Energie desElektrons proportional. Die Photonen erzeugen in

der Zählerelektronik einen ihrer Anzahl proportio-nalen Spannungsimpuls.Das Spektrum der Abb. 485.2 entsteht also durchdie Registrierung vieler g-Quanten, die einen Pho-toeffekt hervorgerufen haben. Ferner werden Er-eignisse registriert, die auf dem Compton-Effektberuhen.

7 Ein Körper der Masse m und der Geschwindigkeit vstoûe elastisch mit einem ruhenden Körper derMasse x.Bestimmen Sie unter Verwendung der Beziehun-gen für den elastischen Stoû zweier Körper, fürwelche Masse x die abgegebene Energie ein Maxi-mum hat.

Lösung:Nach den Formeln für den zentralen elastischenStoû zweier Körper ergibt sich für den gestoûenenKörper die Geschwindigkeitvx 2mv=(m x).

Die übertragene Energie ist am gröûten, wenn diekinetische Energie des gestoûenen Körpers amgröûten ist. Suche also das Maximum für die kine-tische Energie:Es ist

Ekin;x 12 xv2

x 12 x 4m2 v2=(m x)2

12 mv2 4 m x=(m x)2

Ekin 4 mx=(m x)2,

und abgeleitetE0kin;x Ekin 4m (mÿ x)=(m x)3,

alsoxe m.

Die übertragene Energie ist am gröûten, wenn dergestoûene Körper gleiche Masse hat.

10.1 Grundlagen 171

Page 172: Metzler Physik

10.2.1 Die Photonenverteilung hinter dem Doppelspalt

1 Die kleinste Lichtintensität, die das menschlicheAuge noch wahrnehmen kann, liegt bei10ÿ10 W=m2. Wie viele Photonen (l 560 nm)treten bei dieser Intensität pro Sekunde in einePupille der Fläche A 0,5 cm2 ein?

Lösung:Energie eines Photons Eph 3,55 ´ 10ÿ19 J 2,2 eV.Zahl der pro Sekunde durch die Pupille tretendenPhotonen n 14 077.

2 Ein Laser habe eine Strahlungsleistung von 1 mWbei 632,8 nm und einen Strahlquerschnitt von4 mm2.a) Wie groû ist die Anzahl der Photonen, die pro

Sekunde auf 1 mm2 treffen?b) Vergleichen Sie die Intensität des Laserlichtes

mit der des Sonnenlichts (I 1,36 kW=m2).

Lösung:a) Für die Intensität gilt

I E=(DADt) n EPh=(DADt) und I P=DA.

Mit DA1 4 mm2 und DA2 1 mm2 ergibt sichn PDA2 D t=(DA1 EPh) 7,95 ´ 1014

b) IL 250 W=m2, Is 5,44 IL.

3 Warum kann man aus der Art und Weise, in der einFilm geschwärzt ist, schlieûen, dass die Energiedes Lichtes ungleichmäûig über die Wellenfrontverteilt ist?

Lösung:Wäre die Energie des Lichtes gleichmäûig übereine Wellenfront verteilt, so dürften im belichtetenFilm nicht unveränderte Kristalle unmittelbar ne-ben veränderten liegen.

Zusatzaufgabe

4 Monochromatisches Licht der Wellenlänge 550 nmund der Intensität 1 kW=m2 fällt auf eine Metall-schicht und löst Elektronen aus.a) Wie groû ist die Anzahl der Photonen, die pro

Sekunde auf einen cm2 treffen?b) Welche Energie wird durch die ausgelösten

Elektronen pro Sekunde und cm2 abgeführt,wenn die Austrittsenergie E 2 eV beträgt,und 15% der auftreffenden Photonen einenPhotoeffekt bewirken?

Lösung:a) Für die Intensität gilt mit E nEPh

I nEPh=(DADt):

Daraus ergibt sich mit EPh h f und c l fn IDADt l=(hc) die Anzahl der Photonenn 2,82 ´ 1017.

b) Anzahl der ausgelösten Elektronenne 4,22 ´ 1016, gesamte AustrittsenergieEges 8,45 ´ 1016 eV 0,0135 J, gesamte Ener-gie der Photonen, die einen Photoeffekt bewir-ken, Eges ne h f 0,015 J, Energie der Elekt-ronen Ee 1,49 ´ 10ÿ3 J.

10.2.2 Photonenverteilung bei geringer Intensität

1 a) Welche Aussage kann man auf Grund derKenntnis der Intensitätsverteilung hinter einemDoppelspalt über einzelne Photonen machen?

b) Welche weitere Aussage lässt sich über sehrviele Photonen machen?

Lösung:a) Die Wahrscheinlichkeit, ein einzelnes Photon

in einem Teilvolumen einer Registriereinrich-tung nachzuweisen, ist proportional zur Inten-sität der Strahlung.

b) Die Energieverteilung, die sich bei der Regis-trierung sehr vieler Photonen ergibt, zeigt denVerlauf der Intensitätsverteilung der Strahlung.

Seite

383

Seite

384

10 Einführung in die Quantenphysik172

Page 173: Metzler Physik

10.3.1 De-Broglie-Wellen

1 Berechnen Sie die De-Broglie-Wellenlänge vonElektronen, die verschiedene Beschleunigungs-spannungen durchlaufen haben:U1 l V; U2 l03 V; U3 106 V.Geben Sie dazu in Tabellenform kinetische Ener-gie, Geschwindigkeit, Impuls und De-Broglie-Wellenlänge an.

Lösung:Eges EruhEkin, Eruh 0,511 MeV

Für relativistische Teilchen gilt

pE2

gesÿE2ruh

q=c und v c

1ÿ (Eruh=Eges)

2q

,

für nichtrelativistische gilt

p2m0e Ekin

qund v p=m

U in V 1 103 106

Ekin in eV 1 103 106

Eges in MeV 0,511 0,512 1,511

p in MeV=c 1,01 ´ 10ÿ3 3,20 ´ 10ÿ2 1,42

v in c 0,002 0,062 0,941

l in pm 1230 38,8 0,875

2 Berechnen Sie für Lichtquanten und Elektronender Wellenlänge l 0,1 nm den Impuls und diekinetische Energie.

Lösung:Photonen:p 6,6 ´ 1024 m kg=s, Ekin 1,24 ´ 104 eV.Elektronen:p 6,6 ´ 1024 m kg=s, Ekin 150 eV.

3 a) Angenommen, der Wert der Planck'schen Kon-stanten h ändert sich um den Faktor 10 (um 1

10 )Wie würden sich in dem Fall die Versuchser-gebnisse von Versuch 1 verändern?

b) Mit welchem angenommenen Wert von h wä-ren keine Interferenzerscheinungen zu beob-achten?

Lösung:Wegen l h=p ist die De-Broglie-Wellenlänge derElektronen bei konstantem Impuls p proportionalzu h. Wird h um den Faktor 10 vergröûert, verstärktsich die Ablenkung zum ersten Maximum. Mit ei-nem kleineren Anstand L ergibt sich ein gleichwer-tiges Interferenzbild. Bei einem um den Faktor 1

10geänderten Wert von h ist es schwierig, das breiteHauptmaximum vom ersten Nebenmaximum zutrennen.

10.3.2 Welleneigenschaften von Elektronen

1 Zum Versuch von Jönsson: Elektronen werdenmit einer Spannung von UA 54,7 kV beschleu-nigt, bewegen sich durch einen sehr feinen Doppel-spalt mit dem Spaltabstand d 2 mm und werdenim Abstand von e 40 cm registriert. BerechnenSie die Wellenlänge l der Elektronen und den Ab-stand a der Interferenzmaxima auf dem Bild-schirm.

Lösung:Die De-Brogie-Wellenlänge l 5,24 pm (klas-sisch, also nicht relativistisch gerechnet) führt zuMaxima in Abstand a 1,05 mm. Die Angabenim Aufgabentext beziehen sich auch einen verein-fachten Versuch. Jönsson benutze in seinem Expe-riment elektrische Linsen zur Fokussierung.

2 Der Versuch von Jönsson (Aufgabe 1) soll mitProtonen statt mit Elektronen bei sonst gleichenAufbauwerten durchgeführt werden. BerechnenSie die Wellenlänge und den Abstand der Interfe-renzmaxima.

Lösung:Die De-Brogie-Wellenlänge l 0,122 pm bei Pro-tonen führt zu Maxima in Abstand 24,5 nm.

3 Berechnen Sie die De-Broglie-Wellenlänge einerMetallkugel der Masse 1 g, die sich mit v 1 m=sbewegt. Warum ist es praktisch unmöglich, in die-sem Fall Interferenzen nachzuweisen?

Lösung:Die De-Brogie-Wellenlänge l 6,6 ´ 10ÿ31 m ist zuklein, als dass an Beugungsobjekten selbst mitatomaren Abständen messbare Abstände von Inter-ferenzen zu erzeugen sind.

Seite

387

Seite

388

10.3 Ausbreitung von Elektronen 173

Page 174: Metzler Physik

10.4.1 Das Unschärfeprinzip

1 Bei einem Farbmonitor werden Elektronen mitU 25 kV beschleunigt, treten durch eine Strei-fenmaske, deren Spaltöffnungen 0,25 mm betra-gen, und erreichen nach einem Weg von 1 cmden Leuchtschirm.a) Vergleichen Sie die De-Broglie-Wellenlänge

der Elektronen mit der Spaltbreite.b) Wie groû ist die Breite des mittleren Streifens

des Beugungsbildes auf dem Leuchtschirm auf-grund der Unschärferelation?

Lösung:a) Mit p

2me eUp

ist l h=p 7,76 pm. DerWert ist sehr klein im Vergleich zur Spaltbreitevon 0,25 mm.

b) Mit Dx Dp 12 und Dx 125 mm ist Dp

2,65 10ÿ30 m kg=s. Für die Breite des Haupt-maximums d in y 10 mm Abstand ergibt sichmit px=p d=y und p 8,54 10ÿ23 m kg=s derWert d 310 pm. Die Welleneigenschaften derElektronen lassen sich in diesem Fall vernach-lässigen.

10.4.2 Messung der Unschärfe bei Photonen

1 Das Licht von der Sonne besitzt eine Kohärenz-länge von 2 Dx 600 nm bei einer mittleren Wel-lenlänge von 500 nm.a) Wie groû ist die Frequenzunschärfe Df ?b) Welche Wellenlängen begrenzen das Spekt-

rum?

Lösung:a) Die Frequenzunschärfe beträgt mit Dx Dp 1

2und Df c=h Dp 5 ´ 1014 Hz.

b) Das Sonnenspektrum wird begrenzt durch272 nm und 3000 nm.

2 Die spektrale Breite der Linie eines He-Ne-Lasersbeträgt 2 Df 100 Hz. Welche Kohärenzlänge istzu erwarten?

Lösung:Die Kohärenzlänge beträgt 2 Dx 6 ´ 106 m.

3 Für eine Na-Dampf-Lampe wird eine Kohärenz-länge von 0,3 m gemessen. Wie lange dauert derEmissionsvorgang eines Photons? Wie groû ist dieEnergieunschärfe?

Lösung:Die Dauer der Emission ist 2 Dt 1 ns. Die Ener-gieunschärfe beträgt DE 6,6 ´ 10ÿ25 J 4 meV.

Seite

393

Seite

395

10 Einführung in die Quantenphysik174

Page 175: Metzler Physik

11.1.1 Die quantenhafte Absorption

1 Angeregte Quecksilberatome senden UV-Strah-lung der Wellenlänge l 253,6 nm aus.a) Vergleichen sie den Wert der Wellenlänge mit

der gemessenen Anregungsenergie.b) Wie ändert sich die Intensität der UV-Strah-

lung, wenn die Beschleunigungsspannung UB

von 0 V auf 30 V erhöht wird?

Lösung:a) Mit DE hc=l erhält man 7,83 ´ 10ÿ19 J

4,9 eV. Der Wert entspricht genau der gemes-senen Energiedifferenz von 4,9 eV.

b) Die Intensität der UV-Strahlung vergröûert sichstufenweise mit jedem Vielfachen der Anre-gungsenergie bzw. Vielfachen von 4,9 V derBeschleunigungsspannung.

2 a) Welche Geschwindigkeit hat ein Elektron nacheiner Beschleunigung mit 10 V in der Franck-Hertz-Röhre? Warum sind Fallunterscheidun-gen erforderlich?

b) Zeichnen sie ein Zeit-Geschwindigkeit-Dia-gramm.

Lösung:a) Die Formel 1

2 mv2 eU liefertv 1,88 ´ 106 m=s.

b) Bis die Potentialdifferenz von 6,9 V durchlau-fen ist, steigt die Geschwindigkeit näherungs-weise linear bis 1,31 ´ 106 m=s an. Im Fall einesunelastischen Stoûes geht sie auf Null zurückund steigt mit der gleichen Steigung.

11.1.2 Die quantenhafte Emission

1 a) Welche Frequenzen ergeben sich aus der Bal-mer-Formel für m 7 und m 8?

b) Welche Grenzfrequenz f bzw. Grenzwellen-länge l ergibt sich für groûe Werte von m?

Lösung:a) f7 7,55 ´ 1014 Hz, f8 7,71 ´ 1014 Hzb) Für 1=m 0 erhält man f 8,22 ´ 1014 Hz bzw.

l 365 nm.

2 Welche Spektrallinien des Wasserstoffs sind nachder Balmer-Formel (1) im Infrarotbereich zu er-warten?

Lösung:Da mit steigendem Wert von m die Wellenlängekürzer wird, ist m 3 die längste Wellenlänge.Ihr Wert liegt bei 656 nm im sichtbaren Bereichdes Spektrums. Nach der Balmer-Formel kann eskeine längeren Wellenlängen geben.

*3 Im Spektrum des einfach ionisierten Heliums He+

findet man Linien mit l1 656 nm, l2 541 nm,l3 486 nm, l4 454 nm und l5 434 nm. Wel-cher mathematische Zusammenhang ähnlich der

Balmer-Formel beschreibt die Folge der zugehöri-gen Frequenzen?

Lösung:Durch systematisches Probieren erhält man mit derFormel f 4C (1=42ÿ 1=m2 für m 6, 7, 8, 9, 10die genannten Frequenzen bzw. Wellenlängen.

Zusatzaufgabe

4 a) Welche Werte von m in der Balmer-Formel (1)sind den Wasserstofflinien Ha bis Hd zuzuord-nen?

b) Welche Energie haben die Photonen der Was-serstofflinien Ha bis Hd?

Lösung:a) Für m sind die Werte 3, 4, 5, 6 einzusetzen.b) Mit der Formel E h f hC (1=22ÿ 1=m2)

erhält man die Energiewerte 3,0 ´ 10ÿ19 J;4,1 ´ 10ÿ19 J; 4,6 ´ 10ÿ19 J und 4,8 ´ 10ÿ19 J bzw.1,89 eV; 2,55 eV; 2,86 eV und 3,02 eV.

Seite

402

Seite

403

11.1 Energieaustausch mit Atomen 175

Page 176: Metzler Physik

11.1.3 Die Resonanzabsorption

1 a) Die Na-Dampflampe sendet Licht der Wellen-länge l 589 nm aus. Welche Energie über-trägt ein Photon dieser Strahlung?

b) Die Photonen der Strahlung mit l 589 nmwerden in Versuch 2 nicht auf dem Schirmnachgewiesen. Wohin wird ihre Energie trans-portiert?

Lösung:a) Mit E hc=l erhält man

E 3,37 ´ 10ÿ19 J 2,1 eV.b) Die Energie wird mit der Strahlung, die sich

ausgehend von der Na-Dampf-Röhre isotropim Raum verteilt, wegtransportiert.

2 In Versuch 2 wird die Halogenlampe durch eineNa-Lampe ersetzt. Diese Lampe sendet ausschlieû-lich das in Versuch 1 gezeigte gelbe Licht aus.a) Wie ändert sich das Spektrum gegenüber Ver-

such 2?b) Wie erscheint der Glaskolben mit Na-Dampf

im Licht der Na-Dampflampe?

Lösung:a) Das Spektrum ist dunkel, von einer schwachen

gelben Natriumlinie abgesehen.b) Der Na-Dampf, der im weiûen Licht durchsich-

tig erscheint, leuchtet nun milchig gelb. Dieaufgenommene Energie wird in alle Raumrich-tungen isotrop abgegeben.

Seite

404

11 Atomphysik176

Page 177: Metzler Physik

11.2.2 Der Rutherford'sche Streuversuch

1 Warum muss die Folie, die man für den Ruther-ford'schen Streuversuch verwendet, sehr dünnsein?

Lösung:Damit aufeinander folgende Streuprozesse eineseinzelnen a-Teilchens an verschiedenen Atomker-nen (Mehrfachstreuung) statistisch nicht ins Ge-wicht fallen.

2 Vor dem Beobachtungsmikroskop im Rutherford'-schen Streuversuch wird eine enge, spaltförmigeBlende eingesetzt. Wie ändern sich die Messergeb-nisse, wenn diese Blende fehlt?

Lösung:Die Blende vor dem Beobachtungsmikroskopgrenzt den Beobachtungsbereich auf Teilchen mitnahezu gleichem Ablenkwinkel ein. Ein zu breiterBereich ohne die Blende mittelt über den beobach-teten Bereich des Ablenkwinkels und verfälscht da-durch die Ergebnisse.

3 Ein a-Teilchen stöût zentral auf einen ruhendenGoldkern. Die Geschwindigkeit des a-Teilchensbeträgt in groûer Entfernung vom Kern2 ´ 107 m=s. Welchen Impuls und welche Ge-schwindigkeit bekommt der Goldkern durch denStoû?

Lösung:Der nächste Punkt ist erreicht, wenn die kinetischeEnergie vollständig als potentielle Energie vor-liegt:

Aus 12 ma v2 Qa Qgold=(4pe0 r) mit

r Qa Qgold 2=(4pe0 ma v2) wird

r 2 1,602 10ÿ19 C 79 1,602 10ÿ19 C 24 p 8,82 10ÿ12 C(Vm)ÿ1 6,64 10ÿ27kg (2 107 m=s)2

r 2,75 10ÿ14 m.

Impuls:p 2ma v 2 6,64 10ÿ27 kg 2 107 m=sp 2,656 10ÿ19 kg m=s.

Geschwindigkeit:pmAu vAu)vAu

pmAu 2,656 10ÿ19 kg m=s

196,97 1,6603 10ÿ27 kg

812 km=s.

Im Experiment stöût das a-Teilchen nicht mit ei-nem freien Goldkern, sondern mit einem Goldatomin einem Kristallverband. Für den Stoûvorgang isthier die Masse des Kristalls anzusetzen, die als sehrgroû anzusehen ist. Die Geschwindigkeit des Kri-stalls ist nach dem Stoû deshalb Null.

*4 Begründen Sie, warum es nicht sinnvoll ist,Rutherford'sche Streuversuche in Gasen der Ele-mente H, He, Li, N oder O usw. statt mit Metallfo-lien durchzuführen.

Lösung:Da die Atome der ersten Elemente im Periodensys-tem Massen gleicher Gröûenordnung wie die der a-Teilchen haben, erhält man einen groûen Energie-austausch. Die Streuzentren sind dann nicht mehrruhend.

*5 Vergleichen Sie die De-Broglie-Wellenlängen vona-Teilchen und Elektronen gleicher Energie.Warum sind a-Teilchen für Streuexperimente bes-ser geeignet?

Lösung:Mit le h=

2me Ep

und la h=2ma Ep

lässt sichle=la 85,7 bestimmen. Die De-Broglie-Wellen-länge des a-Teilchens ist bei gleicher Energie umden Faktor 86 kleiner. Das gleiche Beugungsbildwie bei der Elektronenbeugung erhält man bei derBeugung von a-Teilchen an einer um den Faktor 86kleineren Struktur.

11.2.3 Das Atommodell von Rutherford

1 Angenommen, die Kernmaterie wäre im gesamtenAtom gleichmäûig verteilt. Welche Ergebnisse wä-ren dann bei der Streuung von a-Teilchen an Ato-men zu erwarten? Vergleichen Sie mit Abb. 407.1.

Lösung:Auûerhalb des Atoms wäre die Bahn der a-Teil-chen gleich mit Abb. 407.1. Innerhalb des Atomswäre eine sehr viel geringere Ablenkung zu ver-zeichnen, da sich die positiven Ladungen des Kernsteilweise kompensieren.

Seite

406

Seite

407

11.2 Entwicklung der Atommodelle 177

Page 178: Metzler Physik

*2 Die Hüllenelektronen eines neutralen Atoms habenden gleichen Betrag der elektrischen Ladung wieder Kern. Warum kann man die Elektronen bei derStreuung von a-Teilchen an Atomen vernachlässi-gen?

Lösung:Da die Masse der Hüllenelektronen 4 ´ 1836-malgeringer ist als die der a-Teilchen, können sie diea-Teilchen nicht beeinflussen.

11.2.4 Das Bohr'sche Atommodell

1 Berechnen sie die Radien der ersten drei Bohr'-schen Bahnen im Wasserstoffatom.

Lösung:Mit der Formel rn h2 e0 n2=(pme e2) sind dieWerte für die Radien mit n 1, 2, 3 in der Reihen-folge r1 5,29 ´ 10ÿ11 m, r2 2,12 ´ 10ÿ10 m undr3 4,76 ´ 10ÿ10 m.

2 Berechnen Sie die potentielle und die kinetischeEnergie eines Elektrons auf der ersten und zweitenBahn des Wasserstoffatoms.

Lösung:Auf der ersten Bohr'schen Bahn istEkin 2,18 ´ 10ÿ19 J 13,6 eV,Epot ± 4,36 ´ 10ÿ19 J ± 27,2 eV.Für die zweite Bohr'sche Bahn istEkin 5,45 ´ 10ÿ19 J 3,4 eV,Epot ± 1,09 ´ 10ÿ18 J ± 6,8 eV.

3 Berechnen Sie das Verhältnis von Coulomb-Kraftund Gravitationskraft zwischen Elektron und Kernin einem Wasserstoffatom.

Lösung:FCoulomb e2=(4pe0 r2), FGrav: g me mp=r2,

FCoulomb 8,24 10ÿ8 N undFGrav: 3,63 10ÿ47 N,

FCoulomb : FGrav: 2 1039.

*4 Bestimmen Sie die Umlauffrequenz des Elektronsauf der ersten Bahn im Wasserstoffatom. Verglei-chen Sie das Ergebnis mit der Frequenz des Pho-tons, das für die Ionisierung eines Wasserstoff-atoms im Grundzustand benötigt wird.

Lösung:Die mit E h f berechnete Frequenz des Photonsvon f 3,29 ´ 1015 Hz ist halb so groû wie die Fre-quenz f1 6,58 ´ 1015 Hz für die erste Bohr'scheBahn.

11.2.5 Die Spektralserien des Wasserstoffatoms

1 Wie groû ist die kleinste vom Wasserstoffatomemittierte Wellenlänge?

Lösung:Die kürzeste Wellenlänge ist l h c=E 91 nm.

2 In den Spektren des H-Atoms und des He+-Atomsist eine Linie bei 656 nm zu finden. Welche Werteder Quantenzahlen n und m sind jeweils zugeord-net?

Lösung:Die Wellenlänge l 656 nm tritt beim Wasserstoffzwischen den Zuständen n 2 und m 3 bzw.beim Helium zwischen n 4 und m 6 auf.

3 Beim Übergang von n 2 auf n 1 werden Photo-nen emittiert, deren Energie von der Zahl Z derProtonen im Kern abhängt. Berechnen Sie dieEnergien für Z 1; 10 und 100. Um welche Ele-mente handelt es sich?

Lösung:Die Energiewerte vergröûern sich mit Z2. Für Z 1,10, 100 erhält man 10,2 eV, 1,02 keV und 102 keVfür die Elemente H (Wasserstoff), Ne (Neon) undFm (Fermium)

Zusatzaufgaben

4 Wenden Sie die Bohr'schen Postulate auf Ionenmit nur einem Elektron in der Hülle an. BestimmenSie die Gesamtenergie dieser Elektronen En imn-ten Quantenzustand für He, Li und Be.

Lösung:

He: En ÿme e4 4

8e20 h2 n2

ÿ54,4 eV1n2

.

Li: En ÿme e4 9

8e20 h2 n2

ÿ122,4 eV1n2

.

Seite

409

Seite

411

11 Atomphysik178

Page 179: Metzler Physik

Be: En ÿme e4 168e2

0 h2 n2ÿ217,6 eV

1n2

.

*5 Die Aufgabe: ¹Bestimmen Sie die Energieniveausdes Heliumatomsª fehlt hier.Warum eigentlich?

Lösung:Beim Heliumatom befinden sich zwei Elektronenin der Hülle. Sie stoûen sich gegenseitig ab, werdenaber beide vom Kern angezogen. Dieses sogenannte Dreikörperproblem ist mit der Bohr'-schen Theorie nicht zu behandeln.

11.2 Entwicklung der Atommodelle 179

Page 180: Metzler Physik

11.3.1 Der lineare Potentialtopf

1 Betrachten sie das Wasserstoffatom in erster Nähe-rung als einen linearen Potentialtopf mit der Längea 10ÿ10 m. Wie groû ist die Energie im Grund-zustand?Wiegroû ist sie imKern,wenna10ÿ15 mbeträgt?

Lösung:a 10ÿ10 m in E h2 n2=(8m a2) eingesetzt er-gibt E 37,6 eV. Mit a 10ÿ15 m ist E 376GeV.

2 Wie verändern sich die Energiedifferenzen zweieraufeinanderfolgender Zustände im linearen Poten-tialtopf mit zunehmendem Wert der Quanten-zahl n?

Lösung:E (n 1)ÿE (n) h2 (1 2 n)=(8 m a2).

Die Energiebeträge werden mit steigendem n grö-ûer, für groûe Werte von n nahezu proportionalzu n.

3 Statt Elektronen können auch ein Protonen oderNeutronen in einem linearen Potentialtopf einge-schlossen sein. Berechnen Sie die ersten drei Ener-giestufen für Protonen und Neutronen mita 10ÿ15 m.

Lösung:E h2 n2=(8 mP a2) liefert für n 1, 2, 3 die WerteE1 2,05 ´ 108 eV, E2 8,18 ´ 108 eV und E3 18,43 ´ 108 eV.Die Werte für Neutronen unterscheiden sich davonnur geringfügig.

11.3.2 Anwendungen des Potentialtopfmodells

1 H2 ist ein ionisiertes H2-Molekül. Es besteht ausnur einem Elektron und zwei Protonen. Verglei-chen Sie die Bindungsenergie von H2 mit dervon H2. Kann es ein zweifach ionisiertes MolekülH2

2 geben?

Lösung:Aus dem Text: E1 9,4 eV, E2 2,35 eV,DE 7,05 eV. Die Bindungsenergie ist geringer,da nur ein Elektron durch den gröûeren Aufent-haltsbereich die Energiedifferenz von 7,05 eV ab-geben kann. Die experimentellen Ergebnisse zei-gen eine Bindungsenergie von 2,65 eV.Ein stabiles H2

2 -Molekül kann es nicht geben, dadas Molekül keine Elektronen mehr besitzt, diedurch eine Vergröûerung des Potentialtopfes Ener-gie abgeben können.

2 Der angeregte Cyanin-Farbstoff besitzt ein Elekt-ron im Zustand n 5. Mit welcher Energie lässtsich der Zustand mit n 6 anregen? Wie groûmüsste die Wellenlänge der Strahlung sein?

Lösung:Zur Anregung mit E 2,83 eV wird Strahlung derWellenlänge l 439 nm benötigt.

3 a) Ein Farbstoffmolekül hat eine typische Längevon a 1,0 nm. Berechnen Sie die ersten sechsEnergiestufen. Welche Energiedifferenzenführen zu einer Absorption im sichtbaren Be-reich des Spektrums (ca. 1,8 eV bis 3,0 eV)?

b) Wie ändern sich Energie und Wellenlänge derAbsorption von n 5 auf n 6, wenn dieLänge a von 1,0 nm auf 1,5 nm vergröûertwird?

Lösung:a) Die ersten sechs Energiewerte sind 0,376 eV;

1,5 eV; 3,38 eV; 6,02 eV, 9,4 eV und 13,5 eV.Der sichtbare Bereich des Spektrums wird mit1,8 eV und 3 eV eingegrenzt. Die Energiediffe-renz ausgehend vom Zustand mit n 1 in die-sem Bereich hat den Wert 1,13 eV, die für n 2den Wert 1,88 eV. Ab n 3 treten keine Werteim sichtbaren Bereich auf.

b) Die Energie DE ändert sich von 4,14 eV auf1,84 eV, sie verringert sich also deutlich. Diezugehörigen Wellenlängen sind 300 nm und674 nm.

Seite

415

Seite

417

11 Atomphysik180

Page 181: Metzler Physik

11.3.3 Schrödinger-Gleichung

1 Berechnen Sie die Ergebnisse des dritten Itera-tionsschritts in Abb. 419.1 für E 1 ´ 10ÿ18 J.

Lösung:Iterationsvorschrift für den linearen Potentialtopf:Y(x)Y(xÿDx)Y 0xÿDxDxY 0(x)Y 0(xÿDx)Y 00xÿDxDxY 00x ÿ1,6382 1038 E YxStartwerte mit E 1 10ÿ18 und Dx 10ÿ11:x 0;Y0 0;Y 00 1;Y 000 0

Ergebnisse des 1. Iterationsschrittes:xDx; YDx 1 10ÿ11; Y 0Dx 1;

Y 00Dx ÿ1,6382 109

Ergebnisse des 2. Iterationsschrittes:x 2Dx; Y2Dx 2 10ÿ11;

Y 02 Dx 0,9836; Y 002 Dx ÿ3,2764 109

Ergebnisse des 3. Iterationsschrittes:x 3Dx; Y3Dx 2,9836 10ÿ11;

Y 03 Dx 0;9509; Y 003 Dx ÿ4,8878 109

2 Wiederholen Sie die Iteration entsprechendAbb. 419.1 für gröûere Energiewerte. Welches istder nächstgröûere Energiewert, für den Y(a) 0gilt?

Lösung:Die nächste Lösung ergibt sich bei der EnergieE2 4 E1 24 ´ 10ÿ18 J.

*3 Vergleichen Sie den in Aufgabe 2 berechnetenEnergiewert mit E n2 h2=(8 me a2) (! 11.3.1).

Lösung:Die Lösung von Aufgabe 2 ist der Wert der Formel

E h2 n2=(8 ma2)

für n 2.

Zusatzaufgabe

4 Berechnen Sie die Ergebnisse des dritten Itera-tionsschritts in Abb. 419.2 fürE ÿ15 1,602 10ÿ19 J.

Lösung:Iterationsvorschrift zum WasserstoffatomY(r)Y(rÿDr)Y 0rÿDrDrY 0(r)Y 0(rÿDr)Y 00rÿDrDrY 00r ÿ2Y 0r=rÿ 1,6382 1038E

2,30708 10ÿ28=rYrStartwertemit Eÿ151,60210ÿ19 undDr10ÿ12:r0 10ÿ12; Yr0 1; Y 0r0 ÿ1010;

Y 00r0 ÿ1,7401 1022

Ergebnisse des 1. Iterationsschrittes:r r0Dr; Yr0Dr 0,99;

Y 0r0Dr ÿ2,7401 1010;

Y 00r0Dr 9,0823 1021

Ergebnisse des 2. Iterationsschrittes:r r0 2Dr; Yr0 2Dr 0,9626;

Y 0r0 2Dr ÿ1,8319 1010;

Y 00r0 2 Dr 4,6432 1020

Ergebnisse des 3. Iterationsschrittes:r r0 3Dr; Yr0 3Dr 0,9443;

Y 0r0 3Dr ÿ1,7854 1010;

Y 00r0 3 Dr 3,7669 1020.

11.3.5 Analytische Lösung der Schrödinger-Gleichung für den linearen Potentialtopf

1 Ein Potentialtopf hat die Länge a 10ÿ10 m.a) Zeigen Sie, dass die Summe der Antreffwahr-

scheinlichkeiten w1(x) in Intervallen der LängeDx 10ÿ11 m den Wert eins ergibt.

b) Wie groû ist die Antreffwahrscheinlichkeitw1(x) im Bereich vom x 0 bis x 3 ´ 10ÿ11 m?

Lösung:a) Siehe Tab. 420.1. Die Summe der Werte ist

eins.b) Die Summe der Werte für die Intervalle um

0,5 ´ 10ÿ11 m bis 2,5 ´ 10ÿ11 m ist 0,146, die An-treffwahrscheinlichkeit beträgt etwa 15%.

2 Berechnen Sie für einen Potentialtopf der Längea 10ÿ10 m die Wahrscheinlichkeiten w2(x) in In-tervallen der Länge Dx 10ÿ11 m für den angereg-ten Zustand mit n 2.

Lösung:Die normierten Antreffwahrscheinlichkeiten fürn 2 in den Intervallen von 0,5 ´ 10ÿ11 m bis9,5 ´ 10ÿ11 m betragen 0,0190983; 0,130902; 0,2;0,130902; 0,0190983; 0,0190983; 0,130902; 0,2;0,130902; 0,0190983.

Seite

419

Seite

421

11.3 Das Atommodell der Quantenphysik 181

Page 182: Metzler Physik

3 Zeigen sie, dass 12 (x ± sin(x) cos(x)) eine Stamm-

funktion von sin2(x) ist. Berechnen sie damit A ausder Gleichung

A2

Za

0

sin2

pxa

dx 1.

Lösung:Die Ableitung von 1

2 (x ± sin(x) cos(x)) nach x liefertmit der Produktregel und dem Ersetzen von cos2(x)durch 1ÿ sin2(x) den Funktionsterm sin2(x).Zur Berechnung des Integrals ist die Stammfunk-tion a=p 1

2 (p x=a ± sin(p x=a) cos(p x=a)) in denGrenzen 0 und a zu berechnen. Mit dem Ergebnisa=2 folgt A

2=a

p.

4 Betrachtet man die Antreffwahrscheinlichkeitfür Elektronen im angeregten Zustand nach

Abb. 421.2, dann stellt sich folgende Frage: Wiekönnen die Elektronen vom linken Bereich in denrechten und umgekehrt gelangen, wenn die An-treffwahrscheinlichkeit in der Mitte null ist?

Lösung:Die Ausbreitung von Quantenobjekten wie Elekt-ronen oder Photonen wird durch Wellen beschrie-ben. Bei einer stehenden Welle ist die Amplitudean einem Knotenpunkt null. Sich vorzustellen, dasses sich um Teilchen handelt, die sich schnell imPotentialtopf bewegen, führt zu gedanklichenSchwierigkeiten ähnlicher Art wie bei der Be-schreibung der Bewegung von Elektronen durcheinen Doppelspalt (! 10.2). Offensichtlich ent-spricht diese Vorstellung nicht der physikalischenRealität.

11.3.6 Analytische Lösung für Wasserstoff

1 Mit welcher Wahrscheinlichkeit trifft man einElektron im Grundzustand im Intervall0 < r < 5 ´ 10ÿ11 m an?

Lösung:Die Wahrscheinlichkeit berechnet sich aus derSumme der Intervallwahrscheinlichkeiten zu0,293 29,3%.

2 Für welchen Bereich beträgt die Wahrscheinlichkeit50%? Wie groû ist dieWahrscheinlichkeit für r0?

Lösung:Durch Addieren der Werte in Tabelle 423.1 erhältman mit guter Genauigkeit den Bereich von 0 m bis7 ´ 10ÿ11 m für 50% Antreffwahrscheinlichkeit.Für r 0 ist die Antreffwahrscheinlichkeit null, dadas Antreffvolumen in wY2 DV null ist.

11.3.7 Die Winkelabhängigkeit der Antreffwahrscheinlichkeit im H-Atom

1 a) Bestimmen Sie die Anzahl der unterschied-lichen Zustände für n 1 bis n 2 ausAbb. 424.2 und geben sie die zugehörigen vierQuantenzahlen an.

b) Vergleichen Sie die Ergebnisse mit dem Auf-bau des Periodensystems der Elemente.

Lösung:a) Für n 1 sind 2 Zustände vorhanden:

n 1, l 0, m 0, s 1=2 undn 1, l 0, m 0, s ± 1=2Für n 2 sind 8 Zustände vorhanden:

n 2, l 0, m 0, s 1=2 undn 2, l 0, m 0, s ±1=2,n 2, l 1, m ±1, s 1=2 undn 2, l 1, m ±1, s ±1=2,n 2, l 1, m 0, s 1=2 undn 2, l 1, m 0, s ±1=2,n 2, l 1, m 1, s 1=2 undn 2, l 1, m 1, s ±1=2

b) Die Zahl der Zustände mit den Quantenzahlenn 1 und n 2 entspricht der Zahl der Elementemit der Periode 1 bzw. 2 (siehe Abb. 426.1)

11.3.9 Das Periodensystem der Elemente

1 Im H-Atom ist die Energie eines 2 s-Elektronsund eines 2 p-Elektrons gleich. Warum ist dieEnergie der 2 s- und 2 p-Elektronen beim Element

Li unterschiedlich, wo doch in beiden Fällen diefür die Energie bedeutsame Quantenzahl n 2ist?

Seite

423

Seite

425

Seite

427

11 Atomphysik182

Page 183: Metzler Physik

Lösung:Im Wasserstoffatom wird das eine vorhandeneElektron durch keine weiteren Elektronen beein-flusst. Es hat die durch die Quantenzahl n festge-legte Energie. Im Lithiumatom wird das Elektronim Zustand n 2 durch die Elektronen im Zustandn 1 beeinflusst. Abhängig vom Zustand 2 s oder2 p ist dieAbschirmung des positivgeladenenKernsdurch die inneren Elektronen unterschiedlich.

*2 Das Periodensystem der Elemente im Anhang zeigtfür das Element Kalium eine scheinbare Unregel-mäûigkeit bei der Besetzung der Zustände. Um

welche Unregelmäûigkeit handelt es sich? WelcheUnterschiede zeigt die Abschirmung für s- und p-Elektronen?

Lösung:Das Element Kalium hat ein Elektron im Zustandn 4, obwohl die Zustände mit n 3 noch nichtvollständig aufgefüllt sind. Die Abschirmung derinneren Elektronen wirkt sich ähnlich wie inAbb. 427.1 gezeigt aus. Die Zustände mit n 3spalten energetisch auf, und zwar so weit, dassdas 3 f-Niveau oberhalb des 4 s-Niveaus liegt. Des-halb wird 4 s vor 3 f besetzt.

11.4.2 Absorption von Röntgenstrahlung

1 Bestimmen Sie Energie, Frequenz und Wellen-länge der charakteristischen Röntgenstrahlung ei-ner Silberplatte.

Lösung:

Mit der Formel Eme (Zÿ 1)2 e4

8 e20 h2

1n2

wird mit Z 47 die Energie für den Übergang vonn2aufm1berechnet.DE3,46 ´ 10ÿ15 J21,6keV. Die Frequenz der zugehörigen Strahlung istf 5,22 ´ 1018 Hz, die Wellenlänge l 57,4 pm.

2 Eine genaue Untersuchung zeigt, dass die charak-teristische Röntgenstrahlung eines Elements meh-rere diskrete Spektrallinien enthält. Wie lässt sichdieses Phänomen erklären?

Lösung:Die einzelnen Linien der charakteristischen Rönt-genstrahlung lassen sich den möglichen Zustands-änderungen hin zum Zustand n 1 zuordnen. Es

sind die Übergänge n 2 nach n 1, dann n 3nach n 1 und n 4 nach n 1 möglich. Die Folgeder Linien endet, wenn die Zustände nicht mehrbesetzt sind, z. B. bei Silber ab n 6.

*3 Das Element Sr zeigt neben der Absorptionskantebei 78 pm eine weitere bei deutlich gröûerer Wel-lenlänge l2. Wie ist sie zu erklären? Berechnen Sieden Wert von l2?

Lösung:Eine gröûere Wellenlänge weist auf einen Über-gang zwischen Zuständen hin, an denen der Grund-zustand nicht beteiligt ist. Die Rechnung mit n 2zeigt, dass für m 6 bzw. m 1 Strahlung mitWellenlängen in der Gröûenordnung 300 pm auf-tritt.Mit n 2 und m 6: f 1,00 ´ 1018 Hz;l 300 pm.Mit n 2 und m 1: f 1,13 ´ 1018 Hz;l 266 pm

11.4.3 Spektren im sichtbaren Bereich

1 Berechnen Sie mit den Angaben aus Abb. 430.1 dieEnergie des 3 p1-Zustands.

Lösung:Aus Abb. 430.1 lässt sich E ± 3,1 eV ablesen.

*2 Die Spektren der Elemente K, Rb und Cs sind hiernicht gezeigt. Suchen Sie nach einem dieser Spekt-ren in der Fachliteratur. Vergleichen Sie mit demNa-Spektrum und schätzen Sie die effektive Kern-ladungszahl Z* ab.

Lösung:Die gesuchten Spektren findet man beispielsweisein Bergmann-Schäfer: Experimentalphysik, dieWerte für Abschirmzahlen im Handbuch der expe-rimentellen Schulphysik Bd. 8. Für die effektiveKernladungszahl Z* Z ± s sind für die Abschirm-zahl s die Werte 0,98 bei Z 20, 0,90 bei Z 30,0,86 bei Z 40 und 0,24 bei Z 50 angegeben. Fürhöhere Werte von Z erhält man negative Werte fürs. Hier überwiegt der Einfluss der äusseren Elekt-ronen die Abschirmung durch die inneren.

Seite

429

Seite

430

11.4 Leistung der Atommodelle 183

Page 184: Metzler Physik

11.4.4 Lumineszenz

1 Die Anregung der Lumineszenz durch beschleu-nigte Elektronen (Elektrolumineszenz) wird beiBildschirmen (Fernseher, Monitor, Oszilloskop)benutzt, um ein optisches Bild zu erhalten.a) Mit welcher Spannung müssen die auftreffen-

den Elektronen mindestens beschleunigt wer-den?

b) Welche Verweildauer im metastabilen Niveauist bei einer Fernsehbildröhre mit für 50 Hzbzw. 100 Hz Bildwechselfrequenz günstig?

Lösung:a) Zur Erzeugung von sichtbarem Licht sind ca.

2 V erforderlich, da die Energie der PhotonenE h f h c=l etwa 2 eV beträgt. Die Be-schleunigungsspannung ist mit ca. 25 kV sehrviel gröûer.

b) Eine Nachleuchtdauer von 0,02 s (0,01 s) be-wirkt, dass der jeweilige Bildpunkt mit 50 Hz(100 Hz) Bildwechselfrequenz genau dann neuangeregt wird, wenn das Nachleuchten der vor-herigen Anregung beendet ist.

11.4.5 Der Helium-Neon-Laser

1 Ein He-Ne-Laser hat einen Spiegelabstand von0,5 m. Bestimmen Sie den Frequenzabstandbenachbarter Resonanzfrequenzen dieses opti-schen Resonators im Bereich der Laserlinie(l 633 nm). Vergleichen Sie die so abzuschät-zende Linienbreite mit der natürlichen Linienbreiteder Neonlinie.

Lösung:Die Länge l ist das 789 889,42fache der Laser-wellenlänge l 633 nm. Die Wellenlänge kannalso nicht erzeugt werden. Die nächstgelege-nen Wellenlängen sind 789 889,5 ´ l1 0,5 m mitl1 632,999932269 nm und 789 889,0 ´ l2 0,5 mmit l2 633,000332958 nm. Der Frequenzabstandist Df 3,0 ´ 108 Hz. Der Linienabstand der Laser-linien ist 300 MHz, die Linienbreite der Laserlinieaus dem Text beträgt 800 MHz. Es können alsomehrere unterschiedliche Frequenzen die Laserbe-dingung erfüllen.

*2 Will man monochromatische Laserlinien erhalten,bevorzugt man kurze optische Resonatoren von0,1 m bis 0,2 m Länge. Welche Konsequenzenhat die Länge des Lasers für die möglichen stehen-den Wellen?

Lösung:Wird der Resonanzraum kleiner gemacht, wächstdie Frequenzdifferenz zwischen benachbarten Li-nien. Wird der Frequenzabstand der Linien gröûer

als die natürliche Linienbreite, dann kann nur nochdie eine Linie innerhalb der Linienbreite der Laser-linien die Laserbedingung erfüllen.

3 Nach welchen Gesichtspunkten sollte man Gasefür einen Gas-Laser aussuchen?

Lösung:Eine Emissionsline des Pumpgases muss mit einerAbsorptionslinie des Lasergases, die bei der Her-stellung der Besetzungsinversion benötigt wird,nahezu übereinstimmen.

4 Ein Puls-Laser sendet Impulse von 10 ns Dauer beieiner Strahlungsleistung von 10 GW aus. WelcheEnergie enthält ein Puls? Warum ist ein solcherPuls gefährlich?

Lösung:Mit E P t erhält man aus P 1010 W und t 10ÿ8 s die Energie E 100 J. Die Energie wird aufeine kleine Fläche in einer kurzen Zeit abgegeben.In dieser Zeit findet keine Wärmeabgabe an dieUmgebung statt, das Material verdampft.

5 Quick-Switch-Laser können Impulse von nur 3 psDauer bei 0,5 ps Anstiegszeit abgeben. Wie genaukönnte man damit eine Entfernung nach dem Lauf-zeitprinzip messen?

Lösung:In 0,5 ps bewegt sich das Licht 0,15 mm weit.

Seite

431

Seite

433

11 Atomphysik184

Page 185: Metzler Physik

11.4.6 Berechnung der Absorptionsspektren von Farbstoffmolekülen

1 Laserdioden emittieren im Bereich um 800 nm. DieStrahlung soll auf einer beschreibbaren CD mit ei-nem Farbstoff absorbiert werden. Nach welchenGesichtspunkten ist der Farbstoff auszuwählen?

Lösung:Der Farbstoff sollte sein Absorptionsmaximum beider Wellenlänge haben, bei der der Laser emittiert.In diesem Fall wird die Energie bevorzugt vomFarbstoff aufgenommen.

2 Welcher Farbstoff eignet sich zur Absorption derHe-Ne-Laserlinie bei 632,8 nm?

Lösung:Die Absorptionswellenlänge berechnet sich nach

l 8me c l2

h(z 2 f )2

(z 4)

für einen Farbstoff mit f 1,83 und l 139 pm inAbhängigkeit von der Zahl der Kettenglieder z zuz 3, l 404 nmz 5, l 531 nmz 7, l 658 nm

z 9, l 785 nmz 11, l 913 nmDer Farbstoff mit z 7 Kettengliedern ist zur Ab-sorption der Laserlinie bei 632,8 nm geeignet.Weitere Informationen zu Farbstoffen finden sichz. B. im Handbuch der experimentellen Physik,Bd. 8.

*3 Cyanin-Farbstoffe sind empfindlich gegenüberUV-Strahlung. Sie verringern ihre Kettenlängebeispielsweise um zwei Elemente. Welchen Ver-lauf hat das geänderte Absorptionsspektrum dieserFarbstoffe?

Lösung:Wenn der Farbstoff seine Kettenlänge um zweiKettenglieder verkürzt, verringert sich die Wellen-länge der Absorption um ca. 127 nm hin zu niedri-geren Wellenlängen. Ein Farbstoffgemisch absor-biert also bei der Wellenlänge, die sich aus demlängsten Farbstoffmolekül ergibt sowie bei um127 nm verringerten Wellenlängen.

Seite

435

11.4 Leistung der Atommodelle 185

Page 186: Metzler Physik

12.1.1 Halbleiter und Dotierung

1 Warum wird bei sehr hohen Temperaturen dotiert?

Lösung:Nur bei sehr hohen Temperaturen tritt eine merk-liche Diffusion der Atome im Festkörper auf, sonstsind die Atome ortsfest.

2 Wie verändert sich der Leitungsmechanismus ei-nes dotierten Halbleiters (p-Leiter oder n-Leiter)bei sehr hohen bzw. sehr niedrigen Temperaturen?

Lösung:Bei sehr hohen Temperaturen verringert sich derEinfluss der durch Dotierung hinzugefügten La-dungsträger. Bei sehr niedrigen Temperaturen ver-ringert sich die Leitfähigkeit.

12.1.3 p-n-Übergang und Dioden

1 Zeichnen Sie die Diodenkennlinie mit I0 10 pAfür die Temperaturen T ± 50 C, 20 C und100 C.

Lösung:Die Spannung U an der Diode in Abhängigkeit vonder Temperatur gibt

U (T) kT=e

an. Für den Diodenstrom ID gilt

ID I0 eUD=U T I0 eeUD=kT

Mit I0 10 pA erhält man für T1 223 K,T2 293 K und T3 373 K die folgenden Graphen:

0,4

0,8

0,2 0,6 0,8

0,4

0,2

0,6

U in V

I in A223 K 293 K 373 K

00

Die Graphen zeigen richtig die Verschiebung desKurvenverlaufs bei Temperaturänderungen. Tat-sächlich verringert sich die Spannung U bei kon-stantem Strom mit steigernder Temperatur, was aufdie Temperaturabhängigkeit von I0 zurückzufüh-ren ist.

2* Wie ändert sich die Spannung an einer Diode beifestem Strom durch Temperaturerhöhung? WelcheSpannungsänderung verursacht eine Temperatur-differenz von 1 K bei T 293 K und U 0,6 V?

Lösung:Die Spannung verringert sich bei Temperaturerhö-hung.Die Steigung des Graphen von U(T) istdU=dT (k=e) ln (I=I0).Mit I 1 A (als typischem Durchlassstrom) undI0 10 pA (aus Aufgabe 1) erhält man den reali-stischen Wert dU=dT 2 mV=K. Andere Werteliegen in dieser Gröûenordnung. Bei einer Tempe-raturerhöhung um 1 K verringert sich UD um etwa2 mV.

3* Welche elektrischen Eigenschaften hat eine Diodea) aus einem undotierten und einem n-dotierten

Halbleiter,b) aus einem Metall und einem p-dotierten Halb-

leiter?

Lösung:a) In einem undotierten Halbleiter sind p- und n-

Leitung gleichzeitig vorhanden. Ein Kontaktmit einem n-Leiter führt zu einer Sperrschichtfür die p-Ladungsträger des undotierten Halb-leiters. Die n-Leitung zwischen den beiden Be-reichen erhöht den Sperrstrom.

b) Ein Metall verhält sich ähnlich einem n-Leiter.Der Kontakt mit einem p-leitenden Materialzeigt vergleichbare Eigenschaften wie ein p-n-Übergang. Allerdings vergröûert sich derSperrstrom gegenüber einem p-n-Übergangdeutlich.

Seite

441

Seite

443

12 Elektronik und Festkörperphysik186

Page 187: Metzler Physik

12.1.4 Der bipolare Transistor

1 In der Verstärkerschaltung von Abb. 445.1 sollenalle Widerstände um den Faktor 10 vergröûertwerden.a) Wie ändert sich der Verstärkungsfaktor?b) Wie ändern sich Eingangs- und Ausgangswi-

derstand?c) Welche Konsequenzen ergeben sich bei der

Anpassung an eine hochohmige Signalspan-nung bzw. an ein niederohmiges Bauelementam Ausgang?

Lösung:a) Wenn die Widerstände um den Faktor 10 grö-

ûere Werte haben, ist der EingangsstromIB 4,3 mA und IC 0,923 mA. Die Werteder Ströme verringern sich um den Faktor 10,die Spannungswerte bleiben konstant. Die Ver-stärkung bleibt damit gleich.

b) Der Eingangswiderstand ist wie der Ausgangs-widerstand um den Faktor 10 angestiegen.

c) Für eine hochohmige Signalquelle ist der Ein-gangswiderstand groû zu wählen. Bei gleicherVerstärkung sind dann alle Widerstände umden gleichen Faktor zu vergröûern. Beim Ver-kleinern um den gleichen Faktor lässt sich einniederohmiges Bauelement (Lastwiderstand)am Ausgang anpassen.

2* In der Verstärkerschaltung von Abb. 445.1 wirdder Transistor durch einen anderen mit der Strom-verstärkung B 300 ersetzt. Wie ändert sich dieVerstärkung und die Amplitude der Ausgangs-wechselspannung?

Lösung:Der Anstieg der Stromverstärkung von B1 220auf B2 300 erfordert die Anpassung des Wertesvon R1 von 330 kW auf 450 kW. Bei gleichen Aus-gangswiderstand und gleicher Verstärkung erhöhtsich der Eingangswiderstand um den FaktorB2=B1 1,36.

12.1.5 Der Feldeffekttransistor

1 Der Eingangswiderstand der Schaltung inAbb. 464.4 ergibt sich aus der Parallelschaltungvon R1 1 MW und R2 240 kW.a) Wie groû ist der Eingangswiderstand der Schal-

tung? Wie groû ist der Eingangswiderstand derMOSFET?

b) Wie lässt sich der Eingangswiderstand 10fachvergröûern, ohne die Verstärkung der Schal-tung zu verändern?

Lösung:a) Der Eingangswiderstand beträgt 194 kW. Der

Eingangswiderstand vom MOSFET ist unend-lich.

b) Der Eingangswiderstand wird 10fach, wenn diebeiden Widerstandswerte R1 und R2 ebenfallsverzehnfacht werden.

2 Berechnen Sie die Leistungsverstärkung der Schal-tung von Abb. 446.4.

Lösung:Die Leistungsverstärkung des MOSFET ist unend-lich, da der Eingangsstrom und damit die Ein-gangsleistung null sind. Durch die Beschaltungmit Widerständen sinkt der Eingangswiderstandauf 194 kW (siehe Aufgabe 1), der Ausgangswider-

stand ist 1 kW, wenn der Widerstand der Drain-Source-Strecke vernachlässigt wird. Mit der Span-nungsverstärkung V 210 erhält man

VP (U2A=U

2E) (RE=RA) 2102 194=1 8,6 106.

3 Im eingeschalteten Zustand wird bei einem MOS-FET die Spannung UDS 0,1 V bei einem StromIDS 10 A gemessen. Wie groû ist der Widerstanddes MOSFET?

Lösung:Der Widerstand der MOSFET beträgt 0,01 W.

4 Stellen Sie Vorzüge und Nachteile eines MOSFETgegenüber einem bipolaren Transistor zusammen?

Lösung:Der MOSFET besitzt den groûen Vorteil, dass seinEingangsstrom null ist. Dieser Vorteil ist nur beiGleichspannung und Wechselspannung niedrigerFrequenz entscheidend. Bei höheren Frequenzenist der MOSFET dem bipolaren Transistor durchseine groûe Eingangskapazität im Nachteil.Ein weiterer Vorteil des MOSFET ist die geringeTemperaturabhängigkeit der Parameter.

5* Im Gegensatz zu bipolaren Transistoren lassen sichMOSFETs problemlos parallel schalten. Welche

Seite

445

Seite

447

12.1 Halbleiterschaltungen 187

Page 188: Metzler Physik

Eigenschaften der Parallelschaltung bleiben imVergleich zu einem einzelnen MOSFET erhalten,welche ¾nderungen sind zu erwarten?

Lösung:Bei der Parallelschaltung zweier MOSFETs ver-doppelt sich die Steilheit S der Strom-Spannungs-Kennlinie, halbiert sich der Ausgangswiderstand.

Leistungs-MOSFET bestehen intern aus einer Pa-rallelschaltung von MOSFET-Strukturen und er-reichen so eine hohe Steilheit und einen kleinenEinschaltwiderstand.Für hohe Frequenzen ist die verdoppelte Eingangs-kapazität der Parallelschaltung nachteilig.

12 Elektronik und Festkörperphysik188

Page 189: Metzler Physik

12.2.1 Zustände im Elektronengas

1 Die Energieniveaus in Abb. 449.1 sollen mit dreiElektronen aufgefüllt sein.a) Welche Energie hat das energiereichste Elek-

tron in einem 3D-Potentialtopf mit den Kanten-längen 1 nm.

b) Welche Möglichkeiten bestehen für die Vertei-lung der Antreffwahrscheinlichkeit? Skizzie-ren Sie eine ausgewählte Verteilung.

Lösung:a) Die Energie im Zustand (2, 1, 1) beträgt

3,61 ´ 10ÿ19 J.b) Die Skizze ergibt sich aus der Überlagerung

von zwei Verteilungen im Grundzustand(Abb. 448.1 a) und einer Verteilung (Abb.448.1 b), die im Raum unterschiedlich orien-tiert sein kann.

2 In Abb. 449.1 sind die ersten sechs Energieniveauseingezeichnet. Welche Tripel von Quantenzahlengehören zum nächsthöheren Energieniveau?

Lösung:Nach nx, ny, nz 1, 2, 3 mit 1 4 9 14 folgt 2, 2,3 mit 4 4 9 17. Die drei Tripel der Quanten-zahlen sind also (3, 2, 2), (2, 3, 2) und (2, 2, 3).

3 Ein 3D-Potentialtopf mit den Kantenlängen 1 nmist mit einem Elektron besetzt. Welches ist dieniedrigste Anregungsenergie für diesen Zustand?

Lösung:Die niedrigste Anregungsenergie ist DE1,8 ´ 10ÿ19 J vom Zustand (1, 1, 1) in der Zustand(2, 1, 1).

12.2.2 Die Fermi-Energie

1 Berechnen Sie die Werte der Fermi-Energie fürAluminium mit einer Elektronendichte vonne 18,1 ´ 1028 mÿ3. Wie ändert sich die Fermi-Energie, wenn die Zahl der freien Elektronen z. B.durch Temperaturerhöhung zunimmt?

Lösung:Die Fermi-Energie von Aluminium beträgt 11,7 eV.Sie steigt mit den Zahl der freien Elektronen.

*2 Berechnen Sie mit den Angaben aus Tab. 451.1den Abstand zweier Energieniveaus nahe derFermi-Energie für einen Kupferwürfel mit derKantenlänge 10 nm.

Lösung:In einem Kupferwürfel der Kantenlänge a 10 nmund EF 7,04 eV 1,23 ´ 10ÿ18 J folgt mitnx ny nz n0 aus

E h2

8me a2(n2

x n2y n2

z)

aufgerundet n0 25. Die Energie des nächsten Zu-stands (nx, ny, nz) (25, 25, 26) ist 1,16 ´ 10ÿ18 J.Damit beträgt der Abstand der EnergieniveausDE 3 10ÿ20 J 0,2 eV.

12.2.3 Supraleitung

1 Berechnen Sie DEG von Quecksilber.

Lösung:Die Energiedifferenz für Quecksilber beträgt1,3 meV.

2* Welche magnetische Feldstärke B erreichen supra-leitende Magnete beim DESY in Hamburg?

Lösung:Informationen aus dem Internet liefern den Wertvon B 24 T.

12.2.4 Absorptionsverhalten von Festkörpern

1 Welche Halbleiter (Tab. 454.2) zeigen eine Ab-sorptionskante im sichtbaren Bereich des Spekt-rums? Welche Farbe erscheint in der Durchsicht?

Lösung:Nur CdS und Se zeigt im sichtbaren Spektrum zwi-schen 1,8 eV und 3 eV eine Absorptionskante. Inder Durchsicht erscheint der CdS-Kristall Gelb-Rot(DE 2,3 eV), der Se-Kristall Rot (DE 2,1 eV).

Seite

449

Seite

451

Seite

452

Seite

454

12.2 Das quantenphysikalische Modell des Festkörpers 189

Page 190: Metzler Physik

2 Ein Selen-Kristall wird von Licht durchstrahlt.Welchen Wellenlängen- bzw. Frequenzbereichumfasst das durchgelassene Licht?

Lösung:Mit l hc=DE erhält man 590 nm. Strahlung mitWellenlängen gröûer 590 nm wird von Se durch-gelassen.

3 Begründen Sie die Durchlässigkeit der Halbleiterwie Si und Ge für Infrarotstrahlung.Lösung:Halbleiter wie Si (DE 1,15 eV) und Ge (DE 0,65 eV) absorbieren unterhalb von 1,9 mm bzw.1,1 mm.

Die Strahlung mit Wellenlängen oberhalb von1,9 mm (Ge) bzw. 1,1 mm (Si) wird durchgelassen,weil die Energie der Photonen kleiner als die mate-rialabhängige Mindestenergie ist.

4 Warum sind Metalle für Mikrowellen (l 1 mm)bzw. Radiowellen (l 1 mm) undurchlässig?

Lösung:In einem makroskopischen Metallstück sind dieEnergieniveaus in einem Abstand von 10ÿ22 eVangeordnet. Radiowellen mit der Wellenlängel 1 m liefern Photonen mit der EnergieDE h c=l 1,2 ´ 10ÿ6 eV.

12.2.5 Leitfähigkeit von Halbleitern

1 Will man die Eigenleitung von Halbleitern bestim-men, muss man bei tiefen Temperaturen sehr aufVerunreinigungen durch andere Stoffe achten.Warum ist dies bei hohen Temperaturen nicht sowichtig?

Lösung:Bei tiefen Temperaturen ist die Eigenleitung auûer-ordentlich gering, der Halbleiter wird zum Isolator.Verunreinigungen in geringster Konzentrationwürden das Ergebnis bereits verfälschen. Bei hö-heren Temperaturen nimmt die Eigenleitung ummehrere Zehnerpotenzen zu, sodass die Leitungdurch Verunreinigungen kaum ins Gewicht fällt.

2 Glas wird bei höherer Temperatur zum Halbleiter,bei sehr hoher sogar zum Leiter. Wie ist dies zuerklären?

Lösung:Das verbotene Band wird bei hohen Temperaturenvon einer zunehmenden Anzahl von Elektronenüberbrückt. Die Elektronen im Leitungsband tra-gen zur Leitung bei.

3 Nicht alle Halbleitermaterialien eignen sich alsFotowiderstand. Worauf ist zu achten?

Lösung:Die materialabhängige Mindestenergie DE solltegeringer sein als die Energie der Photonen an derlangwelligen Grenze der Strahlung.

12.2.8 Kontaktspannung

1 Welche Kontaktspannung entsteht, wenn sich dieMetalle Barium mit EA1 5,2 eV und NickelEA2 2,5 eV berühren?

Lösung:Es entsteht eine Kontaktspannung von(EA1ÿEA2)=e (5,2 eVÿ 2,5 eV)=e 2,7 V.

2 Untersuchen Sie den Aufbau des Peltier-Elements(Abb. 461.2) hinsichtlich Parallel- und Serien-schaltung in elektrischer und thermischer Sicht.Welche Konsequenzen hat die gewählte Anord-nung?

Lösung:Die Halbleiterkontakte des Peltierelement sind inelektrischer Hinsicht in Serie geschaltet. Damitlässt sich bei einer Spannung von einigen mV ander Kontaktstelle eine Betriebsspannung von eini-gen Volt verwenden. In thermischer Sicht sind dieElemente parallelgeschaltet. Die Parallelschaltungder Wärmewiderstände vergröûert den möglichenStrom der Wärmeenergie bei gleicher Temperatur-differenz.

Seite

455

Seite

461

12 Elektronik und Festkörperphysik190

Page 191: Metzler Physik

12.3.1 Der Operationsverstärker

1 Mit welcher Beschaltung lässt sich bei einem OVdie Verstärkung 1; 10; 100 einstellen?

Lösung:Mit R1 1 kW ist für die Verstärkung von 1, 10,100 der Wert von R2 zu 0 W, 9 kW und 99 kW be-stimmt.

2 Welche Verstärkung ergibt sich in Abb. 463.1 fürR2 0 bzw. R1 0?

Lösung:Für R2 0 ist die Verstärkung V 1. Für R1 0ist die Verstärkung nach der Verstärkungsformelunendlich. Hier gilt die VereinfachungV0 ´ R1=(R1 R2)

1nicht mehr. Deshalb ist die reale Verstärkung end-lich und hat den Wert V0 der Verstärkung des Ope-rationsverstärkers.

Seite

463

12.3 Analoge Signalverarbeitung 191

Page 192: Metzler Physik

12.4.1 Computerbaugruppen

1 Berechnen Sie Taktzeiten und Taktfrequenzen fürdie beiden Mikroprozessoren in Abb. 469.1 undAbb. 469.2.

Lösung:Die Taktfrequenz des 4004 Prozessors ist beiT 10,8 ms Taktzeit f 1=T 92,6 kHz. DieTaktzeit bei einem 300 MHz Pentium-Prozessorist 3,3 ns.

*2 Zur Leistungssteigerung von Computersystemenlassen sich 2 bzw. 4 Prozessoren parallel an einem

gemeinsamen Speicher betreiben. In welchenFällen sind zwei Prozessoren doppelt so schnellwie ein einzelner?

Lösung:Durch mehrere Prozessoren lässt sich die Rechen-zeit verkürzen, wenn die eine Teilrechnung nichtdie Ergebnisse der zweiten verwendet. Es ist des-halb bei aufwendigen Rechnungen ein Ziel der In-formatik, die Teilrechnungen zur parallelisieren.

12.4.2 Digitale Grundschaltungen

1 Vergleichen Sie mit Abb. 470.2 und Abb. 471.1:a) Wie lässt sich eine NAND-Schaltung mit

CMOS-Transistoren realisieren?b) Wie lässt sich eine NOR-Schaltung mit bipola-

ren Transistoren aufbauen?

Lösung:

VDD VDD

U1

2

a

T

U

1

2

U

′T ′

2

T1

T

R2R1

U1 U2

3,6 kΩ 3,6 kΩ

aU2T1 T

+3,6 V

CR1,5 kΩ

2 In welchen Anwendungen empfiehlt sich ist derEinsatz von TTL-Schaltungen bzw. CMOS-Schal-tungen?

Lösung:TTL-Schaltungen eignen sich für schnelle Schalt-anwendungen. Bei niedrigen Frequenzen bzw. ei-ner starken Miniaturisierung sind CMOS-Schal-tungen aufgrund ihrer geringeren Verlustleistungbesser geeignet.

3 Die Exclusive-Oder-Schaltung (XOR) gleicht derODER-Schaltung bis auf Q L bei A B H.Stellen Sie die XOR-Funktion aus Grundschaltun-gen her.

Lösung:Wie Abb. 472.1 a mit den Eingängen A und B unddem Ausgang S.

Seite

469

Seite

471

12 Elektronik und Festkörperphysik192

Page 193: Metzler Physik

12.4.3 CPU und Rechenschaltungen

1 Eine Spannungsquelle U lädt über einen Wider-stand R den Kondensator C auf. Von welchen Grö-ûen hängt die im Widerstand umgesetzte Energieab?

Lösung:Aus I(t)U

Reÿ

tRC , UR(t) R I(t) und

PR(t)UR(t) I(t) lässt sich PR(t)U2

Reÿ

2 tRC

herleiten. Das Integral von PR(t) ist die Energie

ER10

PR(t) 12 CU2

Die Energie hängt quadratisch von der Spannung Uab und ist unabhängig vom Widerstand R. Um dieEnergie zur reduzieren muss die Spannung U ver-ringert werden.

12.4.4 RAM und ROM

1 Ermitteln Sie für einen PC die Geschwindigkeit derDatenübertragung in MB=s von der CPU zumCPU-Cache, zum externen Cache, zum Hauptspei-cher und zur Festplatte und vergleichen Sie mit derjeweiligen Speicherkapazität.

Lösung:Typische Angaben entsprechend den aktuellenDatenblättern (siehe z. B. www.intel.com,www. asus.com.tw) sind:1000 MB=s von der CPU zum ersten Cache

100 MB=s vom ersten Cache zum zweiten Cache50 MB=s von zweiten Cache zum RAM10 MB=s vom RAM zur Festplatte

Speicherkapazität:Für den ersten Cache: 16 kBFür den zweiten Cache: 250 kBFür den Hauptspeicher: 128 MBFür die Festplatte: 10 GBDer gröûeren Speicherkapazität steht die geringereDatenübertragungsrate gegenüber.

Seite

473

Seite

475

12.4 Digitalelektronik 193

Page 194: Metzler Physik

13.1.1 Ionisierende Wirkung radioaktiver Strahlung

Zusatzaufgaben

1 Aus welchem Grunde ist der axiale Draht einesZählrohres im allgemeinen mit dem Pluspol derSpannungsquelle verbunden? Berücksichtigen Siebei Ihrer Überlegung die Form des Feldes und dieBedeutung der Elektronen.

Lösung:Bei einem im Auslösebereich arbeitenden Zählrohrruft jedes Strahlungsteilchen eine Entladung her-vor. Diese entsteht dadurch, dass die vom Strah-lungsteilchen bei der Ionisation des Zählrohrgaseserzeugten Elektronen im elektrischen Feld zwi-schen Zählrohrmantel und Zählrohrdraht beschleu-nigt werden, Energie aufnehmen und durch Ionisa-tion weitere Ladungsträger erzeugen. Die positivenIonen sind aufgrund ihrer geringeren Beweglich-keit weit weniger wirksam.Da der Draht sehr dünn ist, ist die elektrische Feld-stärke in der unmittelbaren Umgebung des Drahtessehr hoch. In diesen Raumbereich gelangen alle imZählrohr erzeugten Elektronen und können somitdurch die Energieaufnahme aus dem Feld zur Stoû-ionisation beitragen.

2 a) Erklären sie das Zustandekommen der Totzeiteines Zählrohres. Die Totzeit beginnt im Au-genblick des Anstiegs eines Spannungsimpul-ses, der durch die von einem Teilchen erzeug-

ten Ladungen entsteht, und endet, wenn nahezualle Ladungen abgeflossen sind.

b) Die Totzeit einer Zählapparatur betrage 10ÿ4 s.Die angezeigte Impulsrate sei 1200 sÿ1. Wieviel Impulse werden nicht angezeigt?

Lösung:a) Die im Zählrohr vorhandenen Elektronen er-

zeugen weitere Ladungsträger durch Stoûioni-sation in unmittelbarer Umgebung des positi-ven Zählrohrdrahtes. Aufgrund der geringerenBeweglichkeit der positiven Ionen bildet sichum den Draht eine positive Raumladungs-wolke. Werden nun von einem weiteren Strah-lungsteilchen im Raum zwischen der Zählrohr-wand und der positiven Wolke Ionen und Elekt-ronen erzeugt, so befinden sich diese in einemBereich sehr geringer Feldstärke und beeinflus-sen den Vorgang der Entladung nicht. DieserZustand bleibt bestehen, bis alle positiven La-dungsträger zur Zählrohrwand abgewandertsind.

b) Bei der Registrierung von 1200 Entladungspro-zessen in einer Sekunde ist das Zählrohr insge-samt 1200 ´ 10ÿ4 s 0,12 s nicht zählbereit. Essind also 1200 Prozesse in 0,88 s ausgelöst wor-den, wozu (1200=0,88 1364) Strahlungsteil-chen, die einen Prozeû ausgelöst hätten, einge-fallen sind. Demnach wurden 164 Impulse nichtangezeigt.

13.1.3 Eigenschaften der Strahlungen

1 Bei der vollständigen Abbremsung von a-Teilchenin Luft wird je 35 eV ein Ionenpaar erzeugt, obwohldie zur Ionisation erforderliche Energie nur 15 eVbeträgt.a) Welche Energie haben a-Teilchen, die bei ei-

nem Ionisationsvermögen von 2000 Ionenpaa-ren=mm eine Reichweite von 6,8 cm besitzen?

b) Welche anderen Prozesse auûer der Ionisationkönnen für den Energieverlust der a-Teilchenbeim Durchgang durch Luft verantwortlichsein?

c) Welche Sättigungsstromstärke ist in einer Ioni-sationskammer möglich, wenn ein Präparat proSekunde 108 a-Teilchen emittiert?

Lösung:a) E 2000 ´ 68 mm ´ 35 eV=mm 4,76 MeVb) Es findet Anregung von Atomen und Molekü-

len statt, wobei Elektronen der Hülle auf höhereEnergieniveaus gehoben werden.

c) I DQ=Dt108 68mm 2000 2 1,6 10ÿ19As=(s mm) 4,352 mA

2 Für die Reichweite von a-Teilchen in Gasen giltdie Formel R a v3 (R Reichweite, v Geschwin-digkeit). Für Luft ist a 9,67 ´ 10ÿ24 s3=m2. WelcheEnergie haben a-Teilchen eines Polonium-Präpa-rats, deren Reichweite in Luft 3,7 cm beträgt?

Lösung:v 1,56 ´ 107 m=s, Ekin 5,08 MeV

Seite

479

Seite

484

13 Kernphysik194

Page 195: Metzler Physik

3 Warum ist das Ionisationsvermögen für b-Teilchen(Elektronen) mit einer Energie von 8 MeV mit nur6,5 Ionenpaaren=mm so viel geringer als das vona-Teilchen mit gleicher Energie?

Lösung:Die Ionisation findet aufgrund der Coulomb-Wechselwirkung statt. Diese ist bei a-Teilchenmit ihrer doppelten Ladung verglichen mit dervon Elektronen und wegen ihrer geringeren Ge-schwindigkeit erheblich gröûer.

4 Bei der Untersuchung der b-Strahlung eines Ra-diumpräparats in einem Magnetfeld der StärkeB 2,25 ´ 10ÿ2 T ergibt sich eine Verschiebungdes Maximums der Impulsrate um 20. Die unterdiesem Winkel registrierten b-Teilchen bewegensich auf einer Kreisbahn mit dem Radiusr 11,7 cm.Berechnen Sie unter Berücksichtigung der relativi-stischen Massenveränderlichkeit die Geschwindig-keit der b-Teilchen.

Lösung:m v2=r evB, m0 v=

1ÿ v2=c2

p evB;

v er Bc2=m2

0 c2 e2 r2 B2p

2,5 ´ 108 m=s

5 Zur Ausfilterung von b-Teilchen mit einer be-stimmten Geschwindigkeit durchlaufen diese einWien-Filter aus zwei gekreuzten Feldern. Zur Er-zeugung des elektrischen Feldes wird eine Span-nung von U 2,8 kV an zwei Platten mit einemAbstand von d 0,9 mm gelegt. Senkrecht zumelektrischen Feld besteht ein Magnetfeld vonB 0,011 T.Welchen Radius hat eine Kreisbahn, auf der sichdie ausgefilterten b-Teilchen in einem weiterenMagnetfeld mit der Stärke B 0,036 T bewegen?Bei der Rechnung ist die relativistische Massenver-änderlichkeit zu berücksichtigen.

Lösung:eE evB, v E=B, v 2,83 108 m=s.

Aus mv er B, m gm0 und g 3folgt r 0,13 m.

*6 b-Teilchen eines Präparats werden radial in eineflache zylindrische Kammer geschossen, die voneinem regelbaren Magnetfeld durchsetzt ist.a) Zeigen Sie, dass diese Anordnung zur Auf-

nahme eines Energiespektrums geeignet ist.b) Durch ein Geschwindigkeitsfilter ausgeson-

derte Teilchen von v 0,6 c durchlaufen dieAnordnung bei B 6,85 mT. Berechnen Siedie spezifische Ladung dieser Teilchen.

Lösung:a) Ein Energiespektrum ist eine Darstellung der

Intensität in Abhängigkeit von der Energie.Im Magnetfeld B durchlaufen die b-Teilcheneinen Kreisbogen vom Radius R mitr=R tan(a=2). Für den Impuls p der Teilchengilt p m v e R B. Also gehören zu jedermagnetischen Feldstärke B Teilchen mit be-stimmtem Impuls p. Die Energie der Teilchenergibt sich aus E2ÿ p2 c2 m2

0 c4.

r

Rα2

α = 30°

b) e=m v=(RB) mit mm0=1ÿ v2=c2

pfolgt

e=m v=RB1ÿ v2=c2

p 1,76 1011 As=kg

13.1.4 Strahlungsdetektoren

1 Zwischen Halbleiterdetektoren und Ionisations-kammern besteht eine gewisse Analogie: Alle ent-stehenden Ladungsträger werden auf den Elektro-den gesammelt. Worin bestehen die Unterschiededer beiden Nachweisinstrumente und welche Fol-gerungen ergeben sich daraus?

Lösung:Die Dichte ist sehr verschieden, so dass imFestkörper die Strahlung völlig absorbiert werdenkann. Pro erzeugtes Ladungsträgerpaar werdenim Halbleiter nur ca. 3 eV, im Gas jedoch ca.35 eV benötigt, worauf die gröûere Energieauf-lösung der Halbleiterdetektoren beruht. Daher wer-

Seite

487

13.1 Natürliche Radioaktivität 195

Page 196: Metzler Physik

den Halbleiterdetektoren als Spektrometer einge-setzt.

2 Detektoren sind Geräte, die nur zum Nachweis vonTeilchen geeignet sind. Als Spektrometer bezeich-net man Geräte, die es ermöglichen, spezielle Ei-genschaften der Teilchen, wie z. B. Masse, Energieoder Impuls zu bestimmen. Überlegen Sie, welcheder bisher besprochenen Geräte bzw. Versuchsan-ordnungen als Detektoren, welche als Spektrome-ter anzusehen sind.

Lösung:Detektoren: GM-Zählrohr, IonisationskammerSpektrometer: Anordnungen zur Ablenkung im

Magnetfeld, Szintillationszähler,Halbleiterdetektor

3 WasverstehtmanunterdemEnergieauflösungsver-mögen eines Detektors? Worauf beruht das höhereAuflösungsvermögen eines Halbleiterdetektors?

Lösung:Das Energieauflösungsvermögen ist der QuotientDE=E, wobei DE die Breite des Fotopeaks in hal-ber Höhe ist. Das höhere Auflösungsvermögen ei-nes Halbleiterdetektors beruht darauf, dass zur Er-zeugung eines Ladungsträgerpaares nur der kleineEnergiebetrag von ca. 3 eV nötig ist und ferner alleerzeugten Ladungsträger auch registriert werden.

*4 Monoenergetische a-Strahlung erzeugt in einemSzintillationszähler einen Photopeak (Abb. 485.2)und ein kontinuierliches Spektrum, das auf der Ent-stehung von Elektronen durch den Compton-Effektberuht. Welche Energie können g-Quanten von1 MeV höchstens auf Elektronen im Compton-Effekt übertragen?

Lösung:Für den Compton-Effekt gilt:

1f 0ÿ 1

f h

m0e c2(1ÿ cosò).

Für ò 180 ergibt sich:

1h f 0ÿ 1

h f 2

m0e c2.

Umformungen liefern:

h f 0 h f1

1 2h fm0e c2

DE h f ÿ h f 0 h f

1ÿ 1=

1 2 h f

m0e c2

DE h f 0,8 0,8 MeV.

*5 Ein Americium-Präparat emittiert auûer den a-Teilchen mit den in Tab. 487.2 angegebenen Ener-gieng-QuantenderEnergien0,026 eV;0,033 MeV;0,043 MeV; 0,0595 MeV; 0,099 MeV; 0,103 MeVund 0,125 MeV. Wie ist diese Strahlung zu er-klären?

Lösung:Durch Emission von a-Teilchen wandelt sichAm241 in Np237 um. Dabei entstehen nicht nurNeptuniumkerne im Grundzustand, sondern auchin höheren Energiezuständen. Entsprechend denin Tab. 487.2 angegebenen Energien der a-Teil-chen existieren fünf verschiedene Energiezuständedes Neptuniumkerns. Beim Übergang von einemZustand zu einem niedrigeren wird dann ein g-Quant emittiert.

0,125 MeV0,099 MeV

0,103 MeV

0,043 MeV0,595 MeV

0,026 MeV 0,033 MeVNp 237 Am 241

5,38

7 M

eV5,

442

MeV

5,48

4 M

eV

5,51

1 M

eV5,

543

MeV

α1 2 3 4 5α α α α

γ1

2

3

4

5

γ

γ

γ

γ 6

7

γ

γ

13.1.5 Wechselwirkung von Strahlung mit Materie

1 Wie erklärt man, dass das spezifische Ionisations-vermögen von a-Teilchen längs ihrer Bahnzunimmt und kurz vor ihrem Ende ein Maximumerreicht?

Lösung:Mit dem zurückgelegten Weg verringert sich auf-grund ihrer Wechselwirkung mit der Materie dieGeschwindigkeit der a-Teilchen, sodass sie sichzunehmend länger in der Nähe eines zu ionisieren-den Atoms oder Moleküls befinden und auf dieseüber die Coulomb-Kraft einwirken können.

Seite

491

13 Kernphysik196

Page 197: Metzler Physik

2 Woran liegt es, dass auch b-Teilchen einheitlicherEnergie keine einheitliche Reichweite besitzen?

Lösung:Die b-Teilchen werden aufgrund ihrer geringenMasse bei Wechselwirkungen mit Atomen oderMolekülen stark aus ihrer ursprünglichen Bewe-gungsrichtung abgelenkt und in verschiedeneRichtungen gestreut.

3 Wieso wächst die Absorption von g-Strahlung mitder Ordnungszahl des Materials?

Lösung:Die Absorption von a-Strahlung beruht u. a. aufihrer Wechselwirkung mit Elektronen (Fotoeffekt,Compton-Effekt). Mit zunehmender Ordnungszahlder Atome des Materials wächst auch die Dichteder Elektronen und damit die Wahrscheinlichkeitder Wechselwirkung.

4 Die Halbwertsdicke für Röntgenstrahlen der Wel-lenlänge 12 pm ist bei Aluminium dH 1,44 cm.a) Welchen Wert hat der Schwächungskoeffizi-

ent?b) Welcher Bruchteil der Strahlung durchdringt

eine Aluminiumschicht von 2,5 cm Dicke?

Lösung:a) n=2 n eÿk ´ 1,44 cm, k 0,48 cmÿ1

b) n n0 eÿk ´ 2,5 cm, n 0,3 n0

*5 Für den Energieverlust DE geladener Teilchenbeim Durchgang durch Materie längs der StreckeDx gilt: DE=Dx ist proportional zur Dichte ne derElektronen im Stoff.Zeigen Sie, dass für Materie der Dichte r, die ausAtomen der Ordnungszahl Z besteht, der Zusam-menhang ne (Z=A) r NA gilt und dass der Ener-gieverlust DE pro r Dx für alle Stoffe annäherndkonstant ist.

Lösung:Für die Dichte der Elektronen gilt ne Z ´ nA, wennnA die Dichte der Atome mit nA NA=Vmol ist. DasMolvolumen ergibt sich zu Vmol A=r, sodass gilt:nA (NA ´ r)=A.Wenn DE=Dx proportional zu ne ist undne Z ´ (NA ´ r)=A ist, so ist die Behauptung be-wiesen.

Zusatzaufgaben

6 Bei einem Röntgengerät wird ein mit Luft gefüllterKondensator durchstrahlt. Das durchstrahlte Volu-men beträgt 10 cm3 , die Stromstärke ist 6 ´ 10ÿ10 A.Berechnen Sie die Ionendosisleitung (Ionendo-sis=Zeit) in den Einheiten A=kg und Röntgen=sund die Energiedosisleistung mit Hilfe der Bezie-hung D (34,1 Gy=(C=kg)) ´ Dion.

Lösung:Masse des durchstrahlten Volumensm 1,293 ´ 10ÿ6 kg=cm3 ´ 10 cm3 1,293 ´ 10ÿ5 kg,in der Sekunde erzeugte Ladung eines VorzeichensQ 3 ´ 10ÿ10 C,IonendosisleistungDion=Dt 3 ´ 10ÿ10 C=(1,293 ´ 10ÿ5 kg ´ s)

2,32 ´ 10ÿ5 A=kg 0,09 R=sEnergiedosisleistungD=Dt 2,32 ´ 10ÿ5 A=kg ´ 34,1 Gy=(As=kg)

7,9 ´ 10ÿ4 Gy=s

7 Worüber macht die ¾quivalentdosis eine Aussage?

Lösung:Die ¾quivalentdosis charakterisiert die pro Zeit-einheit von einer Strahlung verursachte Schädi-gung.

13.1 Natürliche Radioaktivität 197

Page 198: Metzler Physik

13.2.2 Proton und Neutron als Kernbausteine

1 Aus Streuversuchen ergaben sich folgendeKernradien: rHe 3 ´ 10ÿ15 m; rO 4,6 ´ 10ÿ15 m;rSr 7 ´ 10ÿ15 m; rSb 7,8 ´ 10ÿ15 m; rAu 8,5 ´ 10ÿ15 m und rBi 8,9 ´ 10ÿ15 m.a) Untersuchen Sie, ob der Radius der Kerne pro-

portional zur dritten Wurzel aus der Massen-zahl ist.

b) Bestimmen Sie aus der Gleichung r r0 A1=3

den Radius eines Nukleons.

Lösung:a) und b) Zur Untersuchung der Proportionalitätbildet man den Quotienten r=A1=3 r0. Es ergebensich folgende Werte für r0 in 10ÿ15 m: 1,89; 1,83;1,58; 1,57; 1,46; 1,50. Also liegt näherungsweiseProportionalität vor.

2 Beim Beschuss von Goldfolie mit a-Teilchender Energie Ekin 5,49 MeV werden einigea-Teilchen um 180 gestreut. Da a-Teilchen dieserEnergie noch nicht in den Wirkungsbereich derKernkräfte geraten, kann man den Radius desGoldkerns abschätzen, indem man den kürzestenAbstand der a-Teilchen vom Kern berechnet.Bestimmen Sie den Abstand, in dem die gesamtekinetische Energie in potentielle Energie umge-wandelt ist.

Lösung:Aus Ekin Epot 1=(4 pe0) Q1 Q2=rfolgt r 41 10ÿ15 m

*3 Um zu zeigen, dass sich in einem Atomkern keineElektronen aufhalten können, vergleicht man diepotentielle Energie eines Elektrons, das sich imAbstand r0 2 ´ 10ÿ15 m um ein Nukleon bewegt,mit der kinetischen Energie eines Elektrons. Diesebestimmt man mit Hilfe der Welleneigenschaftendes Elektrons, das sich in einem linearen Potential-topf der Länge 2 r0 bewegt. Für Systeme aus zweiKörpern, zwischen denen eine anziehende Kraftexistiert, gilt Epot ±2 Ekin.Bestimmen Sie die potentielle Energie und überden Impuls unter Anwendung der relativistischenImpuls-Energie-Beziehung die kinetische Energiedes Elektrons und vergleichen Sie die Energien.

Lösung:Aus Epot 1=(4 p e0) e2=r folgt Epot 0,72 MeV.Die kinetische Energie ergibt sich aus Ekin E ±ERuh und E (p2 c2 ± m2

0 c4)1=2 mit p h=l undl=2 2 r0 zu Ekin 154,6 MeV.Es ist also keinesfalls Epot 2 Ekin, woraus folgt,

dass sich keine Elektronen im Kern aufhaltenkönnen.

*4 Neutronen übertragen bei einem zentralen Stoû mitWasserstoff- bzw. Stickstoffatomen diesen einenTeil ihrer kinetischen Energie. Aus den dabei ineiner Nebelkammer entstehenden Spuren lässt sichdiese zu EH 5,7 MeV und EN 1,6 MeV bestim-men.a) Stellen Sie unter der Annahme eines zentralen

Stoûes für den Stoû der Neutronen mit Wasser-stoff bzw. mit Stickstoff unter Anwendung derErhaltungssätze für Impuls und Energie zweiGleichungssysteme auf.

b) Lösen Sie diese Gleichungssysteme unter Ver-wendung der (nicht-relativistischen) Impuls-Energie-Beziehung nach den unbekanntenkinetischen Energien der Neutronen auf und be-stimmen Sie unter Verwendung der angebenenMesswerte die Masse der Neutronen.

Lösung:Stoû mit H:Ekin E0kinEH, p p0 pH;Stoû mit N:Ekin E00kinEN, p p00 pN;Aus Ekin p2=2 m; E0kin p02=2m(pÿ pH)2=2 m folgt:p pH p2

H=2EH m und mitp2

H 2 mH EH;p EH (mHm)=

(2mH EH)p

.Daraus folgt Ekin EH (mHm)2=(4 mmH).Ebenso ergibt sich Ekin EN (mN + m)2=(4 m mN).Aus den beiden Termen für Ekin ergibt sich m (mH

mNEHp

± mNmHENp

)=(mHENp ÿ

mNEHp

)und daraus mit mH 1 u, mN 14 u,EH 5,7 MeV und EN 1,6 MeV die Masse desstoûenden Teilchens zu m 1,14 u.

Zusatzaufgaben

5. Die atomare Masseneinheit u ist als 112 der

Masse eines Atoms des Kohlenstoffisotops 12Cdefiniert. Andererseits ist bekannt, dass die Zahlder Teilchen in 1 kmol eines Elements NA 6,02 ´ 1026 kmolÿ1 ist. Daraus berechnet sich dieatomare Masseneinheit zu u 1=NA kg=kmol.a) Wie viele Nukleonen bzw. Protonen enthalten

1 g 12C?

Seite

495

13 Kernphysik198

Page 199: Metzler Physik

b) Wie viele Neutronen gibt es in 1 g 1H, 1 g 16O,1 g 235U?

Lösung:a) 1 g 12C enthält 5,02 ´ 1022 Kohlenstoffatome,

also 6,02 ´ 1023 Nukleonen und 3,01 ´ 1023 Pro-tonen.

b) 1 g 1H enthält keine Neutronen, 1 g 16O enthält3,01 ´ 1023 und 1 g 235U enthält 3,66 ´ 1023 Neut-ronen.

6. Bestimmen Sie für die Kerne aus Aufgabe 1 dieDichten.

Lösung:Die Dichte ergibt sich zu rm=V mit V 4=3 p r3.m und V sind Masse und Volumen des Kerns.Es ergeben sich folgende Dichten in 1016 kg=m3:He: 5,8; O: 6,5; Sr: 10,1; Sb: 10,2; Au: 12,7;Bi: 11,7.

13.2 Aufbau und Systematik der Atomkerne 199

Page 200: Metzler Physik

13.3.1 Die natürlichen Zerfallsreihen

1 a) Stellen Sie mit Hilfe der Nuklidkarte aus demAnhang die Zerfallsreihe für Americium 241auf.

b) Stellen Sie mit Hilfe der Nuklidkarte aus demAnhang die Zerfallsreihe für Radium 226 auf.

c) Warum kann kein Nuklid gleichzeitig zwei ver-schiedenen Zerfallsreihen angehören?

Lösung:a) 241Am!a 237Np!a 233Pa!b 233U!a 229Th!a

225 Ra!b 225Ac!a 221Fr!a 217At!a 213Bi!b213 Po!a 209Pb!b 209Bi

b) 226Ra!a 222Rn!a 218Po!a 214Pb!b 214Bi!b214Po!a 210Pb!b 210Bi!b 210Po!a 206Pb

c) Die Massenzahlen der Nuklide einer Zerfalls-reihe genügen alle derselben Formel 4 n k mitk 0, 1, 2, 3. Es kann jedoch nie 4 n k 4 m l sein mit k, l 2 0, 1, 2, 3 und k 6 l, denn 4teilt nie (k ÿ l).

2 Welches Isotop entsteht aus Thorium 232 nach viera- und zwei b-Zerfällen?

Lösung:216Po

3 Worin liegt die Ursache dafür, dass die Elementeder Neptuniumreihe in der Natur nicht mehr gefun-den werden?

Lösung:Die Halbwertszeit der Elemente der Neptunium-reihe ist verglichen mit dem Alter der Erde klein,sodass bereits alle Elemente der Reihe zerfallensind.

*4 Erklären Sie unter Verwendung von Abb. 497.1,warum in den Zerfallsreihen b-Zerfälle vorkom-men.

Lösung:Beim a-Zerfall werden stets zwei Protonen undzwei Neutronen emittiert, sodass der relative Neut-ronenanteil des Tochterkerns gröûer ist als der desMutterkerns. Der Neutronenüberschuss kann nurdurch b-Zerfall abgebaut werden.

13.3.2 Gesetz des radioaktiven Zerfalls

1 Bei der Umwandlung von Protaktinium 234 inUran 234 unter b-Emission wurden folgende be-reits um die Nullrate bereinigte Zählraten ge-messen:

t in s 30 60 90 120 150 180 210 240 270

n in 110 s 236 173 128 98 71 56 38 30 19

a) Bestimmen Sie mit einer graphischen Darstel-lung die Zerfallskonstante l und die Halbwerts-zeit.

b) Berechnen Sie Zählraten nach 6 min. Nachwelcher Zeit beträgt die Zählrate nur noch 1

10bzw. 1

100 ihres Anfangswertes?

Lösung:a)

ln (n)

100

1

2

3

4

5

6

050 150 200 250 300 t0

l 0,01 sÿ1, tH 68,1 s

b) n(6 min) 12,7=(10 s), t 213 s, t 427 s

2 Welcher Bruchteil einer Menge 226Ra mittH 1600 a ist nach 10 Jahren noch nicht zerfallen?

Lösung:n=n0 eÿln2=tH t, n=n0 99,6%

3 Wie viel Blei ist seit Bestehen der Erde (4,55 ´ 109a)aus 1 kg 238U mit tH 4,5 ´ 109 a entstanden?

Lösung:238 g Uran 6,02 ´ 1023 Teilchen,1 kg Uran 2,53 ´ 1024 Teilchen,Zahl der zerfallenen Kerne:n0 ± n n0(1 ± eÿ(ln2=tH) t) 1,27 ´ 1024 2,12 mol(Kerne 206Pb), mPb 436 g

4 Aus der Messung der Masse und der Aktivität lässtsich die Halbwertszeit der sehr langlebigen Sub-stanz 232Th zu tH 1,39 ´ 1010 a bestimmen.Wie groû ist die Zahl der in einer Sekunde zerfal-lenden Kerne bei einer Thoriummasse von 1 g?

Lösung:A l n (ln 2=tH) n 4103 sÿ1

Seite

499

Seite

501

13 Kernphysik200

Page 201: Metzler Physik

5 a) Berechnen Sie die Masse eines Americium 241-Präparats mit einer Aktivität von 333 kBq.

b) Welche Aktivität haben 2 mg 210Po (tH 138 d)?

Lösung:a) n A=(ln 2=tH) 6,56 ´ 1015

1,09 ´ 10-8 mol 2,63 mgb) A (ln 2=tH) n 333 MBq

*6 Radon220 entsteht durch a-Zerfall aus Radium24und zerfällt in Polonium216.a) Zeigen Sie an diesem Beispiel, dass in einer

Zerfallsreihe mit den Kernen (1), (2) und (3)für die Anzahlen N1, N2 und N3 die Beziehungl1 N1 l2 N2 l3 N3 gilt.

b) Bestimmen Sie unter Anwendung der inAufgabenteil a) gegebenen Formel, welcheRadiummenge in einer alten Lagerstätte auf1 g Uran238 entfällt (tH, U238 4,5 ´ 109 a,tH, Ra226 1600 a).

Lösung:a) In der Zeit Dt besteht für die zerfallenden

Kerne Radon und Radium Gleichgewicht, alsoDNRa DNRn. Mit DN l N Dt folgtlRa NRa lRn NRn usw.

b) Da Gleichgewicht besteht, giltNRa (NU tH, Ra)=tH, U, mRa 0,34 mg

Zusatzaufgaben

7. Natürliches Uranerz enthält ein Radiumatom auf2,79 ´ 106 Uranatome. Berechnen Sie die Halb-wertszeit des Urans, wenn die Halbwertszeit desRadium 1600 a beträgt.

Lösung:Nach der Gleichgewichtsbedingung ergibt sichtH 4,46 ´ 109 a.

13.3 Kernumwandlung und radioaktive Strahlung 201

Page 202: Metzler Physik

13.4.1 Massendefekt und Bindungsenergie

1 a) Berechnen Sie die Bindungsenergie der Nuk-lide Tritium 3H und Helium 3He. EntnehmenSie dazu die Massen aus der Tabelle der Nuk-lide im Anhang.

b) Erklären Sie das Ergebnis des Aufgabenteils a).

Lösung:Dm Z (mp me) mn ± mA, EB Dm c2

a) 3H: Dm 9,1 ´ 10ÿ3 u, EB 8,48 MeV;3He: Dm 8,3 ´ 10ÿ3 u, EB 7,72 MeV

b) Die Bindungsenergie des Heliumkerns ist ge-ringer als die des Tritiumkerns, weil der He-liumkern zwei Protonen enthält. Diese besitzenzusammen potentielle elektrische Energie,sodass beim Zusammenfügen der Nukleonenweniger Energie frei wird.

2 Berechnen Sie für folgende Nuklide die Bindungs-energie pro Nukleon nach dem Massendefekt: 14N,

16O, 20Ne, 24Mg und 28Si. Entnehmen Sie die Mas-sen der Nuklidtabelle (! S. 577).

Lösung:Dm Z (mpme)mnÿmA, EB Dmc2

EB=A(14N) 7,48 MeV;EB=A(16O) 7,98 MeV;EB=A(20Ne) 8,03 MeV;EB=A(24Mg) 8,26 MeV;EB=A(28Si) 8,45 MeV

3 Berechnen Sie die Bindungsenergie des zweitenNeutrons eines 4He-Kerns aus den Massen der be-teiligten Teilchen.

Lösung:Dmm (3He)mnÿ 2 meÿ (m (4He)ÿ 2 me),EB 20,6 MeV

13.4.2 Das Tröpfchenmodell des Atomkerns

1 In einem Heliumkern haben die Nukleonen vonei-nander einen mittleren Abstand von 10ÿ14 m.a) Wie groû sind die abstoûenden elektrischen

Kräfte zwischen zwei benachbarten Nukleo-nen?

b) Wie groû sind die Gravitationskräfte zwischenden Teilchen?

Lösung:Fab e2=(4 p e0 r2) 2,3 N;Fan g m2=r2 1,8 ´ 10ÿ36 N

2 Berechnen Sie für die Nuklide 6Li, 7Li, 12C, 17O dieBindungsenergien pro Nukleon nach der FormelEB=A 14 MeVÿ 13 MeV Aÿ1=3ÿ 0,12 MeV A2=3.

Lösung:6Li: 6,45 MeV; 7Li: 6,76 MeV;12C: 7,69 MeV; 17O: 8,15 MeV

*3 a) Berechnen Sie die Bindungsenergie pro Nuk-leon für 235U mit dem Massendefekt auf zwei-erlei Weise: einmal, indem Sie sich den Kernaus einzelnen Nukleonen zusammengesetztdenken, und zum anderen, indem Sie sich denUrankern durch den Einbau weiterer Nukleo-nen aus einem Kern des Bleiisotops 207Pb ent-standen denken.

b) Wie sind die unterschiedlichen Ergebnisse zuerklären?

Lösung:a) Bei Zusammenbau des Kerns aus einzelnen

Nukleonen ergibt sich EB=A 7,59 MeV, denktman sich den Urankern aus dem 207Pb-Kern und28 weiteren Nukleonen aufgebaut, so ergibtsich EB=A 5,53 MeV.

b) Der erste Wert gibt die mittlere Bindungsener-gie pro Nukleon an, ohne zu berücksichtigen,wo die Nukleonen im Kern liegen. Beim zwei-ten Wert geht in die Berechnung die Masse desBleikerns ein, in der sich die auf der abstoûen-den Coulomb-Kraft zwischen den Protonen be-ruhende potentielle elektrische Energie starkbemerkbar macht. Die Rechnung ergibt, dassbeim Einbau der letzten Nukleonen eines gro-ûen Kerns im Mittel ein geringerer Wert derBindungsenergie frei wird, als wenn man dasMittel über die Gesamtzahl der Nukleonen bil-det.

*4 Wenn ein Atomkern ein a-Teilchen emittiert, soverwandelt er sich in einen Kern mit einer jeweilsum 2 niedrigeren Zahl von Protonen und Neutro-nen. Dabei stellt man sich vor, dass sich das a-Teil-chen vor seiner Emission im Kern als eigenständi-ges Teilchen gebildet hat.a) Berechnen Sie die Bindungsenergie bzw. die

Separationsenergie für ein a-Teilchen, das

Seite

503

Seite

505

13 Kernphysik202

Page 203: Metzler Physik

von einem 220Rn-Kern emittiert wird, nach derFormelEB (a) EB(N, Z)ÿEB(Nÿ 2,Zÿ 2)

ÿEB(N 2,Z 2).b) Geben Sie eine Begründung für die Formel.c) Unter welcher Bedingung nur kann von einem

Kern ein a-Teilchen emittiert werden?

Lösung:a)EB(a) EB(134,86)ÿEB(132,84)ÿEB(2,2)

1697,8 MeVÿ 1675,9 MeVÿ 28,3 MeVÿ6,42 MeV

b) Um ein a-Teilchen von einem Kern aus N Neut-ronen und Z Protonen abzutrennen, ist Energienötig. Diese Energie ist gleich der Differenz derBindungsenergien des Ausgangskerns EB(N, Z)und der Summen der entstehenden KerneEB(Nÿ 2, Zÿ 2)ÿEB(N 2, Z 2). Wenndiese Differenz EB(N, Z)ÿ EB(Nÿ 2, Zÿ 2)ÿEB(N 2, Z 2) negativ ist, wird zusammenmit der a-Emission Energie frei.

c) Ein Kern emittiert nur dann a-Teilchen, wenn

die Differenz der Bindungsenergien (s. Auf-gabe b)) negativ ist, das bedeutet, dass dieMasse des Ausgangskerns gröûer sein mussals die Summe der Massen der entstehendenKerne.

*5 Suchen Sie für die Bindungsenergie pro Nukleoneinen von der Massenzahl A abhängigen Term un-ter der Annahme, dass jedes Nukleon im Kern mitjedem anderen wechselwirkt.

Lösung:Die mittlere Bindungsenergie pro Nukleon ist mitungefähr 8 MeV ungefähr konstant.Wenn jedoch in einem Kern mit A Nukleonen einNukleon mit allen anderen (A ± 1) Nukleonenwechselwirkte, würde die Bindungsenergie pro-portional zu (A ± 1) sein. Da dies für alle A Nukleo-nen gälte, wäre die Bindungsenergie proportionalzu A(A ± 1) und die Bindungsenergie pro Nukleonproportional zu (A ± 1). Da dies nicht der Fall ist,wechselwirken nicht alle in einem Kern vorhande-nen Nukleonen miteinander.

13.4.5 Der a-Zerfall

1 Welche kinetische Energie hat ein a-Teilchen, dasaus einem 226Ra-Kern emittiert wird?

Lösung:Mit den Werten mRa 226 226,0254 u undmRa 222 222,0176 u ergibt sich Ekin 4,94 MeV.

2 Das von einem 235U-Kern emittierte a-Teilchen hateine kinetische Energie von 4,58 MeV. WelcheMasse hat der entstehende 231Th-Kern?

Lösung:Masse des Thorium 231-Kerns: 230,987 u.Masse des Thorium 231-Atoms: 231,036 u.

*3 Schätzen Sie mit Hilfe der Unschärferelation dieGröûenordnung der kinetischen Energie eines Teil-chens in einem Atomkern ab.

Lösung:Dp Dx h, Dp 2 p, Dx 10ÿ14 m, alsop h=(2 Dx) 3,3 ´ 10ÿ20 kgm=s,Ekin p2=(2m) 2,1 MeV.

*4 Die folgende Tabelle enthält Energien von a-Teil-chen und die Halbwertszeiten von Elementen, diedem a-Zerfall unterliegen:

Element Energie der a-Teilchenin MeV

Halbwertszeit

216Rn218Rn220Rn222Rn

8,057,126,295,49

45 ms30 ms55,6 s3,824 d

219Rn222Ra225Th

6,556,566,5

4 s38 s8 min

Alle Nuklide im oberen Teil der Tabelle habengleiche Kernladungszahl, dagegen nimmt dieKernladungszahl der drei Nuklide im unteren Teilder Tabelle zu. Wie erklärt man die unterschiedli-chen Halbwertszeiten unter Anwendung des Poten-tialtopfmodells des Atomkerns?

Lösung:Die ersten vier Nuklide haben aufgrund der glei-chen Kernladungszahl für ein a-Teilchen einengleichen Potentialtopf. Diesen Topf kann das a-Teilchen im Tunnel-Effekt verlassen. Die Breitedes zu durchtunnelnden Walls ist jedoch um so ge-ringer, je gröûer die kinetische Energie des a-Teil-chens ist. Also nimmt die Zerfallswahrscheinlich-keit mit der Energie zu und die Halbwertzeit ab.

Seite

508

13.4 Energie der Atomkerne; Kernmodelle 203

Page 204: Metzler Physik

Die drei letzten Kerne emittieren a-Teilchen glei-cher Energie, jedoch nimmt bei diesen Kernen dieKernladungszahl und die Höhe des Potentialwalls

zu, sodass dementsprechend die Zerfallswahr-scheinlichkeit ab- bzw. die Halbwertszeit zu-nimmt.

13 Kernphysik204

Page 205: Metzler Physik

13.5.1 Kernreaktionen

1 Beim Beschuss von Li-Kernen mit Deuteronen fin-den folgende Reaktionen statt:63Li2

1 H!74 Be1

0 n, 63Li2

1 H!42 He3

2 H10 n

a) Welche Energiebeträge werden frei ohne Be-rücksichtigung der kinetischen Energie der Ge-schosse?

b) Warum sind die Energiebeträge verschieden?

Lösung:a) E1 3,36 MeV, E2 1,8 MeVb) Je mehr Nukleonen miteinander in Wechsel-

wirkung treten, um so gröûer ist bei kleinenKernen die mittlere Bindungsenergie pro Nuk-leon. Die Bindungsenergie des Be-Kerns istgröûer als die Summe der Bindungsenergiender beiden He-Kerne.

2 Wie groû muss die Mindestenergie der g-Quantensein, damit die angegebenen Reaktionen ablaufen?a) 7

3Li!63 Li1

0 n,b) 9

4Be! 242He1

0 n

Lösung:a) E 7,25 MeVb) E 1,58 MeV

3 a) Wie hoch ist die Mindestenergie, die zur Ab-spaltung eines Protons bzw. eines Neutrons voneinem 4He-Kern erforderlich ist?

b) Warum sind die Energien verschieden?

Lösung:a) Zur Abspaltung eines Neutrons werden 20,6

MeV benötigt, zur Abspaltung eines Protons19,3 MeV.

b) Die Energie zur Abspaltung eines Protons istwegen der elektrostatischen Abstoûung gerin-ger.

*4 Beim Stoû eines Deuterons der Energie E 1,2 MeV mit einem ruhenden Deuteron findet dieReaktion 2

1H 21 H! 3

1 H 11 p statt. Dabei stehen

die Bewegungsrichtungen des stoûenden Deute-rons und des entstehenden Protons senkrecht auf-einander.a) Wie groû ist die Reaktionsenergie?b) Wie groû ist die kinetische Energie des Pro-

tons?

Lösung:

pD

PT

pp

a) Aus dem Massendefekt ergibt sich die Reak-tionsenergie ER 4,54 MeV.

b) Die Reaktionsenergie und die kinetische Ener-gie des Deuterons verteilen sich als kinetischeEnergie auf das Proton und den TritiumkernETEp 1,2 MeV 4,54 MeV.

Für die Impulse der an der Reaktion beteiligtenTeilchen gilt p2

T p2D p2

p.Unter Verwendung der Beziehung p2 2mEkin

ergibt sich 2mT ET 2mD ED 2mp Ep mit ED 1,2 MeV.Bei Verwendung der Massen der Teilchen ergibtsich Ep 2,96 MeV.

13.5.2 Künstliche Radioaktivität

1 Natürliches Silber besteht zu 52% aus dem Isotop107Ag und zu 48% aus 109Ag. Ein Silberzylinderwird in einer Neutronenquelle bestrahlt. Anschlie-ûend wird das Abklingen der Aktivität untersucht(Nulleffekt : 24 minÿ1):

t in s 0 10 20 30 40

n 0 240 430 600 730

t in s 50 60 70 80 90

n 850 950 1040 1120 1190

t in s 100 110 120 150 180

n 1250 1310 1360 1492 1609

t in s 210 240 270

n 1711 1801 1882

a) Erstellen Sie aus der Messreihe eine Tabelle fürdie Impulsrate in Abhängigkeit von der Zeit,und stellen Sie den Logarithmus der Impulsratein Abhängigkeit von der Zeit graphisch dar.

b) Bestimmen Sie unter Berücksichtigung derTatsache, dass sich hier die Zerfallsvorgänge

Seite

510

Seite

512

13.5 Künstliche Kernumwandlung und Kernspaltung 205

Page 206: Metzler Physik

zweier Nuklide überlagern und dass das Isotop108Ag schneller zerfällt als 110Ag, die Halb-wertszeiten beider Isotope.

c) Stellen Sie die für die Aktivierung und für denZerfall geltenden Reaktionsgleichungen auf.

Lösung:a) Durch Differenzbildung ergeben sich die Im-

pulsraten zu den jeweiligen Intervallmitten:

t in s 5 15 25 35 45 55 65 75

Dn=Dtin (10 s)ÿ1

240 190 170 130 120 100 90 80

t in s 85 95 105 115 135 165 195 225 255

Dn=Dtin

(10 s)ÿ1

70 60 60 50 44 39 34 30 27

Unter Berücksichtigung des Nulleffekts von4=(10 s) ergeben sich die Zählraten, die halb-logarithmisch dargestellt sind.

100

ln ∆nt

1(10 s)in∆

5+ + + + + + + +

++++++++++

1234

6

00 200 t in s300

10847

11047

Ag

Ag

b) Verbindet man die Punkte, die sich aus der Dar-stellung der Wertepaare mit den höheren Zeitenergeben, so ergibt sich eine Gerade. Extrapo-liert man diese für die niedrigen Zeiten, so kannman die Zählraten entnehmen, die allein auf-grund des Zerfalls von 108Ag entstehen unddiese von den Gesamtwerten subtrahieren. Esergeben sich die Zählraten für das Silberisotop110Ag (siehe Abb.) und eine zweite Gerade. Ausden Geraden entnimmt man die Zerfallskon-stanten. Es ergeben sich die Halbwertzeitenfür 108Ag: TH 142 s und für 110Ag: TH 34 s.

c) 10747Ag 1

0 n!10847 Agÿ!b

ÿ10848Cd

10947Ag 1

0 n!11047 Agÿ!b

ÿ10848Cd.

2 22Na ist ein Positronenstrahler.a) Berechnen Sie die Mindestenergie, die zur Um-

wandlung eines Protons in ein Neutron und einPositron zur Verfügung stehen muss.

b) Sollte der Überschuss an Protonen durch Emis-sion eines Protons abgebaut werden, so müsste

sich 22Na in 21Ne umwandeln. Bestimmen Siedie Differenz der Bindungsenergien dieser bei-den Nuklide (mNa22 21,9944 u, mNe2120,99384 u). Welche Bedeutung hat dieserWert, und was folgt daraus für diesen Prozess?

Lösung:a) 1

1p! 20 n 0

1 e v, E 1,8 MeVb) EB(22Na) 174,14 MeV;

EB(22Ne) 167,41 MeV;DEB 6,73 MeV.Diese Energie müsste zugeführt werden, um einProton aus dem Kern zu lösen. Es ist die Sepa-rationsenergie für ein Proton. Die Emission ei-nes Positrons ist also energetisch günstiger.

3 Mit energiereichen g-Quanten können Elektron-Proton-Paare erzeugt werden. Zeigen Sie, dass un-ter ausschlieûlicher Beteiligung eines g-Quantsund der beiden erzeugten Teilchen nicht zugleichder Energie- und der Impulserhaltungssatz erfülltsein können. (Bei der Rechnung ist der Zusammen-hang E2 (p c)2 m0 c2 zu verwenden.).

Lösung:

pe–

+

hfc α

pe

Ep2 c2m2

0 c4p

Energiesatz: h f Eeÿ Ee ,

Impulssatz:h f!

c p!eÿ p!e

c2 p2e (h f )2 c2 p2

eÿ ÿ 2ch f peÿ cos a,

E2e p2

e c2m20 c4

c2p2e m2

0 c4

|z (h f )2 c2 p2eÿ m2

0 c4

|zE2

e (h f ÿEeÿ )2 E2eÿ

ÿ2ch f peÿ ÿ cosa

(h f )2ÿ 2h f Eeÿ E2eÿ

(h f )2E2eÿ ÿ 2 ch f peÿ cos a

2h f Eeÿ 2h f cpeÿ cosaEeÿ cpeÿ cos aE2

eÿ c2 p2eÿ cos2 a

13 Kernphysik206

Page 207: Metzler Physik

c2 p2eÿ m2

0 c4 c2 p2eÿ cos2 a

cos2 a 1 m20 c4

p2eÿ c2

> 1

Da j cos a j stets [1 ist, ist der Prozess nicht mög-lich.

13.5.3 Anwendung von Radionukliden

1 Die Aktivität lebenden Holzes beträgt aufgrundseines 14C-Gehaltes A 0,208 sÿ1 je Gramm Koh-lenstoff. Die Halbwertszeit von 14C ist tH 5760 a.a) Welche Aktivität hat vor 50 Jahren geschlage-

nes Holz je Gramm Kohlenstoff?b) Welches Alter hat Holz aus einem ägyptischen

Königsgrab mit einer Aktivität von 6,5 minÿ1 jeGramm Kohlenstoff?

Lösung:a) A 0,207 Bq;b) A 5421 a

2 Die Halbwertszeit von Uran beträgt 4,5 ´ 109 a. Be-rechnen Sie das Alter eines Minerals, das ein Blei-atom auf zwei Uranatome enthält!

Lösung:N N0 eÿln 2=tH t, N0 3; N 2;ln(2=3) ± (ln2=tH) t; t 2,6 ´ 199.

13.5.4 Kernspaltung

*1 a) Berechnen Sie unter Anwendung der Formelfür die Bindungsenergie (! 13.4.2) mit denKonstanten b 17 MeV und c 0,69 MeVdie Differenz der Bindungsenergien zweierSpaltbruchstücke (A1 140, Z1 55, A2 95,Z2 37) und des Ausgangskerns.

b) Wie groû sind die ¾nderungen der Oberflä-chenenergie und der Coulomb-Energie? GebenSie dafür eine Erklärung.

Lösung:a) ESp EE(A1, Z1)EB(A2, Z2)

ÿEB(A1A2, Z1Z2)mit EB aAÿ bA2=3ÿ cZ2=A1=3;ESp ÿ164,9 MeV337,4 MeV172,5 MeV

b) Die Oberflächenenergie nimmt bei der Spal-tung des groûen Kerns in zwei kleinere um165 MeV zu, die Coulomb-Energie nimmt um337 MeV ab, sodass insgesamt 172 MeV freiwerden.Die Gesamtoberfläche der Spaltbruchstücke istgröûer als die des ungespaltenen Kerns; es lie-gen relativ mehr Nukleonen an der Oberflächeund wechselwirken daher mit weniger anderenNukleonen, da die Kernkraft nur kurze Reich-weite hat. Die Coulomb-Energie nimmt ab, dainsgesamt jedes einzelne Proton mit wenigeranderen Protonen mit der langreichweitigenCoulomb-Kraft wechselwirkt.

Seite

513

Seite

516

13.5 Künstliche Kernumwandlung und Kernspaltung 207

Page 208: Metzler Physik

13.6.1 Funktionsprinzipien von Reaktoren

1 Welchen Bruchteil kinetischer Energie kann einNeutron beim elastischen Stoû mit den in Ruhe be-findlichen Nukliden 2H, 9Be, 12C, 238U maximal ab-gegeben?

Lösung:Unter Verwendung der Beziehungen für die Ge-schwindigkeiten beim zentralen elastischen Stoûergibt sich für die Energie des gestoûenen Körpers:

Ek 12 mn v2

n

4mn mk

(mnmk)2 alsoEk

En 4mn mk

(mnmk)2 ,

2D: 0,89, 9Be: 0,36, 12C: 0,29, 238U: 0,017.

2 Welche Masse U235 wird bei der Explosion einerKernspaltungsbombe gespalten, deren Sprengkraft30 000 t TNT entspricht? (Bei der Explosion von1 kg TNT werden ca. 4 kJ frei.)

Lösung:m 1,46 ´ 10ÿ3 kg Uran werden gespalten, wennman pro Kernspaltung mit 200 MeV rechnet.

Seite

519

13 Kernphysik208

Page 209: Metzler Physik

14.4.3 Teilchenprozesse im Standardmodell

1 Welche Masse müsste ein Kraftträger-Teilchenhaben, das die Reichweite der Kernkräfte(D s 10ÿ15 m) hat (Yukawa-Teilchen)? Gibt esein Teilchen mit etwa dieser Masse?

Lösung:h=4 p[DE D tmc2 D s=cmc D s, alsom ]h=(4 p c D s 6,626 10ÿ34Js=(4p 3 108 m=s 10ÿ15 m) 1,76 10ÿ28 kg 200me

Dies ist etwa die Masse der Myonen und der Pio-nen.

2 Ergänzen Sie Abb. 537.3 um alle möglichen Gluo-nenaustauschprozesse.

Lösung:Von den farbändernden Gluonen fehlen inAbb. 537.3 lediglich das ¹rot-gelbeª und das ¹grün-gelbeª Gluon:

Quark

Anti-quark

gelb grün

rot blau grün

rot

türkis gelb purpur

gelb

3 Die (nahezu) konstante Kraft zwischen zwei ge-bundenen Quarks beträgt etwa 105 N. Auf welchenAbstand muss man zwei Quarks auseinanderzie-hen, damit die potentielle Energie des Gluonenban-des zur Erzeugung eines geladenen Pions aus-reicht?

Lösung:s W=F m c2=F 206me c2=105 N 1,7 10ÿ16 m

4 Beschreiben Sie die folgenden Teilchenprozesseder schwachen Wechselwirkung wie in Abb. 539.1mit Feynman-Diagrammen:a) m ! e ne nm

b) p ! m nm

c) 0! ppÿ

d) p! n e ne

e) p ne! n e

(erster Nachweis des Neutrinos!)

Lösung:

a)

e+

W +

e

+

µν

ν

µ

b)

d

W++

νµ

π

µ

c)

Ws

u

d p

u

d

u

d

u

π

Λ°

d)

W

u

d

p

d

e

u

νu

+

e

d

n

+

Seite

540

14.4 Das Standardmodell 209

Page 210: Metzler Physik

e)

W

u

d

p

d

e

u

ν

u

+e

d

n

+

5 Beschreiben Sie die folgenden Teilchenprozesseder starken Wechselwirkung wie in Abb. 539.2mit Feynman-Diagrammen:a) D ! p pb) n! ppÿ

c) p n!K 0

Lösung:

a)

u

p

d

g

u

π

u

+

d+

u

u

u

∆ +

b)

d

p

d

gu

π

n

d

u

u

u

d

c)

d

u

gs

π

n

+

d

u

s

u

d

u

d

Λ°

K+

Zusatzaufgaben

6 Die Abbildung zeigt die Nebelkammeraufnahme,mit der Anderson das Positron entdeckte. Die Ne-belkammer befand sich in einem Magnetfeld senk-recht zur Bildebene.

a) Wie bewegte sich das Positron und wie war dasMagnetfeld gerichtet?Die Auswertung der Bahnkurven des Teilchensin den beiden Bahnhälften ergab die WerteBr 2,1 10 5 Gauû cm undBr 7,5 10 4 Gauû cm (es gilt: 1 Gauû 10 ÿ4 T).

b) Zeigen Sie, dass sich das Teilchen mit relativisti-schen Geschwindigkeiten bewegte.

c) Berechnen Sie die Gesamtenergie des Teilchens inden beiden Bahnhälften. Als Ruhemasse soll im-mer die Masse des Elektrons angenommen werden.

Lösung:a) Beim Durchlaufen der Wand wird das Teilchen

langsamer. Es bewegte sich daher von der Seitemit der kleineren Bahnkrümmung zu der Seitemit der gröûeren Bahnkrümmung. Das Magnet-feld war daher in die Bildebene hinein gerich-tet.

b) Klassisch folgtv eB r=me e 0,21Tm=me

3,69 1010 m=s.c) p c m v c e B r c

0,21Tm e 3 108 m=s 63MeVWegen m0 c2 0,511MeVpc folgtE

(pc)2 (m0 c2)

p pc 63 MeV

Ebenso folgt E pc 22,5MeV.

14 Teilchenphysik210

Page 211: Metzler Physik

15.1.1 Optische Astronomie heute

1 Zeichnen Sie den Strahlengang in einem Linsente-leskop (Refraktor) und einem Spiegelteleskop (Re-flektor).

Lösung:

Licht-weg

Okular

Die Objektivlinsefokussiert das Licht, sodassdas Okular vergrößern kann.

Refraktor

Licht-weg

Okular

offenesRohrende

ebener Spiegel oder Prisma,die das Licht zum Okular lenken

ein Konkav-spiegel reflektiertund sammeltdas Licht

Reflektor

B

Stern 1

Stern 2

OkularObjektiv

αα

12

12

1

2

0

Obj Okf f

Strahlengang im Refraktor

Strahlengänge in Reflektoren

2 Was bestimmt die Vergröûerung eines Fernrohrs?

Lösung:Unter der Vergröûerung V eines Fernrohrs verstehtman das Verhältnis der Winkel, unter denen maneinen Gegenstand mit Fernrohr (a) und ohne Fern-rohr sieht (a0): V a=a0.Da die Winkel stets klein sind, kann man bei derHerleitung tan a a setzen. Dem Strahlengang imRefraktor (Aufgabe 1) entnimmt man, dass mandurch das Okular den Abstand B des Sterns 1 vonder optischen Achse unter dem Winkel a sieht. Esgilt: a B=fOk, wobei fOk die Okularbrennweite ist.Für den Winkel a0 gilt a0 B=FObj mit der Objek-tivbrennweite fObj. Daraus folgt

V a=a0 B=fOk=(B=fObj) fObj=fOk.

Diese Formel gilt auch für den Reflektor, wobei fObj

jetzt die Brennweite des Primärspiegels ist.

3 Wie groû ist der theoretische Durchmesser einesSternscheibchens beim 5,08 m-Palomar-Reflektor( f 8,5 m)?

Seite

543

15.1 Die Erforschung des Universiums 211

Page 212: Metzler Physik

Lösung:In 7.3.6 wird der Durchmesser D eines Beugungs-scheibchens zu D 2R b 2,44 l=d angegeben(! Abb. 303.2); für b ist hier die Objektivbrenn-weite fObj 8,5 m und für d der Spiegeldurchmes-ser d 5,08 m zu nehmen. Für grünes Licht(l 550 nm) erhält man

D 8,5m 2,44 550nm=5,08m 2,25mm.

4 Der Primärspiegel des Hubble-Teleskops wurdemit einer Genauigkeit von 1

78 einer Wellenlängegeschliffen und poliert. Angenommen der 2,4 m-Spiegel des Hubble-Teleskops wäre auf 3500 kmvergröûert (Durchmesser Europas). Wie groû wä-ren dann seine Unebenheiten?

Lösung:Wäre Europa so glatt wie der Teleskopspiegel, sohätten es nur Erhebungen von 1,4 cm.

x=3500km 10nm=2,5m; daraus folgt x1,4 cm.

15.1.3 Die Entfernung der Sterne und der Galaxien

1 Procyon hat die Parallaxe p 0,28600. BestimmenSie seine Entfernung in Parsec und in Lichtjahren.

Lösung:dProcyon 1,496 1011 m=0,28600

1,496 1011 m=(0,286 p=180 3600) 1,079 ´ 1017 m 3,50 pc 11,4 Lj

2 Bei gröûter Helligkeit hat d Cephei (291 pc ent-fernt) eine scheinbare Helligkeit von 3,m7. WelcheMagnitudo hätte er in der Magellan'schen Wolke(180 000 Lj entfernt)?

Lösung:Die Entfernung von d Cephei beträgt 291 pc 949 Lj. Die scheinbare Helligkeit eines Sternsnimmt ab mit dem Quadrat der Entfernung. Eingleich heller Stern in der Magellan'schen Wolkehätte daher eine um (180 000 Lj=949 Lj)236 000-mal kleinere scheinbare Helligkeit.

15.1.4 Die Expansion des Universums

1 Wie weit ist eine Galaxie entfernt, deren Spektrumeine Rotverschiebung von z 1,4 zeigt (H 75km=s=Mpc)?

Lösung:Mit der Formel für die Fluchtgeschwindigkeitv=c ((1 z)2ÿ 1)=((1 z)2 1) folgt für z 1,4die Fluchtgeschwindigkeit v 211000 km=s. Mitdem Hubble-Gesetz folgt für die Entfernungr v=H 211000 km=s=(75km=s=Mpc)

2800Mpc 9,2 Mrd. Lj.

2 Berechnen Sie aus der Fluchtgeschwindigkeit v274 000 km=s des Quasars OQ 172 dessen Entfer-nung.

Lösung:Mit dem Hubble-Gesetz folgt für die Entfernungdes Quasars OQ 172

r v=H 274000 km=s=(75km=s=Mpc) 3650Mpc 12 Mrd. Lj.

3 Setzen Sie im Hubble-Gesetz die Fluchtgeschwin-digkeit v c und berechnen Sie so den kosmischenRand r. Was soll dieser Begriff ausdrücken?

Lösung:r c=H 300000 km=s=(75km=s=Mpc)

4000 Mpc 13 Mrd. Lj.

Seit das Universum existiert, kann Licht nur dieseEntfernung von 13 Mrd. Lj zurückgelegt haben(vorausgesetzt die Hubble-Konstante hat den ange-nommenen Wert). Von gröûeren Entfernungenkann uns noch kein Licht bis heute erreicht haben.Daher die Bezeichnung kosmischer Rand.

Seite

547

Seite

549

15 Astrophysik212

Page 213: Metzler Physik

15.2.1 Leuchtkraft und Temperatur der Sterne

1 Zeichnen Sie ein Hertzsprung-Russell-Diagrammund tragen Sie die folgenden Sterne ein: Alderamin(a Cephei) 17 L=8500 K; Schedir (a Cassiopeiae)200 L=4100 K; g Lyrae 200 L=13 500 K; Cygni2000 L=5600 K; Algenib (a Persei) 5400 L=5800 K; Castor (a Geminorum) 22 L=10 600 K; Pollux (b Geminorum) 24 L=4900 K.Entscheiden Sie, zu welchem Typ die einzelnenSterne gehören.

2 Aus dem Maximum des spektralen Strahlungs-stroms der Sonne berechnet man deren Ober-flächentemperatur zu T 5780 K. Welchen Werthat lmax?

Lösung (zu 2):lmax 2,898 10ÿ3 mK=5780K 501 nm

Lösung (zu 1):

4

3

2

10

10

10

Leucht-kraft in L ·

Temperatur in K20000

Überriesen

Riesen

Hauptreihensterne

Weiße Zwerge

–4

–3

–2

10

10

10

–110

125

10

15000 10000 8000 6000 4000

Algenib

Cygni

-Lyraeγ

Castor

Aldemarin

Schedir

Pollux

Sonne

15.2.2 Die scheinbare Helligkeit

1 Um welchen Faktor unterscheiden sich die Strah-lungsströme zweier Sterne, deren scheinbareHelligkeiten sich um Dm 1 unterscheiden?

Lösung:Ist die Gröûenklasse um eins kleiner, so hat derStrahlungsstrom den 2,512fachen Wert:m1ÿm2 ÿ2m,5 log10 (s1=s2)

1m! s1=s2 10ÿ1=2;5 1=2,512.

2 Die Venus erreicht ihre Maximalhelligkeit vonÿ4,m 4 etwa 5±6 Wochen vor oder nach der gröûtenAnnäherung (untere Konjugation) und ist dann136 Mio. km von der Erde entfernt. BerechnenSie den Strahlungsstrom S und die absolute Hellig-keit M der Venus bei dieser Phase.

Lösung:Die Venus hat bei gröûter scheinbarer Helligkeitauf der Erde den 57,5fachen Strahlungsstrom derWega. In einer Entfernung von 10 pc wäre dieVenus für uns nur ein äuûØrst schwacher Stern; ihreabsolute Helligkeit M ist nur von 27. Gröûenklas-sen (Magnitudo).Aus mÿ2,5 lg (SVenus=SWega) folgtSVenus 10ÿ4;4=ÿ2;5 SWega 57,5 SWega.

Mit der Entfernungr136Miokm (1,36´1011m=3,086´1016m)´1pc

4,41 ´ 10ÿ6 pcfolgt die absolute HelligkeitM m 5ÿ 5 lg (r=p c) ÿ 4,4 5 ÿ 5 lg 4,41 ´ 10ÿ6 27,4

3 Berechnen Sie die Leuchtkraft von Regulus, demHauptstern im Sternbild Löwe (a Leonis), der 85 Ljentfernt ist und eine scheinbare Helligkeit von 1,m3besitzt.

Lösung:Mit der Formel für die Leuchtkraft L 4p r2 S, wo-bei r die Entfernung und S der Strahlungsstrom ist,folgt mit den Werten für Wega (a Lyrae)(! Tab. 552.1):L=LWega (r=rWega)2 S=SWega

) L 48,7 L8 (r=25,3Lj)2 10ÿm=2;5

) L 48,7 L8 (85Lj=25,3Lj)2 10ÿ1;3=2;5

166 L8

Regulus (a Leonis) hat die 166fache Leuchtkraftder Sonne.

Seite

551

Seite

552

15.2 Die Sterne 213

Page 214: Metzler Physik

15.2.3 Die Masse der Sterne

1 a Centauri ist mit 4,3 Lj Entfernung der sonnen-nächste Stern. Es ist ein Doppelsternsystem mit denscheinbaren Helligkeiten 0m,3 und 1m,7. Die beidenKomponenten haben einen Abstand von 17,6 Bo-gensekunden und eine Umlaufzeit von 80,1 Jahren.Die Entfernungen zum gemeinsamen Drehzentrumstehen im Verhältnis 9 : 13.a) Berechnen Sie den Abstand der beiden Kompo-

nenten in Astronomischen Einheiten (AE).b) Berechnen Sie die Massen der beiden Sterne.

Lösung:a) Der Abstand der beiden Komponenten beträgt

r 17;600 4,3 Lj 3,7 10ÿ4 Lj 3,47 ´ 1012= 149,6 Mio km ´ 1 AE 23 AE.

b) Für das Massenverhältnis der beiden Kompo-nenten gilt M=m b=a 13=9. Um die MasseM in Sonnenmassen M8 zu berechnen, kannman die Gleichung M 4p2 r3=(gT2(1 a=b))(! 15.2.3) für das System Sonne ± Erde auf-schreiben:M8 4p2(1AE)3=(g (1a)2 (1 0)).Bildet man aus beiden Gleichungen den Quo-tienten, so erhält man:M=M8 (r=1 AE)3 (1a=T)2 1=(1 a

b ) 233 (1=80,1)2 1=(1 9

13 ) 1,1.

Die gröûere Komponente hat die 1,1fache Son-nenmasse und die kleinere die 0,8fache.

15.2.4 Radius und Dichte der Sterne

1 a Leonis oder Regulus (lat., kleiner König) ist 85 Ljentfernt und der hellste Stern 1m,3 im SternbildLöwe. Seine Oberflächentemperatur beträgt13400 K. Berechnen Sie Leuchtkraft und Radiusin Sonneneinheiten.

Lösung:Mit der in Aufgabe 552/3 berechneten LeuchtkraftL 166L8 für a Leonis folgt für den Radius

R R8

L=L8

p(T8=T)2

R8

166p

(5780K=13400K)2 2,4R8.

2 Zeigen Sie, dass die Dichte von Hauptreihenster-nen mit zunehmender Masse abnimmt; es giltRStern MStern.

Lösung:Da der Radius eines Hauptreihensterns proportio-nal zur Masse ist, RStern MStern, gilt für die DichterStern MStern=VStern RStern=R3

Stern 1=R2Stern.

15.2.5 Sterneigenschaften und Masse

1 Geben Sie Leuchtkraft, Temperatur, Radius undLebensdauer zweier Hauptreihensterne in Einhei-ten der Sonne an, welche die 0,2- bzw. 20facheSonnenmasse besitzen.

Lösung:

LM3,5 T M3=8

0,2 M8 0,004 L8 3 160 K

20 M8 36 000 L8 17 800 K

RM t 1=M2,5

0,2 M8 0,2 R8 60 t820 M8 20 R8 0,0006 t8

2 Leiten Sie den Zusammenhang L f(T) für Sterneauf der Hauptreihe im HR-Diagramm her.

Lösung:Aus der Masse-Leuchtkraft-Beziehung LM3,5

und der Beziehung T M3=8,M T8=3 folgtL (T8=3)3,5 und daraus näherungsweise L T9.

Seite

553

Seite

554

Seite

555

15 Astrophysik214

Page 215: Metzler Physik

15.2.6 Wolken aus interstellarer Materie

1 Erklären Sie den Unterschied im Erscheinungsbildund in der Anregung von Reflexions- und Emis-sionsnebeln.

Lösung:Reflexionsnebel entstehen durch das von einerStaubwolke gestreute Licht eines Sterns. Da blauesLicht stärker gestreut wird als rotes Licht (so ent-steht auch das Blau des Himmels durch das ge-streute Sonnenlicht) leuchten Reflexionsnebelbläulich. Bei den Emissionsnebeln wird Gas ±meist Wasserstoff ± von der starken UV-Strahlungeines heiûen Sterns ionisiert. Bei der Rekombina-tion leuchtet es in charakteristischer rötlicherFarbe.

*2 Welcher Bedingung muss die thermische Energieder Teilchen einer Gaswolke genügen, damit dieseunter ihrer Gravitation kollabiert (Jeans'sches Kri-terium)?

Lösung:Wir betrachten dazu eine Gaskugel mit dem RadusR, dem Volumen V, der Masse M und der Dichte r.Ein Atom der Masse m hat denn an der Kugelober-fläche gegenüber einem unendlich weit entferntenPunkt die negative potentielle GravitionsenergieÿgmM=R (! 2.2.2); g ist die Gravitionskon-stante. Soll das Atom aus der Kugel entfernt wer-den, so ist demnach die Energie gmM=R aufzu-wenden. Ist die kinetische Energie der Teilchenaufgrund ihrer Temperaturbewegung im Mittelgröûer, so wird sich die Gaswolke ausdehnen; istsie hingegen kleiner, so zieht sich die Wolke zu-sammen. Mit Ekin 3

2 k T (! 4.2.1), wobei k dieBoltzmannsche Konstante und T die absolute Tem-peratur ist, folgt dann die Bedingung

32 k T < gmM=R.

Aus V M=r 43 pR3 folgt R

3M=(4pr)3

p;

setzt man dies in die obige Ungleichung ein undlöst nach M auf, so folgt die als Jeanssches Krite-rium bekannte Bedingung:

M >

81

32p

k

gm

3s

T3

r

s.

In der ersten Wurzel stehen nur konstante Werte.Die zweite Wurzel zeigt, dass die Masse um so grö-ûer sein muû, je höher die Temperatur des Gasesist; bei gröûeren Gasdichten kann die Gesamtmassedagegen kleiner sein. Setzt man typische Werte fürdie Temperatur T 100 K und die Dichter 10ÿ20 kg=m3 10ÿ23 g=cm3, das entspricht6 H-Atomen=cm3 (mH 1,7 ´ 1027 kg), ein, so folgt

M >

81

32p

1,38 10ÿ23JKÿ1

6,6 10ÿ11 Nm2 kgÿ2 1,7 10ÿ27 kg

3s

(100K)3

10ÿ20 kgmÿ3

s,

M > 1,22 1034 kg 6100 M.

Die anfängliche Gaswolke muss die Masse vonTausenden von Sonnenmassen enthalten, um durchihre eigene Gravitation zu kontrahieren.

Entfernung r

E

pot

kin

Gravitation 2

M, V · p

R

m

mMR

= –

=

1r

F ~

32 kT

E γ

15.2.7 Die Sternentstehung

1 Berechnen Sie aus der Solarkonstanten(1,37 kW=m2) den Massenverlust der Sonne injeder Sekunde.

Lösung:Aus der Solarkonstanten, das ist der auf der Erdegemessenen Strahlungsstrom der Sonne S8 1,37 kW=m2 berechnet man die Leuchtkraft L8,das ist die gesamte abgegebene Strahlungsleistungzu (! 15.2.1):

L8 4p r2 S8

S8 4 p (1,496 1011 m)2 1,37 kW=m2

3,85 ´ 1026 W.

Mit der Formel Em c2 ergibt sich eine Massever-lust von

Dm=Dt L8=c2 3,85 1026 J=s=(3 108 m=s)2

4,3 ´ 109 kg=s 4,3 Mio Tonnen=s.

Seite

557

Seite

559

15.2 Die Sterne 215

Page 216: Metzler Physik

Während ihrer bisherigen Lebensdauer von4,5 Mrd. Jahren hat die Sonne demnach eine Massevon 6,3 ´ 1026 kg verloren; das sind 0,3 Promille derSonnenmasse.

2 Schreiben Sie den Proton-Proton-Zyklus in einzel-nen kernchemischen Reaktionsgleichungen auf.

Lösung:Der Proton-Proton-Zyklus besteht aus den folgen-den Reaktionen:11H 1

1 H! 21 H 0

1 e ne

11H 2

1 H! 32 Heg

32He 3

2 He! 42 He 1

1 H 11 H

Zunächst vereinen sich zwei Wasserstoffkerne 1Hzu einem Deuteriumkern 2H, wobei ein Positronund ein Neutrino entstehen. Anschlieûend ver-einigt sich der Deuteriumkern 2H mit einem Was-serstoffkern 1H zu einem Heliumkern 3He. Schlieû-lich bilden zwei 3He-Kerne einen 4He-Kern undzwei Wasserstoffkerne 1H. Als Nettoeffekt vereini-gen sich demnach vier 1H-Kerne zu einem 4He-Kern, wobei insgesamt die Energie von 25,8 MeVfrei wird:

4 11H! 4

2 He 2 01e 2neg

15.2.8 Endstadien der Sternentwicklung

1 Warum üben bei einem Weiûen Zwerg die Elektro-nen und nicht die Protonen den Entartungsdruckaus?

Lösung:Wegen der rund 2000-mal geringeren Masse derElektronen im Vergleich zur Masse der Protonensind die Energiewerte der Elektronen im Potential-topf nach der Formel En h2 n2=(8mep a2) sehrviel höher als die der Protonen (!Abb. 560.1).Die Energie für die geringen Energiewerte der Pro-tonen könnten von der bei der Kontraktion abneh-menden Gravitationsenergie geliefert werden, so-dass bei einer Verkleinerung der Potentialtopf-breite a kein Druck von den Protonen entstehenkann, der dem Gravitationsdruck des Gleichge-wichts halten würde.

2 Berechnen Sie den Schwarzschild-Radius derErde. Erklären Sie, was der berechnete Wert be-deutet.

Lösung:Mit der Masse der Erde mE 6,0 1024 kg berech-net man den Schwarzschild-Radius zu 9 mm:

RS 2 gM

c2

2 667 10ÿ11 m3

kgs2 6 1024 kg

3 108

ms

2 9 mm.

Würde es gelingen, die Materie der Erde auf eineKugel von 9 mm Radius zu komprimieren, so wäreein Schwarzes Loch entstanden.

3 Wie groû wäre der Schwarzschild-Radius einesSchwarzen Lochs im Zentrum einer Galaxie mit5 106 M?

Lösung:Mit einer Masse von M 5 106 M8 5 106 2 1030 kg 1 1037 kg berechnet man denSchwarzschild-Radius entsprechend Aufgabe 2 zuRS 1,5 1010 m 21R8.Ein Schwarzes Loch, dessen Ausdehnung nichtgröûer als 21 Sonnenradien wäre, könnte sichdurchaus im Zentrum einer Galaxie ¸verbergen`.

Seite

563

15 Astrophysik216